Sunteți pe pagina 1din 310

LAURENŢIU PANAITOPOL

ALEXANDRU GICA

PROBLEME
DE ARITMETICǍ
ŞI TEORIA NUMERELOR

idei şi metode de rezolvare

Editura GIL
EBOOK – Probleme de artimetică și teoria numerelor – idei și metode de
rezolvare

ISBN 978-606-500-156-5

Copyright 2022 © Editura GIL


Toate drepturile asupra prezentei ediţii aparţin Editurii GIL. Nicio
parte din aceastǎ lucrare nu se poate reproduce fǎrǎ acordul scris al Editurii
GIL.

Acest ebook este protejat de legea drepturilor de autor. Fiecare PDF este
securizat în 28 de zone cu watermark invizibil (id comandă, e-mail) pentru
a nu putea fi distribuit pe alte căi virtuale. Sistemul este unul complex şi se
detectează automat persoana care a redistribuit cartea ilegal.

EDITURA GIL
www.gil.ro
tel: 0733 - 677 992
web: ebooks.gil.ro
e-mail: comenzi@gil.ro
Cuprins

Prefaţă 5

Capitolul 1. Numere prime. Numere compuse 7

Capitolul 2. C.m.m.d.c. şi C.m.m.m.c. 13

Capitolul 3. Divizibilitate 17

Capitolul 4. Partea ı̂ntreagă. Funcţia lui Legendre 23

Capitolul 5. Numere raţionale. Numere iraţionale 29

Capitolul 6. Baze de numeraţie. Suma cifrelor 35

Capitolul 7. Funcţii aritmetice 41

Capitolul 8. Distribuţia numerelor prime 51

Capitolul 9. Congruenţe 55

Capitolul 10. Resturi pătratice. Simbolul lui Legendre 63

Capitolul 11. Ordinul unui element (Gaussian) 69

Capitolul 12. Numere speciale 73

Capitolul 13. Teorema lui Dirichlet 77

Capitolul 14. Reprezentări aditive 81

Capitolul 15. Ecuaţii algebrice ı̂n numere ı̂ntregi 87

Capitolul 16. Ecuaţii diofantice exponenţiale 95

Capitolul 17. Diverse 99

Capitolul 1. Numere prime. Numere compuse 103

3
Capitolul 2. C.m.m.d.c şi C.m.m.c. 111

Capitolul 3. Divizibilitate 117

Capitolul 4. Partea ı̂ntreagă. Funcţia lui Legendre 131

Capitolul 5. Numere raţionale. Numere iraţionale 145

Capitolul 6. Baze de numere. Suma cifrelor 159

Capitolul 7. Funcţii aritmetice 167

Capitolul 8. Distribuţia numerelor prime 187

Capitolul 9. Congruenţe 197

Capitolul 10. Resturi pătratice. Simbolul lui Legendre 219

Capitolul 11. Ordinul unui element (Gaussian) 231

Capitolul 12. Numere speciale 239

Capitolul 13. Teorema lui Dirichlet 245

Capitolul 14. Reprezentări aditive 251

Capitolul 15. Ecuaţii algebrice ı̂n numere ı̂ntregi 263

Capitolul 16. Ecuaţii diofantice exponenţiale 279

Capitolul 17. Diverse 295

4
Prefaţă

”Problemele sunt inima matematicii” – Paul Halmos

Problemele sunt mediul natural ı̂n care se mişcă un matematician. Aici este
bucuria cea mai pură a unuia din branşă. Încercând să rezolvi probleme clasice,
retrăieşti istoria matematicii. Gândindu-te la problemele nerezolvate ı̂ncă, ai
vrea să contribui şi tu, după puterile tale, la edificiul matematicii. Acestea au
fost scopurile pe care le-am avut ı̂n minte scriind această carte: de a oferi o
panoramă a teoriei numerelor prin probleme şi de a deschide apetitul cititorului
pentru acest domeniu.
Această carte se adresează tuturor celor pasionaţi de teoria numerelor:
elevilor care se pregătesc pentru olimpiadă, profesorilor care doresc să-şi
”antreneze” cât mai bine elevii pentru concursuri sau celor care-şi pregătesc
lucrările de grad, studenţilor care urmează cursuri de ”Teoria Numerelor”.
Am folosit ı̂n carte multe probleme date la Olimpiada Internaţională de Mate-
matică, la Baraje sau la Balcaniade. Aceasta şi pentru că teoria numerelor
este una din cele patru ramuri ale matematicii vizate de problemele de la
OIM. Pentru a da un singur exemplu: ı̂n 2005, din cele 6 probleme de la OIM,
două au fost de teoria numerelor. În 2004 a fost o singură problemă, dar a fost
cea mai grea din concurs.
Fiecare capitol ı̂ncepe cu câteva rezultate teoretice menite pentru a-l ghida
mai bine pe cititor (şi a-l ajuta să-şi ı̂nsuşească o anumită cultură ı̂n domeniu).
Pentru cel care doreşte să intre mai mult ı̂n detaliile demonstraţiilor rezul-
tatelor clasice folosite ı̂n carte, ı̂i recomandăm să urmeze indicaţiile din text,
care trimit la două cărţi ale autorilor: ”O Introducere ı̂n Aritmetică şi Teo-
ria Numerelor”, Editura Universităţii din Bucureşti, 2001 (citată ı̂n interiorul
acestei cărţi cu denumirea ”curs”) şi ”Probleme celebre de Teoria Numerelor”,
Editura Universităţii din Bucureşti, 1998 (citată ı̂n interiorul acestei cărţi cu
denumirea de ”monografie”).

5
În afara rezultatelor teoretice, capitolele ı̂ncep şi cu câteva probleme re-
zolvate, aceasta pentru a sugera mai bine ideile şi metodele de rezolvare din
acel capitol. În interiorul fiecărui capitol problemele sunt dispuse gradual, ı̂n
funcţie de dificultatea lor. Cele din finalul capitolelor sunt o invitaţie către
matematica superioară, fără a ieşi totuşi din ”elementar”.
De câte ori a fost posibil, am indicat autorul problemei (sau al soluţiei)
sau concursul la care a fost dată problema. Dacă problema este clasică sau
autorul problemei este unul foarte cunoscut, am indicat numele autorului
la ”Enunţuri” (altfel, numele autorului este trecut la ”Soluţii”). Am făcut o
excepţie la ”problemele rezolvate” la ı̂nceput de capitol; menţiunea autorului
şi a concursului apare acolo imediat după enunţ.
Dorim să menţionăm că multe din problemele acestei cărţi aparţin
autorilor. Încheiem adresându-vă ı̂ndemnul de a ı̂ncerca sa rezolvaţi pe cont
propriu problemele. Numai aşa o să ”creşteţi”. Citiţi soluţiile autorilor doar
după ce aţi petrecut suficient timp ı̂ncercând să le rezolvaţi dumneavoastră.
Sperăm să fiţi fermecaţi, ca şi noi, de teoria numerelor.

Aprilie 2006 Autorii

6
CAPITOLUL 1

Numere prime. Numere compuse

Definiţie. Numărul natural p > 1 se numeşte prim dacă din ab = pc,


a, b, c ∈ N rezultă a = pa1 sau b = pb1 (a1 , b1 ∈ N). Numărul p > 1 se numeşte
ireductibil (indecompozabil) dacă din p = ab, a, b ∈ N rezultă a = 1 sau b = 1.
Cele două noţiuni definite mai sus coincid: deci p este prim ⇔ p este
ireductibil. Are loc teorema fundamentală a aritmeticii: orice număr natural
n > 1 are o descompunere unică ı̂n factori primi.

Definiţie. Numărul natural n > 1 se numeşte compus (reductibil, decom-


pozabil) dacă se poate scrie sub forma n = ab, a, b ∈ N, a > 1, b > 1.

Faptul că există o infinitate de numere prime, a fost demonstrat ı̂ncă din
antichitate de către Euclid, printr-un raţionament prin reducere la absurd. O
X∞ 1
altă demonstraţie se bazează pe faptul că seria este divergentă. De
m=1
m
X 1
fapt chiar seria este divergentă. Mai precis, are loc rezultatul următor:
p prim
p
X 1
dacă notăm f (x) = , atunci există o constantă b (b = 0, 26149...) astfel
p prim
p
ı̂ncât lim (f (x) − log log x − b) = 0.
x→∞
Mai multe informaţii ”analitice” despre numerele prime găsiţi ı̂n introduc-
erea de la Capitolul 8 ”Distribuţia numerelor prime”.
În antichitate se testa dacă un număr este prim folosind ”ciurul lui Era-
tostene”. Tot atunci se ştia că dacă un număr n este compus, atunci el are un

divizor prim mai mic sau egal cu n. O altă metodă de ”ciuruire” clasică este
cea a lui Fermat, numită a ”diferenţelor pătratelor”: pentru numărul natural

n > 1 se consideră numerele x ∈ N, x > [ n]. Dacă x2 − n = y 2 , y ∈ N,
x − y > 1, atunci n este compus, căci n = x2 − y 2 = (x − y)(x + y). Una
din cele mai sofisticate metode de găsire a unui factor prim al unui număr
dat este ”metoda curbei eliptice” a lui H. Lenstra. Interesul pentru găsirea
descompunerii ı̂n factori primi ai unui număr dat este şi de ordin practic. Cel

7
mai cunoscut sistem de criptare astăzi, RSA (de la numele celor care l-au
inventat: Rivest, Shamir şi Adleman), se bazează pe un număr natural n, care
este produsul a două numere prime mari. A ”sparge” (decripta) acest cod
ı̂nseamnă de fapt (ı̂n esenţă) a descompune pe n ı̂n factori primi.

Problema 1. Fie numerele prime n1 < n2 < · · · < n31 astfel ı̂ncât n41 +
n42 + · · · + n431 = 30n, n ∈ N. Să se arate că ı̂n secvenţa de mai sus, există 3
numere prime consecutive.
(Baraj 2003 juniori, Vasile Berghea)

Rezolvare. Vom arăta că n1 = 2, n2 = 3 şi n3 = 5. Dacă n1 > 2, atunci


toate numerele ni sunt impare, i = 1, 31 şi atunci n41 + n42 + · · · + n431 ar trebui
31
X
să fie impar. Însă n4i = 30n este un număr par şi deci n1 = 2.
i=1
Să presupunem acum că n2 > 3. Atunci n4i = 3mi + 1, mi ∈ N, ∀ i = 1, 31.
Aceasta rezultă imediat ţinând cont că k 2 = M3 + 1, dacă k este un număr
ı̂ntreg care nu este multiplu de 3. Obţinem contradicţia
31
X 31
X
30n = n4i = (3mi + 1) = M3 + 31 = M3 + 1.
i=1 i=1

Deci n2 = 3. Să presupunem acum că n3 > 5. Atunci n4i = 5qi + 1, qi ∈ N,


∀ i = 1, 31 (deoarece ni nu este multiplu de 5, conform presupunerii făcute).
31
X 31
X
Obţinem contradicţia 30n = n4i = (5qi + 1) = M5 + 31 = M5 + 1.
i=1 i=1
Deci n3 = 5 şi enunţul este demonstrat.

Problema 2. Să se rezolve ecuaţia 2q = 1999 + p2 , p şi q fiind numere


prime.
(Olimpiadă Rusia, 1999)

Rezolvare. Vom folosi observaţiile evidente 23k = M7+1, 23k+1 = M7+2,


23k+2 = M7 + 4 (∀ k ∈ N). Evident că q = 3 nu este soluţie pentru problema
noastră. Dacă q = 3k + 1, k ∈ N, atunci p2 = 2q − 1999 = 23k+1 − M7 − 4 =
M7 + 2 − 4 = M7 + 5. Aceasta este o contradicţie căci restul ı̂mpărţirii lui p2
la 7 nu poate fi decât 0, 1, 2 sau 4. Deci q = 3k + 2, k ∈ N şi p2 = 2q − 1999 =
23k+2 − M7 − 4 = M7 + 4 − M7 − 4 = M7. Deci p este multiplu de 7. Fiind
număr prim, deducem că p = 7 şi 2q = 1999 + 72 = 2048 = 211 , q = 11.
Deci singura soluţie a problemei este p = 7, q = 11.

8
1. Să se determine numerele prime a şi b, pentru care ab + 1 şi ab − 1
sunt, de asemenea, prime.

2. Să se determine numerele prime p, pentru care:


a) 2p2 + 1 este prim;
b) 2p2 − 3 şi 2p2 + 3 sunt prime;
c) p + 4, p + 24, p2 + 10, p2 + 34 sunt prime.

3. Ştiind că a, b, c şi a4 +b4 +c4 −3 sunt prime, să se arate că a2 +b2 +c2 −1
este, de asemenea, prim.

4. Să se determine n, pentru care toate numerele n + 1, n + 3, n + 7, n + 9,


n + 13, n + 15 să fie prime.

5. Dacă p şi q sunt prime şi a este natural, a2 = p2 + q 2 + 1, atunci a este


prim.
. .
6. Există numere prime distincte p, q, r astfel ca p2 + 10 .. qr, q 2 + 10 .. pr
.
şi r2 + 10 .. pq? Dar dacă 10 se ı̂nlocuieşte cu 11?

7. Să se găsească toate numerele prime p pentru care 2p + p2 este număr


prim.

8. Care este cel mai mare număr de numere prime printre 10 numere
consecutive?

9. Să se arate că ı̂ntr-o progresie aritmetică de numere naturale, există o


infinitate de numere compuse, dacă raţia nu este 0.

10. Să se determine numerele naturale care nu pot fi scrise ca sumă a


două numere compuse.

24n+2 + 1
11. Să se arate că numărul este ı̂ntreg şi compus pentru n ≥ 2.
5
12. Să se arate că 13n + 2 este număr compus dar nu este pătrat perfect
pentru orice număr natural nenul n.

13. Dacă m, n ∈ N, n ≥ 2, atunci m4 + 4n4 nu e prim.


(OIM 1969)

14. Să se arate că ı̂n şirul (an )n≥1 , an = n4 + (n + 1)4 există o infinitate
de numere compuse.

9
15. Să se descompună ı̂n factori primi 104060401.

16. Să se arate că numărul n = 1.280.000.401 este compus.

17. Să se descompună ı̂n factori primi numărul 7519 folosind metoda
pătratelor a lui Fermat.

18. Fie E(n) = |11 {z


. . . 1}.
n cifre
a) Să se arate că pentru 3 ≤ n ≤ 10, E(n) este compus.
b) Dacă E(n) este prim, atunci n este prim.

a−b b−c c−a


19. Dacă a, b, c sunt numere prime şi numărul E = + +
c a b
este ı̂ntreg, să se arate că E = 0.

20. Fie numărul xn = 101010 . . . 1 cu n cifre de 1. Să se determine nu-


merele n pentru care xn este prim.

21. Dacă n se scrie ı̂n două moduri ca sumă de două pătrate, atunci n
este compus.

22. Să se arate că dintre trei numere consecutive mai mari ca 7, cel puţin
unul are doi factori primi distincţi.

23. Să se arate că dintre 24 de numere consecutive mai mari ca 6, există
unul cu cel puţin 3 factori primi distincţi.

24. Fie m număr natural nenul. Să se arate că ı̂n şirul (xn )n≥1 , xn = 2n −1
există m termeni consecutivi care sunt numere compuse.
n +k
25. Fie numerele naturale n ≥ k ≥ 1. Să se arate că numărul 22 +1
este compus.

26. Dacă a şi b sunt numere naturale, atunci există o infinitate de numere
naturale n pentru care 2n + 2a + 2b + 1 este compus.

27. Dacă n ≥ 2, p, q sunt prime şi pn + q n = r2 , să se arate că r este


compus.
(Baraj 2004)

28. Să se arate că există o infinitate de numere naturale a, astfel ı̂ncât
numerele M1 = n3 + 4n + a şi M2 = n2 + 3n + a să fie compuse pentru orice
n natural.

10
29. Să se determine cel mai mic număr natural a, astfel ı̂ncât pentru orice
n = 0, 9, numerele xn = 210n + a să fie prime.

30. Să se determine cel mai mare număr natural care nu poate fi scris ı̂n
cel puţin zece moduri ca sumă de două numere compuse (nu se face distincţie
ı̂ntre n = a + b şi n = b + a).

31. Găsiţi ı̂ntregii n ≥ 2 pentru care toate numerele naturale, care se


scriu cu n − 1 cifre 1 şi o cifră 7, sunt prime.
(Propusă OIM, 1990)

32. Descompuneţi numărul 51985 − 1 ı̂n trei factori mai mari ca 5100 .
(Propusă OIM, 1985)
p
33. Să se arate că pentru orice număr prim p, numărul C2p − 2 se divide
2
la p .
p
34. Să se arate că pentru orice număr prim p, numărul C3p − 3 se divide
la p2 .

35. Să se arate că polinomul f (a, b, c) = a3 + b3 + c3 − 3abc furnizează


toate numerele prime, cu excepţia lui 3.
n
36. Să se determine m, 0 ≤ m ≤ 2n , dacă numărul 22 + 2m + 3 este
prim.

11
CAPITOLUL 2

C.m.m.d.c. şi C.m.m.m.c.

Pentru a ∈ Z, a 6= 0, notăm Da = {n ∈ N | n | a}.


Definiţie. Pentru a, b numere ı̂ntregi nenule, cel mai mare divizor comun
al numerelor a, b este cel mai mare element al mulţimii Da ∩ Db . Se notează
cu d = (a, b). Dacă b = 0, a 6= 0, definim (a, 0) = |a|. Se arată că d = (a, b) are
următoarele proprietăţi:
i) d|a şi d|b;
ii) dacă d0 ∈ Z este astfel ı̂ncât d0 |a şi d0 |b, atunci d0 |d.
k
Y k
Y βi
Dacă a = pαi i , b= pi , unde p1 < p2 < · · · < pk sunt numere prime, αi ,
i=1 i=1
k
Y min(αi ,βi )
βi ∈ N, αi + βi ≥ 1, ∀ i = 1, k, atunci (a, b) = pi . O altă formulă de
i=1
calcul (algoritmul lui Euclid) se bazează pe teorema ı̂mpărţirii cu rest:

a = bq1 + r1 , 1 ≤ r1 < b
b = r1 q2 + r2 , 1 ≤ r2 < r1
..
.
rn−2 = rn−1 qn + rn , 1 ≤ rn < rn−1
rn−1 = rn qn+1 .

Atunci (a, b) = rn (am presupus mai sus că b ∈ N∗ ).


Dacă (a, b) = 1 spunem că a şi b sunt prime ı̂ntre ele. Dacă a şi b nu sunt
ambele zero, se arată că există α, β ∈ Z astfel ı̂ncât (a, b) = aα + bβ.
Un rezultat pe care-l vom folosi frecvent ı̂n carte este următorul: dacă
(a, b) = 1 şi a · b = ck (k ∈ N, k ≥ 2), atunci a = ±ak1 şi b = ±bk1 , a1 , b1 ∈ Z.
Pentru a ∈ Z, a 6= 0, notăm Ma = {n ∈ N∗ | a | n}.

Definiţie. Pentru a şi b numere ı̂ntregi nenule, cel mai mic multiplu comun
al numerelor a, b, notat cu [a, b], este cel mai mic element al mulţimii Ma ∩ Mb .
Dacă a şi b sunt numere naturale nenule, atunci [a, b] · (a, b) = a · b.

13
k
Y k
Y βi
Dacă a = pαi i , b = pi , unde p1 < p2 < · · · < pk sunt numere prime
i=1 i=1
k
Y max(αi ,βi )
şi αi , βi ∈ N, αi + βi ≥ 1, ∀ i = 1, r, atunci [a, b] = pi .
i=1

21n + 4
Problema 1. Să se demonstreze că fracţia este ireductibilă pentru
14n + 3
orice număr natural n.
(OIM, 1959)

Rezolvare. Trebuie arătat că (21n + 4, 14n + 3) = 1, ∀ n ∈ N. Aceasta


rezultă imediat din egalitatea 3(14n + 3) − 2(21n + 4) = 1 şi din faptul că (a, b)
divide ka + lb, ∀ k, l ∈ Z.
Problema 2. Să se demonstreze că şirul an = 2n − 3 (n ≥ 2) conţine o
infinitate de numere prime ı̂ntre ele două câte două.
(OIM, 1971)

Rezolvare. Construim prin inducţie secvenţa căutată: n1 = 2, a1 = 1,


n2 = 3, a2 = 5. Presupunem că am construit numerele naturale 2 = n1 < n2 <
· · · < nk astfel ı̂ncât (ani , anj ) = 1, ∀ 1 ≤ i < j ≤ k. Fie m = a1 · a2 · . . . · ak .
Considerăm numerele 20 , 21 , . . . , 2m şi resturile ı̂mpărţirii lor la m. Deducem că
există 0 ≤ i < j ≤ m astfel ı̂ncât m|2j −2i (aceasta deoarece avem m+1 resturi
din mulţimea {0, 1, 2, . . . , m − 1}. Deoarece m este impar (an = 2n − 3 este
impar ∀ n ∈ N, n ≥ 2), rezultă că m divide 2j−i −1. Alegem nk+1 = t(j −i)+2,
unde t ∈ N∗ este ales astfel ı̂ncât nk+1 > nk . Deoarece m|2j−i − 1, deducem
€ Š
că restul ı̂mpărţirii lui ank+1 la m este 22 − 3 = 1. Deci ank+1 , m = 1 şi de
aici rezultă pasul de inducţie.

1. Fie a, b numere naturale nenule. Să se arate că numerele (a, b) şi [a, b]
sunt pătrate perfecte dacă şi numai dacă a şi b sunt pătrate perfecte.

2. Dacă a, b, c sunt numere naturale impare, să se arate că


 
a+b b+c c+a
(a, b, c) = , , .
2 2 2
3. Dacă a, b, c sunt numere naturale, [a, b] = [a, c] şi (a, b) = (a, c), atunci
b = c.

4. Dacă a, b ∈ N∗ şi 3[a, b] = a2 − b2 , atunci a = 2b.

5. Dacă a, b ∈ N∗ şi [a, b] = a2 − b, atunci a2 = 2b.

14
6. Să se determine numerele naturale nenule a şi b dacă a2 + b2 = [a, b] +
7(a, b).

7. Dacă a, b sunt numere naturale şi m = [a, b], atunci aa + bb > mm dacă
şi numai dacă a | b sau b | a.

8. Fie a, b, n numere naturale nenule pentru care a2 + b2 = n[a, b] + (a, b).


Să se arate că numerele a şi b sunt consecutive dacă şi numai dacă n = 2.

9. Să se determine numerele naturale a, b, n pentru care avem [a, b] =


(a, b)3 şi a + b = 30n .

10. Dacă suma a trei numere naturale este egală cu cel mai mic multiplu
comun al lor, atunci unul dintre numere este egal cu suma celorlalte două.

11. Dacă a, b, c sunt numere naturale, atunci avem:


[a, b, c] · (a, b) · (b, c) · (c, a) = abc(a, b, c).

12. Fie a, b, c numere naturale nenule astfel ı̂ncât: (a, b, c) · [a, b, c] = ab.
Să se arate că (a, b) divide c şi c divide [a, b].

13. Să se determine numerele naturale a, b, c pentru care:


i) (a, b, c) = 6, [a, b, c] = 1440, a + b + c = 234;
ii) (a, b, c) = 12, [a, b, c] = 420, ab + bc + ca = 65520.

m
14. Dacă m, n sunt numere naturale nenule şi este impar, să se
(m, n)
arate că (3m − 1, 3n + 1) = 2.

15. Dacă a şi n sunt numere naturale, să se calculeze


 2 +3n−1 2 +3n+2

a2n − 1, an +1 .

16. Pentru m şi n numere naturale, să se arate că


(2m − 1, 2n − 1) = 2(m,n) − 1.

17. Fie An = n9 − n. Să se afle cel mai mare divizor comun al numerelor
A1 , A2 , A3 , . . . , A2005 .

18. Fie An = 23n + 36n+2 + 56n+2 . Să se afle cel mai mare divizor comun
al numerelor A0 , A1 , . . . , A2005 .

19. Să se arate că [1, 2, 3, . . . , n] ≤ nπ(n) .

15
20. Se consideră şirul de numere naturale nenule (an )n≥1 , an+1 = an +
(an , an−1 ) pentru orice n ≥ 2. Să se determine termenul general.
21. Fie a şi n numere naturale şi d = (a, n). Să se arate că dintre ele-
mentele mulţimii A = {ak | 1 ≤ k ≤ n} exact d se divid cu n.
22. Fie a, n numere naturale cu a > 1. Să se arate că
„ Ž
an − 1
,a − 1 = (a − 1, n).
a−1

23. Fie a număr natural şi (xn )n≥1 , xn = an + n pentru orice n ≥ 1.


Să se arate că pentru orice n avem (xn , xn+1 , xn+2 ) = 1.
24. Să se arate că există o infinitate de triplete de numere naturale (a, b, c),
astfel ı̂ncât (a, b) = (a, c) + 1 = (b, c) + 2.
€ Š
25. Să se arate că avem Cnm , Cnm+1 = 1, dacă şi numai dacă m ∈ {0, n−1}.
26. Dacă a, b, n sunt numere naturale nenule, să se arate că dacă (a, b) = 1,
avem [a + n, b + n, n2 ] > n2 .
27. Fie a, b, m ∈ N∗ , (a, b) = 1. Să se arate că există o infinitate de k ∈ N
astfel ı̂ncât (a + kb, m) = 1.
28. Să se arate că dacă P ∈ Z[X], grad P ≥ 2, P (0) = P (1) = 1 şi
x0 ∈ Z, xn+1 = P (xn ), atunci dacă i 6= j avem (xi , xj ) = 1.
29. Fie {an }n≥1 un şir de numere naturale nenule. Notăm xn = (an , an+1 ),
yn = [an , an+1 ] pentru orice n ≥ 1. Dacă şirurile {xn }n≥1 , {yn }n≥1 sunt progre-
sii geometrice şi există trei termeni consecutivi ai şirului {an }n≥1 ı̂n progresie
geometrică, atunci şirul {an }n≥1 este o progresie geometrică.
30. Să se arate că nu există un şir strict crescător de numere naturale
(an )n≥1 pentru care, oricare ar fi i, j avem [ai , aj ] < (ai , aj )2 , ı̂nsă există
pentru orice n numerele 1 ≤ a1 < a2 < · · · < an , astfel că pentru i, j ≤ n
avem [ai , aj ] < (ai , aj )2 .
31. Fie a1 , a2 , . . . , ak , n numere naturale cu 1 < a1 < a2 < · · · < ak < n
1 1 1
şi [ai , aj ] > n pentru orice i, j ≤ k, i 6= j. Să se arate că + + · · · + < 2.
a1 a2 ak
32. Fie numerele naturale 1 ≤ a0 < a1 < a2 < · · · < an . Să se arate că
X n 1 1
≤ 1 − n+1 .
[a , a ]
k=0 k k+1
2

16
CAPITOLUL 3

Divizibilitate

Definiţie. Dacă a şi b sunt numere ı̂ntregi, spunem că a divide b dacă
.
există c ∈ Z astfel ı̂ncât b = ac. Notăm acest lucru prin a|b, b..a sau b = Ma.

Faptul că b nu se divide cu a se notează prin a¤|¤b, b..a,
¦ b 6= Ma.

Problemă. Fie n ∈ N, n ≥ 3 şi notăm S = {a ∈ N | 1 < a < n, n | aa−1 −1}.


Să se arate că dacă S = {n − 1}, atunci n = 2q, unde q e număr prim.
(Baraj 2002, Mihai Cipu şi Nicolae Ciprian Bonciocat)

Rezolvare. Pasul 1. Arătăm că n este par. Să presupunem că n ar fi


impar. Deoarece n − 1 ∈ S deducem că n|(n − 1)n−2 − 1. Dar (n − 1)n−2 − 1 =
Mn + (−1)n−2 − 1 = Mn − 2 (deoarece n e impar) şi deducem că n|2, n = 1;
contradicţie. Evident că numărul 4 are proprietăţile din enunţ şi ı̂n continuare
vom presupune că n > 4 este ca ı̂n problemă.
Pasul 2. Arătăm că nu există q prim astfel ı̂ncât q 2 |n. Să presupunem că
n
∃ q prim astfel ı̂ncât q 2 |n. Vom arăta că +1 ∈ S. Pentru aceasta să observăm
q
 k  k  k−2
n n n n
că n , ∀ k ∈ N, k ≥ 2. Într-adevăr, = · 2 · n este multiplu
q q q q
n
de n, deoarece q 2 |n. Notând a = + 1 avem că
q
n
 n  k  2
n q X
q
n n n
aa−1 − 1 = +1 −1 = C kn + · C 1n + 1 − 1 = M = Mn,
q k=2
q q q q q

n n
conform observaţiei precedente. Pe de altă parte, 1 < +1=a≤ +1<n
q 2
n n
şi deci a ∈ S = {n − 1}. Deducem că + 1 = n − 1 şi cum q|n şi q (deoarece
q q
n n
q 2 |n), rezultă că q|2, q = 2, + 1 = n − 1, = 2, n = 4; contradicţie, căci am
2 2
presupus că n > 4. Am arătat deci că ¶ ∃ q prim astfel ı̂ncât q 2 |n.

17
Pasul 3. Arătăm că n = 2q, q prim. Din cele arătate anterior deducem
că n = 2q1 q2 . . . qr , unde 3 ≤ q1 < q2 . . . qr sunt numere prime distincte.
Trebuie să arătăm că r = 1. Să presupunem că r ≥ 2. Considerăm numerele
n n
s · , unde s parcurge mulţimea {0, 1, 2, . . . , q1 − 1}. Deoarece q1 ¤|¤ , rezultă
q1 q1
că resturile ı̂mpărţirii numerelor de mai sus la q1 vor coincide (ca mulţime)
cu {0, 1, 2, . . . , q1 − 1}. Deducem că ∃ t ∈ {0, 1, 2, . . . , q1 − 1} astfel ı̂ncât
n n
t · = bq1 + q1 − 2, b ∈ N. Vom arăta că a = t · + 1 ∈ S. Dacă a = 1 rezultă
q1 q1
n q1 − 1
că t = 0, q1 |2, q1 = 2; contradicţie. Deci a > 1 şi a = t · + 1 ≤ n+1 =
q1 q1
 
1 n
1− n + 1 < n (ultima inegalitate este echivalentă cu 1 < , q1 < n;
q1 q1
aceasta este evident). Avem că
„ Žt n
q1
a−1
n n n
a −1= t +1 −1=M +1−1=M .
q1 q1 q1

n
Deoarece t + 1 = q1 (b + 1) − 1 = Mq1 − 1, deducem şi că aa−1 − 1 =
q1
  n
tq
n 1 tn tn
t +1 − 1 = (Mq1 − 1) q1 − 1 = Mq1 + (−1) q1 − 1 = Mq1 (am folosit
q1
 
n a−1
n a−1 n
că 2 ). Deci q1 |a − 1, a − 1 şi cum q1 , = 1 (deoarece q12 ¤|¤n)
q1 q1 q1
n
rezultă că n|aa−1 − 1. Deci a ∈ S = {n − 1}, t + 1 = n − 1. Cum q2 |n şi
q1
n
q2 , din ultima egalitate rezultă contradicţia q2 |2, q2 = 2.
q1
Deci r = 1 şi enunţul este demonstrat.
Observaţie. Afirmaţia reciprocă nu este adevărată. Luăm de exemplu n =
26 şi avem că 5 ∈ S, 5 6= 25. Avem că 5 ∈ S deoarece 26|54 − 1 = 24 · 26.

1. Să se arate că numărul k = an an−1 . . . a1 a0 se divide cu 8 dacă şi numai


dacă 4a2 + 2a1 + a0 se divide cu 8.

2. Să se arate că numărul k = an an−1 . . . a1 a0 se divide cu 49 dacă şi


numai dacă 2an an−1 . . . a2 + a1 a0 se divide cu 49 şi se divide cu 17 dacă numai
dacă 2 an an−1 . . . a2 − a1 a0 se divide cu 17.

18
3. Să se arate că numărul natural k = an an−1 . . . a1 a0 se divide cu 101
dacă şi numai dacă a1 a0 − a3 a2 + a5 a4 − . . . se divide cu 101.

4. Formulaţi criteriul de divizibilitate cu 103 şi cu 97.

5. Să se găsească cifra a ∈ {0, 1, . . . , 9} astfel ı̂ncât numărul n = 123a456


să fie divizibil la 7, 11, 13, 37.

3n2 + n − 1
6. Pentru ce numere ı̂ntregi n, numărul a = este ı̂ntreg?
2n + 1
7. Să se determine cel mai mic număr n ≥ 1 pentru care fracţia
3n2 + 2
E(n) = 3 se simplifică.
2n + 1

3n2 + a
8. Dacă a, b ∈ Z şi 4a3 + 27b2 = ±1, atunci fracţia E(n) = este
2n3 + b
ireductibilă.

9. Dacă m şi n sunt numere naturale nenule şi 2n + 3m se divide cu 5,


atunci 2m + 3n se divide cu 5.

10. Dacă n ≥ 6 este număr compus, atunci (n − 2)! se divide la n.


n
X
11. Fie n ≥ 2. Să se arate că n! se divide la k dacă şi numai dacă
k=1
n + 1 este compus.

12. Să se arate că dacă a, b, c sunt numere ı̂ntregi şi 7 | a3 + b3 + c3 , atunci
7 | abc.

13. Să se arate că orice număr natural prim cu 10 admite un multiplu
format numai cu cifre de 1.
1 1 1 1
14. Dacă p = 6m + 5 este număr prim şi 1 − + − +···− +
2 3 4 4m + 2
1 a
= atunci p | a.
4m + 3 b
15. Să se arate că pentru k, n ∈ N∗ , k impar, avem că 1 + 2 + 3 + · · · + n
divide 1k + 2k + · · · + nk .

16. Să se găsească numerele n ∈ N∗ pentru care 2n | 3n − 1.


(Baraj 2005)

19
17. Determinaţi un număr n care are exact 2005 divizori primi distincţi
şi pentru care n | 2n + 1.
(prelucrare OIM, 2000)

18. Pentru ce numere n ∈ N∗ se ı̂ntâmplă că [ n] | n?

19. Dacă a, b sunt numere naturale nenule, să se determine numerele


n+a n+b
naturale n pentru care + este număr natural.
b a
20. Să se găsească a, b ∈ N astfel ı̂ncât a2 + b2 = q(a + b) + r, q, r ∈ N,
r < (a + b) şi q 2 + r = 1977.
(OIM, 1977)

21. Fie n ∈ N∗ . Să se calculeze numărul de perechi (a, b) (a, b ∈ N∗ )


1 1 1
pentru care + = .
a b n
22. Să se determine a ∈ N∗ dacă există n ∈ N∗ astfel ca an+1 + 2n+1 + 1
se divide cu an + 2n + 1.
(Baraj 1998)

2a + 1 2a − 1
23. Fie a şi b naturale, b > 2. Să se arate că numerele şi
2b − 1 2b + 1
nu sunt ı̂ntregi.

24. Să se arate că oricare ar fi m şi n naturale, numărul 2002m + 2003
nu divide pe 2146n .

25. Fie numerele ı̂ntregi a şi b. Să se arate că există ı̂ntregii A şi B astfel
ı̂ncât pentru orice număr ı̂ntreg n, an + A şi bn + B sunt prime ı̂ntre ele.
(Propusă OIM, 1984)

26. Să se rezolve ı̂n N∗ ecuaţia (a + b)3 = k(a2 + b2 ) (k este număr natural
nenul fixat).

27. Dacă a, b, c, m, n, p sunt numere naturale nenule cu am |bn , bp |cm , cn |ap


şi b = 72, să se calculeze a şi c.

28. Fie a, b, c, d numere ı̂ntregi a > b > c > d > 0. Se presupune că
ac + bd = (b + d + a − c)(b + d − a + c). Să se arate că numărul ab + cd nu este
prim.
(OIM, 2001)

20
29. Să se găsească toate numerele a, b, c cu proprietatea că 1 < a < b < c
abc − 1
şi ∈ N.
(a − 1)(b − 1)(c − 1)
(OIM, 1992)

30. Se consideră şirul (an )n≥1 pentru care a1 = 2 iar pentru n ≥ 1, an+1
este cel mai mare divizor prim al numărului a1 a2 · . . . · an + 1. Să se arate că
5 nu se află ı̂n şir.

31. Fie a1 ∈ N∗ . Se defineşte an+1 ca fiind produsul factorilor primi ai


numărului an + 1. Să se arate că există un n0 ∈ N∗ astfel ı̂ncât an0 = 2.

32. Fie un şir de numere naturale (an )n≥1 . Dacă a1 are şase cifre şi pentru
orice n, an+1 este divizor prim al lui an + 1, să se calculeze a19 + a20 .

33. Fie a1 ∈ N∗ şi an+1 este cel mai mare divizor prim al numărului
an + n, ∀ n ∈ N∗ . Să se arate că şirul an este nemărginit.

34. Fie o progresie aritmetică de numere naturale cu raţia nenulă. Să se


arate că există o infinitate de numere naturale n, pentru care suma primilor
n termeni ai progresiei să fie un termen al progresiei.

35. Să se arate că pentru orice k ı̂ntreg, k ≥ 2, există o infinitate de puteri
de ordin k, ce nu se pot scrie ca o sumă dintre un număr prim şi o putere de
ordin k.

36. Dacă n numere prime formează o progresie aritmetică, atunci raţia


progresiei se divide la orice număr prim p < n.
(Cantor )

37. Fie numărul real a ≥ 1, astfel ı̂ncât pentru orice pereche (m, n) cu
.. .
m . n, rezultă [ma]..[na]. Să se arate că a este natural.
(Iberoamerican, 1996)

38. Să se găsească un factor cuprins ı̂ntre 1000 şi 5000 al numărului
233 − 219 − 217 − 1.
(Noam Elkies)

39. Fie n ∈ N, n ≥ 2, d1 = 1 < d2 < d3 < · · · < dk = n toţi divizorii


naturali ai lui n şi D = d1 d2 + d2 d3 + · · · + dk−1 dk . Să se arate că D < n2 şi
să se găsească acele numere n pentru care D|n2 .
(OIM, 2002)

21
40. Fie un şir (an )n≥0 cu a0 = 1 şi an+1 = 2an + n · 2n .
a) Să se determine n pentru care an este o putere a lui 2.
b) Să se determine n pentru care an este pătrat perfect.

a2 + b2 + c2
41. Fie a, b, c numere ı̂ntregi pentru care este număr ı̂ntreg.
a+b+c
an + bn + cn
Să se arate că există o infinitate de numere naturale n astfel ı̂ncât
a+b+c
este număr ı̂ntreg.
(Baraj, 1990)

42. ∀ n numărul xn = 78557 · 2n + 1 este compus.


(Selfridge)

43. Fie k ∈ N, k ≥ 3 şi sirul (an )n≥1 definit prin relaţia de recurenţă
an+k = an+1 + an , ∀ n ∈ N∗ ; aj = 0, ∀ j = 1, k − 2, ak−1 = k − 1, ak = k.
Să se arate că pentru orice prim p, ap este multiplu de p.

22
CAPITOLUL 4

Partea ı̂ntreagă. Funcţia lui Legendre

Definiţie. Pentru numărul real x, se numeşte partea ı̂ntreagă a lui x cel


mai mare ı̂ntreg care nu-l depăşeşte pe x. Partea ı̂ntreagă a lui x se notează
cu [x]. Notăm {x} = x − [x]; {x} se numeşte partea fracţionară a lui x. Avem
inegalităţile x − 1 < [x] ≤ x, 0 ≤ {x} < 1.
" # " #

[x] x
Pentru orice n ∈ N avem egalitatea = , valabilă pentru orice
n n
x ∈ R.

Definiţie. Pentru n ∈ N∗ şi p număr prim, se notează cu ep (n) exponentul


lui p din descompunerea ı̂n factori primi a lui n!. Aceasta este funcţia lui
" #
X∞ n
Legendre. Acesta a arătat că ep (n) = .
i=1
pi
(" # )
√ 1
Problema 1. Să se arate că n + n + |n ∈ N∗ = N\{α2 |α ∈ N}.
2
(" # )
√ 1
Rezolvare. Fie m ∈ N∗ , astfel ı̂ncât m 6∈ k + k + |k ∈ N∗ .
2
√ 1
Deoarece funcţia f (n) = n + n + , f : [0, ∞) → R este strict crescătoare şi
2
1
f (0) = , deducem că există n ∈ N astfel ı̂ncât
2
√ 1 √ 1
n+ n+ <m≤n+1+ n+1+ .
2 2
" # " #
√ 1 √ 1
Din cele de mai sus m ≤ n + 1 + n + 1 + . Dar m 6= n + 1 + n + 1 +
2 2
" #
√ 1
conform presupunerii făcute şi rezultă că m + 1 ≤ n + 1 + n + 1 + ≤
2
√ 1 √ 1 √ 1
n+1+ n + 1 + . Deducem inegalităţile n + < m − n ≤ n + 1 + .
2 2 2

23
 
1 2 1 3
Notând t = m − n ∈ Z, obţinem că n < t − ≤ n + 1, − < t2 − t − n ≤ .
2 4 4
2 2 2
Cum t −t−n ∈ Z, din ultimele inegalităţi deducem că t −t−n = 0, n = t −t,
m = n + t = t2 . În acest moment problema este rezolvată.
Problema 2. Fie m, n ∈ N∗ , (m, n) = 1, m par şi n impar. Să se arate că
n−1  
1 X mk
suma + (−1)[ mk
n ] nu depinde de m şi n.
2n k=1 n
(Baraj 2005, Bogdan Enescu)

Rezolvare. Fie k ∈ {1, 2, . . . , n − 1}. Scriem teorema ı̂mpărţirii cu rest şi


avem că mk = nqk + rk , 0 ≤ rk ≤ n − 1. Nu putem avea că rk = 0 deoarece ar
rezulta că n|mk, n|k (deoarece (m, n) = 1); contradicţie. Deci 1 ≤ rk ≤ n − 1.
De asemenea, nu putem avea că rk = rj pentru 1 ≤ j < k ≤ n − 1 deoarece ı̂n
acest caz ar rezulta că n|m(k − j), n|k − j (căci (n, m) = 1); contradicţie.
¦ ©  
mk
Deci rk |k = 1, n − 1 = {1, 2, . . . , n − 1}. Avem că = qk .
n
Pe de altă parte, deoarece m e par şi n e impar, din egalitatea mk =
nqk + rk , deducem că qk şi rk au aceeaşi paritate şi deci (−1)qk = (−1)rk . Din
 
1 n−1 X rk
cele precedente rezultă că suma din enunţ este + (−1)qk (deoarece
2n k=1 n
    ¦ ©
mk rk
= ). Cum (−1)qk = (−1)rk şi rk |k = 1, n − 1 = {1, 2, . . . , n−1},
n n
 
1 n−1
X
j
j 1 1 2 3 4
obţinem că suma din enunţ este + (−1) = − + − + −
2n j=1 n 2n n n n n
2t − 1 2t 1 1
···− + , unde n = 2t + 1. Rezultă că suma cercetată este +t =
n n 2n n
2t + 1 n 1 1
= = . Deci suma este ı̂ntotdeauna .
2n 2n 2 2
{x} x 3
1. Să se rezolve ecuaţia + = .
x [x] 2
3
2. Să se rezolve ecuaţia [x]3 + {x} = .
2

3. Să se rezolve ı̂n R ecuaţia x = 6 [ x] + 1.
3

√ √
4. Să se rezolve ecuaţia [ x] + [ 3 x] = 3.
5. Să se afle polinoamele de gradul al doilea cu coeficienţi ı̂ntregi, pentru
€√ Š €√ Š €√ Š
care numerele f 2 ,f 5 şi f 7 au partea ı̂ntreagă zero.

24
6. Fie a un număr nenul. Să se arate că pentru orice x real avem [a[x]] =
1
[ax], dacă şi numai dacă a = unde n ∈ N∗ .
n
”€ √ Šn —
7. n ∈ N∗ , x = 2 + 3 . Să se arate că x este impar şi că
(x − 1)(x + 3)
= k2 .
12
8. Fie n natural n ≥ 3 şi x real. Dacă {x} = {x2 } = {xn }, atunci x este
ı̂ntreg.

9. Pentru x real şi n ≥ 1 natural avem identitatea:


• ˜ • ˜ • ˜
1 2 n−1
[x] + x + + x+ + ··· + x + = [nx].
n n n
(Hermite)

10. Dacă m şi n sunt numere naturale prime ı̂ntre ele, atunci:
• ˜ • ˜  
m 2m (n − 1)m 1
+ + ··· + = (m − 1)(n − 1).
n n n 2
(Gauss)

11. Dacă m şi n sunt numere naturale impare, prime ı̂ntre ele, atunci:
m−1 n−1
   
X2
hn X2
km m−1 n−1
+ = · .
h=1
m k=1
n 2 2
(Landau)
 
n ” √ —
X n2
12. Să se arate că pentru n natural n ≥ 1 avem k 2 > √ .
k=1 2

13. Să se arate că pentru numărul real x şi numărul natural n ≥ 1 avem:
[x] [2x] [nx]
[nx] ≥ + + ··· + .
1 2 n
14. Fie o progresie aritmetică de numere reale (an )n≥1 de raţie r. Să se
arate că şirul ([an ])n≥1 este progresie aritmetică dacă şi numai dacă r este
număr ı̂ntreg.

15. Fie o progresie geometrică de numere reale (an )n≥1 , cu an ≥ 1. Să se


arate că şirul ([an ])n≥1 este progresie geometrică de raţie q > 1 dacă şi numai
dacă numerele an , n ≥ 1, sunt ı̂ntregi.

25
16. Fie f : (0, ∞) → R, f (x) = [x]x −x[x] . Să se arate că imaginea funcţiei
este Im f = [−1, ∞).
¦” √ — ©
17. Să se arate că pentru orice k ∈ N∗ , ı̂n mulţimea n 2 , n ∈ N
există o infinitate de multipli ai lui k.
¦” √ — ©
18. Să se arate că ı̂n mulţimea n 2 , n ∈ N există o infinitate de
pătrate perfecte.

19. Fie α, β numere reale nenule şi A = {[kα], k ∈ N∗ } şi B = {[lβ], l ∈ N∗ }.


Să se afle condiţiile necesare şi suficiente pentru ca A ∩ B = φ, A ∪ B = N∗ .
(Beatty)

20. Fie şirul crescător (an )n≥1 format din numerele naturale care nu se
” √ —
scriu sub forma m 2 cu m ∈ N. Să se afle termenul general an .

21. Pentru α strict pozitiv notăm S(α) = {[nα], n ∈ N∗ }. Să se arate că
nu există α, β, γ numere strict pozitive, astfel ca S(α) ∪ S(β) ∪ S(γ) = N ∗ şi
S(α) ∩ S(β) = S(β) ∩ S(γ) = S(γ) ∩ S(α) = ∅.
(Putnam, 1995)

22. În câte zerouri se termină M1 = 1000!, M2 = 19783!?

23. Care sunt numerele pentru care n! se termină cu exact 500 de zerouri?
n nu se divide cu 4 dacă şi numai dacă n = 2m .
24. Fie n > 1. C2n

25. Să se arate că 2n nu divide n!, ∀ n ∈ N∗ , şi să se găsească numerele
n ∈ N∗ pentru care 2n−1 | n!.

(m, n)
26. Să se arate că numărul · Cnm este natural.
n
27. Să se arate că următoarele numere sunt ı̂ntregi:
(2m)!(2n)! (6n)!n! (30n)!n!
N1 = , N2 = 2
, N3 = .
(m + n)!m!n! (3n)!((2n)!) (15n)!(10n)!(6n)!
(N1 : OIM, 1972)

28. Dacă n este impar şi compus, n ≥ 15, atunci n! este multiplu de n3 .

29. Pentru n număr natural nenul, să se arate că avem [1, 2, . . . , n] =
” —
Cn1 , Cn2 , . . . , Cnn
dacă şi numai dacă n + 1 este prim.
(Baraj )

26
30. Fie k, h numere naturale, k, h ≥ 2. Să se determine numerele naturale
√ √
n pentru care ecuaţia [ k x] + [ h x] = n nu are soluţii reale.

n
31. Fie (an )n≥1 un şir cu proprietatea că a1 > 0 şi an+1 = an + ,
an
∀ n ≥ 1. Să se arate că există un număr natural nenul n0 , astfel ı̂ncât [an ] = n
pentru orice n ≥ n0 .
(Propusă OIM, 1997)

27
CAPITOLUL 5

Numere raţionale. Numere iraţionale

Vom folosi ı̂n acest capitol câteva noţiuni de algebră liniară. Avem nevoie
de informaţia că R este spaţiu vectorial peste Q. Aceasta ı̂nseamnă că există
X
mulţimea {ei ∈ R|i ∈ I} cu proprietatea că ∀ J ⊆ I, J finită şi aj ej = 0
j∈J
(aj ∈ Q, ∀ j ∈ J) implică aj = 0 ∀ j ∈ J şi cu proprietăţile că ∀ x ∈ R,
X
∃ J ⊆ I, J finită şi x = aj ej (aj ∈ Q, ∀ j ∈ J). De asemenea, vom
j∈J
folosi noţiunea de subspaţiu vectorial generat de o mulţime. Concret, avem
nevoie de următoarea informaţie: dacă {e1 , e2 , . . . , en } ⊆ R şi notăm cu V
( )
n
X
mulţimea V = aj ej |aj ∈ Q, ∀ j = 1, n putem extrage din mulţimea
j=1
{e1 , e2 , . . . , en } o ”bază” a lui V . Aceasta semnifică faptul că ∃ 1 ≤ n1 < n2 <
( )
m
X
· · · < nm ≤ n, m ≤ n astfel ı̂ncât V = bj enj |bj ∈ Q, ∀ j = 1, m şi dacă
j=1
m
X
avem o egalitate de tipul bj enj = 0, bj ∈ Q, ∀ j = 1, m, atunci bj = 0,
j=1
∀ j = 1, m.
Avem nevoie şi de următorul rezultat. Dacă f ∈ Q[X] este un polinom
ireductibil de grad n şi α ∈ C este o rădăcină a lui f , atunci o egalitate de tipul
n−1
X
ai αi = 0, ai ∈ Q, ∀ i = 0, n − 1, implică faptul că ai = 0, ∀ i = 0, n − 1.
i=0
Vom folosi şi criteriul de ireductibilitate al lui Eisenstein: dacă f (X) =
n
X
aj X j ∈ Z[X] şi p este un număr prim cu proprietatea că p¤|¤an , p|aj , ∀ j =
j=0
0, n − 1, p2 ¤|¤a0 , atunci f este un polinom ireductibil ı̂n Q[X].

s
4n − 2
Problema 1. Să se găsească numerele n ∈ Z pentru care numărul
n+5
este raţional.
(Baraj juniori, 2001, Dan Popescu)

29
s Rezolvare. Este evident că n ≤ −6 sau n ≥ 1. Fie n ∈ Z astfel ı̂ncât
4n − 2 4n − 2 a2
∈ Q. Deducem că = 2 , a, b ∈ N∗ , (a, b) = 1. Cum 2|4n − 2
n+5 n+5 b
 
n+5
şi 4¤|¤4n − 2, deducem că n este impar. Notăm d = 2n − 1, . Avem
2
n+5
că d|4 − (2n − 1) = 11, d ∈ {1, 11}. Dacă d = 1, atunci 2n − 1 = a2 ,
2
n+5 n+5
= b2 sau 2n − 1 = −a2 , = −b2 . În cel de-al doilea caz rezultă că
2 2
−4b2 = 2n + 10 = −a2 + 11, a2 = 4b2 + 11 = M4 + 3; contradicţie.
n+5
Deci 2n − 1 = a2 , = b2 , 4b2 = 2n + 10 = a2 + 11, (2b − a)(2b + a) = 11,
2
2b − a = 1, 2b + a = 11, 4b = 12, b = 3, a = 5, n = 13. Dacă d = 11, atunci
n+5 n+5
2n − 1 = 11a2 , = 11b2 , sau 2n − 1 = −11a2 , = −11b2 .
2 2
În primul caz rezultă 44b2 = 2n + 10 = 11a2 + 11, a2 = 4b2 − 1 = M4 + 3;
contradicţie. În cel de-al doilea caz avem −44b2 = 2n + 10 = −11a2 + 11,
(a − 2b)(a + 2b) = 1, a − 2b = a + 2b = 1, b = 0, a = 1, n = −5; contradicţie
(deoarece n ≤ −6 sau n ≥ 1). Deci singura soluţie a problemei este n = 13.

Problema 2. Fie a, b, c ∈ Q astfel ı̂ncât a + b cos 40 + c cos2 40 = 0. Să se


arate că a = b = c = 0.

Rezolvare. Arătăm că cos 40 este rădăcină a polinomului f (X) = 8X 3 −


6X +1 care este ireductibil ı̂n Q[X] şi se aplică unul din rezultatele invocate ı̂n
introducerea de mai sus. Folosim formula cos 3y = cos y(4 cos2 y − 3). Notăm
1
α = cos 40, punem y = 40◦ ı̂n egalitatea precedentă şi obţinem că − =
2
cos 120 = α(4α2 − 3) = 4α3 − 3α. Rezultă că f (α) = 0. Deoarece grad f = 3,
a arăta că f este ireductibil ı̂n Q[X] este acelaşi lucru cu a arăta că f nu are
a
rădăcini raţionale. Dacă f ar avea o rădăcină raţională x = , a ∈ Z, b ∈ N∗ ,
( b )
1 1 1
(a, b) = 1, atunci a|1 şi b|8. Rezultă că x ∈ ± 1, ± , ± , ± . Se verifică
2 4 8
uşor că nici una din cele 8 valori de mai sus nu este rădăcină a lui f şi problema
este rezolvată.

1. Fie n şi k numere naturale, k ≥ 2, astfel ı̂ncât k
n este un număr

raţional. Să se arate că k n este natural.

30
2. Să se determine numărul soluţiilor ı̂n numere naturale ale ecuaţiei
√ √ √
x + y = 1323.
√ √
3. Să se arate că dacă n este natural, atunci numerele 3n + 2 şi 5n + 2
sunt iraţionale.

4. Să se arate că numărul n = 2, 1221222122221 . . . este iraţional.

5. Să se arate că nu există un număr finit de numere raţionale


r1 , r2 , . . . , rn , astfel ca pentru orice număr raţional a să existe numerele ı̂ntregi
x1 , x2 , . . . , xn , astfel ca a = x1 r1 + x2 r2 + · · · + xn rn .
1 1 1
6. Fie a, b, c numere raţionale cu proprietatea că + = .
s a + bc b + ac a + b
c−3
Să se arate că este un număr raţional.
c+1
7. Să se determine toate numerele naturale n cu proprietatea că
√ √ √ √
2n + 1 este număr raţional, iar numerele 2n + 2, 2n + 3, . . . , 3n + 3
sunt iraţionale.
2n2 + 1
8. Să se determine numerele raţionale n, pentru care numărul
n+3
este ı̂ntreg.
2m2 + 1
9. Să se determine numerele iraţionale m, pentru care numerele
m−3
3m2 − 1
şi sunt ambele raţionale.
3m − 5
1 1 1
10. Dacă a, b sunt numere naturale şi + = , a2 + b2 = nc2 , să se
a b c
determine n ştiind că este număr natural.

11. Să se determine numărul prim p şi numărul natural n, dacă 3 pn + 1
este raţional.

12. Să se arate că pentru nici un număr real x numerele 3 − x2 şi
√3
2 − x3 nu sunt simultan raţionale.
(Baraj juniori, 2002)
√ √
13. Să se arate că dacă n este număr natural, atunci n + 3 n + 5 este
iraţional.

14. Să se rezolve ı̂n numere raţionale ecuaţia x3 + y 3 = x2 + y 2 .

31
15. Dacă n este număr natural divizibil cu 3, să se arate că ecuaţia
1 1
x + y + + = n nu are soluţii ı̂n numere raţionale pozitive.
x y
16. Să se arate că ecuaţia x2 + y 2 + z 2 + x + y + z = 1 nu are soluţii ı̂n
numere raţionale.

17. Să se rezolve ı̂n numere raţionale pozitive ecuaţiile:


a) xy = y x ;
b) xx = y y .

a3 + b3
18. Să se arate că ∀ q ∈ Q∗+ , se poate scrie sub forma q = , unde
c3 + d3
a, b, c, d ∈ N∗ .
(Lista scurtă, OIM, 1999)

19. Să se arate că orice număr raţional se scrie ca sumă a trei cuburi de
numere raţionale.

20. Ştiind că numerele x şi y sunt iraţionale iar xy, x2 + y şi y 2 + x sunt
raţionale, să se calculeze x + y.

21. Să se arate că cos 1◦ este iraţional.


π
22. Să se arate că cos este iraţional.
7
1 1
23. Să se arate că arccos √ este iraţional.
π 2003
π
24. Fie n ∈ N∗ . Să se arate că cos ∈ Q, dacă şi numai dacă n = 1, 2, 3.
n
25. Să se arate că există numerele iraţionale a şi b, astfel ı̂ncât ab să fie
raţional.
√ m
26. Să se arate că dacă m, n sunt numere naturale şi 3 − > 0, atunci
n
√ m 1
3− > .
n n(m + 1)
27. Fie a, b, c ∈ Z astfel ı̂ncât fiecare este ı̂n modul mai mic decât 106 şi
√ √ 1
(a, b, c) 6= (0, 0, 0). Să se arate că a + b 2 + c 3 > 20 .
10
(Putnam, 1980)

32
28. Fie n ∈ N∗ şi a1 , a2 , . . . , a2n+1 ∈ R, cu proprietatea că ∀ i = 1, 2n + 1,
{1, 2, . . . , i−1, i+1, i+2, . . . , 2n+1} = G1 ∪G2 , |G1 | = |G2 | = n, G1 ∩G2 = ∅,
X X
ai = ai . Să se arate că a1 = a2 = · · · = a2n+1 .
i∈G1 i∈G2

29. a) Fie S = {x0 = 0 < x1 < x2 < · · · < xn−1 < xn = 1} o submulţime
a lui [0, 1], cu proprietatea că orice distanţă ı̂ntre punctele acestei submulţimi
apare cel puţin de două ori, cu excepţia distanţei 1. Să se arate că xi ∈ Q,
∀ i = 0, n.
(Iran, 1998)

b) Fie S = {x0 = 0 < x1 < · · · < xn−1 < xn = 1} o submulţime, cu


xk + xl
proprietatea că ∀ i = 1, n − 1 xi = , 0 ≤ k, l ≤ n, k 6= i, l 6= i.
2
Să se arate că xi ∈ Q, ∀ i = 0, n.
(Berkeley Math Circle Monthly Contest, 1999-2000)

∞ 1
X
30. Să se arate că este număr iraţional.
n=1
2n2

31. Dacă (an )n≥1 este un şir strict crescător de numere naturale şi an ≥ n,
X ∞ 1 X∞ (−1)n
atunci numerele şi sunt iraţionale.
n=1
(an )! n=1
(an )!

32. Să se arate că există numere zecimale iraţionale 0, a1 a2 . . . astfel ı̂ncât
a2n+1 + a2n+2 = 1 + an , ∀ n ≥ 1.

33. Fie (an )n≥1 un şir de numere naturale nenule, cu proprietatea că an+1
este cel mai mic factor prim al lui an + an−1 pentru orice n ≥ 2. Să se arate
că numărul 0, a1 a2 a3 . . . este raţional.
(Baraj, 2002)
n−1
X €√ Šj
n
34. Fie n ∈ N, n ≥ 2 şi ai ∈ Q, ∀ i = 0, n − 1. Să se arate că aj 2
j=0
este raţional, dacă şi numai dacă aj = 0, ∀ j = 1, n − 1.

35. Fie α un număr real care este rădăcină a unui polinom ireductibil
de grad r ≥ 2, cu coeficienţi numere ı̂ntregi, p ∈ Z, q ∈ N∗ . Atunci există un
p c
număr real c > 0, care nu depinde de p şi q, astfel ı̂ncât α − > r .
q q
(Criteriul lui Liouville)

33
∞ 1
X
36. Să se arate că numărul este transcendent.
n=1
4n!

37. Fie α iraţional. Să se arate că mulţimea {mα + n | m, n ∈ Z} este


densă ı̂n mulţimea numerelor reale.

38. Fie α un număr iraţional pozitiv. Să se arate că mulţimea


{{nα}|n ∈ N∗ } este densă ı̂n [0, 1).

34
CAPITOLUL 6

Baze de numeraţie. Suma cifrelor

Fie un număr natural b ≥ 2 şi fie n ∈ N, n ≥ 1. Atunci există şi sunt


unic determinate numerele naturale k, a0 , a1 , a2 , . . . , ak astfel ı̂ncât n = ak bk +
ak−1 bk−1 +· · ·+a2 b2 +a1 b+a0 , unde aj ∈ {0, 1, 2, . . . , b−1}, ∀ j = 0, k şi ak ≥ 1.
Vom folosi notaţia n = ak ak−1 . . . a1 a0(b) ; aceasta este scrierea lui n ı̂n baza
b. Evident că 0 = 0(b) şi dacă n ∈ Z, n ≤ −1, atunci n are o scriere de tipul
n = −ak ak−1 . . . a1 a0 . Dacă x ∈ R, atunci x = [x] + {x}. Ştim să scriem pe [x]
ı̂n baza b. Trebuie să găsim acum scrierea ı̂n baza b a unui număr real y ∈ [0, 1).
Considerăm x1 = y, xn+1 = bxn − [bxn ] şi an = [bxn ] pentru n ∈ N∗ . Deoarece
xn+1 = {bxn } ∈ [0, 1), deducem că an+1 = [bxn+1 ] ∈ {0, 1, 2, . . . , b − 1},
X∞ a
j
∀ n ∈ N. Rezultă imediat următoarea egalitate: y = j
. Deci scrierea lui
j=1
b
y ∈ [0, 1] ı̂n baza b este y = 0, a1 a2 a3 . . .(b) .

Definiţie. Dacă n = ak ak−1 . . . a1 a0 este scris ı̂n baza 10, notăm s(n) =
k
X
aj (suma cifrelor lui n scris ı̂n baza 10). Se arată că au loc următoarele
j=0
proprietăţi:
1) g|s(n) − n, ∀ n ∈ N;
2) s(m + n) ≤ s(m) + s(n), ∀ n, m ∈ N;
3) s(mn) ≤ ns(m), ∀ m, n ∈ N;
4) s(mn) ≤ s(m)s(n), ∀ m, n ∈ N.

Problema 1. Fie a un număr natural nenul cu proprietatea că numărul


cifrelor lui an (ı̂n baza 10) este impar ∀ n ∈ N∗ . Să se arate că n = 102t , t ∈ N.
(Baraj juniori 2003, Vasile Zidaru)

Rezolvare. Deoarece a are un număr impar de cifre, deducem că ∃ t ∈


N astfel ı̂ncât 102t ≤ a < 102t+1 . Vom arăta prin inducţie că 102t ≤ a <
1
102t+ n , ∀ n ∈ N∗ . Am văzut ceva mai sus că enunţul este adevărat pentru
n = 1. Presupunem că el este adevărat pentru n şi ı̂l vom demonstra pentru
1
n + 1. Avem că 102t ≤ a < 102t+ n . Ridicăm aceste inegalităţi la puterea

35
1
n + 1 şi obţinem că 102t(n+1) ≤ an+1 < 102t(n+1)+1+ n . Dacă cumva avem că
1
102t(n+1)+1 ≤ an+1 < 102t(n+1)+1+ n , rezultă că an+1 are 2t(n + 1) + 2 cifre,
deci un număr par de cifre; contradicţie. Deci 102t(n+1) ≤ an+1 < 102t(n+1)+1 ,
1
102t ≤ a < 102t+ n+1 . Pasul de inducţie a fost demonstrat. Deci 102t ≤ a <
1 1
102t+ n , ∀ n ∈ N. Cum lim 102t+ n = 102t , deducem că a = 102t .
n→∞
Problema 2. a) Să se arate că oricum am alege 39 de numere naturale
consecutive, putem găsi unul dintre ele cu suma cifrelor divizibilă cu 11.
b) Să se găsească primele 38 de numere naturale consecutive astfel ı̂ncât
nici unul din ele nu are suma cifrelor divizibilă cu 11.
(Baraj 1999, Gh. Eckstein)

Rezolvare. a) Pasul 1. Fie b1 , b2 , . . . , b20 , 20 de numere naturale conse-


cutive, astfel ı̂ncât 10|b1 şi 100¤|¤b11 . Arătăm că ∃ 1 ≤ j ≤ 20 astfel ı̂ncât
11|s(bj ). Este evident că dacă 10¤|¤n+1, atunci s(n+1) = s(n)+1. Dacă 10|n+1
şi 102 ¤|¤n + 1, atunci s(n + 1) = s(n) − 8. Dacă 11|s(b1 ), atunci observaţia este
imediată. Dacă s(b1 ) = 11q + r, 2 ≤ r ≤ 10, atunci s(b12−r ) = s(b1 ) + 11 − r =
11(q +1) deoarece am văzut mai sus că s(n+1) = s(n)+1 dacă 10¤|¤n+1 (avem
că 2 ≤ 12−r ≤ 10 şi deci se poate aplica formula precedentă). Cum 11|s(b12−r ),
observaţia este demonstrată. Dacă s(b1 ) = 11q + 1, atunci s(b10 ) = 11q + 10,
s(b11 ) = s(b10 ) − 8 = 11q + 2 (deoarece 10|b11 , 100¤|¤b11 ) şi s(b20 ) = 11q + 11.
Deci şi ı̂n acest caz observaţia este demonstrată.
Pasul 2. Fie 39 de numere naturale consecutive a1 , a2 , . . . , a39 . Dacă 102 ¤|¤aj ,
∀ j = 1, 39, pasul precedent ne arată că putem găsi 1 ≤ j ≤ 39 astfel ı̂ncât
11|s(aj ) (se arată uşor că, de fapt, pe j ı̂l putem găsi ı̂n mulţimea {1, 2, . . . , 29}
ı̂n acest caz). Să presupunem acum că 102 |aj , 1 ≤ j ≤ 39. Dacă j ≤ 20, atunci
considerăm numerele aj , aj+1 , . . . , aj+19 (pe post de b1 , b2 , . . . , b20 ). Evident că
102 ¤|¤aj+10 = b11 şi folosind pasul precedent, deducem că ∃ k, j ≤ k ≤ j + 19
astfel ı̂ncât 11|s(ak ) (evident că k ≤ j + 19 ≤ 39 ı̂n acest caz).
Să presupunem acum că j ≥ 21. Considerăm ı̂n acest caz numerele
aj−20 , aj−19 , . . . , aj−1 (pe post de b1 , b2 , . . . , b20 ). Evident că 102 ¤|¤aj−10 = b11
(deoarece 102 |aj şi aj−10 = aj − 10) şi pasul precedent ne asigură că ∃ k ∈ N,
j − 20 ≤ k ≤ j − 1, astfel ı̂ncât 11|s(ak ).
b) Soluţia punctului a) ne sugerează felul ı̂n care putem găsi primele 38 de
numere naturale consecutive care au proprietatea că suma cifrelor lor nu este
multiplu de 11. Alegem a1 < a2 < · · · < a38 , unde 102 |a20 , s(a20 ) = 11q + 1,
s(a19 ) = 11t + 10, unde q, t ∈ N. Fie acum k ∈ N, k ≥ 2 astfel ı̂ncât 10k |a20 şi
10k+1 ¤|¤a20 . Atunci se deduce imediat că s(a20 ) = s(a19 ) − (9k − 1). Înlocuind

36
ı̂n această formulă pe s(a19 ) şi s(a20 ), deducem că 11q + 1 = 11t + 10 −
(9k − 1). Rezultă că 11|9k + 1 şi cea mai mică valoare pentru care se ı̂ntâmplă
acest fenomen este k = 6. Cele mai mici 38 de numere consecutive care au
proprietatea din enunţ sunt a1 = 106 − 19 = 999981 < a2 = a1 + 1 < · · · <
a20 = 106 < a21 = 106 + 1 < · · · < a38 = 106 + 18 = 1000018.

1. Să se scrie ı̂n bazele 10 şi respectiv 8 numărul 6606(7) .

2. Să se arate că 12, 1(5) = 13, (03)(4) .

3. Să se afle câtul şi restul ı̂mpărţirii numerelor 11011001(2) şi 1011(2) .

4. În ce baze au loc egalităţile:


a) 32 × 14 = 438?
b) 27 × 25 = 708?

5. În ce bază numărul 297 este divizor al lui 792?

6. Să se arate că ı̂n orice bază, b ≥ 6, numărul 12.544 este pătrat perfect.

7. Să se arate că numărul 11211(b) este compus (b ≥ 3).

8. Să se arate că 253(b) este compus. Să se găsească cel mai mic b, astfel
ı̂ncât 253(b) să fie pătrat perfect.

9. Să se arate că numărul 13552(b) : 112(b) este ı̂ntreg şi pătrat perfect.

10. Să se determine x, y, z dacă xyz (7) = zyx(11) .

11. Dacă a > 2 şi (a − 1)2 = xy (a) , să se arate că 2(a − 1) = yx(a) .

12. Câte triplete (a, b, c) există astfel ca ab, c(10) = cb, a(20) ?

13. Notăm cu xn ultima cifră a numărului nn (pentru orice număr natural


nenul). Să se arate că şirul xn este periodic.

14. Să se determine n ştiind că numărul nn are n cifre.

15. Să se determine numerele xyz(b) care sunt pătrate perfecte ı̂n orice
bază b ≥ 10.

16. Să se arate că ı̂ntr-o progresie aritmetică de numere naturale, există
o infinitate de termeni pentru care suma cifrelor este aceeaşi.

17. Fie şirul (xn )n≥1 , x1 = 9 şi xn+1 = 9xn pentru n ≥ 1. Să se arate că
pentru n ≥ 2, ultimele două cifre ale lui xn sunt 8 şi 9.

37
18. Fie n = 123456789(10) şi 1 ≤ a ≤ 9. Să se scrie n · 9a ı̂n baza 10.
r
19. Să se afle primele 2001 zecimale ale numărului 0, 11 . . . 1} .
| {z
2001

20. Din numărul 123456789101112 . . . 9899100 să se elimine 20 de cifre,


astfel ı̂ncât numărul rămas să fie maxim.
(Baraj, OIM, 1968)

21. Să se arate că un număr format din 2n cifre identice, are cel puţin n
factori primi distincţi.

22. Să se afle primele n zecimale ale numărului xn , unde xn = 11 . . . 1} .
| {z
2n
23. Să se arate că un număr scris ı̂n bază 10, cu n ≥ 2 cifre egale, nu este
pătrat perfect.
(Oblath)

24. Un număr natural n are cinci cifre 5 şi cinci cifre 2, restul cifrelor
fiind 0. Să se arate că n nu este pătrat perfect.

25. Să se arate că nu există o putere a lui 2 astfel ı̂ncât, permutându-i
cifrele, să se obţină o altă putere a lui 2 (cu acelaşi număr de cifre).

26. Dacă p este prim şi sp (n) este suma cifrelor numărului n scris ı̂n baza
n − sp (n)
p, atunci funcţia lui Legendre ep (n) verifică relaţia ep (n) = .
p−1
1
27. Să se arate că s(8n) ≥ s(n).
8
28. Să se arate că există o infinitate de numere m, care se scriu ı̂n cel
puţin două moduri sub forma m = x + s(x).

29. Să se arate că ecuaţia s(x2 ) + s(y 2 ) = s(z 2 ) are o infinitate de soluţii.

30. Determinaţi toate numerele n > 0, pentru care există numerele natu-
rale a şi b, astfel ca n = s(a) = s(b) = s(a + b).
(Baraj, OBJ, 2000)
€ € € ŠŠŠ
31. Să se calculeze s s s 20052005 .
(Adaptare după o problemă de la OIM, 1975)

32. Să se arate că dacă n ≥ 2 şi (n, 3) = 1, numărul N = 1 |000...0


{z } 1 000...0
| {z } 1,
n−1 n−1
scris ı̂n baza b ≥ 2, este compus.

38
33. Să se găsească numerele n ∈ N∗ pentru care p(n) = n2 − 15n + 5. Cu
p(n) se notează produsul cifrelor lui n (ı̂n baza 10).
(Adaptare după o problemă de la OIM 1968)

34. Fie M mulţimea numerelor n ∈ N∗ care nu conţin cifra 9 ı̂n scrierea


X 1
lor ı̂n baza 10. Să se arate că seria este convergentă.
n∈M
n
(Lista scurtă, OIM, 1975)

35. Pentru numerele m ∈ N∗ , notăm cu L(m) numărul de cifre mai mari


X∞ L(2n )
sau egale cu 5, din scrierea lui m ı̂n baza 10. Să se calculeze .
n=0
2n

36. Să se arate că ∀ n ∈ N, (3, n) = 1, k ∈ N, k ≥ n, există un multiplu


de n care are suma cifrelor k.
(Lista scurtă, OIM, 1999)

37. Să se găsească toate numerele cu proprietatea că n = s4 (n).

38. Să se arate că există o infinitate de numere naturale n, care nu se pot
scrie sub forma n = a + b, s(a) = s(b).

39. Fie (an )n≥1 o progresie aritmetică de numere pozitive. Să se arate
că există o infinitate de indici n, pentru care numerele [an ] au aceeaşi sumă a
cifrelor.
(Baraj, 2002)

40. Să se determine toate bazele b, astfel ı̂ncât ultima cifră a oricărui
pătrat scris ı̂n baza b, este de asemenea pătrat.

41. Propoziţia ”există numere a de n cifre astfel ı̂ncât s(a) = p(a)” este
adevărată pentru o infinitate de numere n şi falsă pentru o infinitate de numere
n.
(Baraj )

42. Fie M ∈ N∗ . Să se arate că ∃ n ∈ N, astfel ı̂ncât 2n ı̂n baza 10 să
ı̂nceapă cu cifrele lui M .

43. Să se arate că dacă primele două cifre ale lui 2n sunt aceleaşi cu
primele două cifre ale lui 5n , atunci acestea sunt 3 şi 1 (n este număr natural
nenul).

39
CAPITOLUL 7

Funcţii aritmetice

Vom folosi ı̂n acest capitol, cu precădere, funcţiile τ, σ, ϕ şi µ. Pentru


n ∈ N∗ , notăm cu τ (n) numărul divizorilor săi naturali. Dacă descom-
punerea standard ı̂n factori primi a lui n este n = pα1 1 · pα2 2 · . . . · pαr r , atunci
Yn

τ (n) = (αj + 1); τ (1) = 1. Pentru n ∈ N∗ avem τ (n) ≤ 2 n. De fapt,
j=1
τ (n)
pentru orice ε > 0 avem că lim = 0.
n→∞ nε
Pentru n ∈ N∗ , notăm cu σ(n) suma divizorilor săi naturali. Dacă de-
scompunerea standard ı̂n factori primi a lui n este n = pα1 1 pα2 2 . . . pαr r , atunci
Y r pαj +1 − 1
j √
σ(n) = ; σ(1) = 1. Dacă n este compus, avem σ(n) > n + n,
j=1
p j − 1
iar pentru n ≥ 7 avem inegalitatea σ(n) < n log n.
Pentru n ∈ N∗ , notăm cu ϕ(n) numărul numerelor naturale prime cu n,
care nu-l depăşesc pe n. Dacă descompunerea standard ı̂n factori primi a lui
Yr  1

α1 α2 α
n este n = p1 p2 . . . pr , atunci ϕ(n) = n
r 1− ; ϕ(1) = 1. Gauss a
j=1
pj
X √
arătat că ϕ(d) = n. Pentru n ≥ 7 avem că ϕ(n) ≥ n, iar pentru n compus
d|n
√ X
avem ϕ(n) ≤ n − n. Dacă notăm cu f (x) = ϕ(n) (pentru x ∈ R, x ≥ 1),
n≤x
3 2
atunci există o constantă c > 0 astfel ı̂ncât f (x) − x ≤ c x log x.
π2
Funcţia ϕ se numeşte indicatorul lui Euler. Acesta din urmă a arătat că
dacă n ∈ N∗ şi a ∈ Z, (a, n) = 1, atunci aϕ(n) = Mn + 1 (teorema lui Euler).
Funcţia lui Möbius se notează cu µ. Avem că µ(1) = 1 şi µ(n) = 0 dacă ∃ p
prim astfel ı̂ncât p2 |n. Dacă n = p1 p2 . . . pr , unde p1 , p2 , . . . , pr sunt numere
X
prime distincte, atunci µ(n) = (−1)r . Avem că µ(d) este 1 dacă n = 1 şi 0
d|n
X
dacă n > 1. Fie f : N∗ → C şi F funcţia sa sumatorie (F (n) = f (d)). Atunci
d|n

41
 
X n
are loc egalitatea f (n) = µ(d)F , ∀ n ∈ N∗ (teorema de inversiune a
d|n
d
lui Möbius). Dacă f : N∗ → C are proprietatea că f (nm) = f (n) · f (m),
X X Y
∀ n, m ∈ N∗ şi că f (n) este absolut convergentă, atunci f (n) (1 −
n≥1 n≥1 p prim
X Y
f (p)) = 1. În contextul precedent avem şi că µ(n)f (n) = (1 − f (p))
n≥1 p prim
(ı̂n ipoteza mai slabă că f (n · m) = f (n)f (m) ∀ m, n ∈ N∗ , (m, n) = 1).
Dacă f : N∗ → C are proprietatea că f (n · m) = f (n) · f (m), ∀ n, m ∈ N∗ ,
X Y
(n, m) = 1, atunci µ(d)f (d) = (1 − f (p)). Aplicând acest rezultat
d|n p prim
p|n

X µ(d) ϕ(n)
obţinem că = , ∀ n ∈ N∗ .
d|n
d n

Problemă. Un număr ak ak−1 . . . a1 a0 (scris ı̂n baza 10) se numeşte al-


ternat dacă aj şi aj−1 au parităţi diferite ∀ j = 1, k. Să se determine toate
numerele n ∈ N∗ cu proprietatea că un multiplu al lor este alternat.
(OIM, 2005)

Rezolvare. Arătăm că dacă n nu este multiplu de 20, atunci n are un


multiplu care este alternat. Este imediat că orice număr care se divide cu 20
nu admite un multiplu alternat.
Pasul 1. Arătăm că ∀ n ∈ N∗ , (n, 10) = 1, admite un multiplu de forma
101010101 . . . 01, care este evident alternat. Avem că (10, 99n) = 1 şi conform
teoremei lui Euler, avem că 10ϕ(99n) = 99n · m + 1. E uşor de văzut că ϕ(99n)
este par şi deci ϕ(99n) = 2(h + 1), h ∈ N. Avem egalităţile
10ϕ(99n) − 1 102(h+1) − 1
n·m = = = 1 + 102 + 104 + · · · + 102h = 10101 . . . 01
99 102 − 1
(cifra 1 apare de h + 1 ori). Afirmaţia de la acest pas este demonstrată.
Pasul 2. Arătăm că ∀ k ∈ N∗ există un multiplu impar al lui 5k care
este alternat şi care are cel mult k cifre. Probăm acest enunţ prin inducţie.
Pentru k = 1 alegem numărul 5, pentru k = 2 alegem pe 25, iar pentru k = 3
alegem pe 125. Presupunem acum că m = ak−1 ak−2 . . . a2 a1 a0 are calitatea
că este alternat şi multiplu de 5k (a0 = 5, a1 = 2, a2 = 1). Alegem numărul
m1 = ak ak−1 . . . a1 a0 = ak · 10k + m, unde ak ∈ {0, 2, 4, 6, 8} dacă k este impar
şi ak ∈ {1, 3, 5, 7, 9} dacă k este par. Avem că m = 5k · b iar pe ak ı̂l alegem
astfel ı̂ncât 5|ak · 2k + b. Avem că m1 = 5k (ak · 2k + b) = M5k+1 şi este alternat.
Faptul că ı̂l pot alege pe ak ı̂n mulţimile {0, 2, 4, 6, 8} sau {1, 3, 5, 7, 9} astfel

42
ı̂ncât 5|ak · 2k + b, rezultă din faptul că resturile numerelor a · 2k + b la 5 (unde
a ∈ {0, 2, 4, 6, 8} sau a ∈ {1, 3, 5, 7, 9}) coincid (ca mulţime) cu {0, 1, 2, 3, 4}.
Pasul de inducţie a fost demonstrat.
Pasul 3. ∀ k ∈ N∗ , există un număr alternat de forma ak 2ak−1 2 . . . 2a1 2 care
este multiplu de 22k+2 (ı̂n plus, pot alege numerele ai din mulţimea {1, 3, 5, 7}).
Demonstrăm această afirmaţie din nou prin inducţie. Pentru k = 1 alegem
a1 = 3. Presupunem acum că m = ak 2ak−1 2 . . . 2a1 2 este alternat şi m =
22k+2 ·b, b ∈ N∗ . Construim m1 = ak+1 2ak 2 . . . 2a1 2 = m+ak+1 ·22k+1 +2·22k =
22k+1 (2b + 1 + ak+1 ). Fie r ∈ {1, 3, 5, 7} restul ı̂mpărţirii lui 2b + 1 la 8. Alegem
ak+1 = 8 − r şi deducem că m1 este alternat şi este multiplu de 22k+4 . Pasul
de inducţie este demonstrat.
Pasul 4. Dacă n = 5k · m, k ∈ N∗ , (m, 10) = 1, atunci ∃ un multiplu
alternat al lui n. Fie a alternat cu 2k cifre (prima putând fi şi 0), multiplu
impar de 52k (a există conform pasului 2). Considerăm numărul N = aa . . . a,
(102k )h − 1
obţinut prin ”lipirea” lui a de h ori. Avem că N = a = a+a·102k +
102k − 1
a(102k )2 + · · · + a(102k )h−1 . Deoarece (10, (102k − 1)m) = 1, din teorema lui
2k
Euler ştim că 10ϕ(m(10 −1)) = q · m(102k − 1) + 1. Alegem h = ϕ(m(102k − 1))
şi din cele de mai sus rezultă că numărul N este alternat şi multiplu de 52k · m
(deci şi de n = 5k · m). În plus, N este şi impar.
Pasul 5. n = 2 · 5k · m, (m, 10) = 1. La numărul N construit la pasul 4
”lipim” un zero la sfârşit şi obţinem un multiplu alternat de n (N era impar
şi alternat, multiplu de 5k · m).
Pasul 6. n = 2k · m, (m, 10) = 1. Atunci există un multiplu de n care
este alternat. Se procedează ca la pasul 4 ”lipind” pe a de h ori, unde h =
ϕ(m(102k − 1)) iar a este un număr alternat cu 2k cifre, multiplu de 22k (de
fapt chiar multiplu de 22k+2 ); a există conform pasului 3.
Soluţia problemei este ı̂ncheiată ı̂n acest moment.

1. Să se arate că τ (n) este impar dacă şi numai dacă n este pătrat perfect.

τ (n)
2. Să se arate că produsul divizorilor naturali ai lui n este n 2 .

3. a) Să se determine numerele naturale nenule, pentru care media geo-


metrică a divizorilor este număr natural.
Y
b) Care sunt numerele naturale n ≥ 2 pentru care d = n.
d|n
d6=n

43
4. În câte moduri se poate descompune un număr n > 1 ı̂n doi factori
primi ı̂ntre ei?

5. Pentru n număr natural, să se afle numărul de soluţii ı̂n numere na-
1 1 1
turale ale ecuaţiei + = .
x y n
6. Să se arate că τ (ns ) − τ (nt ) se divide cu s − t.

7. Să se arate că τ (ms ) − τ (ns ) se divide cu s.

8. Să se determine cel mai mic număr n care are:


a) 18 divizori naturali;
b) 24 divizori naturali.

9. a) Dacă σ(n) este număr prim, atunci n = pα , unde p şi α + 1 sunt


numere prime.
b) Să se rezolve ecuaţia σ(n) = 31.

10. Să se determine n dacă:


a) ϕ(n) = 4;
b) ϕ(n) = 6;
n
c) ϕ(n) = .
3
11. a) Fie m ∈ N∗ . Să se rezolve ecuaţia mϕ(n) = n.
m−1
b) Fie m ∈ N, m ≥ 2. Să se rezolve ecuaţia ϕ(n) = n.
m
12. Să se arate că n se divide la n − ϕ(n) dacă şi numai dacă n = pα .

13. Pentru fiecare n natural, să se arate că există s = s(n) > 1 astfel
ı̂ncât σ(ns ) se divide cu σ(n).

14. Să se arate că există o infinitate de numere compuse n, pentru care
σ(n) se divide cu τ (n).

15. Să se arate că pentru n ≥ 2 există o infinitate de numere m, astfel


ϕ(n) ϕ(m)
ı̂ncât = .
n m
ϕ(m) ϕ(n)
16. Să se arate că dacă = , atunci m şi n au aceeaşi factori
m n
primi.

44
17. Să se demonstreze că dacă (ai , n) = 1, 1 ≤ a1 < a2 < · · · < aϕ(n) ≤ n,
ϕ(n)
X nϕ(n)
atunci ai = (n ∈ N, n ≥ 2).
i=1
2
n−1  
X 1
18. Să se arate că pentru n ≥ 2 avem ϕ(n) = .
k=1
(n, k)

19. Să se arate că:


a) lim inf σ(n + 1) − σ(n) = −∞.
n→∞
b) lim sup σ(n + 1) − σ(n) = ∞.
n→∞

X 1
20. Să se arate că seria este divergentă, iar seria
n≥1
τ (n)σ(n)
„ Ž2
X τ (n)
este convergentă.
n≥1
σ(n)

X 1
21. Să se arate că seria este convergentă.
n≥1
ϕ(n)σ(n)

22. Dacă M = [m, n] şi d = (m, n), să se arate că ϕ(m)ϕ(n) = ϕ(M )·ϕ(d).

23. Să se arate că:


a) σ(mn) ≤ σ(m) · σ(n).
b) Dacă [m, n] = M şi (m, n) = d, atunci σ(m)σ(n) = σ(M )σ(d).

24. Să se arate că τ (mn) ≤ τ (m)τ (n).

25. Dacă n ≥ 2 avem τ (n)σ(n) ≥ 2n + 2. În ce caz avem egalitate?

26. Să se arate că ϕ(m)ϕ(n) ≤ ϕ(mn) ≤ nϕ(m).

27. Să se rezolve ecuaţia ϕ(mn) = ϕ(m) + ϕ(n).

28. Să se arate că ϕ2 (mn) ≤ ϕ(m2 )ϕ(n2 ).

29. Să se arate că:


a) τ 2 (n)ϕ(n) > σ(n) pentru n ≥ 2.
b) τ (n)ϕ(n) ≥ n pentru n ≥ 1.
c) σ(n)ϕ(n) < n2 pentru n ≥ 2.
d) τ 2 (n)ϕ(n) ≤ n2 pentru n ≥ 5.

30. Să se arate că τ (n)σ(n) < n2 , ∀ n ≥ 13.

45
31. Să se arate că τ (n) + ϕ(n) ≤ n + 1, ∀ n ∈ N∗ , şi să se afle numerele
n pentru care are loc egalitate.

32. Să se arate că σ(n) = ϕ(n) + τ (n) dacă şi numai dacă n este prim.

33. Fie n ∈ N∗ . Să se arate că σ(n) + ϕ(n) = nτ (n) dacă şi numai dacă
n este prim.

34. Să se arate că:


n ln 2
a) ϕ(n) ≥ pentru n ≥ 1.
ln 2n
b) τ (n)ϕ(n) ≥ σ(n) pentru n impar.
6 σ(n)ϕ(n)
c) 2 < < 1.
π n2
d) σ(n)ϕ(n) ≥ nτ (n) pentru n ≥ 3.
e) σ 2 (n) ≥ nτ 2 (n) pentru n ≥ 1.
n σ(k)
X
35. Să se arate că < 2n, ∀ n ∈ N∗ .
k=1
k

n
36. Dacă d > 1 este divizorul minim al lui n, să se arate că τ (n) ≤ + 1.
d
37. Să se arate că există o infinitate de numere compuse n pentru care
σ(n) < 2n şi o infinitate de numere n pentru care σ(n) > 2n.

38. Să se arate că există o infinitate de numere naturale nenule m, pentru
σ(m) σ(k)
care > , ∀ k = 1, m − 1.
m k
39. Fie n ∈ N∗ şi 1 = d1 < d2 < · · · < dk = n toţi divizorii lui n scrişi ı̂n
ordine crescătoare. Să se afle toate numerele n pentru care d21 +d22 +d23 +d24 = n.
„ Ž2
X X
40. Fie n număr natural nenul. Să se arate că τ (d) = τ 3 (d).
d|n d|n

41. Fie a, b ∈ N∗ . Atunci 3τ (a)τ (b) = 4τ (ab) dacă şi numai dacă (a, b) = p
prim şi p3 nu divide ab.
(Baraj )

42. Să se găsească toate numerele n ∈ N∗ pentru care numărul divizorilor



lor este 3 4n.
(Lista scurtă, OIM, 2000)

46
n  n
X √
43. Să se arate că numerele şi [ n] au aceeaşi paritate.
i=1
i

44. Să se arate că dacă n ≥ 1, atunci numărul 103n+1 + 4(−1)n are mai
mult de 10 divizori supraunitari.

45. Fie p şi q două numere prime distincte. Să se arate că există numerele
naturale nenule a şi b, astfel ı̂ncât media aritmetică a divizorilor numărului
n = pa q b să fie număr ı̂ntreg.
(Baraj, OIM, 2002)

46. Determinaţi cel mai mic număr natural care are exact 144 de divizori
şi se divide cu 10 ı̂ntregi consecutivi.
( OIM, 1995)

47. Găsiţi numerele naturale N cu proprietăţile:


i) N are exact 16 divizori 1 = d1 < d2 < · · · < d16 = N .
ii) dd5 = (d2 + d4 )d6 .
(OBJ, 2002)

48. Fie n ∈ N, n = 24k − 1, k ∈ N∗ . Să se arate că σ(n) se divide cu 24.


(Putnam, 1969)

49. Fie q1 , q2 , . . . , qk (k ≥ 2) numere prime distincte. Să se arate că există


numerele naturale nenule α1 , . . . , αk , astfel ı̂ncât media aritmetică a divizorilor
k
Y
naturali ai numărului n = qiαi să fie număr natural.
i=1

50. Fie n ∈ N∗ şi a1 = 1 < a2 < · · · < aϕ(n) , toate numerele prime cu n
mai mici decât n. Să se arate că numerele a1 , a2 , . . . , aϕ(n) formează progresie
aritmetică, dacă şi numai dacă n = 1, n = 6, n = prim, n = 2k .
(OIM, 1991)
X
51. Se defineşte şirul (an )n∈N∗ prin relaţia 2n = ad . Să se arate că
d|n
n|an , ∀ n ∈ N∗ .
(Lista scurtă, OIM, 1989)

52. Să se rezolve ecuaţia τ (a) + τ (b) = τ (ab).


( OIM, 1979)

47
53. Să se găsească toate numerele k ∈ N∗ pentru care există n ∈ N∗ ,
τ (n2 )
astfel ı̂ncât = k.
τ (n)
( OIM, 1998)

54. Să se calculeze probabilitatea ca luând două numere x, y din N∗ , ele


să fie prime ı̂ntre ele.
 
X n
55. Să se arate că µ(k)τ = 1.
k|n
k
X
56. Să se arate că µ(k)τ (k) = (−1)ω(n) pentru n > 1, unde ω(n) este
k|n
numărul factorilor primi ai lui n.
X
57. Să se arate că |µ(k)|τ (k) = 3ω(n) .
k|n
Y
58. Dacă γ(n) = p pentru n > 1 şi γ(1) = 1, să se arate că pentru
p|n
X µ(k)γ(k) X µ(k) ϕ(n)
n > 1, = 0 şi = .
k|n
k k|n
γ(k) n

n  
X n
59. Să se arate că µ(k) = 1.
k=1
k
n  
X n n(n + 1)
60. Să se arate că ϕ(k) = .
k=1
k 2
n n  2
X X n
61. Să se arate că 2 ϕ(k) = µ(k) .
k=1 k=1
k
X
62. Dacă f : N∗ → R, F (n) = f (d), să se arate că
d|n

n n  
X X n
F (k) = f (k) .
k=1 k=1
k
63. Aplicând formula din problema precedentă, să se arate că:
n  
X n
a) µ(k) = 1.
k=1
k
n ϕ(k)
X n µ(k) n
X
b) = .
k=1
k k=1
k k

48
n  
X n n(n + 1)
c) ϕ(k) = .
k=1
k 2

X |µ(n)| π2
64. Să se arate că = .
n=1
ϕ(n)σ(n) 6
∞ xn ϕ(n)
X
65. Să se determine x ∈ (0, 1) dacă = 1.
n=1
1 − xn

49
CAPITOLUL 8

Distribuţia numerelor prime

Pentru x ∈ R, x ≥ 2, notăm cu π(x) numărul numerelor prime care nu ı̂l


depăşesc pe x. În 1896 J. Hadamard şi Ch. de la Vallée Poussin au arătat că
π(x) log x
lim = 1, unul din cele mai cunoscute rezultate din teoria numerelor
x→∞ x
(afirmaţia de mai sus este cunoscută sub numele de teorema elementului prim).
Rezultatul fusese conjecturat de Cebâşev ı̂n 1850 sub această formă şi de Gauss
Z x
π(x) dt
ı̂n 1792 (ı̂ntr-o formă echivalentă: lim = 1, unde Li(x) = ).
x→∞ Li(x) 2 log t
2 n
Cebâşev a arătat că pentru ∀ n ∈ N, n ≥ 3 au loc inegalităţile < π(n) <
3 log n
3 n
. Există forme mai rafinate ale acestor inegalităţi, dintre care amintim
2 log n
x x
aici π(x) < pentru x ≥ 4 şi π(x) > pentru x ≥ 5393.
log x − 1, 12 log x − 1
pn
Teorema elementului prim implică faptul că lim = 1 (vezi şi problema
n→∞ n log n
16 din acest capitol), unde pn indică al n-lea număr prim (p1 = 2, p2 = 3,
p3 = 5, p4 = 7, p5 = 11 . . . ). Este uşor de arătat că lim sup(pn+1 − pn ) = +∞,
n→∞
ı̂nsă este o problemă ı̂ncă deschisă dacă există o infinitate de indici n pentru
care pn+1 − pn = 2 (pn şi pn+1 cu proprietatea că pn+1 − pn = 2 se numesc
numere prime gemene).
În 1845 Bertrand a conjecturat că π(2n) − π(n) ≥ 1 pentru n ≥ 2. Rezul-
tatul a fost demonstrat de Cebâşev ı̂n 1850 şi este cunoscut sub numele de
”Postulatul lui Bertrand” sau ”Teorema lui Cebâşev”. El poate fi pus şi sub
forma următoare: ∀ n ∈ N, n ≥ 2, ∃ p prim astfel ı̂ncât n < p < 2n. Există
variante mai tari pentru teorema lui Bertrand-Cebâşev: π(2n) − π(n) ≥ 2,
∀ n ≥ 6 şi π(2n) − π(n) ≥ 3 pentru n ≥ 9. Ca o consecinţă a acestor ine-
galităţi, rezultă imediat că pn+1 < 2pn ∀ n ∈ N∗ , pn+2 < 2pn , ∀ n ∈ N, n ≥ 4
şi pn+3 < 2pn , ∀ n ∈ N, n ≥ 5.

51
1. Să se arate că pentru orice n ≥ 1, există cel puţin trei numere prime
care au exact n cifre.

2. a) Dacă s este natural n ≥ 4, să se arate că există m natural, astfel ca


n ≤ m ≤ 2n, m = pq, p 6= q, p, q prime.
b) Dacă n este natural n ≥ 15, să se arate că există m natural, astfel
că n ≤ m ≤ 2n, m = pqr cu p < q < r, p, q, r prime.

3. Dacă s este număr natural, să se arate că pentru n suficient de mare,
ı̂ntre n şi 2n se găseşte un număr cu exact s factori primi distincţi. Acest enunţ
generalizează problema precedentă.

4. a) Pentru orice număr natural n, n ≥ 2, există un număr prim p, astfel


n
ı̂ncât < p ≤ n.
2
b) Dacă k, n ∈ N, k ≥ 2, n ≥ 2, ecuaţia n! = xk nu are soluţii naturale.

5. Fie n natural, n ≥ 2 şi numerele ı̂ntregi ai , i¤|¤ai , ∀ i = 2, n. Să se arate


n a
X Xn 1
i
că 6∈ Z. În particular, 6∈ N, ∀ n ∈ N, n ≥ 2.
i=2
i i=2
i

1 1 1
6. Fie n, k ≥ 1 numere naturale. Să se arate că + + ··· +
n n+1 n+k
nu este ı̂ntreg.

7. Găsiţi toate numerele n ∈ N, n ≥ 2, astfel ı̂ncât x2 ≡ 1(n), ∀ x ∈ Z,


(x, n) = 1.
(Lista scurtă, OIM, 2000)

8. Fie qn cel mai mic număr prim pentru care qn ¤|¤n. Să se arate că
qn
lim = 0.
n→∞ n

9. Să se arate că pentru orice k ı̂ntreg, k ≥ 1, elementele mulţimii


{1, 2, 3, . . . , 2k} pot fi ı̂mpărţite ı̂n perechi disjuncte, astfel ca suma numerelor
din fiecare pereche să fie număr prim.

10. Fie (an )n≥1 un şir strict crescător de numere naturale, astfel ı̂ncât
a2n = an + n pentru orice n ≥ 1. Ştiind că dacă an este prim atunci şi n este
prim, să se arate că an = n pentru orice n.

52
11. Să se arate că numărul x = 0, a1 a2 a3 . . . ak . . . , unde
8
>
< 1 dacă n este prim
an = 0 dacă n este compus, este iraţional.
>
:
0 dacă n = 1
12. Fie π(x) numărul numerelor prime care nu depăşesc pe x şi fie n
π(n) π(n − 1)
număr ı̂ntreg, n ≥ 2. Avem că > dacă şi numai dacă n este
n n−1
prim.
13. Să se arate că pentru n ≥ 1 avem π(n!+2n)+π(n) ≤ π(n!+n)+π(2n).
14. Dacă (pn )n≥1 este şirul numerelor prime, să se arate că există o
infinitate de indici n, pentru care pn+1 ≥ 2pk − pk−1 pentru orice k ∈ 2, n.
15. Se consideră m numere naturale distincte ı̂n intervalul [1, n], astfel
ı̂ncât nici unul nu divide produsul celorlalte n − 1. Să se arate că m ≤ π(n).
16. Se notează π(x) numărul numerelor prime care nu depăşesc pe x şi
x
pn al n-lea număr prim. Ştiind că π(x) ∼ , să se arate că pn ∼ n ln n.
ln x
n
X 1
17. Să se arate că pk ∼ npn .
k=1
2
∞ 1
X
18. Să se arate că seria este divergentă.
n=1
pn
(Euler )
n p
X k+1 − pk
19. Să se arate că ∼ n.
k=1
ln pk
√∞ √
X
pn+1 − pn
20. Să se arate că seria este convergentă.
n=1
n
21. Fie (an )n≥1 , (bn )n≥1 şiruri de numere naturale cu limita ∞. Să se
an pan
arate că lim = 1, dacă şi numai dacă lim = 1.
n→∞ bn n→∞ pb
n

22. Care numere n se scriu ca sumă a două numere compuse?


23. Să se găsească toate numerele n ∈ N∗ pentru care:
X X X
a) n = p; b) n = p; c) n = p.
p prim p prim p prim
p<n p≤ n
2 p≤ n
3

53
24. Să se determine n natural, astfel ı̂ncât ı̂n intervalul (n, n2 ) să se
găsească n numere prime.

25. Să se arate că pentru orice n natural, n ≥ 1, există cel puţin n numere
prime cu n cifre.

26. Să se determine numerele naturale n, pentru care π(n) = s(n).

27. Fie f : N∗→ N∗ o funcţie total multiplicativă, pentru care f (pn ) = n+1,
X∞ 1
pentru orice n ≥ 1. Să se arate că 2
= 2.
n=1
f (n)

28. Să se arate că dacă a > b > 0, atunci lim (π(ax) − π(bx)) = ∞.
x→∞
 
p
29. Să se arate că mulţimea A = p, q prime este densă ı̂n R+ .
q
X X
30. Fie θ(x) = ln p şi ψ(x) = ln p funcţiile lui Cebâşev şi a un
p≤x pm ≤x

X
număr real nenul. Să se arate că seria ((θ(n))a − (ψ(n))a ) este convergentă
n=2
1
dacă şi numai dacă a < − .
2

54
CAPITOLUL 9

Congruenţe

Dacă n ∈ N∗ şi a, b ∈ Z, notăm a ≡ b (mod n) (sau uneori a ≡ b(n)) dacă


n|a − b (se spune că a este congruent cu b modulo n). Se arată uşor că ≡ este
o relaţie de echivalenţă pe Z (relaţia este simetrică, reflexivă şi tranzitivă).
Dacă a ≡ b (mod n) şi c ≡ d (mod n), se arată imediat că a + c ≡ b + d
(mod n) şi ac ≡ bd (mod n). Vom nota cu a b mulţimea {b ∈ Z|a ≡ b (mod n)}
b b b Õ
şi cu Zn mulţimea {0, 1, 2, . . . , n − 1}. Se definesc operaţiile ”+” şi ”·” pe Zn
ı̂n felul următor: ab +b b = aÕ+ b, a b ·b c Se arată uşor că operaţiile sunt bine
b = ab.
definite şi (Zn , +, ·) este un inel comutativ cu n elemente, numit inelul claselor
de resturi modulo n. Numărul elementelor inversabile din acest inel (faţă de
operaţia ”·”) este ϕ(n), unde ϕ este indicatorul lui Euler. Zn este corp ⇔ n
este prim. Dacă a ∈ Z, (a, n) = 1, n ∈ N∗ , atunci aϕ(n) ≡ 1 (mod n); aceasta
este teorema lui Euler.
Un caz particular al acestui rezultat este Mica Teoremă a lui Fermat: dacă p
este un număr prim care nu divide pe a ∈ Z, atunci ap−1 ≡ 1 (mod p). Teorema
lui Wilson afirmă că (p−1)! ≡ −1 (mod p) dacă şi numai dacă p este prim. Dacă
n, m ∈ N∗ , (n, m) = 1, atunci inelele Zmn şi Zm × Zn sunt izomorfe. Aceasta
este forma algebrică a lemei chineze a resturilor. În limbajul teoriei numerelor,
ea se formulează ı̂n felul următor: fie m1 , m2 , . . . , ms numere naturale nenule
prime ı̂ntre ele două câte două şi fie numerele ı̂ntregi a1 , a2 , . . . , as .
Atunci sistemul x ≡ aj (mod mj ), ∀ j = 1, s are soluţie şi aceasta este unică
modulo m1 · m2 · . . . · ms .
Fie f ∈ Z[X] şi x0 ∈ Z astfel ı̂ncât f (x0 ) ≡ 0 (mod pi ), unde p este un
număr prim şi i ∈ N∗ . Dacă f 0 (x0 ) 6≡ 0 (mod p), atunci x0 generează o soluţie
f (x0 )
pentru congruenţa f (x) ≡ 0 (mod pi+1 ). Dacă f 0 (x0 ) ≡ 0 (mod p) şi ≡0
pi
(mod p), atunci x0 generează p soluţii pentru congruenţa f (x) ≡ 0 (mod pi+1 ),
f (x0 )
iar dacă f 0 (x0 ) ≡ 0 (mod p) şi 6≡ 0 (mod p), atunci x0 nu generează nici
pi
o soluţie pentru congruenţa f (x) ≡ 0 (mod pi+1 ).

55
Vom folosi ı̂n acest capitol şi teorema lui Lagrange care afirmă că dacă
n
X
f (x) = ak xk , p prim, p¤|¤an , atunci congruenţa f (x) ≡ 0 (mod p) are cel
k=0
mult n soluţii modulo p.

Problema 1. Fie şirul an = 2n + 3n + 6n − 1, n ∈ N∗ . Să se determine


mulţimea M = {m ∈ N∗ |(m, an ) = 1, ∀ n ∈ N∗ }.
(OIM 2005)

Rezolvare. Arătăm că M = {1}. Să presupunem că există m ∈ M , m > 1.


Atunci ∃ p prim, p|m şi cum (an , m) = 1, ∀ n ∈ N∗ , rezultă că (an , p) = 1,
∀ n ∈ N∗ . Deci ∃ p prim, p ∈ M . Cum a1 = 10 şi a2 = 48, este evident
că p 6= 2, 3, 5. Deci p ≥ 7. Vom arăta acum că p|ap−2 , ceea ce constituie o
contradicţie cu faptul că p ∈ M .
Din Mica Teoremă a lui Fermat ştim că 2p−1 ≡ 3p−1 ≡ 6p−1 ≡ 1 (mod p).
p
Atunci 6ap−2 = 3 · 2p−1 + 2 · 3p−1 + 6p−1 − 6 ≡ 3 + 2 + 1 − 6 = 0 (mod p).
Deci p|6ap−2 şi cum p 6= 2, 3, deducem că p|ap−2 , ceea ce am văzut că este
imposibil. Deci M = {1}.

Problema 2. Fie (an )n≥1 un şir de numere ı̂ntregi cu proprietatea că


a1 , a2 , . . . , an constituie un sistem de reprezentanţi modulo n pentru ∀ n∈ N∗ .
Se ştie, de asemenea, că şirul an are o infinitate de termeni pozitivi şi o in-
finitate de termeni negativi. Să se arate că fiecare număr ı̂ntreg apare exact o
dată ı̂n şir.
(OIM 2005)

Rezolvare. Să presupunem că ∃ 1 ≤ i < j astfel ı̂ncât ai = aj . Atunci nu-


merele a1 , a2 , . . . , ai , . . . , aj nu pot constitui un sistem de reprezentanţi modulo
j, deoarece ai ≡ aj (mod j) (are loc chiar egalitate). Deci termenii şirului sunt
distincţi. Fie k ∈ Z, arbitrar. Considerăm şirul bn = an − k. El are aceleaşi
proprietăţi ca şirul iniţial. Pentru a arăta că ∃ n ∈ N∗ astfel ı̂ncât an = k, este
suficient să arătăm că ∃ n ∈ N∗ astfel ı̂ncât bn = 0.
Să presupunem că bn 6= 0, ∀ n ∈ N∗ . Fie k primul indice pentru care
b1 , b2 , . . . , bk au acelaşi semn şi bk+1 are semn opus faţă de b1 . Putem presupune
că bi > 0, ∀ i = 1, k şi bk+1 < 0 (aceasta deoarece şi şirul −bn are proprietăţile
din enunţ). Fie bj = max{b1 , b2 , . . . , bk }. Avem evident că bj ≥ k (deoarece
b1 , b2 , . . . , bk sunt numere naturale nenule distincte). Notăm n = bj − bk+1 şi
avem că n ≥ k + 1 (deoarece bj ≥ k şi ak+1 ≤ −1).

56
Ştim din ipoteză că b1 , b2 , . . . , bn constituie un sistem de reprezentanţi
modulo n. Acest lucru nu este adevărat deoarece n = bj − bk+1 , bj ≡ bk+1
(mod n) şi 1 ≤ j ≤ k < k + 1 ≤ n. Presupunerea făcută a fost deci falsă şi
∃ n ∈ N∗ astfel ı̂ncât bn = 0, an = k.
Problema 3. Găsiţi toate perechile de numere naturale (m, n) pentru care
m ≥ 2, n ≥ 2 şi m divide an − 1 pentru orice a ı̂n mulţimea {1, 2, . . . , n}.
(Baraj 2001, Laurenţiu Panaitopol )

Rezolvare. Fie m şi n cu proprietăţile din enunţ. Fie p prim, p|m. Dacă
n ≥ p, atunci ar trebui ca p|m|pn − 1, ceea ce este evident absurd. Deci
n ≤ p−1. Considerăm polinomul f (X) = (X −1)(X −2) . . . (X −n)−(X n −1)
cu coeficienţii ı̂ntregi de grad n − 1. Coeficientul lui X n−1 este −(1 + 2 +
n(n + 1) n(n + 1)
3 + · · · + n) = − . Dacă p¤|¤ , atunci, conform teoremei lui
2 2
Lagrange, congruenţa f (x) ≡ 0 (mod p) ar trebui să aibă cel mult n − 1 soluţii
modulo p. Însă ea are soluţiile 1, 2, 3, . . . , n care nu sunt congruente modulo
n(n + 1)
p deoarece n ≤ p − 1. Deci neapărat p| şi p|n(n + 1). Dacă p|n,
2
rezultă contradicţia p ≤ n (am arătat mai devreme că n ≤ p − 1). Deci p|n + 1,
p ≤ n+1 ≤ p. Rezultă de aici că n = p−1 şi m = pk , k ≥ 1. Dacă cumva k ≥ 2,
atunci p2 |m|(p − 1)p−1 − 1. Dar (p − 1)p−1 − 1 ≡ −p(p − 1) (mod p2 ) şi rezultă
contradicţia p2 |p(p − 1). Am folosit mai sus că p este impar (p − 1 = n ≥ 2,
p ≥ 3). Deci k = 1 şi singurele soluţii ale problemei sunt perechile (p, p − 1), p
fiind număr prim impar. Reciproca afirmaţiei este evidentă deoarece p|ap−1 −1,
∀ a ∈ {1, 2, 3, . . . , p − 1}, conform Micii Teorema a lui Fermat.

1. Pentru orice numere ı̂ntregi k, x, y, z avem 9k ± 4 6= x3 + y 3 + z 3 .

2. Fie p prim, p > 3. Să se arate că 7p − 6p ≡ 1(mod 43).


(Iran, 1999)

3. Să se factorizeze numărul 1000027.


nn n
4. Să se arate că 1989 nn − nn , ∀ n ∈ N, n ≥ 3.
(Propusă pentru Balcaniada din 1989)

5. Fie n, m ∈ N, m impar. Atunci (2m − 1, 2n + 1) = 1.

6. Pentru orice număr natural impar n, 2n! − 1 este multiplu de n.

7. Să se rezolve congruenţa 237x ≡ 1(mod 1000) cu ajutorul fracţiilor


continue.

57
8. a) Să se rezolve congruenţa 237x ≡ 1(mod 1000).
b) Să se afle ultimele trei cifre ale lui 12371199 .
c) Să se afle ultimele trei cifre ale lui 12361199 .
d) Să se afle ultimele trei cifre ale lui 12351199 .

9. Dacă 3a7 ≡ 5(mod 11), să se arate că a ≡ 3(mod 11).

10. Dacă 2a7 ≡ 1(mod 19), să se arate că a ≡ 13(mod 19).

11. Să se rezolve congruenţa a37 ≡ 3(mod 17).

12. Să se arate că pentru orice număr natural n, restul ı̂mpărţirii lui
10n − 1 la 37 este pătrat perfect.

13. Să se calculeze restul ı̂mpărţirii lui:


a) 100! la 109;
b) 75! · 130! la 211.

14. Să se arate că dacă ultimele patru cifre ale unui pătrat sunt egale,
atunci ele sunt 0.

15. Dacă a este ı̂ntreg a5 ≡ 2(mod 7) şi a41 ≡ 2(mod 13), să se arate că
a ≡ 32(mod 91).

16. Să se afle numerele prime p pentru care:


2 2
a) 3p + 11p ≡ 0(mod p2 );
2 2
b) 42p + 32p ≡ 0(mod p2 ).

4p − 1
17. Dacă p este prim p > 3 şi dacă n = , atunci n divide 2n − 2.
3
(Erdös)

2p + 1
18. Pentru care numere prime p, este ı̂ntreg?
p
19. Fie p prim şi a, b ∈ Z astfel ı̂ncât ap ≡ bp (p). Să se arate că
ap ≡ bp (mod p2 ).

20. Dacă p şi q sunt numere prime diferite şi a este ı̂ntreg, să se arate că
apq − ap − aq + a se divide la pq.

21. Dacă p este prim şi n este număr natural nenul, să se arate că
n  n 2
X X
k 2p+1 ≡ kp (mod p).
k=1 k=1

58
”€ √ Šp —
22. Pentru p prim, p 6= 2, să se arate că 2+ 3 − 3 se divide la p.

23. Dacă p este prim, m ≥ 2 şi 3n − 2n = pm , să se arate că n este prim.
(Concurs Bulgaria)

m+1
24. Dacă m este impar, 2ϕ(m)−1 ı̂mpărţit la m dă restul .
2
25. Dacă a şi b sunt numere naturale nenule coprime, să se arate că
aϕ(b) + bϕ(a) ≡ 1(mod ab).

26. Fie a, m ∈ N∗ , m ≥ 2. Să se arate că m|am − am−ϕ(m) .

27. Să se rezolve sistemele:


a) 2x ≡ 3(mod 7), 3x ≡ 5(mod 8), 2x ≡ 5(mod 9);
b) 2x ≡ 1(mod 3), 5x ≡ 6(mod 7), 7x ≡ 12(mod 13), folosind lema
chineză a resturilor.

28. Să se rezolve congruenţa x3 + x + 4 ≡ 0(mod 343).

29. Să se rezolve congruenţa x2 + x + 1974 ≡ 0(mod 990).

30. Să se rezolve congruenţele:


a) 7x4 + 19x + 25 ≡ 0(mod 27);
b) x3 − x + 2 ≡ 0(mod 32).

31. Să se arate că congruenţa x112 + 3x + 5 ≡ 0(mod 121) nu are soluţii.

32. Să se arate că pentru p număr prim şi n ≥ 1, congruenţa


xp−1 ≡ 1(mod pn )
are p − 1 soluţii.

33. Să se determine numerele naturale a, astfel ı̂ncât pentru orice k ≥ 1


există x, astfel ca x2 ≡ a(mod 2k ).

34. Să se arate că pentru orice număr ı̂ntreg n, orice factor al lui 2n2 +
6n + 5 este de forma 4k + 1.

35. Fie a, b, c numere ı̂ntregi, a, b, c ≥ 1. Dacă abc se scrie ca sumă de


două pătrate şi ab + bc + ca este prim, atunci a, b, c se scriu ca sume de două
pătrate.

36. Dacă a, b sunt numere naturale nenule cu (a, b) = 1 şi ab = x2 + y 2 ,


x, y ∈ Z, atunci există m, n, p, q ∈ Z şi a = m2 + n2 şi b = p2 + q 2 .

37. Fie a, b, x, y numere naturale astfel ı̂ncât ab = x2 + y 2 6= 0 şi a + b


prim. Să se arate că a = m2 + n2 , b = p2 + q 2 cu m, n, p, q numere naturale.

59
m2 + 1
38. Să se arate că nu este număr ı̂ntreg, dacă n este număr
7n2 − 1
natural nenul.
2a2 − 1
39. Să se arate că dacă a şi b sunt numere ı̂ntregi, atunci 2 nu este
b +2
număr ı̂ntreg.

È
40. Fie a, b numere naturale congruente cu 3 modulo 4. Să se arate că
ax2 + by 2 este număr raţional (x, y numere ı̂ntregi), dacă şi numai dacă
x = y = 0.

41. Fie a, b, c numere ı̂ntregi impare. Să se arate că nu există x, y, z numere
ı̂ntregi, astfel ı̂ncât z 2 − c2 = (x2 + a2 )(y 2 + b2 ).

42. Să se arate că dacă n are un divizor prim p = 4k + 3, atunci ecuaţia
x2 + 3 = y n nu are soluţii ı̂ntregi.

43. Dacă p este prim, p = 4k + 3, atunci ecuaţia x4 − py 4 = 1 admite ı̂n


numere ı̂ntregi numai soluţiile (±1, 0).

44. Fie p ≥ 3 număr prim. Pentru k ∈ 1, p − 1 notăm rk restul ı̂mpărţirii


p−1
X
lui kp la p2 . Să se calculeze rk .
k=1

45. Să se determine numerele ı̂ntregi nenule a, pentru care (n + a)n − a


e multiplu de n2 , pentru orice număr natural nenul n.

46. Fie c ∈ N∗ . Să se găsească a, b ∈ N∗ astfel ı̂ncât 2c − 1|2a + 2b + 1.


(OIM, 1990)

47. Să se găsească un număr n ∈ N, 100 ≤ n ≤ 1997 astfel ı̂ncât n|2n + 2.

48. Să se arate că pentru orice p prim, există o infinitate de numere n,
pentru care p|2n + n.

49. a, b ∈ Z, a ≥ 2. Există o infinitate de n ∈ N, astfel ı̂ncât an + b să fie


număr compus.

50. Fie a, b ∈ N. Să se arate că există un număr finit de numere naturale
   
1 n 1 n
n, astfel ı̂ncât a + + b+ ∈ Z.
2 2

60
51. Să se găsească cel mai mic n ∈ N∗ , cu proprietatea că fiind date n
numere ı̂ntregi distincte, se pot găsi ı̂ntotdeauna 4 distincte dintre ele a, b, c, d,
astfel ı̂ncât a + b ≡ c + d(mod 20).
(Lista scurtă, OIM, 1998)

52. Se consideră şirul (an )n≥1 pentru care a1 = 2 şi an+1 este cel mai
mare divizor prim al lui a1 a2 . . . an + 1 pentru n ≥ 1. Să se arate că 5 şi 11 nu
se află ı̂n acest şir.

53. Să se arate că dacă p este prim şi n este un număr natural, n ≥ p,
 
p
n
atunci Cn − este multiplu de p.
p

54. Să se arate că n este prim dacă şi numai dacă Cnk este multiplu de n,
oricare k număr natural, 1 ≤ k ≤ n − 1.

55. Pentru care numere n ∈ N∗ , Cnk este impar ∀ k = 0, n?

56. Fie p, n ∈ N∗ , p ≥ 3, n < p. Să se arate că (n − 1)!(p − n)! ≡


(−1)n (mod p), dacă şi numai dacă p este prim.
 
2p
57. Fie p un număr prim, p ≥ 5 şi k = . Să se arate că p2 Cp1 + Cp2 +
3
· · · + Cpk .
p
58. Fie p prim. Să se arate că p3 C3p − 3 dacă p ≥ 3.
p−1
X
2 1 a
59. Fie p prim, p ≥ 5, = , (a, b) = 1, a, b ∈ N∗ . Să se arate
i=1
i(p − i) b
că p|a.

1 1 1 c
60. Dacă p > 3 este număr prim şi + + · · · + = , c, d ∈ N∗ ,
12 22 (p − 1)2 d
(c, d) = 1, să se arate că p|c.

61. Fie Sk (n) = Sk = 1k + 2k + · · · + nk şi p prim, p > 2. Dacă


S1 , S2 , . . . , S2m−1 se divid cu p, să se arate că S2m+1 se divide cu p2 .

62. Să se determine numerele naturale nenule x, y, z pentru care:


a) xy ≡ 1(mod z), yz ≡ 1(mod x), xz ≡ 1(mod y);
b) xy ≡ −1(mod z), yz ≡ −1(mod x), xz ≡ −1(mod y).

61
63. a) Să se arate că nu există x, y ∈ Z astfel ı̂ncât x2 = y 5 − 4.
(Balcaniadă, 1998)
b) Să se arate că ecuaţia x3 − 3xy 2 + y3 = 2891 nu are soluţii ı̂n Z.
(OIM, 1982)

64. Să se găsească toate numerele n ∈ N∗ pentru care ∃ m ∈ N, astfel


ı̂ncât 2n − 1|m2 + 9.
(Lista scurtă, OIM, 1998)

65. n copii stau ı̂n cerc şi primesc bomboane de la profesorul lor după
următoarea regulă: el dă o bomboană unui copil, sare unul, dă o bomboană
următorului, sare doi copii, dă o bomboană următorului şi aşa mai departe.
Cum trebuie să fie n ∈ N∗ , pentru ca fiecare copil să primească cel puţin o
bomboană?
(APMO, 1991)

66. Fie p un număr prim. Să se determine gradul maxim al polinoamelor


f , cu coeficienţi ı̂n mulţimea {0, 1, . . . , p − 1}, de grad mai mic strict decât
p şi care au proprietatea că, dacă f (n) ≡ f (m)(mod p), atunci neapărat n ≡
m(mod p).
(Concurs Turcia, 2000)

67. Dacă p este prim, p ≥ 3, atunci are loc congruenţa 1p−1 + 2p−1 + . . .
+(p − 1)p−1 ≡ p + (p − 1)!(mod p2 ).
(Lerch)

68. n ∈ N, n ≥ 2. Următoarele afirmaţii sunt echivalente:


a) n şi n + 2 sunt prime;
b) 4((n − 1)! + 1) + n ≡ 0(mod n(n + 2)).
(Clement)

69. a) Dacă m este un număr compus, m ≥ 6, atunci m|(m − 1)!;


b) Să se afle toate numerele prime p pentru care (p − 1)! + 1 = pk .
(Liouville)

70. Să se arate că ∀ n ∈ N∗ , există n numere naturale consecutive, astfel


ı̂ncât nici unul dintre ele să nu fie putere de număr prim.
( OIM, 1989)

71. Să se arate că pentru numărul prim p = 1093 avem 2p−1 ≡ 1(mod p2 ).

62
CAPITOLUL 10

Resturi pătratice. Simbolul lui Legendre

Fie a ∈ Z şi p un număr prim care nu divide pe a.


 
a
Definiţie. Se defineşte simbolul lui Legendre ca fiind 1, dacă ∃ b ∈ Z
p
   
2
a a
astfel ı̂ncât a ≡ b (mod p) şi = −1 ı̂n caz contrar. Dacă = 1,
p p
 
a
spunem că a este rest pătratic modulo p, iar dacă = −1, spunem că a
p
este nerest pătratic modulo p. Dacă p ≥ 3, ı̂n mulţimea {1, 2, . . . , p − 1} există
p−1 p−1
resturi pătratice şi neresturi pătratice modulo p. Următoarele
2 2
proprietăţi permit calculul rapid al simbolului Legendre:
   
a b
1) dacă a ≡ b (mod p), p¤|¤ab, atunci = ;
p p
 2
k
2) dacă p¤|¤k, atunci = 1;
p
    
ab a b
3) dacă p¤|¤ab, atunci = ;
p p p
 
p−1 a
4) dacă p¤|¤a, p 6= 2, atunci a 2 ≡ (mod p) (criteriul lui Euler);
p
 
−1 p−1
5) dacă p 6= 2, atunci = (−1) 2 ;
p
 
2 p2 −1
6) dacă p 6= 2, atunci = (−1) 8 ;
p
   
p q
7) dacă p şi q sunt numere prime impare şi distincte, atunci = ·
q p
p−1 q−1
(−1) 2 · 2 (legea de reciprocitate pătratică).

r
Y αj
Definiţie. Dacă m ∈ N∗ , m ≥ 2, m = pj (p1 < p2 < · · · < pr sunt
j=1
numere prime distincte, αj ∈ N∗ , ∀ j = 1, r) şi a ∈ N∗ , (a, m) = 1, se defineşte

63
   
a Yr a αj
simbolul Jacobi ca fiind = , simbolurile din dreapta fiind
m j=1
pj
simboluri Legendre. Se arată că simbolul Jacobi are proprietăţile de mai sus
(fără proprietatea 4)); la 5) şi la 6) p se ı̂nlocuieşte cu un număr impar, iar la
7) p şi q sunt numere impare prime ı̂ntre ele şi distincte.

Problema 1. Fie p prim, p > 2, p = 3k + 2 (k ∈ N). Fie mulţimea


S = {y 2 − x3 − 1|x, y ∈ Z, 0 ≤ x, y ≤ p − 1}. Să se arate că cel mult p − 1
elemente ale mulţimii S sunt divizibile cu p.
(Balcaniadă, 1999)

Rezolvare. Arătăm ı̂ntâi că funcţia f : Zp → Zp , f (x) = x3 este injectivă


pentru p ≡ 2 (mod 3). Dacă nu ar fi aşa, atunci găsim x, y ∈ Z astfel ı̂ncât
x3 ≡ y 3 (mod p) şi x 6≡ y (mod p). Deducem că p|x2 + xy + y 2 (deoarece
p|x3 − y 3 = (x − y)(x2 + xy + y 2 ) şi p¤|¤x − y). De aici rezultă că p|x şi p|y (vezi
problema 15 din acest capitol), ceea ce constituie o contradicţie căci p¤|¤x − y.
Dacă p¤|¤x, rezultă imediat că p¤|¤y şi p|4x2 + 4xy + 4y 2 = (2x + y)2 + 3y 2 ,
−3y 2 ≡ (2x + y)2 (mod p). Deoarece p¤|¤(2x + y), p¤|¤3, p¤|¤y, deducem că
‚ Œ ‚ Œ  ‚ Œ  
(2x + y)2 −3y 2 −3 y2 −3
1= = = = =
p p p p p
    ‹  ‹  ‹
−1 3 p−1 p p−1 3−1 p 2
= = (−1) 2 (−1) 2 · 2 = = = −1;
p p 3 3 3
contradicţie. Deci p|x şi p|y; contradicţie. Deci funcţia de mai sus este injectivă.
Numărăm acum câte perechi (x, y) există cu x, y ∈ N, 0 ≤ x, y ≤ p − 1 şi
p|y 2 − x3 − 1. Dăm lui y o valoare oarecare din mulţimea {0, 1, 2, . . . , p − 1}.
Deoarece funcţia f (x) = x3 , f : Zp → Zp este injectivă (deci şi bijectivă
fiindcă Zp este finită), rezultă că ∃ un singur x ∈ {0, 1, 2, . . . , p − 1} astfel ı̂ncât
x3 ≡ y 2 − 1 (mod p). Rezultă că există p perechi (x, y) cu 0 ≤ x, y ≤ p − 1,
x, y ∈ N şi p|y 2 − x3 − 1. Din aceste p perechi, două conduc la acelaşi număr
din S, y 2 − x3 − 1, şi anume 12 − 03 − 1 = 32 − 23 − 1 = 0 (e vorba de perechile
(0, 1) şi (2, 3)). Deci cel mult p − 1 elemente ale lui S sunt divizibile cu p.

Problema 2. Fie polinomul f (X) = a1 + a2 X + a3 X 2 + · · · + ap−1 X p−2 ,


 
j
unde p este un număr prim, p ≥ 3 şi aj = , ∀ j = 1, p − 1.
p
a) Să se arate că 1 este rădăcină simplă a lui f ⇔ p ≡ 3 (mod 4);
b) Dacă p ≡ 5 (mod 8), să se arate că 1 este rădăcină de ordin 2 a lui f .
(Baraj 2004, Călin Popescu)

64
p−1  
X j p−1
Rezolvare. a) Avem f (1) = = 0 deoarece există resturi
j=1
p 2
p−1
pătratice şi neresturi pătratice modulo p. Să presupunem că p ≡ 1
2
(mod 4) şi să arătăm că f 0 (1) = 0. Avem că f 0 (1) = a2 + 2a3 + 3a4 + · · · +
 
−1
(p − 2)ap−1 . Avem că aj = ap−j deoarece = 1 ı̂n acest caz. Deci
p
p−1
X
2
(j−1)aj +(p−j−1)ap−j = aj (j−1+p−j−1) = (p−2)aj şi f 0 (1) = (p−2) aj .
j=1
p−1
X
2 p−1
X p−1
X
Notând S = aj , T = aj , avem că S + T = aj = 0 şi S = T
j=1 j= p+1 j=1
2
(deoarece aj = ap−j ). Deducem că S = T = 0 şi f 0 (1) = (p − 2)S = 0 şi 1 nu
este rădăcină simplă pentru f . Să presupunem acum că p ≡ 3 (mod 4).
În acest caz ap−j = −aj şi (j − 1)aj + (p − j − 1)ap−j = aj (j − 1 − p + j + 1) =
p−1
X
2
aj (2j − p) este număr impar. Deci f 0 (1) = aj (2j − p) este număr im-
j=1
p−1
par deoarece se adună un număr impar de numere impare ( este impar
2
deoarece p ≡ 3 (mod 4)). Deci f 0 (1) ≡ 1 (mod 2), f 0 (1) =
6 0 şi 1 este rădăcină
simplă a lui f ı̂n acest caz.
b) Fie p prim, p ≡ 5 (mod 8). Am văzut la punctul a) că f 0 (1) = 0 ı̂n acest
caz. Pentru a demonstra enunţul, trebuie să arătăm că f 00 (1) 6= 0. Avem că
p−1
X
f 00 (1) = (j − 2)(j − 1)aj . În acest caz avem că aj = ap−j .
j=1
Vom arăta că f 00 (1) ≡ 4 (mod 8); aceasta implică f 00 (1) 6= 0 şi deci 1 este
rădăcină dublă a lui f . Pentru a arăta că f 00 (1) ≡ 4 (mod 8), grupăm termenii
8
(j − 2)(j − 1)aj + (p − j − 2)(p − j − 1)ap−j ≡ aj [(j − 2)(j − 1) + (3 − j)(4 − j)] =
8
aj (j 2 − 3j + 2 + j 2 − 7j + 12) ≡ aj (2j 2 − 2j − 2).
Se arată imediat că 2j 2 − 2j − 2 ≡ 2 (mod 8) dacă j ≡ 2, 3 (mod 4) şi 2j 2 −
2j − 2 ≡ −2 (mod 8) dacă j ≡ 0, 1 (mod 4).
8
Ţinând cont de aceste informaţii, rezultă că f 00 (1) ≡ 2(−a1 + a2 + a3 −
a4 − a5 + a6 + a7 − a8 · · · − a4k−3 + a4k−2 + a4k−1 − a4k − a4k+1 + a4k+2 ), unde
p−1
X
2
p = 8k + 5. Am arătat ceva mai sus că aj = 0 dacă p ≡ 1 (mod 4).
j=1

65
Deci −a1 − a4 − a5 − a8 · · · − a4k − a4k+1 = a2 + a3 + a6 + a7 · · · + a4k−2 +
8
a4k−1 + a4k+2 şi deci f 00 (1) ≡ 4(a2 + a3 + a6 + a7 · · · + a4k−1 + a4k+2 ).
Cum a2 + a3 + a6 + a7 · · · + a4k−1 + a4k+2 este număr impar (fiind suma a
2k + 1 numere impare), deducem că f 00 (1) ≡ 4 (mod 8), ceea ce am văzut că
demonstrează enunţul ı̂n acest caz.
 
30
1. Să se calculeze .
211
 p 
3 +1
2. Fie p număr prim, p > 3. Să se calculeze .
p
3. Fie p şi q numere prime impare care se scriu sub forma p = a2 + b2 ,
 2 2   
a c − b2 d2 p
q = c2 +d2 (a, b, c, d fiind numere ı̂ntregi). Să se arate că = .
p q
4. Fie p prim, p > 3 şi 1 = r1 < r2 < · · · < r p−1 ≤ p − 1 toate resturile
2
p−1
X2
pătratice modulo p. Să se arate că rj ≡ 0(mod p).
j=1

5. Fie p prim, p ≡ 1(mod 4). Să se arate că dacă a şi b sunt ı̂ntregi, 1 ≤ a,
b ≤ p − 1, astfel ı̂ncât a + b = p, atunci ele sunt simultan resturi sau neresturi
pătratice modulo p.
p−1
6. a) Fie p prim, p ≡ 1(mod 4). Să se arate că − n(n + 1) este rest
4
pătratic modulo p pentru orice n ı̂ntreg, astfel ı̂ncât p¤|¤(2n + 1).
b) Dacă p ≡ 3(mod 4) este număr prim şi n este număr ı̂ntreg, să se
p+1
arate atunci că + n(n + 1) este rest pătratic modulo p dacă (2n + 1) nu
4
este multiplu de p.
7. Să se arate că 167¤|¤2n + 3m , ∀ n, m ∈ N.
8. Să se arate că numărul 111 . . . 161 nu este pătrat perfect.
9. Fie a, b ∈ N∗ astfel ı̂ncât 15a + 16b şi 16a − 15b sunt ambele pătrate
nenule. Să se afle cea mai mică valoare a lui 16a − 15b cu proprietatea de mai
sus.
(OIM, 1996)

10. Fie k ∈ Z şi p număr prim impar astfel ı̂ncât p¤|¤k. Să se arate că
p−1    
X x(x + k) a
= −1 (facem convenţia că = 0 dacă p|a).
x=1
p p

66
11. Fie p şi q prime astfel ı̂ncât q = 4p + 1. Să se arate că q este divizor
al numărului 4p + 1.

12. Fie p număr prim de forma 12k − 1. Arătaţi că ∃ x ∈ Z aşa ı̂ncât
.
(x2 − k)..p.

13. Fie p prim, p ≡ 3, 5(mod p). Atunci (p − 2)! + 1 nu e pătrat.

14. Fie p prim, p ≡ 3, 5(mod 8). Să se arate că (p − 3)! + 1 nu este pătrat.

15. Dacă p este prim, p = 3k + 2 şi p divide a2 + ab + b2 , să se arate că


p divide a şi p divide b.

16. Fie p un număr prim astfel ı̂ncât p divide n2 + n + 1 (n fiind număr


ı̂ntreg). Să se arate că p = 3 sau p ≡ 1(mod 3).

17. Fie p număr prim, un divizor al numărului n2 + n − 1. Să se arate că


p = 5 sau p ≡ ±1(mod 5).

18. Fie x şi y numere ı̂ntregi, prime ı̂ntre ele, astfel ı̂ncât p|x2 − 3y 2 , p
fiind număr prim. Să se arate că p = 3 sau 3 este rest pătratic modulo p.

19. Fie x şi y numere ı̂ntregi, prime ı̂ntre ele, astfel ı̂ncât p|x2 + 2y 2 , p
fiind număr prim. Să se arate că p = 2 sau p ≡ 1, 3(mod 8).

20. Dacă p este prim şi pn = a2 + 2b2 , b 6= 0, atunci p = x2 + 2y 2 .


(Baraj 1997)

21. Să se rezolve ecuaţia xp−3 ≡ p − 3(mod p), unde p > 3 este număr
prim.

3x2 + 1
22. Să se arate că nu e număr ı̂ntreg pentru orice număr ı̂ntreg
6k − 1
nenul k.
m2 + m + 1
23. Dacă m şi n sunt numere ı̂ntregi, atunci este ı̂ntreg dacă
n3 − n − 1
şi numai dacă n = 0, 1, −1.

3x2 + 1
24. este număr ı̂ntreg doar pentru y = 0, ±1.
y4 − y2 − 1

3x2 + 1
25. Pentru orice numere x, y ı̂ntregi, numărul nu este ı̂ntreg.
3y 2 + 5

67
26. Să se găsească toate numerele naturale nenule m şi n pentru care
3n +1
m
este număr ı̂ntreg.
6 −1
27. Să se găsească toate numerele naturale nenule pentru care
b2 + b + 1
este număr ı̂ntreg.
2a2 + 2a − 1
28. Fie p şi q numere prime impare astfel ı̂ncât q = 2p + 1. Să se arate
că q|Mp = 2p − 1 dacă şi numai dacă p ≡ 3(mod 4).

29. Dacă p ≥ 3 este număr prim, să se arate că orice divizor natural al
 
p+1
lui este rest pătratic mod p.
4
 
q
30. Fie p prim, p > 5. Să se arate că ∃ q prim, q < p, astfel ı̂ncât = 1.
p
31. Fie p prim de forma 7k − 1. Să se arate că ecuaţia xp1 + xp2 + · · · + xpn =
(x1 + x2 + · · · + xn )2 + 2 nu are soluţii ı̂n numere ı̂ntregi.

32. Să se arate că numărul 3n + 2 nu are factori primi de forma 24k + 13
şi 24k + 23.

33. Fie q număr prim impar şi r ∈ N cu proprietatea că q¤|¤r, r ≡ 3(mod 4)
 
−r
şi = 1. Să se arate că 4qk + r¤|¤q n + 1, ∀ k, n ∈ N.
q
34. Se consideră şirul an+1 = 2an + n · 2n , ∀ n ∈ N, a0 = 1. Să se arate
că pentru orice număr prim p = 15k + 7, an nu este multiplu de p.

35. Fie p prim, p ≡ 1(mod 4). Să se calculeze suma


hÈ i hÈ i –r ™
p−1
s= 1·p + 2 · p + ··· + ·p .
4
(OIM )

36. Să se arate că pentru orice număr prim p şi orice număr ı̂ntreg a,
polinomul f (x) = x4 + a2 este reductibil ı̂n Zp [x].

37. Fie p un număr prim, p ≥ 3. Să se arate că cel mai mic nerest pătratic

modulo p este mai mic decât p + 1.

38. Să se arate că ∀ m ∈ N∗ , ∃ n ∈ Z astfel ı̂ncât


f (n) = (n2 + 3)(n2 − 13)(n2 + 39) ≡ 0(mod m).

68
CAPITOLUL 11

Ordinul unui element (Gaussian)

Definiţie. Fie m ∈ N∗ , a ∈ Z, (a, m) = 1. Din teorema lui Euler ştim că


aϕ(m) ≡ 1 (mod m). Cel mai mic număr natural nenul n pentru care an ≡ 1
(mod m) ı̂l notăm cu g = γm (a). El se numeşte gaussian sau ordin (el este
ordinul lui a b ı̂n grupul (U (Zm ), ·)). Au loc următoarele proprietăţi:
1) Dacă an ≡ 1 (mod m), atunci γm (a)|n;
2) γm (a)|ϕ(m);
3) Dacă al ≡ an (mod m), atunci l ≡ n (mod γm (a)).
Pentru fiecare i = 1, m − 1 notăm cu ri ∈ {1, 2, . . . , m − 1} restul ı̂mpărţirii
lui a · i la m. Punem b1 = r1 , bi+1 = rbi şi gaussianul este cel mai mic indice i
pentru care bi = 1.

Definiţie. În contextul de mai sus, a se numeşte rădăcină primitivă modu-


lo m, dacă (a, m) = 1 şi γm (a) = ϕ(m). Dacă p este număr prim, atunci există
ϕ(p − 1) rădăcini primitive modulo p.
Are loc şi următoarea teoremă: pentru m ∈ N, m > 1, există rădăcini
primitive modulo m dacă şi numai dacă m ∈ {2, 4, pα , 2pα }, unde α ∈ N∗ şi p
este un număr prim impar.

Problema 1. Să se găsească toate numerele prime p, q pentru care 2p + 2q


este un multiplu de p · q.

Rezolvare. Să presupunem că p 6= 2 şi q 6= 2. Scriem p − 1 = 2l · n, q − 1 =


2k m, k, l ∈ N∗ , 2¤|¤mn. Deoarece p · q|2p + 2q , deducem că 0 ≡ 2p + 2q ≡ 2p + 2
(mod q) (am folosit Mica Teoremă a lui Fermat). Deci 2p−1 ≡ −1 (mod q).
l
Notăm x = 2n . Avem că x2 ≡ −1 (mod q) şi de aici rezultă că γq (x) =
l+1 l
2l+1 (deoarece x2 ≡ 1 (mod q) şi x2 6≡ 1 (mod q)). Deducem că 2l+1 =
γq (x)|ϕ(q) = q − 1 = 2k · m şi de aici rezultă că l + 1 ≤ k. Analog se arată
că k + 1 ≤ l şi am obţinut o contradicţie: l ≤ k − 1 ≤ l − 2. Deci neapărat
unul din numere trebuie să fie 2; de exemplu q = 2. Avem p|2p + 2q = 2p + 22 ,
0 ≡ 2p + 22 ≡ 2 + 22 = 6 (mod p) şi deci p ∈ {2, 3}. Soluţiile problemei sunt
deci perechile (p, q) = (2, 2), (2, 3), (3, 2).

69
Problema 2. Să se arate că n¤|¤2n−1 + 1, ∀ n ∈ N, n ≥ 2.

Rezolvare. Să presupunem că există n ∈ N, n ≥ 2 astfel ı̂ncât n|2n−1 + 1


şi fie n = pα1 1 . . . pαr r descompunerea sa standard ı̂n factori primi. Este ev-
ident că un astfel de n ar trebui să fie impar. Fie m ∈ N∗ astfel ı̂ncât
pi ≡ 1 (mod 2m ) ∀ i = 1, r şi există un indice j ∈ {1, 2, . . . , r} astfel ı̂ncât
pj 6≡ 1 (mod 2m+1 ). Deoarece pi ≡ 1 (mod 2m ) ∀ i = 1, r, deducem că n ≡ 1
m
(mod 2m ), n − 1 = 2m · t, t ∈ N∗ şi pj |n|2n−1 + 1 = 22 ·t + 1. Notând x = 2t ,
m
avem că (x)2 ≡ −1 (mod pj ). De aici rezultă imediat că γpj (x) = 2m+1 şi deci
2m+1 = γpj (x)|ϕ(pj ) = pj − 1, pj ≡ 1 (mod 2m+1 ); contradicţie. Deci ¶ ∃ n ∈ N,
n ≥ 2, astfel ı̂ncât n|2 n−1 + 1.

1. Să se arate că 176 ≡ 1(mod 7 · 13 · 27).

2. Să se arate că 11 · 31 · 61 divide numărul 2015 − 1.


.
3. Dacă (a, 30) = 1 rezultă a60 − 1..1800.

4. Să se rezolve ecuaţia 2x8 ≡ 3(mod 13).

5. Pentru ce numere a, ecuaţia x27 ≡ a(mod 37) are soluţii?

6. Dacă a > 1, atunci n|ϕ(an − 1), ∀ n ∈ N∗ .

7. Fie p prim şi n ∈ N∗ astfel ı̂ncât (n − 1, p − 1) = 1 şi xn ≡ a(mod p),


xn+1 ≡ a2 (mod p). Să se arate că x ≡ a ≡ 1(mod p) sau x ≡ a ≡ 0(mod p).

(
8. Fie p şi q prime, a număr ı̂ntreg, a 6≡ 0, 1(mod p) astfel ı̂ncât sistemul
xq−2 ≡ a2 (mod p)
are soluţii. Să se arate că q|p − 1.
xq−1 ≡ a(mod p)
9. Pentru ce numere naturale n:
a) 11|5n + 7n ?
b) 37|5n + 7n ?
n
10. Fie p un divizor prim al lui Fn = 22 + 1, n ≥ 2. Să se arate că
p ≡ 1(mod 2n+2 ).

11. a) Să se arate că 216 + 1 = 65537 este prim.


b) Să se arate că 216 + 1 e cel mai mic divizor prim al numărului
15
122 + 1.

12. Să se descompună ı̂n factori primi numărul 233 − 1.

70
13. Să se arate că n¤|¤2n − 1, ∀ n ∈ N, n ≥ 2.

14. Să se arate că dacă n > 1, atunci 3n − 2n nu se divide cu n.

15. Pentru fiecare număr natural nenul k, să se determine n > 0 minim
pentru care 5n ≡ 1 (mod 2k ).

16. Să se calculeze cel mai mic număr natural n astfel ı̂ncât 22005 |3n − 1.

17. Fie k > 1 şi p număr prim p = 4k + 3. Condiţia necesară şi suficientă
ca numărul q = 2p + 1 să fie prim este ca 2p ≡ 1(mod 2p + 1), adică q divide
Mp .

18. Să se arate că pentru orice număr prim p, există un număr prim q,
astfel ı̂ncât np 6≡ p(mod q), ∀ n ∈ N.
(OIM, 2003)

19. Să se găsească toate perechile (n, p) cu proprietatea că n ∈ N∗ , p


număr prim, n ≤ 2p şi np−1 |(p − 1)n + 1.
(OIM, 1999)

20. Fie a ı̂ntreg şi p > 2 este prim, (a, p) = 1 astfel ı̂ncât an + 1 se divide
cu p. Notăm i = γp (a) şi j > 0 minim, astfel ı̂ncât aj + 1 se divide cu p. Să se
i
arate că i este par şi j = .
2
2n + 1
21. Determinaţi numerele n ∈ N∗ pentru care este număr natural.
n2
(OIM, 1990)

22. Demonstraţi că există o infinitate de numere n ∈ N∗ pentru care


3n + 2n
este număr natural.
n2
23. Să se afle a minim, astfel ı̂ncât a să fie rădăcină primitivă modulo 23.
p−1
24. Fie p un număr prim, p ≡ 3, 5 (mod 8), astfel ı̂ncât este de
2
asemenea număr prim. Să se arate că 2 este rădăcină primitivă modulo p.

25. Fie p prim şi a o rădăcină primitivă modulo p. Atunci a sau a + p


este rădăcină primitivă modulo p2 .

71
CAPITOLUL 12

Numere speciale
n
Numerele Fermat Fn sunt definite prin formula Fn = 22 + 1. Fermat a
presupus că ele sunt prime pentru ∀ n ∈ N. Euler a arătat că F5 nu este prim,
demonstrând că 641|F5 . Nu se cunoaşte nici un număr n ∈ N, n ≥ 5, pentru
care Fn să fie prim. Importanţa numerelor prime este dată de următoarea
teoremă a lui Gauss: polinomul regulat cu n ≥ 3 laturi poate fi construit cu rigla
şi compasul dacă şi numai dacă n = 2k p1 · p2 . . . pr , unde 3 ≤ p1 < p2 · · · < pr
sunt numere prime de tip Fermat.
Mn = 2n − 1, n ∈ N, sunt numerele Mersenne. Se arată uşor că dacă n
este compus, atunci şi Mn este compus. Deci pentru ca Mn să fie prim este
necesar ca n să fie prim. Nu este ı̂nsă şi suficient, după cum arată exemplul
M11 = 211 − 1 = 2047 = 23 · 89.
În 1876 Lucas şi Lehmer (ı̂n 1930) au demonstrat următorul rezultat: dacă
p este un număr prim impar, atunci Mp este prim ⇔ Mp |sp−1 , unde numerele
(sn )n≥1 sunt date prin recurenţa s1 = 4, sn+1 = s2n − 2, pentru n ≥ 1.
Nu se ştie ı̂ncă dacă ∃ o infinitate de numere prime p astfel ı̂ncât Mp să
fie număr prim (nu se cunoaşte nici dacă ∃ o infinitate de numere prime p
astfel ı̂ncât Mp să fie compus). Importanţa numerelor Mp este aceea că cele
mai mari numere prime cunoscute astăzi sunt de această formă. Până astăzi
se cunosc 42 de numere prime Mersenne.
”Fabricantul de microprocesoare Intel foloseşte un program de găsire a nu-
merelor prime ale lui Mersenne pentru a testa fiecare Pentium pus ı̂n comerţ”
(informaţie Keith Devlin).
Mersenne a trăit ı̂n secolul 17, ı̂nsă numerele Mp erau cunoscute din anti-
chitate de către Euclid. Un număr natural n se numeşte perfect, dacă este egal
cu suma divizorilor săi proprii. De exemplu 6 = 1+2+3, 28 = 1+2+4+7+14
sunt numere perfecte. Cu definiţiile din capitolele precedente, avem că n
este perfect ⇔ σ(n) = 2n. În antichitate, Euclid a demonstrat că dacă
n = 2k (2k+1 −1) şi 2k+1 −1 este prim, atunci n este perfect. Euler a demonstrat
că dacă n este perfect şi par, atunci n = 2k (2k+1 − 1), 2k+1 − 1 fiind număr

73
prim (Mersenne). Nu se ştie ı̂ncă dacă există vreun număr perfect impar (nici
nu s-a demonstrat că nu există nici un astfel de număr).
Un număr compus n se numeşte pseudoprim dacă 2n ≡ 2 (mod n). Nu-
merele compuse n pentru care an−1 ≡ 1 (mod n) ∀ a ∈ Z, (a, n) = 1 se numesc
numere Carmichael. 341 = 11 · 31 este pseudoprim, dar nu este Carmichael.
Cel mai mic număr Carmichael este 561 = 3 · 11 · 17. Nu se ştie dacă există
o infinitate de numere Carmichael. Un număr compus n = pα1 1 pα2 2 . . . pαr r este
Carmichael ⇔
i) n este impar;
ii) r ≥ 3;
iii) αi = 1 şi pi − 1|n − 1, ∀ i = 1, r.
n(n + 1)
Numerele triunghiulare tn se definesc prin egalitatea tn = , n ∈ N.
2
În acest capitol vom arăta că orice număr natural este suma a trei numere
triunghiulare (problema 18). Pentru demonstraţie avem nevoie de o teoremă
a lui Gauss: n ∈ N∗ se scrie ca suma a trei pătrate dacă şi numai dacă n 6=
4a (8b + 7).
Leonardo Fibonacci a introdus ı̂n 1228 şirul ce-i poartă numele, definit
prin recurenţa u1 = u2 = 1, un+2 ∗
√ = un+1 + u√ n , ∀ n ∈ N . Se arată că
1 1+ 5 1− 5
un = √ (an − bn ), unde a = şi b = . De asemenea, se arată
5 2 2
că dacă n ≥ 5 este astfel ı̂ncât un este prim, atunci n este prim.
1. Să se arate că un număr Fermat nu este pătrat perfect.
2. Să se arate că un număr Fermat nu este cub perfect.
3. Să se arate că fiecare număr impar este divizor pentru o infinitate de
numere Mersenne.
4. Să se arate că pentru m şi n > 1 nu poate avea loc egalitatea Mn = k m .
5. Să se arate că ı̂ntr-o progresie aritmetică de numere naturale cu raţie
nenulă, există o infinitate de numere care nu sunt libere de pătrate.
6. Să se arate că dacă n este un număr perfect, atunci
X 1 X 1
> ln 2 > .
p|n
p−1 p|n
p+1
k
Y
7. Dacă n = pαi i este un număr perfect impar şi p1 < p2 < · · · < pk ,
i=1
atunci p1 ≤ k.

74
8. Să se arate că numărul n = 2 · 73 · 1103 · 2089 este pseudoprim.

9. Să se arate că 15841 şi 101101 sunt numere Carmichael.

10. Să se arate că dacă p şi q sunt numere prime, q > p, atunci pentru
ca numărul pq să fie pseudoprim este necesar ca Mp Mq să fie pseudoprim.

n
11. Să se arate că dacă n este număr Carmichael şi p|n, atunci p−1 −1.
p
¦
12. Pentru k ≥ 1 se notează Ck = n ≥ k|an−k ≡ 1(mod n) pentru
©
(a, n) = 1 (numere Knödel). Să se arate că pentru k ≥ 2, mulţimea Ck este
infinită.

13. Să se determine n şi m ştiind că tn+m − tn este număr prim.

14. Să se arate că, un număr care ı̂n baza 9 se scrie numai cu cifra 1, este
un număr triunghiular.

15. Să se arate că numărul |22 {z


. . . 2} 11 . . . 1} este triunghiular.
| {z
n n

16. Să se determine numerele lui Fermat care sunt şi triunghiulare.

17. Să se arate că ecuaţia x2 − y 2 = z 3 are o infinitate de soluţii cu z ∈ N


şi x, y numere triunghiulare.

18. Să se arate că orice număr natural se scrie ca o sumă a trei numere
triunghiulare.

19. Există o infinitate de numere triunghiulare ce sunt pătrate.

20. Să se arate că există o infinitate de numere naturale n, pentru care
numărul 2n − 1 nu este triunghiular.

21. Să se arate că există o infinitate de numere naturale n, pentru care
2n − 1 nu se poate scrie ca sumă a două numere triunghiulare.

22. Să se rezolve ı̂n numere naturale ecuaţia tx + ty = Fn .

23. Să se arate că orice număr natural nenul este sumă a unor numere
Fibonacci distincte.

X un 1
24. Să se arate că n+1
= .
n=0
10 89

75
25. Să se arate că numerele un ale lui Fibonacci verifică relaţiile:
n
X
a) u2k = u2n+1 − 1;
k=1
b) u2n + u2n = 2un un+1 ;
c) u2n = u2n+1 − u2n−1 ;
2n−1
X
d) uk · uk+1 = u22n ;
k=1
e) u2n+3 − 2u2n+2 − 2u2n+1 + u2n = 0.

26. Se notează cu lj al j-lea număr din N∗ liber de pătrate. Să se arate


că lim (ln+1 − ln ) = ∞.
n→∞

27. Există o infinitate de triplete m, m + 1, m + 2 de numere libere de


pătrate.

76
CAPITOLUL 13

Teorema lui Dirichlet

Fie a, b ∈ Z, a ≥ 1, (a, b) = 1. Există atunci o infinitate de numere prime


ı̂n progresia aritmetică (b + na)n∈N . Acest celebru rezultat a fost demonstrat
ı̂n 1837 de Dirichlet. Demonstraţia iniţială folosea teoria formelor pătratice.
Cele mai multe demonstraţii folosesc argumente de analiză complexă. Dacă se
notează cu π(a, b, x) numărul numerelor prime din progresia (b + na)n∈N care
nu-l depăşesc pe x, atunci se arată că

π(a, b, x) log x 1
lim = .
x→∞ x ϕ(a)

Pentru anumite cazuri particulare ale teoremei lui Dirichlet, există


demonstraţii ”elementare”. A. Rotkiewicz a dat o astfel de demonstraţie pen-
tru b = 1.
În cele ce urmează vom demonstra că există o infinitate de numere prime
de forma pn + 1, p fiind un număr prim fixat. Să presupunem că există un
număr finit de astfel de numere prime şi le notăm cu p1 , p2 , . . . , pr (ı̂n caz că
r
Y
există vreunul). Notăm a = p · pj (a = p dacă ¶
∃ nici un număr prim de
j=1
forma pn + 1). Considerăm numărul N = ap−1 + ap−2 + · · · + a + 1 > 1 şi fie
q un număr prim care-l divide pe N . Avem că q|N (a − 1) = ap − 1, ap ≡ 1
(mod q). Din această ultimă congruenţă rezultă că γq (a) = 1 sau p, deoarece
p este prim (vezi Capitolul 11). Dacă γq (a) = 1, atunci a ≡ 1 (mod q) şi
0 ≡ N = ap−1 + ap−2 + · · · + a + 1 ≡ p (mod q), q|p, q = p, p|N . Aceasta este
o contradicţie căci p|a şi cum p|N , deducem că p|N − ap−1 − ap−2 · · · − a = 1;
contradicţie. Deci γq (a) = p şi p = γq (a)|ϕ(q) = q − 1. Rezultă că q − 1 = pt,
t ∈ N, q = pt + 1. Însă am presupus că există doar un număr finit de numere
prime de forma pn + 1. Deci ∃ 1 ≤ j ≤ r astfel ı̂ncât q = pj . Acest lucru nu
r
Y
este ı̂nsă posibil deoarece q = pj |a = p pk şi q|N = ap−1 + ap−2 + · · · + a + 1.
k=1
La fel ca mai sus se obţine contradicţia q|1. Deci există o infinitate de numere
prime de forma pn + 1, dacă p este un număr prim fixat.

77
1. Să se arate că există o infinitate de numere prime p, pentru care p − 2
şi p + 2 sunt numere compuse

2. Să se arate că pentru orice n natural, există o infinitate de numere


prime care au cel puţin n cifre de 0.

3. Să se arate că pentru orice n natural, există o infinitate de numere


prime care au suma cifrelor mai mare decât n.

4. Să se arate că există o infinitate de numere prime p care conţin toate
cifrele 0, 1, 2, . . . , 9.

5. Pentru m ∈ N∗ , să se arate că există o infinitate de numere prime p,


astfel ı̂ncât 2m kp + 1.

6. Să se arate că există o infinitate de numere prime care sunt suma a
patru cuburi de numere ı̂ntregi.

7. Să se arate că există o infinitate de numere prime p, pentru care există
numerele ı̂ntregi x, y, z, astfel ca 9p = x3 + y 3 + 9z 3 .

8. Să se arate că există o infinitate de numere prime p, care se reprezintă


sub forma p = 3x3 + 3y 3 + z 3 cu x, y, z numere ı̂ntregi.

9. Pentru orice număr ı̂ntreg k, să se arate că există o infinitate de numere
prime de forma a2 + b2 + c2 + k.

10. Să se determine ı̂ntregii a pentru care, pentru orice n ı̂ntreg, există
ı̂ntregii x şi y astfel ca n = (a + 1)xy + 2ax + 2a2 y.

11. Să se arate că există o infinitate de numere prime p, care nu se scriu
sub forma p = 3x − 2y .

12. Să se arate că orice n natural se scrie ı̂ntr-o infinitate de moduri, sub
forma n = p + a2 − b2 cu p prim şi a, b naturale.

13. Fie a şi b două numere naturale nenule şi d = (a, b). Să se arate că
a+p b+p
există o infinitate de numere prime p, pentru care + este ı̂ntreg,
b a
2
dacă şi numai dacă d divide a + b.

14. Să se arate că există o infinitate de numere prime p, astfel ı̂ncât
pentru orice numere naturale m şi n, numărul 2m + n2 nu se divide cu p.

78
15. Să se arate că există o infinitate de numere prime de forma 8k + 5.

16. Fie a şi b numere naturale cu (a, b) = 1 şi s ≥ 1 număr natural. Să
se arate că ı̂n progresia xn = an + b există o infinitate de termeni care sunt
produs de s numere prime distincte.

17. Fie m număr natural nenul. Să se arate că există o infinitate de
numere prime p, pentru care p + 1 şi p − 1 au mai mult de m factori distinţi.

18. Să se arate că pentru orice număr natural m ≥ 1, există o infinitate
de numere naturale n, astfel ı̂ncât m divide pe σ(n), ϕ(n), τ (n).

19. Să se arate că există p prim, astfel ı̂ncât p − 1, p + 1 şi p + 2 au cel
puţin n divizori primi.

20. Fie m un număr natural liber de pătrate, m = 8k + 3. Să se arate


că există o infinitate de numere prime p, astfel ı̂ncât −m este rest pătratic
modulo p.

21. Pentru orice număr natural k ≥ 2 există o infinitate de numere


compuse n, pentru care n divide an−k − 1 pentru orice număr ı̂ntreg a prim
cu n.
(Makowski )

22. Fie r un număr raţional pozitiv care nu este ı̂ntreg. Să se arate că
există o infinitate de numere naturale n, astfel ı̂ncât [nr] este prim.
(Borosh-Hensley)

79
CAPITOLUL 14

Reprezentări aditive

Dacă A şi B sunt submulţimi ale lui C şi h ∈ N∗ , problemele de


h
X
reprezentare aditivă semnifică faptul că ∀ b ∈ B se scrie sub forma b = ai ,
i=1
unde ai ∈ A, ∀ i = 1, h. Există câteva probleme clasice de reprezentare aditivă.
Teorema lui Lagrange afirmă că orice număr natural este suma a patru pătrate
de numere naturale. Un număr n ∈ N∗ se scrie n = a2 + b2 , a, b ∈ N ⇔ orice
divizor prim de forma 4k + 3 al lui n apare la exponent par ı̂n descompunerea
lui n.
Teorema lui Gauss afirmă că n = a2 +b2 +c2 (a, b, c ∈ Z) ⇔ n 6= 4k (8m+7).
În 1770, Waring a conjecturat că ∀ k ∈ N, k ≥ 2, ∃ s ∈ N∗ , un număr care
s
X
depinde doar de k, astfel ı̂ncât ∀ n ∈ N se scrie sub forma n = nki , ni ∈ N,
i=1
∀ i = 1, s. Rezultatul a fost demonstrat ı̂n 1909 de Hilbert.
O altă problemă faimoasă de reprezentare aditivă este conjectura lui Gold-
bach: orice număr par n ≥ 4 se poate scrie sub forma n = p + q, p şi q fiind
numere prime. Cel mai bun rezultat demonstrat până astăzi este că ∃ n0 ∈ N
astfel ı̂ncât ∀ n ∈ N, n ≥ n0 , se poate scrie sub forma n = p + m, unde p este
prim şi m este prim sau produs de două numere prime (teorema lui Chen). De
asemenea, Vinogradov a demonstrat că ∃ n0 ∈ N astfel ı̂ncât ∀ n ∈ N, n ≥ n0 ,
n impar, atunci n = p + q + r, p, q şi r fiind numere prime.

Problema 1. Să se arate că pentru orice număr natural n ≥ 6 există


numerele naturale nenule x1 , x2 , . . . , xn ∈ N∗ astfel ı̂ncât

1 1 1
2 + 2 + · · · + 2 = 1.
x1 x2 xn

Rezolvare. Arătăm că dacă enunţul este adevărat pentru n, atunci el


este adevărat şi pentru n + 3. Fiind adevărat pentru n, ∃ xi ∈ N∗ , i = 1, n
1 1 1
astfel ı̂ncât 2 + 2 + · · · + 2 = 1. Atunci numerele yj = 2xj , j = 1, n,
x1 x2 xn

81
yn+1 = yn+2 = yn+3 = 2 au proprietatea că
n+3
X 1 1X n 1 3 1 3
2 = 2 + = + = 1.
j=1
yj 4 j=1 xj 4 4 4

Pentru a termina demonstraţia mai trebuie arătat enunţul pentru n = 6, 7, 8.


1 1 1 1 1 1 1 1 1 1 1 1 1
Avem + + + + + = 1, + + + + + + = 1 şi
4 4 4 9 9 36 4 4 4 16 16 16 16
1 1 1 1 1 1 1 1
+ + + + + + + = 1 şi raţionamentul prin inducţie este
4 4 4 9 9 81 81 324
X5 1
∃ x1 , x2 , x3 , x4 , x5 ∈ N∗ astfel ı̂ncât
ı̂ncheiat. Este uşor de arătat că ¶ = 1.
x2
j=1 j
Problema 2. Să se găsească cel mai mic n ∈ N∗ pentru care există
n
X
x1 , x2 , . . . , xn ∈ N astfel ı̂ncât x4j = 1998.
j=1
(Baraj juniori 1998, Gh. Iurea)

Rezolvare. Se arată uşor că x4 ≡ 0 (mod 16) sau x4 ≡ 1 (mod 16). De aici
rezultă imediat că n ≥ 14 deoarece 1998 = 16 · 124 + 14 ≡ 14 (mod 16). Dacă
n = 14, atunci numerele xj , j = 1, 14, trebuie să fie toate impare. Ţinând cont
x4k − 1
că 74 = 2401, rezultă că xj ∈ {1, 3, 5}, ∀ j = 1, 14. Fie yk = ∈ {0, 5, 39}
16
14
X
şi avem că yk = 124 = a·5+b·39, a+b ≤ 14. Avem 39b ≡ 124 ≡ 4 (mod 5),
k=1
124 − 39 85
4b ≡ 4 (mod 5), b ≡ 1 (mod 5). Dacă b = 1, atunci a = = = 17 şi
5 5
obţinem contradicţia a + b = 18 > 14. Dacă b ≥ 6, atunci 124 = 5a + 39b ≥
39 · 6 = 234; contradicţie. Deci n ≥ 15.
15
X
Pe de altă parte, 1998 = 2 · 54 + 9 · 34 + 24 + 3 · 14 = x4i .
i=1

Observaţie. În 1770, Waring a conjecturat că orice număr natural n se scrie
19
X
sub forma n = x4i , xi ∈ N, ∀ i = 1, 19. Acest lucru a fost demonstrat ı̂n
i=1
1986 de către R. Balasubramanian, J. H. Deshouillers, F. Dress.

1. Pentru n > 6, există numerele coprime a, b mai mari strict decât 1


astfel ı̂ncât n = a + b.

2. Pentru n > 17, există a, b, c mai mari strict decât 1 şi prime două câte
două, astfel ı̂ncât n = a + b + c.

82
3. Să se arate că orice număr ı̂ntreg k se poate reprezenta sub forma
k = x2 + y 2 − z 2 .

4. Să se arate că pentru orice număr ı̂ntreg a, există o infinitate de numere
ı̂ntregi n, care nu se pot scrie sub forma n = a(x2 − y 2 ) + 2x cu x şi y ı̂ntregi.

5. Să se arate că orice număr natural n se scrie sub forma n = x2 + 2y 2 +


3z 2 + 6t2 (x, y, z, t numere ı̂ntregi).
(Jacobi )

6. Să se rezolve ecuaţia 7 · 4n = a2 + b2 + c2 + d2 .

7. Dacă p este număr prim de forma 6k + 1, atunci 2p2 este suma a trei
bipătrate nenule.
(Makowski )

8. Să se arate că orice ı̂ntreg k se reprezintă ı̂ntr-o infinitate de moduri


sub forma k = x3 + y 3 + s2 − t2 cu x, y, s, t numere ı̂ntregi.

9. Să se arate că numărul 2005 se scrie ı̂ntr-o infinitate de moduri sub
forma x3 + y 3 + s2 + t2 .

10. Există o infinitate de numere ı̂ntregi care nu se scriu sub forma


x4 + y4 − z4.

11. Să se arate că pentru orice k ı̂ntreg, există n natural şi o alegere a
semnelor ”+” sau ”−”, astfel ı̂ncât k = ±12 ± 22 ± · · · ± n2 .
(Erdös-Surany)

12. Să se arate că numărul natural nenul n se scrie sub forma
n = ±12 ± 22 ± · · · ± n2 , dacă şi numai dacă n = 4k sau n = 4k + 1.

13. Pentru ce numere naturale nenule există o alegere a semnelor ±, astfel


ı̂ncât ±12 ± 22 ± · · · ± n2 = 0?

14. Să se arate că pentru orice număr natural nenul n, există numerele
1
naturale nenule x1 , x2 , . . . , xn , cel mult două fiind egale, astfel ı̂ncât 1 = +
x1
1 1
+ ··· + .
x2 xn
1 1 1
15. Să se arate că pentru orice n ≥ 3, ecuaţia + +···+ = 1 are
x1 x2 xn
soluţii ı̂n numere naturale distincte.

83
16. Să se arate că orice număr raţional r ∈ (0, 1) poate fi scris ca suma
inverselor unor numere naturale distincte.

17. Să se arate că ecuaţia X 2 + Y 2 + Z 2 + X + Y + Z = 1 nu are soluţii


ı̂n numere raţionale.

18. Fie n un număr natural care se scrie ca sumă a trei pătrate raţionale.
Să se arate că n se scrie ca sumă a trei pătrate de numere naturale.

a
19. Fie a, b numere naturale nenule. este sumă a trei pătrate raţionale,
b
dacă şi numai dacă ab e suma a trei pătrate naturale.

20. În orice progresie aritmetică an + b, unde a, b ∈ N, a impar, există o


infinitate de numere care nu se scriu ca sumă de trei pătrate.

21. Există o infinitate de numere naturale n cu proprietatea că n şi n + 1


nu se scriu ca sumă de trei pătrate.

22. Nu există nici un număr natural n cu proprietatea că n, n + 1 şi n + 2


nu se scriu ca sumă de trei pătrate.

23. Să se arate că n se scrie ca sumă de trei pătrate, dacă şi numai dacă
4n = a2 + b2 + c2 + d2 cu a + b = c + d.

24. Fie n un număr natural impar. Să se arate că n se poate scrie sub
forma n = a2 + b2 + 2c2 , a, b, c fiind numere ı̂ntregi.

25. Fie n un număr natural impar. Să se arate că n se poate scrie
n = a2 + b2 + c2 + (c + 1)2 , a, b, c fiind numere ı̂ntregi.
(Lionet)

26. Să se determine toate numerele naturale de forma n = aa . . . a(10)


care nu se scriu ca sumă de trei pătrate.

27. Să se determine toate numerele naturale n ≥ 2, cu proprietatea


că oricare ar fi numărul a, astfel ı̂ncât a se scrie ca sumă de trei pătrate şi
n
1 ≤ a ≤ , atunci şi a + n se scrie ca sumă de trei pătrate.
2
28. Fie A un inel unitar cu 1 6= 0 şi cu proprietatea că oricare ar fi x, y, z
din A, astfel ı̂ncât x2 + y 2 + z 2 = 0, atunci neapărat x = y = z = 0. Să se
arate că A are o infinitate de elemente.

84
29. Să se arate că ∃ n0 ∈ N, astfel ı̂ncât ∀ n ∈ N, n ≥ n0 , se poate scrie
ca sumă de pătrate distincte.
(Lista scurtă, OIM, 2000)

30. Fie a, b ∈ N∗ , (a, b) = 1.


a) Să se arate că ab − a − b = max{x ∈ N | x 6= ma + nb, ∀ m, n ∈ N}.
(a−1)(b−1)
b) Să se arate că |{x ∈ N | x 6= ma + nb, ∀ m, n ∈ N}| = .
2
31. Fie a, b, c ∈ N∗ ; (a, b) = (a, c) = (b, c) = 1. Să se arate că
2abc − ab − ac − bc = max {x ∈ N | x 6= kab + lac + tbc, ∀ k, l, t ∈ N} .
(OIM, 1983)

32. Orice număr natural nenul se scrie ca suma a cel mult zece pătrate
impare. Să se arate că numerele de forma 72k + 42 necesită exact zece pătrate
impare.
(Pollak-Turski )

33. Să se arate că ∀ k ∈ N∗ , ∃ k numere naturale consecutive, care nu se


scriu ca sumă de două pătrate.

34. Să se arate că există o infinitate de numere naturale n, care nu se pot
scrie ca suma dintre două puteri distincte ale lui 2 şi un număr prim.
(A. Schinzel )

35. Pentru fiecare n ∈ N se notează cu r(n) numărul de soluţii naturale


ale ecuaţiei n = x + 2y + 3z. Să se arate că r(n) este cel mai apropiat ı̂ntreg
(n + 3)2
de .
12
(Hardy & Wright)

85
CAPITOLUL 15

Ecuaţii algebrice ı̂n numere ı̂ntregi

Domeniul acesta coincide cu ı̂nsăşi istoria matematicii. El mai este cunos-


cut sub numele ”ecuaţii diofantice”. Numele vine de la celebrul aritmetician
al antichităţii Diophantus (sec. 3 d.H.). Domeniul este foarte vast şi ne vom
limita la câteva ecuaţii diofantice clasice abordate ı̂n acest capitol.
În 1637, Fermat a formulat următoarea problemă: dacă n ∈ N, n ≥ 3 şi
x, y, z ∈ Z sunt astfel ı̂ncât xn + y n = z n , atunci x · y · z = 0. Enunţul a
fost demonstrat de A. Wiles ı̂n 1994–1995. În acest capitol vom folosi enunţul
pentru n = 3 şi n = 4 (cazul n = 4 a fost demonstrat de Fermat iar cazul
n = 3 de către Euler). Ne vom ocupa ı̂n acest capitol şi de ecuaţia lui Pell.
Dacă d ∈ N este un număr care nu e pătrat, atunci există x1 , y1 ∈ N, astfel
ı̂ncât x21 − dy12 = 1, (x1 , y1 ) 6= (1, 0) şi ∀ x, y ∈ N cu x2 − dy 2 = 1, atunci
√ € √ Šn
∃ n ∈ N astfel ı̂ncât x + y d = x1 + y1 d . (x1 , y1 ) se numeşte soluţia
minimală a ecuaţiei Pell. Există o procedură de calcul a soluţiei minimale care
√ √ √
foloseşte fracţia continuă a numărului iraţional d. Dacă p + q d = r + s d,

p, q, r, s ∈ Q, atunci p = r şi q = s deoarece d este iraţional. Deci

x = xn1 + Cn2 xn−2


1 y12 d + Cn4 xn−4
1 y14 d2 + . . . şi
y = Cn1 xn−1
1 y1 + Cn3 xn−3
1 y13 d + . . . .

Ne vom ocupa ı̂n acest capitol şi de diverse cazuri particulare ale ecuaţiei
lui Mordell: x2 + k = y 3 (k este un număr ı̂ntreg nenul fixat). Deşi se ştie că o
astfel de ecuaţie are doar un număr finit de soluţii ı̂ntregi, nu se cunoaşte nici
o procedură pentru a determina aceste soluţii.
Vom folosi ı̂n acest capitol de mai multe ori observaţia următoare: dacă
p|a + b2 , p prim, p ≡ 3 (mod 4), atunci p|a şi p|b.
2

Problema 1. Să se arate că pentru orice număr natural n ≥ 3, există a


şi b numere ı̂ntregi impare astfel ı̂ncât a2 + 7b2 = 2n .
(Euler )

87
Rezolvare. Vom demonstra enunţul prin inducţie după n. Pentru n = 3
alegem a = b = 1. Presupunem că am găsit numerele impare a şi b astfel ı̂ncât
a2 + 7b2 = 2n . Înlocuind eventual pe a cu −a şi pe b cu −b, putem presupune
a − 7b a+b
că a ≡ b ≡ 1 (mod 4). Considerăm c = şi d = . Cele două numere
2 2
sunt ı̂ntregi şi deoarece a ≡ b ≡ 1 (mod 4), avem a − 7b ≡ a + b ≡ 2 (mod 4) şi
deci c şi d sunt numere ı̂ntregi impare. Calculăm

(a − 7b)2 + 7(a + b)2 8a2 + 56b2


c2 + 7d2 = = = 2(a2 + 7b2 ) = 2n+1 .
4 4

Pasul de inducţie a fost demonstrat şi cu aceasta şi enunţul.

0
Problema 2. Să se arate că Sn = C2n+1 22n +C2n+1
2 22n−2 3+· · ·+C2n+1
2n 3n

este suma a două pătrate consecutive, ∀ n ∈ N.


(Baraj 1999, Dorin Andrica)

€ √ Š2n+1 √
Rezolvare. Avem 2 + 3 = xn + yn 3, xn = 2Sn . Cum (2, 1) este
soluţia minimală a ecuaţiei Pell x2 − 3y 2 = 1, deducem că x2n − 3yn2 = 1.
Avem x2n − 1 = 3yn2 = (xn − 1)(xn + 1). Dar (xn − 1, xn + 1) = 1 (deoarece
xn este par) şi deducem că xn − 1 = 3α2 , xn + 1 = β 2 sau xn − 1 = α2 ,
xn + 1 = 3β 2 , cu α, β ∈ Z. În primul caz rezultă că 3α2 + 2 = β 2 ≡ 2 (mod 3);
contradicţie. Deci xn − 1 = α2 , xn + 1 = 3β 2 . Cum xn este par, rezultă
că α este impar 2Sn − 1 = (2t + 1)2 = 4t2 + 4t + 1, 2Sn = 4t2 + 4t + 2,
Sn = 2t2 + 2t + 1 = t2 + (t + 1)2 . Enunţul este demonstrat ı̂n acest moment.

Problema 3. Să se rezolve ecuaţia p2 − p + 1 = k 3 , unde k este un număr


ı̂ntreg şi p este număr prim.
(Balcaniadă, 2005)

Rezolvare. Avem că p(p − 1) = k 3 − 1 = (k − 1)(k 2 + k + 1). Este evident


că p 6= 3. Dacă p ≡ −1 (mod 3), atunci k 3 = p2 − p + 1 ≡ 0 (mod 3), 3|k,
4k 3 = 4p2 − 4p + 4 = (2p − 1)2 + 3, 9|4k 3 − (2p − 1)2 = 3; contradicţie
(2p − 1 ≡ −2 − 1 ≡ 0 (mod 3)). Deci p ≡ 1 (mod 3) şi k ≡ k 3 = p2 − p + 1 ≡ 1
(mod 3), k 2 + k + 1 ≡ 0 (mod 3). Rezultă că 9|(k − 1)(k 2 + k + 1) = p(p − 1),
p ≡ 1 (mod 9) şi egalitatea

p−1 k − 1 k2 + k + 1
p· = · . (*)
9 3 3

88
!
k − 1 k2 + k + 1 k2 + k + 1 k−1 k−1
Avem că , = 1 deoarece = k· +2· +
3 3 3 3 3
k−1 k2 + k + 1
1. Din egalitatea (∗) deducem că p sau p (deoarece p este
3 3
k−1
k−1 k−1 p−1
prim). Dacă p , atunci p ≤ şi obţinem contradicţia = 3 ·
3 3 9 p
k2 + k + 1 k2 + k + 1 k+1
≥ ≥ , p − 1 ≥ 3(k + 1) = 3k + 3, p ≥ 3k + 4,
3 3 3
k−1 k−1 k2 + k + 1
≥ p ≥ 3k + 4, k − 1 ≥ 9k + 12, 8k + 13 ≤ 0. Deci p¤|¤ şi p ,
3 3 3
k2 + k + 1 p−1 k−1 k−1
= p · t, t ∈ N∗ . Avem = t· , p = 9t · +1 ≡
3 9 3 3
 
k−1 k2 + k + 1 k−1 k−1
1 mod . Din pt = =k· +2· + 1, deducem că
3 3 3 3
     
k−1 k−1 k−1
p · t ≡ 1 mod . Cum p ≡ 1 mod , rezultă că t ≡ 1 mod .
3 3 3
k−1 k−1
Să presupunem că t > 1. Atunci t − 1, t − 1 > 0, t − 1 ≥ ,
3 3
k−1 k+2 k2 + k + 1 k2 + k + 1 k2 + k + 1
t≥ +1 = . Rezultă că p = ≤ = .
3 3 3t k+2 k+2

3
p−1 k−1 k−1
Dar = t≥ , p − 1 ≥ 3(k − 1), p ≥ 3k − 2 şi deci 3k − 2 ≤ p ≤
9 3 3
k2 + k + 1
. Din ultimele inegalităţi deducem că (k + 2)(3k − 2) ≤ k 2 + k + 1,
k+2
3k 2 + 4k − 4 ≤ k 2 + k + 1, 2k 2 + 3k − 5 ≤ 0, (k − 1)(2k + 5) ≤ 0. Ultima
inegalitate este imposibilă căci k 3 = p2 − p + 1 ≥ 52 − 5 + 1 = 21, k ≥ 3.
k2 + k + 1 p−1 k−1
Deci t = 1, = p, = . Rezultă (din ultima egalitate) că
3 9 3
k2 + k + 1
p = 3k − 2 şi = p = 3k − 2, k 2 + k + 1 = 9k − 6, k 2 − 8k + 7 = 0,
3
(k − 1)(k − 7) = 0, k = 1 sau k = 7. Am văzut mai sus că k ≥ 3 şi deci k = 7,
p = 19. Aceasta este singura soluţie a ecuaţiei din enunţ.

Observaţie. Ljunggren a demonstrat ı̂n 1942 că singurele soluţii ı̂ntregi ale
ecuaţiei x2 − x + 1 = y 3 sunt (x, y) = (0, 1), (1, 0), (19, 7), (−18, 7).

1. Ecuaţia x2 − y 2 = a3 are soluţii ∀ a ∈ Z.

89
2. Să se rezolve ı̂n numere ı̂ntregi ecuaţia x2 + 3x + 5 = 121y 2 .

3. Să se arate că ecuaţia x3 + y 3 = 5z 3 nu are soluţii ı̂n numere ı̂ntregi,


nedivizibile cu 3.

4. Să se rezolve ı̂n Z ecuaţia x(x + 1)(x2 + x + 2) = 2y 2 .

5. Să se rezolve ı̂n numere naturale nenule ecuaţia x2 + y 2 = axy.

6. Fie p număr prim. Ecuaţia p(x3 + y 3 ) = x4 − y 4 are soluţii naturale


nenule, dacă şi numai dacă 2p − 1 = k 2 . În acest caz soluţia este unică.

7. Fie k, a, b, c ∈ Z, a 6= 0, 2a¤|¤b. Să se arate că există un număr finit de


numere n ∈ Z, astfel ı̂ncât a2 n2 + bn + c = k 2 .

n(n + 1)
8. Să se găsească numerele a, b ∈ Z cu proprietatea că a + bn
2
este pătrat, ∀ n ∈ N.

9. Să se rezolve ı̂n numere ı̂ntregi ecuaţia x3 + 3 = 4y(y + 1).

10. Să se rezolve ı̂n numere ı̂ntregi ecuaţiile 5x2 + 16xy + 13y 2 = 23 şi
13x2 + 34xy + 22y 2 = 23.

11. Să se rezolve ı̂n numere ı̂ntregi ecuaţia 19x3 − 84y 2 = 1984.

12. Să se rezolve ı̂n numere ı̂ntregi ecuaţia x2 + 4 = y 5 .

13. Să se rezolve ı̂n numere ı̂ntregi ecuaţia x2 + 5 = y 9 .

14. Să se rezolve ı̂n numere ı̂ntregi ecuaţia 7x3 = 12y 2 − 1.


(
x+y+z =3
15. Să se rezolve ı̂n numere ı̂ntregi sistemul
x3 + y 3 + z 3 = 3.

16. Să se rezolve ı̂n numere ı̂ntregi ecuaţia x2 +3xy +7y 2 +4x−9y −6 = 0.

17. Să se rezolve ı̂n N ecuaţia (x + y)2 + 3x + y + 1 = z 2 .

18. Să se rezolve ı̂n Z ecuaţia m3 = x3 + 9x2 + 8x + 9.

19. Ecuaţia x2 + y 2 = 2z 2 are o infinitate de soluţii ı̂n numere naturale


şi cu z prim.

20. Să se arate că ecuaţia x6 + y 6 + z 4 = t4 are o infinitate de soluţii ı̂n


numere naturale, pentru care (x, y, z, t) = 1.

90
21. Să se rezolve ı̂n numere ı̂ntregi ecuaţia x2 + 3 = y 3 .

22. Să se rezolve ı̂n numere ı̂ntregi ecuaţia x2 + 12 = y 3 .

23. Fie n ∈ N∗ , astfel ı̂ncât ∀ p prim, p|n este de forma p = 4k + 3


u2 + v 2
(k ∈ N). Să se rezolve ı̂n N ecuaţia = 2n.
u+v
24. Există o infinitate de numere naturale n pentru care 2n + 1 şi 3n + 1
sunt pătrate.

25. Să se determine triunghiurile cu laturi numere naturale consecutive


şi cu aria număr natural.

26. Să se rezolve ı̂n numere ı̂ntregi ecuaţia


2„ Ž2 32
y−x
(x2 − 1)(y 2 − 1) = 4 − 15 .
2

27. Să se arate că ecuaţia x3 + y 3 + 1 = z 2 are o infinitate de soluţii ı̂n


numere ı̂ntregi.

28. Să se arate că ecuaţia x2 + y 2 + z 2 + 2xyz = 1 are o infinitate de


soluţii ı̂n numere ı̂ntregi.

29. Să se rezolve ı̂n numere ı̂ntregi ecuaţia 3x2 + 1 = 4y 3 .

30. 3x2 + 16 = y 3 .

31. x(x2 + 4) = y 4 .

32. Să se rezolve ı̂n numere ı̂ntregi ecuaţia x(x2 + 1) = 2y 4 .

33. Fie x, y, z, n numere naturale nenule, n ≥ 2. Să se arate că:


a) Dacă min{x, y} ≤ n, atunci xn + y n 6= z n .
b) Dacă x + y este prim şi n impar, atunci xn + y n 6= z n .
c) Dacă z este prim şi n impar, atunci xn + y n 6= z n .

34. Fie n număr natural. Să se arate că ecuaţia 2n = (x + y)2 + 3x + y


are soluţie unică ı̂n numere naturale.

35. Fie k un număr natural nenul. Ecuaţia


(x1 + x2 + · · · + xn )2
x21 + x22 + · · · + x2n − =k
n
are soluţii pentru orice număr natural n ≥ k, dacă şi numai dacă k este par.

91
36. Fie n un număr natural nenul. Ecuaţia
(x1 + x2 + · · · + xn )2
x21 + x22 + · · · + x2n − =1
n
are soluţii ı̂n numere ı̂ntregi, dacă şi numai dacă n = 4.
(Baraj, 1977)

37. Fie x, y, z numere naturale nenule pentru care x2 + y 2 = z 2 + 1. Dacă


y | z, să se arate că z | xy 2 .

38. Fie numerele naturale x, y, z astfel ı̂ncât x2 + y 2 = z 2 + 1. Dacă y


este par şi y divide pe z, atunci z divide pe y 2 .

39. Dacă x, y sunt numere naturale astfel ca x2 − (n2 − 1)y 2 = 1, n ≥ 2,


.
să se arate că xy ..n.

40. Dacă x, y sunt numere naturale, n ≥ 3, şi x2 − (n2 − 4)y 2 = 1, atunci


 
. n
y .. .
2
41. Să se arate că oricare ar fi n ≥ 2, ecuaţia x21 + x22 + · · · + x2n =
nx1 x2 . . . xn are o infinitate de soluţii ı̂n numere ı̂ntregi.

42. Fie p un număr prim, p ≡ 7 (mod 8). Să se arate că ecuaţia x2 −py 2 = 2
are soluţii ı̂ntregi.

43. Dacă x, y, z ∈ N satisfac egalitatea x2 + y 2 + 1 = xyz, atunci z = 3.

44. Să se arate că pentru n ≥ 3, ecuaţia xn + y n = z n nu are soluţii ı̂n


numere naturale nenule ı̂n progresie aritmetică. Să se studieze cazul n = 2.

45. Să se rezolve ı̂n Q ecuaţia x2 + 432 = y 3 .

46. Ecuaţia 4xy − x − y = z 2 nu are soluţii ı̂n numere naturale nenule,


dar are o infinitate de soluţii ı̂n numere ı̂ntregi negative.
(L. Euler )

47. Ecuaţia x5 + 7 = y 2 nu are soluţii ı̂ntregi.

48. Să se rezolve ı̂n numere ı̂ntregi ecuaţia x2 − y 3 = 7.


(V.A. Lebesgue)

49. Ecuaţia x2 + 36 = y 5 nu are soluţii ı̂ntregi.


(A. Macowski )

92
50. Fie p număr prim, p = 4k+3. Să se arate că ecuaţia x2p +y 2p +z 2p = w2p
nu are soluţii ı̂n numere ı̂ntregi nedivizibile cu p.
(B. Powel )

51. Să se rezolve ı̂n numere ı̂ntregi ecuaţia x2 + 16 = y 3 .

52. Fie M1 mulţimea celor n − 1 plane paralele cu baza unui con, astfel
ı̂ncât ele ı̂mpart conul ı̂n n corpuri cu volume egale, şi fie M2 mulţimea celor
n − 1 plane paralele cu baza conului care ı̂mpart suprafaţa laterală a conului
ı̂n n părţi egale. Să se studieze care sunt condiţiile necesare şi suficiente pentru
ca cele două mulţimi să aibă puncte comune.
(Baraj, 1974)

53. Fie un triunghi echilateral ABC. Notăm cu M1 mulţimea celor n − 1


drepte paralele cu BC care ı̂mpart triunghiul ı̂n n figuri de arii egale, şi cu M2
mulţimea celor n − 1 drepte paralele cu BC care ı̂mpart triunghiul ı̂n n figuri
de perimetre egale. Să se arate că M1 şi M2 sunt mulţimi disjuncte.
(Baraj )

54. Să se găsească toate numerele m, n din N∗ pentru care m + n2 + d3 =


mnd, unde d = (m, n).
(Lista scurtă, OIM, 1995)

a2 + b2
55. Fie a, b ∈ N astfel ı̂ncât k = ∈ N. Să se arate că k este pătrat
1 + ab
perfect.
(OIM, 1988)

93
CAPITOLUL 16

Ecuaţii diofantice exponenţiale

Vom menţiona aici doar o celebră ecuaţie a lui Catalan. Acesta a conjec-
turat, cu peste 150 de ani ı̂n urmă, că ecuaţia xz − y t = 1, unde x, y, z, t sunt
numere ı̂ntregi mai mari sau egale cu 2, are doar soluţia 32 − 23 = 1, x = 3,
z = 2, y = 2, t = 3. Rezultatul a fost demonstrat recent de Preda Mihăilescu,
aceasta fiind probabil cea mai importantă realizare românească ı̂n matematica
ultimilor ani.

Problema 1. Să se rezolve ı̂n numere naturale ecuaţia x2 + 615 = 2n .


(Olimpiadă Vilnius)

Rezolvare. Să presupunem că n este impar. Atunci x2 ≡ x2 + 615 = 2n ≡


(−1)n ≡ −1 (mod 3); contradicţie. Deci n = 2m, m ∈ N, 615 = 22m − x2 =
(2m − x)(2m + x). Rezultă că 2m − x = 1, 2m + x = 615 sau 2m − x = 3,
2m + x = 205 sau 2m − x = 15, 2m + x = 41 sau 2m − x = 5, 2m + x = 123.
În primul caz avem 2m+1 = 616, 2m = 308; contradicţie. În al doilea caz avem
2m+1 = 208; imposibil. În al treilea caz avem 2m+1 = 56, contradicţie. În
ultimul caz avem 2m+1 = 128, m = 6, x = 59, n = 12. Aceasta este singura
soluţie a ecuaţiei din enunţ.

Problema 2. Să se rezolve ı̂n numere naturale ecuaţia 2x = 3y + 5.


(Propusă OIM )

Rezolvare. Avem 23 = 31 + 5, 25 = 33 + 5. Se arată uşor că x 6= 0, 1, 2, 4.


Presupunem ı̂n continuare că x ≥ 6. Atunci 2x ≡ 0 (mod 64) şi 3y ≡ 59
(mod 64). Deoarece 38 ≡ 33 (mod 64) şi 316 ≡ 1 (mod 64), deducem că
γ64 (3) = 16. Deci congruenţa 3y ≡ 59 (mod 64) are o unică soluţie y mo-
dulo 16. Avem că 311 ≡ 38 · 33 ≡ 33 · 33 ≡ 11 · 81 ≡ 11 · 17 = 187 ≡ 59 (mod 64)
şi deci y = 16t + 11, t ∈ N. Avem egalitatea

2x = 3y + 5 = 316t+11 + 5.

Calculăm restul ı̂mpărţirii lui 311 la 7. Avem 33 = 27 ≡ −7 (mod 17), 36 ≡


49 ≡ −2 (mod 17), 39 ≡ 33 ·36 ≡ (−7)(−2) = 14 ≡ −3 (mod 17), 311 = 39 ·32 ≡

95
−33 = −27 ≡ 7 (mod 17).
Din Mica Teoremă a lui Fermat ştim că 316 ≡ 1 (mod 17) şi deci 316t+11 ≡
311 ≡ 7 (mod 17). Rezultă că

2x = 316t+11 + 5 ≡ 7 + 5 ≡ 12 (mod 17)


! ! !x  
12 2x 2 2
şi deci = = = 1x = 1 (avem că = 1 deoarece 17 ≡ 1
17 17 17 17
          
12 4 3 3 17 17−1 3−1 17
(mod 8)). Dar = = = (−1) 2 · 2 = =
17 17 17 17 3 3
 
2 32 −1
= (−1) 8 = −1; contradicţie. Deci nu există soluţii cu x ∈ N, x ≥ 6.
3
Am arătat că singurele soluţii ale ecuaţiei din enunţ sunt (x, y) = (3, 1), (5, 3).

1. Să se rezolve ı̂n numere naturale ecuaţia 3x = 2y + 7.

2. Să se rezolve ı̂n numere naturale ecuaţia 2x − 3y = 1.

3. Să se rezolve ı̂n N ecuaţia 3x − 1 = 2y+1 .

4. Să se arate că dacă p este număr prim astfel ı̂ncât 2p + 3p = mk , m şi
k fiind numere naturale nenule, atunci neapărat k = 1.

5. Să se rezolve ı̂n N ecuaţia 3x + 4x = y 2 .

6. Să se rezolve ı̂n N ecuaţia 3x + 4y = 5z .

7. Să se rezolve ı̂n numere naturale ecuaţia 2x + 3y = 5z .

8. Să se rezolve ı̂n numere naturale ecuaţia 5x + 12y = 13z .

9. Să se rezolve ı̂n numere naturale ecuaţia 3x = 2y + 5.

10. Să se rezolve ı̂n numere naturale ecuaţia 2x − 3y = 7.

11. Să se rezolve ecuaţia |pr − q s | = 1, p şi q fiind numere prime şi r, s
numere naturale mai mari sau egale cu 2.

12. Să se rezolve ecuaţia 5x − 4x = y 2 .

13. Să se rezolve ı̂n numere naturale ecuaţia 2x = x2 + y 2 .

14. Fie n un număr natural n ≥ 2. Să se rezolve ı̂n numere naturale


ecuaţia x2 + 4 = 2y n .

96
15. Să se rezolve ı̂n numere naturale ecuaţia x3 + 1 = py , unde p este un
număr prim.

16. Să se rezolve ı̂n numere naturale ecuaţia 2n + 1 = y m , m ≥ 2.

17. Să se rezolve ecuaţia xm + y m = 2n , unde x, y, m, n sunt numere


naturale nenule, m ≥ 2, n ≥ 2.

18. Să se rezolve ı̂n numere naturale ecuaţia 3m + 4m + 5m = 6n .

19. Produsul a k ≥ 2 numere naturale nenule consecutive nu este putere k.

20. Să se rezolve ecuaţia 1m + 3m + · · · + (2n − 1)m = n2m ı̂n numere


naturale nenule.

21. Să se rezolve ı̂n numere naturale nenule ecuaţia


1m + 3m + 5m + · · · + (2n − 1)m = nm+1 .

22. Produsul a trei numere naturale nenule consecutive nu este putere


k ≥ 2 a unui număr natural.

23. Să se rezolve ı̂n N ecuaţia 5x + 2 = 7y .

24. Să se rezolve ı̂n numere naturale ecuaţia 3x = 2y + 11.

25. Să se rezolve ı̂n N ecuaţia 2 + 11x = 13y .

26. Fie n un număr natural, n > 1. Să se rezolve ı̂n numere naturale
impare ecuaţia xn + 2n−1 = y 2 .

27. Fie n număr natural, n > 1. Să se rezolve ecuaţia xn = 2n + y 2 ı̂n


numere naturale impare.

28. Fie p prim, p ≡ 1 (mod 4). Să se arate că ecuaţia xp + 2p = p2 + y 2


nu are soluţii naturale.

29. Fie n un număr natural, n ≡ 3 (mod 4). Să se rezolve ecuaţia xn−y 2 = 5.

30. Fie q un număr natural mai mare sau egal cu 3. Să se rezolve ı̂n
numere naturale nenule ecuaţia aq + bq = q(a + b).

31. Să se rezolve ecuaţia xn + y n = (x + y)m dacă x, y sunt numere


naturale nenule distincte, iar m şi n sunt numere naturale mai mari sau egale
decât 2.

97
32. Fie n număr natural. Să se rezolve ı̂n numere naturale nenule ecuaţia
xn + y n = (x − y)n+1 .

33. Să se rezolve ı̂n N ecuaţia xy − y x = x + y.

34. Să se rezolve ı̂n N ecuaţia n2 + 3n = a2 .


(OBJ, 2000)
2
35. Să se rezolve ı̂n N∗ ecuaţia ab = ba .
(OIM, 1997)

36. Să se rezolve ı̂n numere naturale nenule ecuaţia xn+1 −(x+1)n = 2001.
(Putnam, 2001)

37. Să se rezolve ı̂n numere naturale ecuaţia 10n + 89 = x2 .

98
CAPITOLUL 17

Diverse

Deoarece acesta este un capitol compozit, ne mărginim aici să prezentăm


două probleme care conţin idei asemănătoare cu câteva din problemele acestei
secţiuni.

Problema 1. Nu există n ∈ N∗ astfel ı̂ncât {n, n + 1, n + 2, n + 3, n + 4, n +


Y Y
5} = A ∪ B, A ∩ B = ∅ şi a= b.
a∈A b∈B
(OIM 1970)

Soluţie. Să presupunem că ∃ n ∈ N∗ cu proprietăţile din enunţ. Notăm


Y Y
x= a= b. Cum cel mult unul din numerele n, n + 1, n + 2, n + 3, n + 4,
a∈A b∈B Y Y
n + 5 se divide cu 7, rezultă că nu putem avea simultan 7 a şi 7 b.
a∈A b∈B
Deci n ≡ 1 (mod 7) şi n(n + 1)(n + 2)(n + 3)(n + 4)(n + 5) ≡ 6! ≡ −1 (mod 7).
Avem că x2 = n(n + 1)(n + 2)(n + 3)(n + 4)(n + 5) ≡ −1 (mod 7). Aceasta
este ı̂nsă imposibil căci x2 ≡ 0, 1, 2, 4 (mod 7).
¦
Problema 2. Fie n ∈ N∗ . Să se găsească d = max |S| S ⊆ {1, 2, . . . , n},
©
∃ x 6= y, x, y ∈ S astfel ı̂ncât (x, y) = 1 şi ¶
¶ ∃ x, y ∈ S, x 6= y, astfel ı̂ncât x|y .
(Balcaniadă 2005)

Rezolvare. Analizăm ı̂ntâi cazul n = 4k, k ∈ N∗ . Arătăm că d = k.


Mulţimea S = {2k + 2, 2k + 4, . . . , 4k} are proprietăţile din enunţ şi |S| = k.
Mai trebuie arătat că ¶ ∃ S cu proprietăţile din enunţ şi cu |S| = k + 1. Să
presupunem prin absurd că ar exista o astfel de mulţime cu k + 1 elemente.
Fie S = {a1 < a2 < · · · < ak+1 }. Dintre toate mulţimile cu k + 1 elemente cu
proprietăţile din enunţ, o alegem pe cea pentru care a1 este maxim.
Arătăm că a1 ≤ 2k. Să presupunem că a1 ≥ 2k + 1. Deoarece (aj , aj+1 ) 6= 1,
rezultă că aj+1 ≥ aj + 2, ∀ j = 1, k. Deducem că ak+1 ≥ ak + 2 ≥ ak−1 + 4 ≥
ak−2 + 6 · · · ≥ a1 + 2k ≥ (2k + 1) + 2k = 4k + 1; contradicţie (ak+1 ≤ n = 4k).
Deci, ı̂ntr-adevăr, a1 ≤ 2k.

99
Considerăm acum mulţimea T = {2a1 , a2 , a3 , . . . , ak+1 } ⊆ {1, 2, . . . , n}
(deoarece 2a1 ≤ 4k = n). Avem |T | = k + 1 deoarece 2a1 6= aj ∀ j = 2, k
(a1 ¤|¤aj , ∀ j ≥ 2, conform proprietăţilor mulţimii S). Cum cel mai mic element
al lui T este mai mare strict decât a1 , rezultă că T nu are proprietăţile din
enunţ. Este limpede că (a, b) 6= 1, ∀ a 6= b, a, b ∈ T . Rezultă că ∃ a 6= b,
a, b ∈ T , astfel ı̂ncât a|b. Datorită faptului că ai ¤|¤aj ∀ i 6= j, rezultă că singura
posibilitate este că există j ≥ 2 astfel ı̂ncât aj |2a1 . Deci 2a1 = aj · k, k ∈ N∗ .
k
Dacă k = 1, rezultă contradicţia a1 |aj . Deci k ≥ 2 şi a1 = aj · ≥ aj ;
2
contradicţie. Deci d = k.
La fel ca mai sus se arată că dacă n = 4k + 1, k ∈ N∗ , atunci d = k (pentru
n = 1, d = 1). Pentru n = 4k + 2, d = k + 1, iar pentru n = 4k + 3, d = k + 1.
1. Dacă ı̂ntr-o progresie aritmetică de numere naturale există un pătrat
perfect, atunci ı̂n această progresie există o infinitate de pătrate perfecte.
2. Să se arate că există a1 , a2 , . . . , an ∈ {−1, 1}, astfel ca a1 a2 + a2 a3 +
.
··· + a a + a a = 0, dacă şi numai dacă n..4.
n−1 n n 1

3. Fie şirul (an )n≥1 , a1 = 4, an+1 = a2n − 3an + 3, pentru orice n ≥ 4. Să
se arate că:
a) Dacă m > n atunci (am , an ) = 1.
b) an an+1 − 1 este cub.
c) an nu este cub.
n 1
X 1
d) < .
a
k=1 k
2
4. Pentru n ∈ N, n ≥ 2, cu descompunerea canonică n = pα1 1 pα2 2 . . . pαk k ,
1 1 1 X∞
notăm an = + + · · · + . Să se arate că a2 · a3 · . . . · ak < 1.
p1 p2 pk k=2
(Baraj, 1996)

5. Fie {a1 , a2 , . . . , an } = {n + 1, n + 2, . . . , 2n}. Să se arate că pentru


1 X n k
n ≥ 51, numărul x = are prima zecimală 3.
n k=1 ak

6. a) a ∈ N, a = 4k − 1 (k ∈ N∗ ), an = 1 − Cn2 a + Cn4 a2 − Cn6 a3 . . . ,


∀ n ∈ N∗ . Să se arate că 2n−1 |an , ∀ n ∈ N∗ .
n
X
2k+1 3k
b) Fie n ∈ N şi b = C2n+1 2 . Să se arate că b nu este multiplu de 5.
k=0
(OIM 1974)

100
5 − a2n+1
7. Fie şirul (an )n∈N , an+2 = , a0 = a1 = 1. Să se arate că an
an
este număr ı̂ntreg pentru orice n.

8. Există o infinitate de numere prime p, astfel ı̂ncât perioada scrierii


1 ¦
zecimale a lui să fie multiplu de 3. Să se calculeze max ai + ai+k + ai+2k |
p
1 ©
i = 1, k, p este ca mai sus şi = 0, (a1 , a2 , . . . , a3k ) .
p
(Lista scurtă, OIM, 1999)
p−1
Y
9. Fie p prim şi E = k 2k−p−1 . Să se arate că E este număr ı̂ntreg.
k=1
(Baraj 1988)

10. Să se determine numerele naturale nenule a şi n cu proprietatea că


an+1 + 2n+1 + 1 se divide cu an + 2n + 1.
(Baraj 1998)

a2
11. Să se găsească toate numerele naturale a şi b pentru care
2ab2 − b3 + 1
este număr natural.
( OIM, 2003)

12. Fie A ⊆ {1, 2, . . . , 126}, |A| = 7. Să se arate că ∃ a, b ∈ A, astfel ı̂ncât
b
1 < ≤ 2.
a
13. Fie A ⊆ {1, 2, . . . , 3n}, |A| = 2n + 1, n ∈ N∗ . Să se arate că există
trei elemente ı̂n A care să fie ı̂n progresie aritmetică.
  ¦
n+1
14. Fie n ∈ N, n ≥ 2. Să se arate că + 1 = min k ∈ N∗ |
2 ©
∀ A ⊆ {1, . . . , n}, |A| = k, ∃ a, b, c ∈ A astfel ı̂ncât a + b = c .

15. Dacă n este număr natural nenul şi X o mulţime de n + 2 numere


ı̂ntregi distincte, cu proprietatea că pentru orice x ∈ X avem |x| ≤ n, să se
arate că ı̂n X există trei numere distincte a, b, c cu a + b = c.
(Concurs India, 2000)

16. Pentru ce numere n ∈ N∗ , mulţimea {1, 2, . . . , n} se poate scrie


ca reuniunea a trei mulţimi A, B, C, disjuncte două câte două, astfel ı̂ncât
X X X
x= x= x.
x∈A x∈B x∈C

101
17. Să se arate că ∀ n ∈ N∗ , n = 18k (k ∈ N∗ ), are proprietatea că ∃
mulţimile A, B, C, astfel ı̂ncât {12 , 22 , . . . , n2 } = A ∪ B ∪ C, A ∩ B = A ∩ C =
X X X
B ∩ C = ∅, x= x= x.
x∈A x∈B x∈C

18. Fie A ⊆ {1, 2, . . . , 2n}, |A| = n + 1. Să se arate că există a şi b ı̂n
mulţimea A astfel ı̂ncât a 6= b şi a|b.
¦
19. Să se calculeze max k | ∃ A ⊆ {1, 2, . . . , 2003}, |A| = k şi a · b 6∈ A,
©
∀ a, b ∈ A .

20. Fie n ≥ 3 şi a1 , a2 , . . . , an numerele {1, 2, . . . , n} ı̂ntr-o ordine ar-


bitrară. Notăm bi = iai , i ∈ 1, n. Să se arate că numerele b1 , b2 , . . . , bn nu
formează o progresie aritmetică sau geometrică.

21. Pentru n ≥ 2 se consideră x1 < x2 < · · · < xn numere naturale nenule


consecutive, astfel ı̂ncât xk | kCnk , ∀ k = 1, n − 1. Să se arate că x1 ∈ {1, 2}.

22. a) Fie n natural, n ≥ 2. Să se găsească numerele naturale m,


m
X m
Y
a1 , a2 , . . . , am astfel ı̂ncât ai = n şi ai să fie maxim.
i=1 i=1
b) Fie k, n numere naturale cu k ≥ 2, n ≥ 1. Să se găsească numerele
m
X m
Y
naturale m, a1 , a2 , . . . , am astfel ı̂ncât aki = n şi ai să fie maxim.
i=1 i=1
(generalizarea unei probleme de la OIM, 1976)

Fie n ∈ N, n ≥ 2. Să se arate că dacă k 2 + k + n este prim ∀ k,


23. s
n
0≤k≤ , atunci acest lucru este adevărat pentru ∀ k, 0 ≤ k ≤ n − 2.
3
( OIM, 1987)

24. Fie numărul xn = |11 {z


. . . 1} 22 . . . 2} 5 scris ı̂n baza b. Să se arate că xn
| {z
n−1 n
este pătrat perfect pentru orice n, dacă şi numai dacă b = 10.
(Lista scurtă, OIM, 2003)

25. Fie M o mulţime infinită ı̂n plan, cu proprietatea că distanţa dintre
oricare două puncte ale ei este un număr natural. Să se arate că M este o
submulţime a unei drepte.
(Erdös, 1945)

102
CAPITOLUL 1

Numere prime. Numere compuse

1. ab + 1 este impar, deci a = 2 (sau b = 2). Din 2b + 1 şi 2b − 1 prime


. .
rezultă b = 2 sau b = 3. Pentru b ≥ 5 avem 2b − 1 .. 3 sau 2b + 1 .. 3.
.
2. a) p = 3. Pentru p > 3 avem p = 3l ± 1 şi 2p2 + 1 .. 3.
b) p = 2 sau p = 5. Pentru p > 5 avem p = 5k ± 1 sau 5k ± 2 şi unul dintre
numere se divide cu 5.
c) p = 7. Pentru p > 7 se arată că unul dintre numere se divide cu 7.

3. n = a4 + b4 + c4 − 3 este impar. Dacă a ≤ b ≤ c rezultă a = 2, deci


.
n = b4 + c4 + 13. Dacă b = 3k ± 1 şi c = 3h ± 1 rezultă n .. 3. Deci b = 3.
Rezultă n = c4 + 94. Dacă c = 5k + α, α ∈ {±1, ±2}, rezultă c4 = M5 + 1 şi
.
n .. 5. Deci c = 5, n = 719 care este prim şi a2 + b2 + c2 − 1 = 37 care este, de
asemenea, prim.

4. Notând cu a1 , a2 , a3 , a4 , a5 , a6 numerele date, se verifică uşor că pentru


n ∈ {1, 2, 3} cel puţin unul dintre ele este compus. Pentru n = 4 toate numerele
sunt prime. Fie n ≥ 5. Dacă n = 5k, a6 este compus. Dacă n = 5k + 1, a4 este
compus. Dacă n = 5k + 2, a5 este compus. Dacă n = 5k + 3, a3 este compus,
iar pentru n = 5k + 4, cu k ≥ 1, numărul a1 este compus. Deci n = 4.

5. i) Fie a impar. Dacă p, q sunt impare, rezultă contradicţia M8 + 1 =


M8 + 3. Dacă p şi q sunt pare, atunci p = q = 2 şi a = 3, deci număr prim.
ii) Fie a par. Avem M4 − 1 = p2 + q 2 . Rezultă p = 2 şi contradicţia
q 2 = M4 − 1.
(L. Panaitopol )

6. Fie p < q < r. Din r|p2 + 10 şi r|q 2 + 10, rezultă r|(q − p)(q + p). Cum
r > q − p > 0, rezultă r|p + q. Deoarece p + q < 2r, rezultă p + q = r şi deci
p = 2. Din qr|p2 + 10, rezultă contradicţia qr|14. Înlocuind 10 cu 11, după
p = 2 şi r = q + 2, urmează qr|15 şi se găsesc p = 2, q = 3, r = 5 care verifică
condiţiile.

103
7. Pentru p ≥ 5 avem că
2p + p2 = (3 − 1)p + (3k ± 1)2 = M3 + (−1)p + 1 = M3,
2p + p2 > 3 şi deci 2p + p2 este compus. Avem că 22 + 22 = 4, 23 + 32 = 17 şi
deci singura soluţie a problemei este p = 3.

8. Fie numerele consecutive k +1, k +2, k +3, . . . , k +10. Pentru k ∈ {0, 2}


avem patru numere prime. Pentru k = 1 există cinci numere prime. Fie k ≥ 3.
Cinci dintre numere sunt pare şi deci compuse, iar din trei numere impare
consecutive unul este multiplu de 3 şi deci este compus. Aşadar, pentru k ≥ 3
cel mult patru dintre numere sunt prime. Răspunsul este deci 5 şi k = 1.
.
9. a1 ≥ 0, a2 ≥ 1, a3 ≥ 2, an+3 = a3 +rn. Dacă n = ka3 , rezultă an+3 .. a3 ,
an+3 > a3 (pentru k > 0, r > 0).

10. Arătăm că pentru n ≥ 12, n se scrie ca sumă a două numere compuse.
Dacă n = 2k, k ≥ 6, avem scrierea n = 6 + 2(k − 3). Dacă n = 2k + 1, k ≥ 6,
avem n = 9 + 2(k − 4). Prin verificare se arată că numerele 1, 2, 3, 4, 5, 6, 7,
9, 11 nu pot fi scrise ca sumă a două numere compuse.
€ Š
42n+1 + 1 (4 + 1) 42n − 42n−1 + · · · + 1
11. x = = ∈ N. Apoi
5 5
€ Š€ Š
22n+1 + 1 + 2n+1 22n+1 + 1 − 2n+1
x= .
5
12. Avem 13n +2 = 13n −1+3 = (13−1)(13n−1 +13n−2 +· · ·+1)+3 = M3.
Presupunem 13n +2 = k 2 . Avem k = 13h+α cu |α| ≤ 6 şi 13n +2 = (13h+α)2
şi de aici α2 − 2 = M13, contradicţie.

13. m4 +4n4 = (m2 +2n2 )2 −4m2 n2 = (m2 +2n2 +2mn)(m2 +2n2 −2mn),
m2 + 2n2 + 2mn > m2 + 2n2 − 2mn = (m − n)2 + n2 ≥ n2 ≥ 4 > 1.
Prima inegalitate de mai sus este strictă dacă m > 0. Dacă m = 0, numărul
m4 + 4n4 = 4n4 este evident compus.

14. Pentru n = 17k + 1, k ∈ N∗ se obţine an = M17 şi an > 17, adică an


este număr compus.

15. Avem că C40 = C44 = 1, C41 = C43 = 4, C42 = 6, ceea ce ne arată calea
pe care trebuie să o urmăm.
104060401 = (102 )4 + 4(102 )3 + 6(102 )2 + 4 · 102 + 1 = (102 + 1)4 = 1014 ;
aceasta este descompunerea căutată.

104
16. Avem n = 207 + 202 + 1 = 207 −20+202 +20+1 = 20(203 −1)(203 +1) +
” — .
202 + 20 + 1 = (202 + 20 + 1) (203 + 1)(202 − 20) + 1 şi deci n .. 421.
”√ —
17. 7519 = 86. Avem că 872 − 7519 = 50, 882 − 7519 = 225 = 152 .
De aici deducem că 7519 = 882 − 152 = 73 · 103, aceasta fiind descompunerea
ı̂n factori primi cătată.
. . . .
18. a) E(5) .. 41, E(7) .. 239. b) Dacă n .. m, atunci E(n) .. E(m).

(b − a)(b − c)(c − a)
19. E = . Dacă a ≥ b ≥ c, atunci a|b − a, sau a|b − c,
abc
sau a|c − a, şi deci una din paranteze este 0.

20. Avem

2 4 2(n−1)
102n − 1 (10n − 1)(10n + 1)
xn = 1 + 10 + 10 + · · · + 10 = = .
102 − 1 99
100k − 1 100k − 1
Atunci, x2k = (102k + 1) şi cum ∈ N∗ , pentru k ≥ 2 avem
99 99
100k − 1
> 1 şi 102k + 1 > 1, adică x2k este compus. Pentru k ≥ 1 avem
99
102k+1 − 1 102k+1 + 1 102k+1 − 1 102k+1 + 1
x2k+1 = · . Numerele , sunt na-
9 11 9 11
turale şi mai mari ca 1, deci x2k+1 este compus. Singurul număr prim este
x2 = 101.

21. n = a2 + b2 = c2 + d2 . Dacă a şi b au aceeaşi paritate, n este par, deci


compus. Deci a şi b, respectiv c şi d sunt numere naturale distincte cu parităţi
a−c d+b u
diferite. Fie a şi c pare, b şi d impare. Avem = = , (u, ν) = 1.
d−b a+c ν
hν + ku
Rezultă a + c = hν, b + d = hu, a − c = ku, d − b = kν şi deci a = ,
2
hu − kν (u2 + ν 2 )(k 2 + h2 )
b= şi n = .
2 4
22. Fie numerele n, n + 1, n + 2. Dacă n = 6k, 6k − 1 sau 6k − 2, unul
dintre numere are factorii 2 şi 3.
Dacă n = 6k + 1, k ≥ 2, avem n + 1 = 2(3k + 1) şi n + 2 = 3(2k + 1).
Arătăm că nu putem avea simultan 3k + 1 = 2a , 2k + 1 = 3b cu a ≥ 2, b ≥ 2.
.
Avem 3b+1 − 2a+1 = 1 şi cum 3b+1 − 1 .. 4, rezultă că b + 1 este par. b + 1 = 2c şi
deci (3c −1)(3c +1) = 2a+1 şi de aici 3c +1 = 2x , 3c −1 = 2y şi deci 2x −2y = 2,

105
adică 2x−1 − 2y−1 = 1. Rezultă y = 1 şi apoi c = 1, b = 1, ceea ce contrazice
b ≥ 2.
Dacă n = 6k + 3, numerele sunt 3(2k + 1), 2(3k + 2), 6k + 5, k ≥ 1. Dacă
2k + 1 = 3a şi 3k + 2 = 2b cu a ≥ 1, b ≥ 3, rezultă 2b+1 − 3a+1 = 1 şi deci
.
2b+1 − 1.. 3, adică b + 1 = 2c, c ≥ 2. Rezultă (2c − 1)(2c + 1) = 3a+1 şi deci
2c − 1 = 3x , 2c + 1 = 3y şi apoi 3y − 3x = 2, adică x = 0, c = 1, şi de aici
contradicţie cu c ≥ 2.
Dacă n = 6k + 2, numerele sunt 2(3k + 1), 3(2k + 1), 2(3k + 2). Cum 3k + 1
şi 3k + 2 nu pot fi ambele puteri ale lui 2, se obţine din nou contradicţie.

23. Fie an , n ∈ 1, 24 numerele date. Avem an = a1 + n − 1 şi a1 = 36k + α


cu α = 7, 42 şi k ∈ N. Facem observaţia că numerele 36k + 30 = 6(6k + 5) şi
36k + 42 = 6(6k + 7) cu k ∈ N au cel puţin trei factori primi distincţi, deoarece
6k + 5 şi 6k + 7 sunt prime cu 2 şi 3.
Pentru 7 ≤ α ≤ 30 avem a31−α = 36k + α + 31 − α − 1 = 36k + 30 şi
1 ≤ 31 − α ≤ 24 şi deci a31−α se află printre cele 24 de numere.
Pentru 31 ≤ α ≤ 42 avem a43−α = 36k + α + 43 − α − 1 = 36k + 42 şi
1 ≤ 43 − α ≤ 12. Enunţul este justificat.
.
24. Fie nk = (m + 1)! + k cu k ∈ 2, m + 1. Rezultă nk .. k, nk > k. Avem
.
xnk = 2nk − 1 = 2ak − 1 .. 2k − 1 şi deci 2nk − 1 este compus pentru k ≥ 2.
.
25. Pentru k impar rezultă 22 +k + 1 .. 3. Pentru k = 2s t, s ≥ 1 şi t impar
n

n s s n−s
din n ≥ k rezultă n > s. Avem 22 +2 t + 1 = 22 (2 +t) + 1 şi cum 2n−s + t
s n
este impar, rezultă 22 + 1 divide 22 +k + 1.

26. Alegem n = b + am cu m număr natural impar.

2n + 2a + 2b + 1 = 2b ((2a )m + 1) + 2a + 1 =
€ Š
= 2b (2a + 1) 2a(m−1) − 2a(m−2) + · · · + 1 + 2a + 1 =
= M(2a + 1).

(L. Panaitopol )

27. Presupunem că r este prim. Rezultă imediat că r 6= 2. Atunci unul
dintre numerele p şi q este 2, de exemplu q = 2 (de aici rezultă p ≥ 3). Dacă n
este par, avem n = 2y, y ∈ N∗ . Deci p2y = (r − 2y )(r + 2y ) şi deci r − 2y = pa ,
r + 2y = pb , a < b, a + b = 2y. Rezultă că 2y+1 = pa (pb−a − 1), a = 0, b = 2y.
Avem 1 + 2y = r = p2y − 2y şi deci 1 + 2y+1 = p2y ≥ 32y .

106
Am obţinut o contradicţie, căci se arată imediat prin inducţie că 32y > 1+2y+1 ,
∀ y ∈ N∗ . Dacă n este impar, din pn + 2n = r2 rezultă 1 < p + 2 < r2 , p + 2|r2 ,
adică p + 2 = r şi deci (p + 2)2 = pn + 2n ≥ p3 + 23 , adică p3 − p2 − 4p + 4 ≤ 0
şi deci (p − 1)(p2 − 4) ≤ 0. Aşadar p = 2, ceea ce nu convine. Deci ecuaţia nu
are soluţii dacă r este prim. Aşadar r este compus.

28. Luăm a = k 3 + 4k, k ∈ N, k ≥ 2;

M1 = n3 + 4n + k 3 + 4k = (n + k)(n2 − nk + k 2 + 4).

Cum n+k ≥ 2, n2 −nk+k 2 > 1, rezultă că M1 este compus. M2 = n(n+3)+a.


Numărul n(n+3) este par, deci dacă a este par şi a ≥ 4, M2 este par şi M2 > 2.
Deci M2 este compus. Vom lua a = k 3 + 4k = k(k 2 + 4) cu k = 2h, h ≥ 1 şi
deci a = 8h(h2 + 1), h ≥ 1.
(L. Panaitopol )

29. Avem x0 = a şi deci a este prim. Dacă a ∈ {2, 3, 5, 7}, atunci a|x1
şi deci x1 este compus. Dacă a = 11, x1 = 221 = 13 · 17. Aşadar a > 11.
Avem xn ≥ a > 11 şi deci este necesar ca 11¤|¤xn pentru orice n ∈ 0, 9. Avem
xn = 11 · 19 · n + n + a şi deci este necesar ca a, a + 1, a + 2, . . . , a + 9 să nu
se dividă cu 11. Rezultă a + 10 = 11k şi deci a = 11k − 10. Cum a este prim,
rezultă a ∈ {23, 67, 89, 199, . . . }.
Pentru a = 23 avem x8 = 13 · 131. Pentru a = 67 avem x3 = 17 · 41.
Pentru a = 89 avem x1 = 13 · 23. Pentru a = 199 numerele xn , n ∈ 0, 9
sunt toate prime. Aşadar a minim este 199.

30. Fie π(n) numărul numerelor prime mai mici sau egale cu n. Vom
demonstra următoarea
n
Lemă. π(n) ≤ − 11, pentru orice n ≥ 54.
2
54
Demonstraţie. Vom utiliza inducţia. Avem π(54) = 16 ≤ − 11 şi π(55) =
2
55
16 < − 11. Dacă n + 1 = 2k, atunci
2
2k − 1 n+1
π(n + 1) = π(2k) = π(2k − 1) ≤ − 11 < − 11.
2 2
Dacă n + 1 = 2k + 1, atunci
2k − 1 n+1
π(n + 1) ≤ π(2k − 1) + 1 ≤ − 11 + 1 = − 11
2 2
şi demonstraţia este ı̂ncheiată.

107
Revenind la problemă, vom arăta că orice număr n ≥ 52 se poate scrie ı̂n
cel puţin 10 moduri, ca sumă de două numere compuse. Avem:
52 = 2k + (52 − 2k), k = 2, 13, adică 12 scrieri;
53 = 4 + 49 = 8 + 45 = 9 + 44 = 14 + 39 = 15 + 38 = 18 + 35 = 20 + 33 =
21 + 32 = 25 + 28 = 26 + 27 are 10 scrieri.
Fie acum n ≥ 54 şi c1 , c2 , . . . , ck numerele compuse mai mici sau egale cu
n. Avem n = k +π(n)+1. Dintre numerele n−c1 , n−c2 , . . . , n−ck , unul poate
fi 1, cel mult π(n) sunt prime şi deci cel puţin k − π(n) − 1 sunt compuse. Cum
 
n
k = n − π(n) − 1, rezultă că cel puţin n − 2π(n) − 2 ≥ n − 2 − 2 − 11 = 20
2
sunt compuse. Nefăcând distincţie ı̂ntre scrierile n = a+b şi n = b+a, obţinem
cel puţin 10 scrieri distincte.
În concluzie, numărul căutat este 51. Se arată cu uşurinţă că sunt doar
nouă scrieri pentru n = 51, şi anume:

9 + 42 = 6 + 45 = 12 + 39 = 15 + 36 = 16 + 35 =
= 18 + 33 = 21 + 30 = 24 + 27 = 25 + 26.

(L. Panaitopol )

31. Răspunsul este n = 2 şi n = 3, numerele fiind 7, 17 şi 71. Pentru


n ≥ 4 construim un număr cu n cifre, ı̂n condiţiile date, care este compus.
Pentru n = 3k suma cifrelor numărului este 7 + 3k − 1 = M3 şi deci orice
număr se divide cu 3.
Amintim criteriul de divizibilitate cu 7, 11, 13. Pentru M = an an−1 . . . a1 a0 ,
M se divide cu 7, 11 sau 13 dacă M1 = a2 a1 a0 −a5 a4 a3 +a8 a7 a6 −a11 a10 a9 +. . .
se divide cu 7, 11 sau 13. Avem cazurile:
a) a = 6k + 1. Numărul N = 7111 . . . 1 se divide cu 7 deoarece
.
N = 111 − 111 + 111 − · · · + 7 .. 7.
1
b) n = 6k + 2. Numărul N = 111 . . . 1711 se divide cu 13 deoarece
N1 = 711 − 111 + 111 − · · · − 111 + 11 = 611 = M13.
.
c) n = 6k + 4. Numărul N = 711 . . . 11..13 deoarece N = 111−7 = 104 = M13.
1
d) n = 6k + 5. Numărul N = 11 . . . 1711 se divide cu 7 deoarece
N1 = 711 − 111 + 111 + · · · − 11 = 700 = M7.

32. Fie n = 5397 şi avem

N = 51985 − 1 = M5 − 1 = (n − 1)(n4 + n3 + n2 + n + 1).

108
Cum n − 1 > 5100 rămâne să descompunem
N1 = n4 + n3 + n2 + n + 1 = (n2 + 3n + 1)2 − 5n(n + 1)2 .
Cum n = 5m2 cu m = 5198 , avem
N1 = (25m4 + 15m2 + 1)2 − 25m2 (5m2 + 1)2 =
= (25m4 + 15m2 + 1 − 25m3 − 5m)(25m4 + 15m2 + 1 + 25m3 + 5m)
şi factorii sunt mai mari ca 5100 .
p €
X Š2 p−1
X€ Š2
p
33. C2p = Cpk = 2+ Cpk . Cum p|Cpk , ∀ k = 1, p − 1, deducem
k=0 k=1
p
din ultima egalitate că p2 | C2p − 2.
k
34. Punând ı̂n identitatea Cm+n = Cn0 Cm
k + C 1 C k−1 + · · · + C k · C 0 ,
n m n m
n = k = p, m = 2p, obţinem că
p p p−1
C3p = C2p + Cp1 C2p + · · · + Cpp−1 C2p
1
+ 1,
€ Š
p p p−1 p−2
C3p − 3 = C2p − 2 + Cp1 C2p + Cp2 C2p + · · · + Cpp−1 C2p
1
.
Din ultima identitate, din faptul că p | Cpk , ∀ k = 1, p − 1, p | C2p k ,
2 p
∀ k = 1, p − 1 şi din exerciţiul precedent (care ne asigură că p | C2p − 2),
p
deducem că p2 | C3p − 3.
Observaţie. O rezolvare a unui enunţ similar mai puternic se găseşte la
capitolul ”Congruenţe”.

35. Alegem b = c = a + 1 şi f (a, b, c) = 3a + 2. Dacă b = c = a − 1, avem


f (a, b, c) = 3a − 2. Se arată că
1 € Š
f (a, b, c) = (a + b + c) (a − b)2 + (b − c)2 + (a − c)2 = 3
2
nu are soluţii.

36. Dacă n = 0, atunci m = 0 sau m = 1 şi N = 2m + 5. Convine doar


m = 1 şi N = 7. Presupunem că m > 2. Atunci
n € Š Y
n−1
i
 €€ α Ša Š
N = 22 − 1 + 4 2m−2 + 1 = 22 + 1 + 4 22 +1 ,
i=0
α
unde m − 2 = 2α
· a, a impar. De aici deducem că N este multiplu de 22 + 1
α
deoarece α ≤ n − 1(2α < m = a · 2α + 2 ≤ 2n ). Cum N > 22 + 1, obţinem că
n
N nu este prim. Dacă m = 0, N = 22 + 4 este compus. Dacă m = 1,
n n
N = 22 + 5 = (3 − 1)2 + 5 = M3 + 6 = M3

109
1
(pentru n ≥ 1). Deci 3 | N , N ≥ 9 şi deci N este compus. 22 + 22 + 3 = 11
şi deci m = 2 este soluţie a problemei. Am arătat că singurele soluţii ale
problemei sunt m = 1 şi m = 2.

110
CAPITOLUL 2

C.m.m.d.c şi C.m.m.c.

1. Fie d = (a, b). Avem a = dα, b = dβ, [a, b] = dαβ şi (α, β) = 1. Dacă
d = x2 şi dαβ = y 2 , rezultă αβ pătrat perfect cu (α, β) = 1. Adică α = A2 ,
β = B 2 şi deci a = (xA)2 şi b = (xB)2 . Reciproca este imediată.

2. Fie a = dα, b = dβ, c = dγ cu (α, β, γ) = 1. Rămâne de arătat că


   
α+β β+γ γ+α α+β β+γ γ+α
(α, β, γ) = , , , adică δ = , , = 1.
2 2 2 2 2 2
α+β β+γ γ+α α+β
Avem δ + + = α + β + γ. Din δ rezultă δ|α + β, deci
2 2 2 2
δ|(α + β + γ) − (α + β) = γ. Analog, δ|α şi δ|β adică δ|(α, β, γ) = 1 şi deci
δ = 1.

3. Avem [a, b] · (a, b) = ab şi apoi ab = ac, etc.

3αβ
4. a = dα, b = dβ, (α, β) = 1. Rezultă [a, b] = dαβ şi d = . Cum
α2 − β2
(αβ, α2 − β 2 ) = 1, rezultă α2 − β 2 |3, adică α = 2, β = 1.

5. Din a = dα, b = dβ, (α, β) = 1 rezultă dα2 = β(α + 1) şi cum


(α2 , β) = 1 şi (α2 , α + 1) = 1, rezultă α = 1, d = 2β, a2 = 2b.

αβ + 7
6. Din a = dα, b = dβ, (α, β) = 1 rezultă d = ≤ 4. Pentru d = 1
È α2 + β 2
β± 28 − 3β 2
rezultă α = şi deci β ∈ {1, 2, 3}. Pentru d = 2 şi d = 3 nu
2
avem soluţii, iar pentru d = 4 rezultă α = β = 1. Perechile a, b sunt (1, 3),
(3, 1), (2, 3), (3, 2), (4, 4).

7. Dacă a ≤ b rezultă 2bb > mm cu m = kb. Presupunem k ≥ 2 şi avem


2bb > (2b)2b = 22b · b2b şi deci 1 > 22b−1 · bb > 1. Din această contradicţie
rezultă k = 1 şi deci b = m, adică a | b.

8. Dacă b = a + 1, atunci (a, b) = 1 şi deci [a, b] = ab şi (a, b) = 1. Relaţia


din enunţ se scrie a2 +a2 +2a+1 = na(a+1)+1 şi din a(a+1)(2−n) = 0, rezultă

111
n = 2. Dacă n = 2 notăm (a, b) = d şi avem a = dα, b = dβ, (α, β) = 1. Rezultă
2α · β + 1
[a, b] = dαβ şi avem relaţia d(α2 + β 2 ) = 2αβ + 1. Din d = ≥ 1
α2 + β 2
2
2α2 + 1 1
rezultă (α − β) ≤ 1. Dacă α = β avem d = = 1 + ∈
/ N şi deci
2α2 2α2
α − β = ±1 şi d = 1. Avem deci dα − dβ = ±1, adică a − b = ±1, şi deci a şi
b sunt numere consecutive.

9. Fie a = dα, b = dβ, (α, β) = 1. Rezultă [a, b] = dαβ şi dαβ = d3 ,


adică αβ = d2 . Cum (α, β) = 1, rezultă α = A2 , β = B 2 cu AB = d. Rezultă

de aici a = A3 B şi b = B 3 A şi apoi AB(A2 + B 2 ) = 30n . Cum A2 + B 2 ..3
¦ şi
(A, B) = 1, avem A = 3n x şi deci Bx(9n x2 + B 2 ) = 10n cu (x, B) = 1.
. .
Dacă x .. 2, rezultă x .. 2n şi contradicţia 10n > 9n · x3 ≥ 9n · 2n . Aşadar, x
este impar. Analog, se arată că x nu se divide cu 5 şi B nu se divide nici cu 2,
nici cu 5. Cum Bx|10n , rezultă B = x = 1 şi 9n + 1 = 10n . Deci n = 1, a = 27,
b = 3 sau n = 1, a = 3, b = 27.

10. Fie (a, b, c) = d şi deci a = dα, b = dβ, c = dγ şi din a+b+c = [a, b, c]
rezultă α + β + γ = [α, β, γ] cu (α, β, γ) = 1. Dacă (α, β) = x > 1, rezultă
x|[α, β, γ] şi deci contradicţia x | γ. Aşadar (α, β) = 1, (α, γ) = 1, (β, γ) = 1 şi
deci [α, β, γ] = αβγ. Avem deci α + β + γ = αβγ. Fie 1 ≤ α < β < γ. Avem
α+β α+β α+β
γ> , deci γ = > şi deci αβ < 3. Rezultă α = 1, β = 2,
2 αβ − 1 2
γ = 3 şi deci a = d, b = 2d, c = 3d şi c = a + b.
k
Y k
Y k
Y
yi
11. Dacă a = pxi i , b = pi , c = pzi i , xi , yi , zi ≥ 0, rămâne de
i=1 i=1 i=1
arătat că max(xi , yi , zi ) + min(xi , yi ) + min(yi , zi ) + min(zi , xi ) = xi + yi +
zi + min(xi , yi , zi ). Simetria relaţiei permite alegerea xi ≤ yi ≤ zi şi egalitatea
devine zi + xi + yi + xi = xi + yi + zi + xi .

12. Cu notaţiile din problema precedentă, avem

min(xi , yi , zi ) + max(xi , yi , zi ) = xi + yi ⇔ min(xi , yi ) ≤ zi ≤ max(xi , yi ).

13. i) Fie a = 6α, b = 6β, c = 6γ cu (α, β, γ) = 1. Rezultă [α, β, γ] =


24 · 3 · 5 şi α + β + γ = 39. Fie α = 16k. Rezultă k impar şi 16k < 39, deci
k = 1 şi α = 16. Avem deci β + γ = 23 şi 3 · 5 | αβγ şi de aici β = 15, γ = 8
sau β = 20, γ = 3 sau β = 18, γ = 5 (ultima soluţie nu convine), adică avem
soluţiile (96, 90, 48); (96, 120, 18) şi permutările circulare.

112
ii) Avem a = 12α, b = 12β, c = 12γ cu (α, β, γ) = 1. rezultă [α, β, γ] = 5·7
şi αβ + βγ + γα = 455. Fie α = 7A. Avem 7A(β + γ) + βγ = 7 · 65 şi deci
7 | βγ. Fie β = 7B. Rezultă A(7B + γ) + Bγ = 65 (1) şi 7¤|¤ABγ (2). Dacă 5|γ,
din (1) rezultă 5|AB. Dacă 5¤|¤γ, din [α, β, γ] = 35 rezultă 5|AB. Aşadar 5|AB.
Din (1) rezultă 7AB < 65, adică AB ≤ 9 şi deci AB = 5.
Fie A = 5 şi B = 1. Din (1) avem 35 + 5γ + γ = 65 şi deci γ = 5. Avem
soluţia a = 420, b = 84, c = 60 şi permutările ciclice.
14. Evident 2|3m − 1 şi 2|3n + 1. Fie d = (m, n). Avem m = dx, n = dy cu
x impar. Dacă D = (3m − 1, 3n + 1), avem 3m − 1 = DX şi 3n + 1 = DY , adică
3dx = DX + 1 şi 3dy = DY − 1. Ridicăm egalităţile la puterile y, respectiv
x. Rezultă 3dxy = (DX + 1)y = (DY − 1)x , adică MD + 1 = MD − 1 şi deci
D|2. Aşadar D = 2.
€ 2 2
Š
15. Fie D = a2n +3n+1 − 1, an +3n+2 + 1 . Avem a(n+1)(2n+1) − 1 = MD
şi a(n+1)(n+2) +1 = MD şi deci a(n+1)(2n+1) = MD+1 şi a(n+1)(n+2) = MD−1.
Ridicăm egalităţile la puterile n+2 şi respectiv 2n+1. Rezultă (MD +1)n+2 =
(MD − 1)2n+1 , adică MD + 1 = MD − 1 şi deci D|2. Dacă a este par, avem
D = 1, iar dacă a este impar, avem D = 2.
16. Dacă m = da şi n = db, (a, b) = 1, 2m − 1 = 2da − 1 = (2d )a − 1 =
(2d − 1)(2d(a−1) + 2d(a−2) + · · · + 1).
Reciproc, există λ, µ ∈ N astfel ca mλ − nµ = d. Dacă D = (2m − 1, 2n − 1)
avem 2m = Dk+1, 2n = Dh+1 şi 2mλ = (Dk+1)λ = MD+1 şi 2nµ = MD+1.
Avem MD + 1 = 2mλ = 2nµ+d = 2d (MD + 1) şi deci D|2d − 1.
. .
17. Avem An = n(n4−1)(n4+1). Se arată că n(n4 − 1)..5 şi (n−1)n(n+1)..6.
Aşadar 30|An . Avem A2 = 30 · 17 şi A3 = 3 · 80 · 82 şi deci A3 nu se divide cu
17. Aşadar (A1 , A2 , A3 , . . . , A2005 ) = 30.
18. Avem A0 = 35 = 5 · 7, A1 = 6569 + 58 şi deci 5 nu divide A1 . Arătăm
că 7|An . Avem
An = 8n + 9 · 272n + 25(125)2n =
= (7 + 1)n + 9(M7 − 1)2n + 25(M7 − 1)2n =
= M7 + 1 + 9(M7 + 1) + 25(M7 + 1) = M7.
19. Fie p prim, p ≤ n şi ip exponentul său ı̂n [1, 2, 3, . . . , n]. Din pip ≤ n
rezultă
Y Y
[1, 2, 3, . . . , n] = pip ≤ n = nπ(n) .
p≤n p≤n

113
20. Fie d = (a1 , a2 ) cu a1 = dα, a2 = dβ, (α, β) = 1, a3 = a2 + (dβ, dα) =
dβ + d = d(β + 1), a4 = a3 + (d(β + 1), dβ) = d(β + 1) + d = d(β + 2).
Prin inducţie se arată că pentru n ≥ 2, an = d(β + n − 2) şi deci
an = dβ + (n − 2)d = a2 + (n − 2)(a1 , a2 ).
ak bk
21. Fie a = db, n = dm, (b, m) = 1. Avem = şi deci n|ak ⇔
n m
m|bk ⇔ m|k deoarece (m, b) = 1 şi deci k = mt cu t ∈ {1, 2, . . . , d} deoarece
md = n ≥ k.

22. Fie a − 1 = dA, n = dm, (A, m) = 1. Avem


 n ‹   ‚ Œ
a −1 (dA + 1)n − 1 M(dA)2 + ndA
,a − 1 = , dA = , dA =
a−1 dA dA
= (MdA + n, dA) = (n, dA) = (n, dA) = (n, a − 1).

23. Presupunem că există p prim p|an + n, p|an+1 + n + 1, p|an+2 + n + 2.


Avem an = kp − n, an+1 = hp − n − 1, an+2 = lp − n − 2 şi de aici a(kp − n) =
hp − n − 1, a(hp − n − 1) = lp − n − 2. Rezultă p|n(a − 1) − 1, p|n(a − 1) + a − 2
şi deci p|n(a − 1) + a − 2 − (n(a − 1) − 1) = a − 1. Cum p|n(a − 1) − 1, rezultă
contradicţia p|1. Deci (xn , xn+1 , xn+2 ) = 1.

24. Luăm a = 2k(2k + 1), b = (2k + 1)(2k − 1), c = 2k(2k − 1) cu k ∈ N∗ .


Avem (a, b) = 2k + 1, (a, c) = 2k şi (b, c) = 2k − 1.

25. Dacă m ∈ {0, n − 1} condiţia se verifică. Fie 1 ≤ m ≤ n − 2. Avem


€ Š
(m + 1)Cnm+1 = (n − m)Cnm . Dacă Cnm , Cnm+1 = 1, rezultă Cnm | m + 1 şi
deci m + 1 ≥ Cnm ≥ n, deci m ≥ n − 1, ceea ce intră ı̂n contradicţie cu
€ Š
m ≤ n − 2. Deci condiţia Cnm , Cnm+1 = 1 este ı̂ndeplinită dacă şi numai dacă
n ∈ {0, n − 1}.

26. Presupunem prin absurd că [a + n, b + n, n2 ] = n2 , adică a + n|n2 şi


b + n|n2 (1). Arătăm că (a + n, b + n) = 1. Presupunem că există p număr prim
cu p|a + n şi p|b + n. Din relaţiile (1) rezultă p|n şi apoi contradicţia p|a şi p|b.
Aşadar, (a + n, b + n) = 1 şi de aici (a + n)(b + n)|n2 , ceea ce este imposibil,
deoarece (a + n)(b + n) > n2 .

27. Rezultatul este o consecinţă imediată a teoremei lui Dirichlet, pri-


vitoare la numerele prime din progresiile aritmetice, ı̂n care raţia şi primul
termen sunt prime ı̂ntre ele. Există ı̂nsă o demonstraţie elementară pe care o
prezentăm ı̂n cele ce urmează. Este suficient să demonstrăm că ∃ un k ∈ N
astfel ı̂ncât (a + kb, m) = 1, căci atunci (a + (k + lm)b, m) = 1, ∀ l ∈ N.

114
Fie (a, m) = n, Dm = pα1 1 . . . pαr r , unde pi , i = 1, r, sunt toate numerele
prime ce apar ı̂n descompunerea lui n şi αi = epi (m).
m
Considerăm k = ∈ N∗ şi arătăm că (a + kb, m) = 1. Să presupunem că
Dm
n-ar fi aşa şi fie q un număr prim astfel ı̂ncât q|a + kb şi q|m.
m
Cazul I : q|k = . Ţinând cont de definiţia lui Dm , rezultă că q¤|¤a. Dar
Dm
q|a + kb şi q|k implică contradicţia q|a.
m
Cazul II : q¤|¤k = . Cum q|m, din construcţia lui Dm rezultă q|a. Din
Dm
q|a şi q|a + kb obţinem q|kb. Dar q este prim şi q¤|¤k, ceea ce implică q|b. Am
obţinut contradicţia q|(a, b) = 1.

28. P = x(x − 1)a + 1, a ∈ Z. Se arată prin inducţie după i că ∀ i ∈ N∗ ,


xi −1 = x0 x1 . . . xi−1 ki . Dacă i > j, din relaţia precedentă rezultă că (xi , xj ) =
1.

29. Avem an an+1 = xn yn şi deci şirul {an an+1 }n≥1 este o progresie
an+1 an+2 an+2
geometrică cu raţia q = = . Din an+2 = qan , ∀ n ≥ 1, rezultă
an an+1 an
a2n+1 = a1 q n şi a2n = a2 q n−1 . Există k astfel ı̂ncât ak ak+2 = a2k+1 .
Pentru k = 2n avem a2n a2n+2 = a22n+1 , adică a2 q n−1 · a2 q n = a21 q 2n şi deci
 2
a2
q= .
a1
Pentru k = 2n − 1 avem a2n−1 · a2n+1 = a22n şi deci a1 q n−1 a1 q n = a22 q 2n−2
 2
a2 ak+1
şi avem de asemenea q = . Calculăm .
a1 ak
a2n a2 q n−1 a2
Pentru k = 2n − 1 avem = = .
a2n−1 a1 q n−1 a1
 
a2n+1 a1 q n a1 a1 a2 2 a2
Pentru k = 2n avem = = q= = .
a2n a2 q n−1 a2 a2 a1 a1
a2
Aşadar, {an }n≥1 este o progresie geometrică de raţie r = . Se arată uşor că
a1

a1 |a2 şi deci r ∈ N .

30. Pentru i = 1 avem aj ≤ [a1 , aj ] ≤ a21 , ∀ j, ceea ce constituie o


contradicţie, deoarece şirul este nemărginit. Fie ak = (2n + k)! pentru k ∈ 1, n.
Deoarece a1 |a2 | . . . |an , avem (ai , aj ) ≥ (2n+1)! şi [ai , aj ] ≤ (3n)!. Prin inducţie
se demonstrează că (3n)! < ((2n + 1)!)2 , ceea ce justifică enunţul.

115
31. Pentru i 6= j, ai şi aj nu au multipli comuni ≤ n. În caz contrar am
 
n
avea [ai , aj ] ≤ n. Numărul multiplilor lui ai care nu ı̂l depăşesc pe n este .
ai
Xk  n Xk n 
Xk 1
Rezultă de aici că ≤ n şi deci n > −1 =n − k.
i=1
ai i=1
ai a
i=1 i
k 1
X n+k
Rezultă < < 2 deoarece n > ak > k.
i=1
ai n

32. Vom demonstra prin inducţie. Pentru n = 1 avem [a0 , a1 ] ≥ 2, deci


1 1 1
≤ = 1 − . Dacă an+1 ≥ 2n+1 , avem [an , an+1 ] ≥ 2n+1 şi din ipoteza
[a0 , a1 ] 2 2
n−1
X 1 1
de inducţie ≤ 1 − n rezultă
k=0
[ak , ak+1 ] 2
n
X 1 1 1 1 1 1
≤1− n + ≤ 1 − n + n+1 = 1 − n+1 .
[a , a ]
k=0 k k+1
2 [an , an+1 ] 2 2 2

ab
Fie an+1 < 2n+1 . Deoarece [a, b] = şi |a − b| ≥ d, avem
(a, b)
n
X n
X n
1 (ak , ak+1 ) X ak+1 − ak
= ≤ =
[a , a ] k=0 ak ak+1
k=0 k k+1 k=0
ak ak+1
n  
X 1 1 1 1 1
= − = − ≤ 1 − n+1 .
k=0
ak ak+1 a0 an+1 2
(Boris Ivlev, Kvant, 1984)

116
CAPITOLUL 3

Divizibilitate

1. n = 1000 · an an−1 . . . a3 + 100a2 + 10a1 + a0 = M8 + 4a2 + 2a1 + a0 .

2. n = 100an an−1 . . . a2 + a1 a0 . Dar 100 = 2 · 49 + 2 şi 100 = 6 · 17 − 2.

3. 104k = M101 + 1 şi 104k+2 = M101 − 1.

4. Se ţine seama că 103 = 100 + 3 şi 97 = 100 − 3.

5. Notând b = 7 sau 11, sau 13, avem că n ≡ 456 − 23a + 1 = 457 − 23a =
227 − a (mod b). Dacă b = 7, deducem că 3 ≡ a(7) şi deci a = 3. Dacă b = 11,
rezultă că a ≡ 7(11) şi deci a = 7. Pentru b = 13, obţinem a ≡ 6(13) şi
a = 6. Avem că n ≡ 456 + 23a + 1 = 687 + a ≡ 21 + a(37). Condiţia 37|n
este satisfăcută doar dacă a ≡ 16 (mod 37). Însă nici un număr din mulţimea
{0, 1, 2, . . . , 9} nu are această proprietate şi deci n nu se divide cu 37.

3
6. 4a = 6n − 1 − ∈ Z, deci n = 0, −1, 1, −2 şi a = −1, 1 − 3.
2n + 1
7. Din d|3n2 + 2 şi d|2n3 + 1 rezultă d|6n3 + 4n şi d|6n3 + 3, deci d|4n − 3.
Din d|4n − 3 şi d|3n2 + 2 rezultă d|12n2 − 9n şi d|12n2 + 8 şi de aici d|9n + 8.
Din d|4n − 3 şi d|9n + 8 rezultă d|36n − 27 şi d|36n + 32. De aici d|59 şi cum
59 este prim, rezultă d = 59|4n − 3 şi n minim este 45.

8. Presupunem că există p prim cu p|3n2 + a şi p|2n3 + b. Rezultă


p|2n(3n2 +a)−3(2n3 +b) = 2an−3b. Rezultă apoi p|2a(3n2 +a)−3n(2an−3b) =
2a2 + 9bn. În sfârşit se obţine contradicţia din

p|9b(2an − 3b) − 2a(2a2 + 9bn) = −4a3 − 27b2 = ±1.

9. Arătăm mai ı̂ntâi că m şi n au aceeaşi paritate. Dacă n = 2k + 1 şi


m = 2h avem

2n + 3m = 2 · 4k + 9h = 2(5 − 1)k + (10 − 1)h =


= 2(M5 ± 1) + (M5 ± 1) 6= M5.

117
Dacă n = 2k şi m = 2h + 1 avem

2n + 3m = 4k + 3 · 9h = (5 − 1)k + 3(10 − 1)h =


= M5 ± 1 + 3(M5 ± 1) 6= M5.

Fie acum n şi m de aceeaşi paritate. Avem

2m + 3n = (5 − 3)m + (5 − 2)n = M5 + (−1)m 3m + M5 + (−1)n 2n =


= M5 + (−1)m (2n + 3m ) = M5 + (−1)m M5 = M5.

10. n = a · b, a, b ≥ 2. Rezultă a, b ≤ n − 2. Dacă a < b, avem (n − 2)! =


. √ √
1·2·. . .·a·. . .·b·. . .·(n−2) .. ab = n. Dacă a = b, rezultă a = n şi 2 n ≤ n−2
.
pentru n ≥ 9, deci (n − 2)! = 1 · 2 · . . . · a · . . . · 2a · . . . · (n − 2)..a2 = n.

n
X n(n + 1) 2n! 2(n − 1)!
11. Avem că k= , xn = = . Dacă n + 1
k=1
2 n(n + 1) n+1
este compus şi n ≥ 5, din problema precedentă rezultă că (n + 1)|(n − 1)!
4
şi xn ∈ N∗ . Pentru n = 3, x3 = = 1. Dacă n + 1 este prim, evident că
4
n + 1¤|¤2(n − 1)! şi deci xn ∈
/ N.


12. Dacă 7¤|¤abc, a3 , b3 , c3 dau la ı̂mpărţirea cu 7 resturile ±1 şi a3+b3+c3 ..7.
¦

13. Fie (n, 10) = 1. Aplicăm teorema ı̂mpărţirii cu rest. 1 = nq1 + r1 ,


11 = nq2 + r2 , . . . , |11 {z
. . . 1} = nqn+1 + rn+1 . Din cele n + 1 resturi două sunt
n+1 ori
11 . . . 1 = nqi + r;
egale: . Prin scădere obţinem 11 . . . 100 . . . 0 = n(qi − qj ) şi
11 . . . 1 = nqj + r
cum (n, 10) = 1 ⇒ n|11 . . . 1.

14. Avem

4m+3
X (−1)k−1 4m+3
X 1 2m+1
X 1 4m+3
X 1
xm = = −2 = =
k=1
k k=1
k k=1
2k k=2m+2
k
X 
3m+2
1 1
‹ 3m+2
X 1 pc
= + =p = .
k=2m+2
k 6m + 5 − k k=2m+2
k(6m + 5 − k) b

118
n(n + 1) k
15. Trebuie să arătăm că 1 + · · · + nk , n(n + 1)|2(1k + 2k +
n €
2
X Š
· · · + nk ) = j k + (n + 1 − j)k = s. Avem că
j=1
n n
(n+1) X k k
X
s ≡ j + (−j) = (j k − j k ) = 0 (mod (n + 1)) şi
j=1 j=1
n n €
(n) X € k k
Š X Š
s ≡ j + (1 − j) ≡ j k − (j − 1)k = nk ≡ 0 (mod n).
j=1 j=1

Cum (n, n + 1) = 1, rezultă n(n + 1)|s.

16. Dacă n este impar şi 2n |3n − 1, atunci n = 1. Dacă cumva n ≥ 3,


atunci 4|3n −1. Aceasta este o contradicţie, căci 3n −1 ≡ (−1)n −1 ≡ −1−1 =
−2 ≡ 2 (mod 4). Deci singurul număr impar care este soluţie a problemei este
n = 1. Presupunem că n este par; n = 2k t, t ∈ N∗ , 2¤|¤t, k ∈ N∗ . Arătăm
t
prin inducţie că 32 ≡ 1 + 2t+2 (2t+3 ), ∀ t ∈ N∗ . Pentru t = 1, este clar că
t
32 = 1 + 23 ≡ 1 + 23 (24 ). Presupunem că 32 = 1 + 2t+2 + 2t+3 α, α ∈ Z. Atunci
t+1
€ Š 2 € Š k
32 ≡ 1 + 2t+2 ≡ 1 + 2t+3 mod 2t+4 . Avem deci 32 ≡ 1 + 2k+2 (2k+3 ),
k
3n = 32 t ≡ 1+t2k+2 (2k+3 ) (s-a folosit imparitatea lui t). Rezultă e2 (3n −1) =
k+2. Însă k+2 = e2 (3n −1) ≥ e2 (2n ) = n = 2k t ≥ 2k > k+2, ultima inegalitate
fiind adevărată pentru k ≥ 3. Deci nu avem soluţii cu k ≥ 3.
Pentru k = 1 avem 3 = e2 (3n − 1) ≥ e2 (2n ) = n = 2t ≥ 4 pentru t ≥ 2.
Deci, ı̂n acest caz, singura soluţie este t = 1, n = 2.
Pentru k = 2 avem 4 = e2 (3n − 1) ≥ e2 (2n ) = n = 4t > 4 pentru t ≥ 2.
Deci t = 1, n = 4 este singura soluţie ı̂n acest caz. Am arătat că soluţiile sunt
n = 1; n = 2; n = 4.

17. Definim a1 = 3 şi pentru m ∈ N, m ≥ 1, am+1 = 3am p, unde p


este un divizor prim diferit de 3 al lui 22am − 2am + 1. Numărul căutat va fi
n = a2005 . Avem că am este impar şi divizibil cu 3, ∀ m ∈ N∗ . Deci am ≡ 3(6)
şi 22am − 2am + 1 ≡ 26 − 23 + 1 ≡ 3 (mod 9). Cum am ≥ 3, avem că
22am − 2am + 1 = 2am (2am − 1) + 1 ≥ 8 · 7 + 1 = 57.
Din cele de mai sus rezultă că există un divizor prim p 6= 3 al lui 22am −2am +1.
Prin inducţie se arată că am 2am + 1, ∀ m ∈ N∗ . Pentru m = 1 este evident,
căci 3|23 + 1. Dacă am 2am + 1, atunci
23am + 1 2am + 1 22am − 2am + 1
= · ∈N
3am p am 3p

119
.
şi cum 23am p + 1 .. 23am + 1, deducem am+1 2am+1 + 1.
Mai trebuie arătat că numărul de divizori primi ai lui am+1 este m + 1.
Aceasta este echivalent cu a arăta că p¤|¤am . Dacă p | am , cum am 2am +1, atunci
2am ≡ −1 (mod p) şi 22am − 2am + 1 ≡ 3 (mod p). Cum p 22am − 2am + 1,
obţinem contradicţia p = 3.

18. Fie a ∈ N∗ , astfel ı̂ncât a2 ≤ n < (a + 1)2 . Atunci a ≤ n < a + 1
√ √
şi [ n] = a. Cum [ n] | n, deducem că n = ab, b ∈ N∗ . Din a2 ≤ n = ab <
a2 + 2a + 1 1
(a + 1)2 , rezultă că a ≤ b < = a + 2 + ≤ a + 3. Concluzia este
a 2
că b ∈ {a, a + 1, a + 2} şi deci soluţia problemei este formată din numerele a2 ,
a2 + a, a2 + 2a pentru orice a ∈ N∗ .

n+a n+b
19. Presupunem că + = c ∈ N. Fie d = (a, b), a = dα,
b a
b = dβ, (α, β) = 1. Ecuaţia noastră este echivalentă cu α(n + a) + β(n + b) =
d(cαβ − α2 − β 2 ) d2
cdαβ, n = = (cαβ − α2 − β 2 ), unde d1 = (d, α + β),
α+β s
d = d1 d2 , α + β = d1 s, (d2 , s) = 1. Cum n ∈ N şi (s, d2 ) = 1, deducem că
s|cαβ − (α2 + β 2 ) = (c + 2)αβ − (α + β)2 , s|(c + 2)αβ (deoarece s|α + β) şi
s|c + 2 ((s, αβ) = 1, deoarece s|α + β şi (α, β) = 1). Avem c + 2 = sk şi
d2 € Š € Š
n= (c + 2)αβ − (α + β)2 = d2 kαβ − d21 s = d2 kαβ − d2 d21 s =
s
= d2 kαβ − d(α + β) = d2 kαβ − a − b.

20. Ecuaţia dată este echivalentă cu

(2a − q)2 + (2b − q)2 = 2q 2 + 4r = 2(q 2 + r) + 2r = 2 · 1977 + 2r.



Însă 0 ≤ r ≤ 1977 şi deci (2a − q)2 + (2b − q)2 ≤ 4 · 1977, |2a − q| ≤ 4 · 1977 <
89, |2b − q| < 89. Deci |2a − q| ≤ 88, |2b − q| ≤ 88, |2a + 2b| ≤ 2 · 88 + 2q,
|a + b| ≤ 88 + q. Însă q 2 ≤ 1977, q ≤ 44 şi deci a + b ≤ 88 + 44 = 132,
0 ≤ r < a + b ≤ 132. Dacă 0 ≤ q ≤ 42, atunci r = 1977 − q 2 ≥ 1977 − 422 =
1977 − 1764 = 213 şi am obţinut o contradicţie.
Dacă q = 43, avem că r = 1977 − 1849 = 128. Cum 2a − 43 şi 2b − 43 sunt
impare, avem că |2a−43| ≤ 87, |2b−43| ≤ 87, |2a+2b| ≤ 2·87+2·43 = 2·130,
0 ≤ r = 128 < a + b ≤ 130, a + b ∈ {129, 130}. Dar |2a| ≤ 87 + 43 = 130,
|2b| ≤ 130, a, b ≤ 65. Deci (a, b) ∈ {(64, 65), (65, 64), (65, 65)}.
Dacă a = b = 65, avem că 2 · 652 = 65 · 130 + 0 şi r = 0 6= 128. Dacă
a = 64, b = 65, avem că 642 + 652 = 64 · 129 + 65, r = 65, contradicţie.

120
Deci q = 44 şi r = 1977 − 442 = 41. Avem (a − 22)2 + (b − 22)2 = 2 · 222 + 41 =
1009. Cum 1009 este prim, există m şi n numere naturale unice, astfel ı̂ncât
m2 + n2 = 1009. Se arată uşor că m = 28 şi n = 15 şi că b = 50 şi a ∈ {7, 37}.
Avem de asemenea a = 50 şi b ∈ {7, 37}.

21. Fie n = q1α1 · . . . · qrαr , q1 < q2 < · · · < qr , qj prim, αj ∈ N∗ , ∀ j = 1, r.


Vom arăta că numărul cătat este (1 + 2α1 )(1 + 2α2 ) . . . (1 + 2αr ).
Fie a, b o soluţie a ecuaţiei din enunţ. Avem ab = na+nb, ab−na−nb+n2 = n2 ,
1 1 1 1
(a − n)(b − n) = n2 = q12α1 . . . qr2αr . Avem că = + > , a > n, a − n > 0
n a b a
şi la fel se arată că b − n > 0. Deci a − n = q1β1 · q2β2 . . . qrβr , βj ≤ 2αj , ∀ j = 1, r
şi b − n = q12α1 −β1 · q22α2 −β2 . . . qr2αr −βr . De aici rezultă imediat că numărul de
soluţii este (1 + 2α1 )(1 + 2α2 ) . . . (1 + 2αr ) = τ (n2 ).

an+1 + 2n+1 + 1 .
22. Avem n n
− a ∈ Z ⇔ 2n (2 − a) + 1 − a .. an + 2n + 1.
a +2 +1
. .
Pentru a = 1 avem 2n .. 2n + 2. Pentru a = 2 avem −1 .. 2n+1 + 1. Pentru a ≥ 3
şi n ≥ 2 avem de asemenea o contradicţie,
 ‹  ‹n
1 a
0 < 2n (a − 2) + a − 1 < an + 2n + 1 ⇔ (a − 2) 1 + < + 1.
2n 2
 n  
a 1
Este suficient să arătăm că > (a − 2) 1 + n . Aceasta se arată imediat
2 2
2
a +5 14
prin inducţie după n ≥ 2. Deci n = 1, ∈ Z, ∈ Z, a = 4 sau
a+3 a+3
a = 11.
(L. Panaitopol )

23. Avem a = bq + r cu 0 ≤ r ≤ b − 1 şi


2a + 1 2bq+r + 1 2bq+r − 2r + 2r + 1
m= b
= b
= =
2 −1 2 −1 2b − 1
€ Š 2r + 1
= 2r (2b )q−1 + (2b )q−2 + · · · + (2b ) + 1 + b .
2 −1
2r + 1
Deoarece b > 2, avem + 1 ≤ 2r 2b−1 2b
+ 1 < − 1 şi deci 0 < b < 1, adică
2 −1
2a − 1 2bq+r − 1
m∈/ Z. Avem de asemenea n = b = . Pentru q impar
2 +1 2b + 1
2bq+r + 2r −(2r + 1) € Š 2r + 1
r b q−1 b q−2 b q−3
n= = 2 (2 ) −(2 ) + (2 ) − · · · + 1 −
2b + 1 2b + 1

121
2r + 1
şi n ∈
/ Z deoarece 0 < b < 1.
2 +1
Pentru q par avem
€ Šq ‹
2bq+r − 2r + 2r − 1 2r 22b 2
−1 2r − 1
n= = + .
2b + 1 2b +1 2b + 1
€ Šq . 2r − 1
Cum 22b 2
− 1 .. 2b + 1 şi 0 < b < 1, rezultă n ∈
/ Z.
2 +1
.
24. Pentru m = 1 proprietatea este adevărată. Dacă m ≥ 2, 2002m ..4 şi
cum 2003 = 4k + 3, rezultă că 2002m + 2003 = 4h + 3. Un număr de forma
4h + 3 are un factor prim de forma 4t + 3. Cum 2146 = 2 · 29 · 37, rezultă că
2146n nu are un factor prim de forma 4t + 3 şi deci enunţul se verifică.

25. Dacă a = 0 sau b = 0 se consideră A = B = 1. Dacă ab 6= 0, notăm


d = (a, b) şi avem a = dα, b = dβ cu (α, β) = 1. Există A, B ∈ Z astfel ı̂ncât:

αB − βA = 1. (1)

Arătăm că A, B ı̂ndeplinesc condiţiile cerute. Presupunem că există p prim


cu p |αdn + A şi p |βdn + B şi rezultă contradicţia

p | β(αdn + A) − α(βdn + B) = βA − αB = −1.

26. Fie λ = (a, b), a = λα, b = λβ, (α, β) = 1. Avem că λ(α + β)3 =
k(α2 + β 2 ). Deoarece (α, β) = 1, rezultă că

(α + β, α2 + β 2 ) = (α + β, 2αβ) = (α + β, 2) = 1, 2.

Să presupunem că (α+β, α2+β 2 ) = 1 (aceasta se ı̂ntâmplă dacă α 6≡ β (mod 2)).
Atunci ((α + β)3 , α2 + β 2 ) = 1, k = (α + β)3 m, λ = (α2 + β 2 )m, m ∈ N∗ şi
avem soluţiile a = mα(α2 + β 2 ), b = mβ(α2 + β 2 ). Dacă α şi β sunt ambele
impare, atunci (α + β, α2 + β 2 ) = 2, α + β = 2p, α2 + β 2 = 2q, (p, q) = 1,
q impar. Avem că 4λp3 = kq, p3 | k (deoarece (p, q) = 1), k = mp3 , m ∈ N∗ ,
α2 + β 2
4λ = mq, m = 4m1 (deoarece q este impar), λ = m1 q = m1 şi deci
2
αm1 (α2 + β 2 ) βm1 (α2 + β 2 )
a= ,b= . În primul caz k = m(α + β)3 , iar ı̂n
2 2
 
α+β 3 (α + β)3
cel de-al doilea caz k = 4m1 = m1 .
2 2

122
27. Rezultă imediat că a, b, c au aceeaşi factori primi, şi cum b = 23 · 32 ,
factorii primi sunt 2 şi 3. Fie a = 2α1 3α2 , c = 2ν1 · 3ν2 . Din am |bn rezultă:

mα1 ≤ 3n, (1)

mα2 ≤ 2n. (2)


Din bp | cm rezultă:
3p ≤ mν1 , (3)
2p ≤ mν2 , (4)
iar din cn | ap rezultă:
nν1 ≤ pα1 , (5)
nν2 ≤ pα2 . (6)
Din (1) şi (5) prin ı̂nmulţire avem mν1 ≤ 3p şi ţinând seama de (3), rezultă
3p 2p 3n 2n
mν1 = 3p şi deci ν1 = . Analog se arată că ν2 = , α1 = , α2 = .
m m m m
Cum αi , νi ∈ N, rezultă m|2p, m|3p ⇒ m|p şi m|2n, m|3n ⇒ m|n şi deci
p = km, n = hm. Avem deci a = 72h , c = 72k , cu k, h ∈ N∗ . Astfel, avem:
bn = k1 am , cm = k2 bp , ap = k3 cn şi deci bnp = k1p amp , amp = k3m cnm şi deci
bnp = k1p k3m cnm (∗). Din cm = k2 bp deducem cmn = k2n bpn (∗∗) şi ı̂nmulţind (∗)
cu (∗∗) avem k1p k2n k3m = 1 şi deci k1 = k2 = k3 = 1. Avem bn = am , cm = bp şi
n p
deci a = 72 m , c = 72 m , unde m|n şi m|p.
(L. Panaitopol )

28. Relaţia se scrie:

a2 − ac + c2 = b2 + bd + d2 . (1)

De aici avem

(ac + bd)(b2 + bd + d2 ) = ac(b2 + bd + d2 ) + bd(a2 − ac + c2 ) =


= (ab + cd)(ad + bc).

Dacă p = ab + cd şi p este prim, atunci:


1) p|ac + bd sau 2) p|b2 + bd + d2 .
1) Deoarece p = ab + cd, rezultă p|ac + bd − ab − cd = −(a − d)(b − c) şi deci
p|a − d sau p|b − c, adică p ≤ a − d sau p ≤ b − c. Dar p = ab + cd > a > a − d
şi p > a > b − c. Am obţinut o contradicţie.
2) Avem inegalităţile b2 + bd + d2 < 2ab + cd < 2p, adică b2 + bd + d2 = p
şi cum p = ab + cd, rezultă b2 + bd + d2 = ab + cd ⇔ b(b + d − a) = d(c − d).

123
Pentru că p este prim, rezultă (b, d) = 1 şi din egalitatea precedentă deducem
b|c − d. Deci b ≤ c − d < c < d şi am obţinut din nou o contradicţie.

5 √ x 5 √
29. Deoarece < 3 2, deducem că ∀ x ≥ 5 avem ≤ < 3 2.
4 x−1 4
Dacă numerele a, b, c satisfac condiţiile din enunţ şi a ≥ 5, atunci
√3
√ √
abc − 1 2 32 32 abc − 1
0< ≤ (abc − 1) · · =2 < 2.
(a − 1)(b − 1)(c − 1) a b c abc
abc − 1
Singura posibilitate este ca = 1. Această ultimă egali-
(a − 1)(b − 1)(c − 1)
tate conduce la egalitatea ab + ac + bc = a + b + c care este contradictorie
(deoarece a > 1, b > 1, c > 1). Deci a ∈ {2, 3, 4}. Dacă a = 2, deducem că
2bc − 1 2bc − 1
∈ N. Pentru b ≥ 5, avem că < 3 (ultima ine-
(b − 1)(c − 1) (b − 1)(c − 1)
galitate este echivalentă cu bc − 3b − 3c + 4 > 0, (b − 3)(c − 3) > 5; pentru
b ≥ 5, avem c ≥ 6 şi (b − 3)(c − 3) ≥ 2 · 3 = 6 > 5). Deci pentru b ≥ 5
2bc − 1
singura posibilitate pentru ca să fie număr natural este aceea
(b − 1)(c − 1)
2bc − 1
ca = 2 (valoarea 1 este exclusă, conform consideraţiilor prece-
(b − 1)(c − 1)
dente). Din ultima egalitate rezultă că −2b − 2c + 3 = 0, ceea ce este imposibil.
Deci b ∈ {3, 4} şi cum b este par (căci b − 1 divide numărul impar 2bc − 1),
8c − 1 8c − 1 7
deducem că b = 4. Avem că ∈ N, ∈ N, ∈ N, c = 8.
3(c − 1) c−1 c−1
3bc − 1
Dacă a = 3, se arată la fel ca mai sus că < 2 pentru
2(b − 1)(c − 1)
b ≥ 7. Avem b ∈ {4, 5, 6}. Cum b este impar (căci 2|3bc − 1), rezultă b = 5,
15c − 1 15c − 1 14
∈ Z, ∈ Z, ∈ Z, c = 15 (deoarece c este impar, c > 5).
8(c − 1) c−1 c−1
4bc − 1
Pentru a = 4 se arată la fel ca mai sus că < 2 pentru b ≥ 6.
3(b − 1)(c − 1)
4bc − 1
Deci ı̂n acest caz nu avem soluţii, căci nu poate fi 1 (conform
3(b − 1)(c − 1)
unei observaţii precedente).
Concluzia tuturor consideraţiilor precedente este că singurele soluţii ale
problemei sunt (a, b, c) = (2, 4, 8), (3, 5, 15).

30. Avem a1 = 2, a2 = 3, a3 = 7, a4 = 43. Rezultă că oricare ar fi n,


(a1 a2 . . . an+1, 6) = 1 pentru n ≥ 2 şi dacă an+1 = 5, atunci a1 a2 . . . an + 1 = 5k ,

124
ceea ce este ı̂n contradicţie cu a1 a2 . . . an 6= M4, deoarece a1 = 2 iar pentru
k ≥ 2, ak este impar.

31. Dacă există m ∈ N∗ , astfel ı̂ncât am ∈ {1, 2, 3, 4}, atunci deducem


imediat enunţul. Să presupunem că k = min{an | n ∈ N∗ } ≥ 5. Există i ∈ N∗
cu proprietatea că ai = k. Unul din numerele consecutive k + 1, k + 2, k + 3,
k+4 este divizibil cu 4. Fie j ∈ {1, 2, 3, 4} astfel ı̂ncât 4|k+j. Deducem imediat
k+j k+4
că ai+j ≤ ≤ < k şi obţinem o contradicţie cu definiţia numărului
2 2
k.
(Dorel Miheţ)

32. Avem a1 < 106 şi deci a2 < 106 . Presupunem că ak > 2, ∀ k ≤ 17.
ak + 1 106
Cum ak este impar (k ≥ 2), rezultă ak+1 ≤ şi deci a3 ≤ ,
2 2
10 6 106 1
+ 1 106 1 2
+ + 1 106 1 1
a4 ≤ 2 = 2 + şi a5 ≤ 2 2 = 3 + 2 + . Rezultă
2 2 2 2 2 2 2
14
106 X 1
a17 ≤ 15 + j
< 30, 52 + 1.
2 j=1
2
Aşadar a17 ≤ 31 şi deci a17 ∈ {2, 3, 5, 7, 11, 13, 17, 19, 23, 29, 31}. Rezultă
a18 ∈ {3, 2, 7, 5}, a19 ∈ {2, 3}. De aici obţinem că a19 + a20 = 5.
Dacă există k ≤ 17 astfel ı̂ncât ak = 2, atunci {a19 , a20 } = {2, 3} şi
a19 + a20 = 5, ceea ce constituie acelaşi rezultat ca mai sus.

33. Să presupunem că şirul (an )n≥1 ar fi mărginit. Fie q = max{an |n ≥ 2},
q fiind număr prim, conform definiţiei şirului an , q = pj (al j-lea număr prim).
Fie n = (p1 p2 . . . pj )2 . an+1 este cel mai mare divizor prim al numărului an +n.
Cum an+1 |n, deducem că an+1 |an , an+1 = an = pk , 1 ≤ k ≤ j. Deci pk este
cel mai mare divizor prim al numărului
€ Š
pk + p21 p22 . . . p2j = pk 1 + p21 . . . p2k−1 · pk · p2k+1 . . . p2j .
De aici deducem că numărul 1 + p21 . . . p2k−1 · pk · p2k+1 . . . p2j > 1 are toţi factorii
primi mai mici sau egali cu pk . Acest fapt ne conduce imediat la o contradicţie,
căci ps ¤|¤1 + p21 . . . p2k−1 · pk · p2k+1 . . . p2j , ∀ s ≤ k.
(L. Panaitopol )

34. Cu notaţiile obişnuite avem


(2a1 + r(n − 1))n
= a1 + rm ⇔ 2a1 (n − 1) = r(2m − n(n − 1)).
2

125
Alegem n−1 = 2kr şi avem 4a1 k = 2m−2kr(2kr −1), m = 2a1 k +kr(2kr −1).

35. Dacă a, x ∈ N∗ sunt astfel ı̂ncât (x + a)k − xk este un număr prim,


atunci a = 1. Considerăm funcţia F (x) = (x + 1)k − xk . Să presupunem că
F (x0 ) = q, unde q este număr prim pentru un x0 ∈ N∗ . Fie xn = x0 + nq
cu n ∈ N∗ . Avem F (xn ) ≡ (x0 + 1)k − xk0 = q ≡ 0 (mod q) şi F (xn ) >
F (x0 ) = q deoarece F este o funcţie crescătoare pentru x ≥ 0. F 0 (x) =
€ Š
k (x + 1)k−1 − xk−1 ≥ 0. Aşadar avem F (xn ) > q şi F (x) ≡ 0 (mod q),
adică F (xn ) nu este prim.

36. Fie a, a+r, a+2r, . . . , a+(n−1)r progresia. Pentru r = 0, enunţul este


trivial. Fie r ≥ 1. Arătăm că a ≥ n. Dacă cumva a ≤ n − 1, atunci a + ar =
a(1 + r) este termen al progresiei şi evident că nu este prim, contradicţie.
Aşadar a ≥ n, pentru p prim p ≤ n − 1, considerăm numerele a, a + r, a +
2r, . . . , a + (p − 1)r. Cum a ≥ n, rezultă că aceste numere sunt prime şi mai
mari ca p, aşa ı̂ncât la ı̂mpărţirea cu p, nici unul dintre ele nu dă restul 0.
Există deci două dintre aceste p numere care dau acelaşi rest la ı̂mpărţirea cu
p. Aşadar p|(a + hr) − (a + kr), unde 0 ≤ k < h ≤ p − 1. Cum p|r(h − k) şi
0 < h − k < p, rezultă p¤|¤(h − k) şi deci p|r.

37. Presupunem că a ∈ / N. Fie n natural astfel ı̂ncât na ∈


/ N, n ≥ 2 şi
1
deci {na} = na − [na] > 0. Fie b ∈ N∗ maxim, astfel ca {na} < . Rezultă
b
1 1 1 1
{na} ≥ şi deci ≤ {na} < ⇔ 1 ≤ (b + 1)na − (b + 1)[na] < 1 + .
b+1 b+1 b b
Pentru m = (b + 1)n se obţine contradicţia [ma] = (b + 1)[na] + 1 6= M[na].

38. În identitatea x3 +y 3 +z 3 −3xyz = (x+y+z)(x2 +y 2 +z 2 −xy−xz−yz)


punem x = 211 , y = −26 şi z = −1. Calculăm x3 + y 3 + z 3 − 3xyz şi obţinem

233 − 218 − 1 − 3 · 217 = 233 − 5 · 217 − 1 = 233 − 219 − 217 − 1,

adică exact numărul din enunţ. Din identitatea de mai sus deducem că un
factor al acestui număr este 211 − 26 − 1 = 1983 = 3 · 661. Se poate deduce uşor
ı̂n acest moment că descompunerea ı̂n factori primi a lui 233 − 219 − 217 − 1
este 33 · 13 · 661 · 37021.
n
39. = dk+1−i , ∀ i = 1, k şi
di
n n n n n n
D= · + · + ··· + · =
dk dk−1 dk−1 dk−2 d2 d1

126
 
1 1 1
= n2 + + ··· + ≤
d1 d2 d2 d3 dk−1 dk
   ‹
2 1 1 1 2 1
≤n + + ··· + =n 1− < n2 .
1·2 2·3 (k − 1)k k

Am folosit mai sus faptul că di ≥ i, ∀ i = 1, k. Pentru cea de-a doua parte
a problemei, să observăm că numerele prime n satisfac proprietatea indicată.
Într-adevăr, ı̂n acest caz, D = d1 d2 = n|n2 . Vom arăta că doar numerele prime
au această proprietate. Fie n un număr care nu este prim şi arătăm că D¤|¤n2 .
Să presupunem că D|n2 . Deci n2 = a · D, a ∈ N, a > 1 (deoarece am arătat
anterior că D < n2 ).
 
1 1 1 1
n2 = a · D = an2 + + ··· + > an2 ,
d1 d2 d2 d3 dk−1 dk d2 d2

deoarece k ≥ 3 (n nefiind prim). Din inegalităţile precedente rezultă 1 < a < d2 .


Fie p un număr prim care divide pe a. Din n2 = aD deducem că p|n. Însă
p ≤ a < d2 şi obţinem o contradicţie cu definiţia lui d2 , care este cel mai mic
divizor supraunitar al lui n.

40. a) Punem an = 2n bn şi avem b0 = a0 = 1. De asemenea 2n+1 bn+1 =


n
2 · 2n bn + n · 2n şi deci bn+1 = bn + . Rezultă
2

b1 = b0
1
b2 = b1 +
2
..
.
n−1
bn = bn−1 + .
2

n(n−1)
Adunăm aceste relaţii şi obţinem bn = 1+ şi deci an = 2n−2 (n2 −n+4).
4
Fie an = 2k ⇒ n2 − n + 4 = 2h . Discriminantul ecuaţiei trebuie să fie
pătrat şi deci D = 2h+2 − 15 = l2 . Pentru h impar,

h = 2α − 1 ⇒ D = 22α+1 − 15 = 2(4)α − 15 = 2(5 − 1)α − 15 = M5 ± 2 6= l2

deoarece:
dacă l = 5k, atunci l2 = M5,
dacă l = 5k ± 1, atunci l2 = M5 + 1,

127
dacă l = 5k ± 2, atunci l2 = M5 − 1.
Dacă h este par, h = 2α. D = l2 ⇔ 4α+1 − l2 = 15,
( (
€ Š€ Š 2α+1 − l = 1 2α+1 − l = 3
α+1 α+1
2 −l 2 + l = 15 ⇒ sau
2α+1 + l = 15 2α+1 + l = 5.

Avem 2α+2 = 16 sau 2α+2 = 8 şi deci α = 2 sau α = 1. Rezultă h = 4 sau


h = 2, adică n2 − n + 4 = 16, n = 4 sau n2 − n + 4 = 4 şi deci n = 0 sau n = 1.

b) Dacă n = 2α+1, an = k 2 ⇔ 22α−1 (4α2 + 2α + 4) = k 2 ⇔ 4α (2α2 + α + 2) =


k 2 ⇔ 2α2 + α + 2 = k 2 . Discriminantul ecuaţiei trebuie să fie pătrat şi deci
D = 1 − 16 + 8k 2 = 8k 2 − 15 = h2 . Raţionând modulo 3, deducem că −k 2 = h2
(mod 3) ⇒ 3|k 2 + h2 ⇒ 3|k şi 3|h ⇒ 8k 2 − h2 = M9 6= 15. Dacă n = 2α,
atunci 4α−1 (4α2 − 2α + 4) = k 2 ⇔ 4α2 − 2α + 4 = y 2 . Pentru α = 0, n = 0;
pentru α = 1 nu sunt soluţii, iar pentru α = 2 avem n = 4, a4 = 64 = 82 .
Dacă α ≥ 3, atunci (2α − 1)2 < 4α2 − 2α + 4 < (2α)2 şi deci 4α2 − 2α + 4 6= y 2 .

41. Arătăm că numerele n = 2k au proprietatea din enunţ. Pentru aceasta


vom arăta prin inducţie că:
k.
1) a2 + b2 + c2 ..(a + b + c);
k k

” —
k .
2) 2 (ab)2 + (bc)2 + (ac)2 ..(a + b + c).
k k

Facem verificarea pentru k = 0. Afirmaţia 1) este evidentă.


a2 + b2 + c2 2ab + 2ac + 2bc 2(ab + ac + bc)
Din = a+b+c− ∈ Z, deducem că
a+b+c a+b+c a+b+c
este ı̂ntreg, adică afirmaţia 2) pentru k = 0.
Să presupunem că afirmaţiile 1) şi 2) sunt adevărate pentru k şi le demon-
străm pentru k + 1. Avem
€ k k k
Š2
a2
k+1 k+1
+ b2 + c2
k+1
a2 + b2 + c2 k k
(ab)2 + (bc)2 + (ac)2
k

= −2 ∈Z
a+b+c a+b+c a+b+c
ţinând cont de ipoteza de inducţie. Avem
k k k (a + b + c)m
(ab)2 + (bc)2 + (ac)2 = ∈ Z, ; m ∈ N
2
şi
” k k k
—2
(ab)2
k+1
+ (bc)2
k+1
+ (ac)2
k+1
(ab)2 + (bc)2 + (ac)2
2 =2 −
a+b+c a+b+c
€ k k k
Š
2k
a2 + b2 + c2 (a + b + c)m2 ka
2 + b2 + c2 k k k

− 4(abc) = − 4(abc)2 .
a+b+c 2 a+b+c

128
m(a + b + c) (a + b + c)m2
Din ∈ Z deducem că ∈ Z şi din formula precedentă
2 k k k
2
2 2
a +b +c 2
(ţinând cont că ∈ Z), rezultă afirmaţia 2) pentru k + 1.
a+b+c
(L. Panaitopol )

42. xn se divide cu cel puţin unul dintre numerele 3, 5, 7, 13, 19, 37, 73 şi
. . .
anume: dacă n par, xn .. 3; dacă n = 4h + 1, xn .. 5; dacă n = 12h + 7, xn .. 7;
.
dacă n = 12h + 11, xn .. 13. Pentru cazul n = 12h + 3 se consideră subcazurile:
.
a) n = 36l + 3 pentru care xn .. 73;
.
b) n = 36l + 15 pentru care xn .. 19;
.
c) n = 36l + 27 pentru care x .. 37. n

43. Ecuaţia caracteristică a şirului este xk −x−1 = 0 cu rădăcinile simple


x1 , . . . , xk . Avem an = α1 xn1 + · · · + αk xnk , ∀ n ∈ N∗ , unde α1 , α2 , . . . , αk sunt
rădăcinile sistemului:
8
>
> α1 x1 + α2 x2 + · · · + αk xk = 0
>
> ..
>
< .
k−2
α1 x1 + · · · + αk xk = 0 k−2 (*)
>
>
> α k−1
+ · · · + αk xk−1
>
> 1 x1 k =k−1
: k k
α1 x1 + · · · + αk xk = k
Determinantul Vandermonde al sistemului este nenul deoarece rădăcinile
x1 , . . . , xk sunt distincte şi nenule. Soluţia sistemului este deci unică. Arătăm
k
X j
că αj = 1, ∀ j = 1, k. Notăm Sj = xi , j = 1, k şi cu sj polinomul simetric
i=1
fundamental de ordin j ı̂n x1 , . . . , xk . Avem formulele lui Newton:
S1 = s1
S2 = s21 − 2s2
S3 = s31 − 3s1 s2 + 3s3
S4 = s41 − 4s21 s2 + 4s1 s3 + 2s22 − 4s4
..
.
De aici deducem că S1 = S2 = · · · = Sk−2 = 0 (folosim faptul că s1 = s2 =
· · · = sk−2 = 0). Sk = S1 + k = k,
k
X 1 sk−1
Sk−1 = k + =k+ = k − 1.
i=1
xi sk

129
Din toate consideraţiile precedente, rezultă că α1 = α2 = · · · = αk = 1 este
soluţia sistemului (*).
Fie p prim. Avem
ap = xp1 +xp2 +· · ·+xpk = (x1 +x2 +· · ·+xk )p −pf (x1 , x2 , . . . , xp ) = −pf (x1 , . . . , xk ),
unde
X (p − 1)!
f (x1 , . . . , xk ) = xn1 . . . xnk k
n1 +n2 +···+nk =p
n1 !n2 ! . . . nk ! 1
0≤nj <p

este polinom simetric ı̂n x1 , . . . , xk cu coeficienţi ı̂ntregi. Din teorema fun-


damentală a polinoamelor simetrice, rezultă că f (x1 , . . . , xk ) = g(s1 , . . . , sk ),
unde g este un polinom cu coeficienţi ı̂ntregi. Cum s1 = s2 = · · · = sk−2 = 0,
sk−1 = k−1, sk = k, deducem că f (x1 , . . . , xk ) ∈ Z. Deci ap = −pf (x1 , . . . , xk )
este multiplu de p.

130
CAPITOLUL 4

Partea ı̂ntreagă. Funcţia lui Legendre

{x} [x] + {x} 3


1. [x] 6= 0, deci x ∈
/ [0, 1). Avem + = şi deci
[x] + {x} [x] 2
„ Ž2 „ Ž √
{x} {x} − 3 ± 17
2 +3 − 1 = 0, {x} = [x]. Pentru x ≥ 1, {x} =
[x] [x] 4
√ 8 √ √ € √ Š9
17−3 < 1 + 17 1 + 17 3 1 + 17 =
[x] < 1 şi deci [x] ∈ {1, 2, 3} şi x ∈ , , .
4 : 4 2 4 ;

3 + 17 −4
Dacă x < 0, atunci {x} = − [x] < 1 şi deci [x] > √ =
√ 4 17 + 3
3 − 17
> −1, contradicţie.
2

3
2. Funcţia f : R → R, f [x] = [x]3 + {x}, este injectivă. x0 = este
2
soluţie a problemei şi observaţia precedentă ne asigură că este singura soluţie
a ecuaţiei din enunţ.
”√ —
3. Evident că x ∈ Z, x = 6k+1, k ∈ Z, k = 3 6k + 1 . Pentru k ≥ 3 avem
”√ — √ √
că k = 3 6k + 1 ≤ 3 6k + 1 < 3 7k < k (deoarece k 2 > 7 pentru k ≥ 3).
Deci nu există soluţii cu k ≥ 3. Deoarece k(48 − k 2 ) > 0, ∀ k ∈ Z, k ≤ −7,
√ k
deducem că 3 6k > pentru k ≤ −7. Obţinem, pentru k ≤ −7, contradicţia
2
”√ — √ √ k
k = 3 6k + 1 > 3 6k + 1 − 1 > 3 6k − 1 > − 1 > k. Deci soluţiile k trebuie
2
cătate ı̂n mulţimea {−6, −5, . . . , 2}. O verificare simplă arată că soluţiile sunt
k = −3, 1, 2 şi deci x = −17, 7, 13.
√ √ √ √
4. Rezultă x > 1 şi deci x > 3 x. Deducem că [ 3 x] = 1 şi [ x] = 2 şi
√ √
deci 3 x < 2 şi 3 > x ≥ 2. Rezultă că x < 8 şi 9 > x ≥ 4, adică x ∈ [4, 8).

5. Fie f (x) = ax2 + bx + c, a, b, c ∈ Z, a 6= 0, un polinom cu proprietatea


” √ — ” √ — ” √ —
din enunţ. Deducem b 2 = −2a − c, b 5 = −5a − c, b 7 = −7a − c

131
” √ —
” √ —
” √ — ” √ — ” √ — ” √ — 3
b 2 − b 5 ” √ —
şi 3a = b 2 − b 5 , 2a = b 5 − b 7 , = ” √ — ” √ — , 5 b 5 =
2 b 5 − b 7
” √ — ” √ —
2 b 2 + 3 b 7 . Din ultima egalitate rezultă, ţinând cont de inegalităţile
x ≥ [x] > x − 1, că
€ √ Š ” √ — ” √ — ” √ — √ √
5 b 5 − 1 < 5 b 5 = 2 b 2 + 3 b 7 ≤ 2b 2 + 3b 7

şi
√ ” √ — ” √ — ” √ — € √ Š € √ Š
5b 5 ≥ 5 b 5 = 2 b 2 + 3 b 7 > 2 b 2 − 1 + 3 b 7 − 1 .
€ √ √ √ Š
Din inegalităţile de mai sus deducem că −5 < b 5 5 − 2 2 − 3 7 < 5.
Cum
√ √ √ 2
5 5 − 2 2 − 3 7 > 11, 18 − 2, 83 − 7, 94 = 11, 18 − 10, 77 = 0, 41 > ,
5
5 25
rezultă că |b| < = , |b| ≤ 12. Făcând verificările, rezultă că singurele
2 2
5
soluţii sunt (a, b, c) = (−1, 4, 3), (−2, 8, −7), (1, −4, 4), (2, −8, 8).
(L. Panaitopol )
2 3 2 3
1 [x] x
6. Pentru a = este cunoscută egalitatea 4 5 = 4 5 (curs pagina 31).
n n n
    
1 1 1
Pentru a > 0 punem x = şi avem a = 1 şi deci a ≥ 1, adică
a a a
     
1 1 1 1 1 1 1
≥ . Cum ≤ , rezultă = şi = n ∈ N∗ .
a a a a a a a
1
În cazul a < 0 procedăm astfel: ∃ n ∈ N∗ astfel ı̂ncât a < − (deoarece
n
 
1 1 1
− % 0). Alegem x = − şi avem că [xa] = − = −1. Pe de altă parte,
n na n
1 1
avem că 0 < x = − < 1 (deoarece a < − ) şi deci [x] = 0. Avem că
na n
[a[x]] = [a · 0] = 0 şi am obţinut o contradicţie, căci [a[x]] = 0 şi [ax] = −1.
€ √ Šn √ € √ Šn √
7. 2 + 3 = an +bn 3, 2 − 3 = an −bn 3, an , bn ∈ N, a2n −3b2n = 1
€ √ Š€ √ Š
(deoarece 2 + 3 2 − 3 = 1).
€ √ Šn € √ Šn
2+ 3 + 2 − 3 = 2an .

132
€ √ Šn
Cum 0 < 2 − 3 < 1, rezultă că x = 2an − 1 este impar.

(x − 1)(x + 3) (an − 1)(an + 1) a2 − 1


= = n = b2n .
12 3 3
8√
( 1± ∆>
x2
−x=k <
8. . Din prima egalitate rezultă că x= .
n
x −x=h > 2
: ∆ = 1 + 4k

Avem k ∈ N şi deci ∆ = 1 + 4k ≥ 1. Notăm ε = ±1 şi avem egalitatea


‚ √ Œn √
1+ε ∆ 1+ε ∆
− = h.
2 2
√ € Š
Dacă ∆ ∈ R − Q, atunci ε Cn1 + Cn3 ∆ + Cn5 ∆2 . . . = 2n−1 ε. Rezultă că

Cn1 + Cn3 ∆ + Cn3 ∆2 · · · = 2n−1 = Cn1 + Cn3 + Cn5 . . .



şi deci Cn3 (∆ − 1) + Cn5 (∆2 − 1) · · · = 0. Dar ∆ ≥ 1 şi deci ∆ = 1, ∆ = 1 ∈ Q,
√ p
ceea ce constituie o contradicţie. Deoarece ∆ ∈ Q, rezultă că x ∈ Q, x = ,
q
(p, q) = 1, q ∈ N∗ , p ∈ Z. Avem p2 −pq−kq 2 = 0 şi deci q | p2 , q = 1, x = p ∈ Z.
(L. Panaitopol )
" # !
n−1
X k 1
9. Fie funcţia f : R → R, f (x) = x+ −[nx]. Deoarece f x+ =
k=0
n n
" ! " #
1 1 k
f (x), este suficient că studiem pe f pe 0, . Pentru 0 ≤ x < , x+ = 0,
n n n
[nx] = 0 şi deci f (x) = 0.
m
Consecinţă: pentru x = rezultă
n
• ˜ • ˜ • ˜
m m+1 m+n−1
+ + ··· + = m.
n n n

10. Fie ı̂n plan punctele A(n, 0), B(n, m), C(0, m). În interiorul drep-
tunghiului OABC se află (m−1)(n−1) puncte de coordonate ı̂ntregi. Deoarece
(m, n) = 1, pe diagonala OB nu se află astfel de puncte. Sub diagonală se află
(m − 1)(n − 1)
puncte de forma (k, h) cu h, k ∈ N∗ . Pentru k fixat, există
2
  n−1  
km X km
puncte. În total, sub diagonala OB sunt puncte.
n k=0
n

133
8 9
<    
n−1 m−1 =
11. Fie M = km − hn k ∈ 1, 2, . . . , , h ∈ 1, 2, . . . , .
: 2 2 ;

m−1 n−1
Elementele lui M sunt distincte, nenule şi |M | = · .
2 2
8
Numărul elementelor 9
pozitive (km > hn) se obţine fixând k şi găsind
" # " # n−1 " #
< km = km X2 km
h ∈ 1, 2, . . . , , adică elemente pentru k fixat. În total
: n ; n n
k=1
m−1 " #
X2 hn
elemente pozitive. Analog se obţin elemente negative.
h=1
m
n ” √ —
X n2 ¦ √ ©
12. Se arată prin inducţie că Sn = k 2 > √ . Dacă n 2 = α,
k=1 2
" #
1 ” √ Š n2 ” √ — (n + 1)2
pentru α ∈ 0, 1 − √ ∪ 2 − 2, 1 se arată că √ + (n + 1) 2 ≥ √ .
2 2 2
!
1 1
Pentru α ∈ 1 − √ , , se arată că
2 2
(n − 1)2 ” √ — ” √ — (n + 1)2
√ + n 2 + (n + 1) 2 ≥ √ .
2 2
" !
1 √
Pentru α ∈ , 2 − 2 , avem
2
(n − 2)2 ” √ — ” √ — ” √ — (n + 1)2
√ + (n − 1) 2 + n 2 + (n + 1) 2 > √ .
2 2
(L. Panaitopol )

13. Este suficient să considerăm x ∈ [0, 1). Modificările numerelor [kx],
p
k ≤ n, se produc pentru x = , (p, q) = 1, 1 ≤ p ≤ q − 1, 2 ≤ q ≤ n. Avem
q
kp = qak + bk , 0 ≤ bk ≤ q − 1 şi rămâne de arătat că
a2 an
an ≥ a1 + + ··· + , (1)
2 n
b1 , b2 , . . . , bq−1 sunt distincte şi deci {b1 , b2 , . . . , bq−1 } = {1, Ì
2, . . . , q − 1}.
!
1 b1 b2 bq−1 q−1 b1 b2 . . . bq−1
Din inegalitatea mediilor + +···+ ≥ =1
q−1 1 2 q−1 (q − 1)!
şi deci
b1 b2 bq−1
+ + ··· + ≥ q − 1. (2)
1 2 q−1

134
kp − bk
Înlocuind ak = , relaţia (1) devine
q
n
X bk
≥ bn . (3)
k=1
k
Ţinând seama de (2) şi de bn ≤ q − 1, n > q − 1, relaţia (3) este adevărată.

14. Fie an+1 = an + r şi [an+1 ] = [an ] + r0 . Rezultă an+1 − [an+1 ] =


an −[an ]+(r−r0 ), adică {an+1 } = {an }+(r−r0 ) şi deci {an+1 } = a1 +(r−r0 )n.
Dacă r = r0 , rezultă r = [an+1 ]−[an ] ∈ Z. Dacă r 6= r0 , atunci şirul a1 +(r−r0 )n
este nemărginit şi se obţine o contradicţie, căci a1 +(r −r0 )n = {an+1 } ∈ [0, 1),
∀ n ∈ N. Reciproc, dacă r ∈ Z, avem an+1 = an + r şi deci [an+1 ] = [an + r] =
[an ] + r şi deci ([an ])n≥1 este o progresie aritmetică.
(L. Panaitopol )

15. Dacă an ∈ Z, rezultă că [an ] = an şi deci şirul ([an ])n≥1 este progresie
geometrică. Dacă ([an ])n≥1 este progresie geometrică, avem an+1 = an q şi
[an+1 ] an q − {an q}
[an+1 ] = [an ]q 0 . Deoarece [an ] 6= 0, avem = = q 0 şi deci
[an ] [an ]
q[an ] + q{an } − {an q} q{an } − {an q}
= q 0 , deci = q 0 − q.
[an ] [an ]
Cum (q{an } − {an q})n≥1 este mărginit şi lim [an ] = +∞, rezultă q = q 0 .
n→∞
Din an+1 = an q şi [an+1 ] = [an ]q, prin scădere rezultă {an+1 } = {an }q şi
deci {a2n } = {an }q n . Cum {a2n } ∈ [0, 1) şi lim q n = ∞, rezultă {an } = 0,
n→∞
∀ n ≥ 1, adică an ∈ Z.
(L. Panaitopol )

16. Fie x = k + α cu k ∈ N şi α ∈ [0, 1) şi g(α) = k k+α − (k + α)k cu


α ∈ [0, 1). Pentru k = 0 avem
g(α) = −1. (1)
Pentru k = 1 avem
g(α) = 1 − 1 − α = −α ∈ (−1, 0]. (2)
Pentru k = 2 arătăm că g(α) ≥ −1, adică 22+α ≥ α2 + 4α + 3,
(α + 2) ln 2 ≥ ln(α2 + 4α + 3). Pentru h(α) = (α + 2) ln 2 − ln(α2 + 4α + 3),
2α + 4 2α + 4 3
avem h0 (α) = ln 2 − 2 < 0, deoarece 2 > > ln 2.
α + 4α + 3 α + 4α + 3 4
Deoarece h : [0, 1) → R este decrescătoare, avem h(α) ≥ h(1) = 0 şi deci
22+α ≥ α2 + 4α + 3, adică g(α) ≥ −1.

135
k+α k
Pentru k ≥ 3 avem g(α) > 0 ⇔ k k+α > (k + α)k ⇔ ≥ , ceea
ln(k + α) ln k
x
ce este adevărat, deoarece pentru x ≥ e, y(x) = este crescătoare. Rezultă
ln x
că f (x) ≥ −1.
Pentru k fixat, k ≥ 3, avem g(0) = k k − k k = 0 şi
‚  ‹k Œ
k+1 k k 1 kk
lim g(α) = k − (k + 1) = k k− 1+ > ,
α→1
α<1
k 5
!k
1 14
deoarece 1+ <e< .
k 5
kk
Fie M > 0. Pentru k ≥ max{3, M } avem > M şi lim g(α) > M . Deci
5 α→1
α<1
[0, M ] ⊂ Im (g). Aşadar Im f = {−1} ∪ (−1, 0] ∪ [0, ∞) = [−1, ∞).
(L. Panaitopol )
€√ Šm €√ Šm
2+1 + 2−1
17. Considerăm ecuaţia 2x2 = y 2 +1. Fie xm = √ ,
€√ Šm €√ Šm 2 2
2+1 − 2−1
ym = , m impar. Rezultă că xm , ym ∈ N şi 2x2m =
2
2 + 1. [Motivaţia alegerii numerelor x şi y rezultă din rezolvarea ecuaţiei
ym m m
non Pell (curs, pagina 134)]. Avem 2k xm = k 2 ym
2 2 2 + k 2 < (ky + 1)2 pentru
m
2
k −1 √
ym > (∗) şi cum 2k 2 x2m > k 2 ym
2 , rezultă ky < kx
m m 2 < kym + 1 şi
” 2k√ —
deci kxm 2 = kym dacă condiţia (∗) este ı̂ndeplinită. Enunţul rezultă acum
imediat, ţinând cont că lim ym = +∞.
m→∞
(L. Panaitopol )

18. Fie xm şi ym numerele construite ı̂n problema precedentă. Din


€ Š
2x2m = ym 2 + 1 rezultă că 2x2 y 2 = y 4 + y 2 < y 2 + 1 2 . Avem că
m m m m m

4
€ Š2 √
ym < 2x2m ym 2
< ym 2
+ 1 , ym 2
< xm ym 2 < ym 2
+1
” √ —
şi deci xm ym 2 = ym 2 .

(L. Panaitopol )

19. Să presupunem că A ∩ B = ∅ şi A ∪ B = N∗ . Notăm An = A ∩ [1, n],


Bn = B ∩ [1, n] pentru ∀ n ∈ N∗ . |An | = k ∈ N, unde [αk] ≤ n < [α(k + 1)].
Din aceste inegalităţi deducem că αk − 1 < [αk] ≤ n < [α(k + 1)] ≤ α(k + 1)
n+1 1
şi că k < < k + 1 + ≤ k + 2 (faptul că α ≥ 1 şi β ≥ 1 rezultă imediat
α α

136
" #
n+1
din faptul că 0 6∈ A ∪ B). Din inegalităţile precedente, rezultă că este
α
" #
n+1
egal cu k sau k + 1 şi deci |An | = k = − αn , unde αn ∈ {0, 1}.
α
" #
n+1
Analog se arată că |Bn | = − βn , unde βn ∈ {0, 1}.
β

 
1 1
n = |An ∪ Bn | = |An | + |Bn | = (n + 1) + − γn − δn − αn − βn ,
α β
( ) ( )
n+1 n+1
unde γn = şi δn = . Împărţind ultima egalitate la n şi
α β
1 1
făcându-l pe n să tindă la infinit, obţinem că + = 1.
α β
1 1
Să arătăm că α 6∈ Q. Dacă cumva α ∈ Q, atunci din + = 1, deducem
α β
m k
că β ∈ Q. α = , β = , k, m, n ∈ N∗ , mk = [nkα] = [nmβ] şi se contrazice
n n
1 1
faptul că A ∩ B = ∅. Condiţiile necesare sunt α, β 6∈ Q şi + = 1. Aceste
α β
condiţii sunt şi suficiente. Să presupunem că α, β satisfac condiţiile de mai
n+1
sus. |An | este cel mai mare k, pentru care kα < n + 1. Deoarece 6∈ N
" # " # α
n+1 n+1
(α 6∈ Q), rezultă că |An | = . Analog |Bn | = .
α β
" # " #
n+1 n+1 n+1 n+1
Avem că = + ε, = + η, unde ε, η ∈ (0, 1) (ε 6= 0 şi
α α β β
η 6= 0 deoarece α 6∈ Q, β 6∈ Q). Adunând cele două egalităţi precedente, rezultă
" # " #
1 1 n+1 n+1
(deoarece + = 1), că n + 1 = + + ε + η. De aici rezultă că
α β α β
ε + η ∈ Z şi deci ε + η = 1 (deoarece 0 < ε + η < 2). Această egalitate ne arată
" # " #
n+1 n+1
că n = + şi deci |An | + |Bn | = n, ∀ n ∈ N∗ . De aici rezultă că
α β
A ∩ B = ∅ şi că A ∪ B = N∗ . Dacă ∃ n ∈ N∗ , astfel ı̂ncât n = [kα] = [lb], atunci
n = |An | + |Bn | = 2 + |An−1 | + |Bn−1 | = 2 + (n − 1) = n + 1, contradicţie.
Deci An ∩ Bn = ∅ şi |An ∪ Bn | = n = |{1, 2, . . . , n}|. De aici, A ∩ B = ∅ şi
A ∪ B = N∗ .

137
20. Conform problemei 19, rezultă că termenii lui an se scriu [βm] cu
1 1 √ ”€ √ Š — ” √ —
√ + = 1 şi deci β = 2 + 2. Avem an = 2 + 2 n = 2n + n 2 ,
2 β
n ∈ N∗ .
21. Să presupunem că există α, β, γ cu proprietăţile din enunţ. Nu putem
avea două dintre ele egale şi atunci, după o eventuală renumerotare, pot pre-
supune că 0 ≤ α < β < γ. Deoarece 0 ∈ / S(α) ∪ S(β) ∪ S(γ), rezultă că α ≥ 1.
Deoarece 1 ∈ S(α) ∪ S(β) ∪ S(γ), neapărat α < 2. Să observăm că distanţa
dintre două numere consecutive din S(α) este cel mult 2. Într-adevăr
[(k + 1)α] − [kα] < (k + 1)α − (kα − 1) = α + 1 < 3.
β β m
Nu putem avea ∈ Q, căci ı̂n acest caz am avea m, n ∈ N∗ astfel ı̂ncât = ,
γ γ n
β
nβ = mγ, [nβ] = [mγ] şi aceasta contrazice S(β) ∩ S(γ) = ∅. Deci ∈ / Q şi
γ
 
β
atunci mulţimea m + n | m, n ∈ Z este densă ı̂n R. Există deci m, n ∈ Z
γ
β 1
astfel ı̂ncât 0 < m + n < , 0 < |mβ + nγ| < 1. Deoarece β > 1, γ > 1,
γ γ
rezultă că m 6= 0 şi n 6= 0.
Să presupunem că m > 0 şi n < 0 (celălalt caz posibil, m < 0 şi n > 0,
se tratează analog). mβ = [mβ] + ε, −nγ = [−nγ] + η, ε, η ∈ [0, 1]. mβ +
nγ = [mβ] − [−nγ] + ε − η şi inegalitatea 0 < |mβ + nγ| < 1 ne asigură că
[mβ] − [−nγ] ∈ {0, ±1}. Valoarea 0 nu este posibilă deoarece S(β) ∩ S(γ) = ∅.
Deci [mβ] − [−nγ] ∈ {±1} şi rezultă că [mβ] şi [−nγ] sunt două numere
consecutive care nu sunt ı̂n S(α), deoarece S(α) ∩ (S(β) ∪ S(γ)) = ∅. Însă,
conform unei observaţii demonstrate anterior, din două numere consecutive,
cel puţin unul este ı̂n S(α) (deoarece 1 ≤ α < 2). Am obţinut o contradicţie
şi problema este rezolvată.
22. Trebuie calculat e5 (1000) şi e5 (19783). Pentru M1 rezultatul este 249
iar pentru M2 rezultatul este 4943.
" # " #
n n n n n
23. e5 (n) = 500 ⇔ 500 = + 2 + · · · < + 2 + 3 + . . . şi deci
5 5 5 5 5
n
5 n
500 < = adică n > 2000.
1 4
1−
5

138
" #

X n
Pentru n ≥ 2010 avem că ≥ 501. Convin numerele
k=1
5k
2005, 2006, 2007, 2008, 2009.
n şi n = 2m a, cu
24. Fie i exponentul lui 2 din descompunerea lui C2n
(a, 2) = 1. Avem
∞ • ˜ • ˜‹ ∞ ‚– m+1 ™ • m ˜Œ
X 2n n X 2 a 2 a
i = e2 (2n) − 2e2 (n) = −2 k = −2 =
k=1
2k 2 k=1
2k 2k
• ˜ • ˜
a a
= 2m a+2m−1 a+ · · · +a+
+ 2 + · · · − 2(2m−1 a+2m−2 a+ · · · + a)−
2 2
• ˜ • ˜ ‹ • ˜ • ˜ ‹
a a a a
−2 + 2 + ... = a − + 2 + ... .
2 2 2 2
" # " #
a a−1
Pentru că a este impar, rezultă că k = . Pentru a = 1 avem i = 1.
2 2k
Pentru a ≥ 3 notăm h = [log2 (a − 1)] şi avem
∞ •
X ˜ h •
X ˜ h
X ∞
X
a−1 a−1 a−1 a−1
= ≤ < =a−1
k=1
2k k=1
2k k=1
2k k=1
2k
.
n .. 4 ⇔ a ≥ 3. Avem că
şi deci i > a − (a − 1) = 1, adică i ≥ 2 şi deci C2n

C n ..4
2n ¦ ⇔ a = 1, adică n = 2m .
(L. Panaitopol )

25. Fie k ∈ N astfel ı̂ncât 2k ≤ n < 2k+1 . e2 (n), exponentul lui 2 ı̂n n! este
       
n n n n n
egal cu + 2 + · · · + k . Pentru fiecare i = 1, k avem că i = i − αi ,
2 2 2 2 2
αi ∈ [0, 1). Dacă cumva 2n |n!, atunci e2 (n) = n + β, β ≥ 0.
k n k   k
X X 1 X
Ar rezulta deci că − α i = n + β = n 1 − − αi . Obţinem
i=1
2i i=1 2k i=1
n Xk
imediat contradicţia + β + αi = 0 (toate numerele din membrul stâng
2k i=1
n
sunt pozitive şi ≥ 1).
2k
Fie n ∈ N astfel ı̂ncât 2n−1 |n!. Folosind notaţiile precedente, rezultă că

  k k
1 X n X
e2 (n) = n 1 − k − αi = n − 1 + β, β ≥ 0. Deci 1 = k + β + αi .
2 i=1
2 i=1
n
Cum β ≥ 0, αi ∈ [0, 1), ∀ i = 1, k şi ≥ 1, rezultă din egalitatea precedentă
2k

139
că n = 2k , β = 0, αi = 0, ∀ i = 1, k. Se verifică imediat că n = 2k este soluţie
a problemei noastre.

26. Fie p număr prim, p|n şi n = pα k, m = pβ h, (kh, p) = 1. Arătăm


(m, n) m
că exponentul lui p din scrierea lui Cn este pozitiv. Dacă α ≤ β, acest
n
lucru este evident. Putem presupune că β < α. Exponentul de mai sus este
∞      
X n m n−m
xp = β − α + i
− i − =
i=1
p p pi
 α  – β ™ " β € α−⠊ #!

X p p k−h
p k p h
=β−α+ − − .
i=1
pi pi pi

pα k
Pentru i ∈ 1, β parantezele sunt 0. Dacă β + 1 ≤ i ≤ α, i
∈ N şi deci
€ Š p
 α   β   β α−β
p k pα k p h pβ h p p k−h  pα k − hpβ
= . Avem ı̂nsă < şi < şi
pi pi pi pi pi pi
deci parantezele sunt cel puţin 1. Cum ı̂n intervalul [β + 1, α] există α − β
indici i, rezultă xp ≥ β − α + α − β = 0.
(L. Panaitopol )

27. Pentru calcularea lui N1 considerăm p număr prim. Trebuie arătat


că ep (2m) + ep (2n) − ep (m + n) − ep (m) − ep (n) ≥ 0, adică
         
X 2m 2n m+n m n
i
+ i − i
− i − i ≥ 0.
i≥1
p p p p p

m n
Se notează x = i
şi y = i şi se arată că [2x] + [2y] − [x + y] − [x] − [y] ≥ 0.
p p
Celelalte exerciţii se rezolvă analog.

28. Fie p un divizor prim al lui n care apare la exponent α ı̂n descom-
punerea lui n. Trebuie să arătăm că ep (n) ≥ 3α. Analizăm ı̂ntâi cazul n = pα .
Dacă p ≥ 5, atunci
5α − 1
ep (n) = pα−1 + pα−2 + · · · + p + 1 ≥ 5α−1 + · · · + 5 + 1 = ≥ 3α.
4
Ultima inegalitate este echivalentă cu 5α ≥ 12α+1, care este adevărată pentru
orice α ≥ 2 (se arată prin inducţie); α este mai mare sau egal cu 2 deoarece n
este compus.
3α − 1
Dacă p = 3, atunci α ≥ 3 şi e3 (n) = 3α−1 + · · · + 3 + 1 = ≥ 3α (se
2
arată prin inducţie că 3 ≥ 6α + 1, ∀ α ≥ 3). Analizăm acum cazul n = e · pα ,
α

140
e ≥ 3, p¤|¤e. Avem că
€ Š € Š 3α − 1
ep (n) ≥ e pα−1 + pα−2 + · · · +p+1 ≥ 3 3α−1 + · · · +3+1 = 3 ≥ 3α
2
(ultima inegalitate este echivalentă cu 3α ≥ 2α + 1; se arată imediat prin
inducţie că este adevărată pentru α ≥ 1). Enunţul este demonstrat.
(L. Panaitopol )

k
Y
29. Fie E1 = [1, 2, . . . , n] = pαi i , unde αi este exponentul maxim
i=1
  ” — k
ln n Y
pentru care pαi i ≤ n şi deci αi = . Fie E2 = Cn1 , Cn2 , . . . , Cnn = pβi i
ln pi i=1
cu βi ≥ 0. Cum pβi i |E2 şi cum αi este maxim, cu pαi i ≤ n, rezultă βi ≤ αi şi
E2 |E1 . Exponentul lui p ı̂n descompunerea lui Cnk este
t ‚– ™ – ™ – ™Œ
X n k n−k
m − m − ,
m=1
pi pi pm
i

 
ln n
unde t = . Vom avea αi = βi dacă şi numai dacă
ln pi
t ‚– ™ – ™ – ™Œ
X n k n−k
m − m − = t.
m=1
pi pi pm
i

Deoarece [x + y] − [x] − [y] ≤ 1, avem αi = βi dacă şi numai dacă


– ™ – ™ – ™
n k n−k
− m − = 1, (1)
pm
i pi pm
i

pentru orice m ∈ 1, t.
a) Fie n + 1 compus. Alegem p|n + 1 şi rezultă că n + 1 = py r cu (p, r) = 1.
 
ln n
Dacă r > 1, atunci py < n şi deci y ≤ . Cum p = pi , ı̂n (1) facem m = y şi
ln p
 y     y     
p r−1 k p r−1−k k −k−1
avem − y − = 1, adică r−1− y −r− =1
py p py p py
   
k k+1
şi deci y + 2 − y = 0. Avem deci că
p p
       
k k 1 k 1
0= y + 2− y − y = 2− − y
p p p py p

141
   
k 1 k 1
şi că 1 < + ≤ 2. Ultimele inegalităţi sunt imposibile căci + ≤
py py py py
py − 1 1
y
+ y = 1. Dacă r = 1, avem că y ≥ 2 (deoarece n + 1 este compus),
p p
py−1 < n; alegem m = y − 1 şi se ajunge la contradicţie ı̂n acelaşi mod ca mai
sus.
 
ln n
b) Să presupunem că n + 1 este prim. Fie p prim, p ≤ n, α = .
ln p
     
n k n−k
Alegem k = n + 1 − pα şi vrem să arătăm că m − m − = 1,
p p pm
   
k n−k
∀ m = 1, α. Aceasta este echivalent cu a arăta că + = 1.
pm pm
Însă
   α   
n−k p −1 −1 pm − 1
= = = şi
pm pm pm pm
     
k n + 1 − pα n+1 1
m
= m
= m
≥ m,
p p p p
căci restul ı̂mpărţirii lui n+1 la pm nu poate fi zero (deoarece n+1 este număr
prim şi p < n + 1). De aici rezultă că
     
n−k k pm − 1 n+1 pm − 1 1
+ = + ≥ + m =1
pm pm pm pm pm p
   
k n−k
şi că + = 1, ceea ce trebuia demonstrat.
pm pm
(L. Panaitopol )

30. Pentru n = 0 ecuaţia are soluţii (pentru orice x ∈ [0, 1)). Fie n ≥ 1
√ √
astfel ı̂ncât ecuaţia din enunţ are soluţii şi fie [ k x] = a, [ h x] = n − a, evident
√ √
0 < a < n. Avem, a ≤ [ k x] < a + 1, n − a ≤ [ h x] < n − a + 1 şi de aici
” Š ” Š
deducem că x ∈ ak , (a + 1)k ∩ (n − a)h , (n − a + 1)h . Rezultă că intersecţia
celor două intervale este nevidă şi aceasta se ı̂ntâmplă dacă şi numai dacă
(n − a)h < (a + 1)k şi ak < (n − a + 1)h .
Fie d = (k, h), k = dk1 , h = dh1 , (k1 , h1 ) = 1 şi din cele de mai sus
k1 k1 k1
rezultă că n − a < (a + 1) h1 şi a h1 < n − a + 1. Notăm f (b) = b h1 + b − 1
şi inegalităţile de mai sus se transcriu ca f (a) < n < f (a + 1). Cum [1, ∞) =

S
[f (b), f (b + 1)) (deoarece f este strict crescătoare şi limita la +∞ este +∞;
b=1
f (1) = 1), deducem că ecuaţia din enunţ nu are soluţii dacă şi numai dacă
k1
există c ∈ N∗ astfel ı̂ncât n = f (c) = c h1 + c − 1.

142
k1
Deoarece c h1 ∈ N∗ şi (k1 , h1 ) = 1, rezultă că c = dh1 , d ∈ N∗ şi n =
dk1 + dh1 − 1.
(L. Panaitopol )

n
31. Să presupunem că an ≥ n + 1, ∀ n ≥ 2. Atunci an+1 = an + ≤
an
n 1
an + = an + 1 − , ∀ n ≥ 2. Adunând aceste inegalităţi pentru
n+1 n+1
Xm 1
n = 2, m − 1, deducem că am ≤ a2 + (m − 2) − . Dar am ≥ m + 1 şi
j=3
j
m 1
X m 1
X
obţinem inegalităţile m + 1 ≤ am ≤ a2 + m − 2 − , a2 ≥ 3 + .
j=3
j j=3
j
m
X1
Ultima inegalitate nu poate avea loc pentru orice m, deoarece = +∞.
j=3
j
Există deci n ≥ 2 astfel ı̂ncât an < n + 1.
Fie n0 cel mai mic număr natural, n0 ≥ 2, cu această proprietate. Avem
n
deci an0 < n0 + 1. Pentru n0 ≥ 3, atunci an0 −1 ≥ n0 şi an0 = an0 −1 + ≥
an0 −1
1
an0 −1 ≥ n0 , [an0 ] = n0 . Dacă n0 = 2, atunci a2 = a1 + ≥ 2 (din inegalitatea
a1
mediilor) şi [an0 ] = [a2 ] = 2 = n0 . Arătăm acum prin inducţie că [an ] = n,
∀ n ≥ n0 . Verificarea pentru n = n0 am făcut-o mai sus. Să presupunem că
n
[an ] = n. Atunci n ≤ an < n + 1 şi an+1 = an + < (n + 1) + 1 = n + 2.
an
n
Inegalitatea an+1 ≥ n + 1 este echivalentă cu an + ≥ n + 1 ⇔ a2n + n ≥
an
nan + an ⇔ (an − n)(an − 1) ≥ 0. Ultima inegalitate este evidentă deoarece
an ≥ n ≥ 1. Deci n + 1 ≤ an+1 < n + 2, [an+1 ] = n + 1 şi pasul de inducţie
este demonstrat.
(L. Panaitopol )

143
CAPITOLUL 5

Numere raţionale. Numere iraţionale

√ a
1. Din k
n = , (a, b) = 1, rezultă bk |ak cu (ak , bk ) = 1 şi deci b = 1.
b
√ √ √ √ 1323 + x − y
2. Avem y = 21 3 − x şi de aici 3x = ∈ Q.
42
2 2
Din problema 1 rezultă 3x = a (a ∈ N) şi apoi x = 3A , 0 ≤ A ≤ 21,
y = 3(21 − A)2 . Numărul soluţiilor este 22.
3. Se presupune 3n + 2 = m2 , unde m = 3k sau m = 3k ± 1. Se ajunge la
√ √
contradicţia m2 ≡ 2(3). Deci 3n + 2 ∈ R\Q. La fel se arată că 5n + 2 este
număr iraţional.
4. Un număr cu o infinitate de zecimale este raţional dacă şi numai dacă
este o fracţie periodică. Presupunem că numărul n are o perioadă de lungime
k. Deoarece există ı̂n scrierea numărului o secvenţă de 2k cifre 2, rezultă că
perioada este formată numai din cifra 2, ceea ce este fals. Deci n este iraţional.
pi
5. Presupunem ri = , pi , qi ∈ Z, ∀ i ∈ 1, n. Fie p număr prim care
qi
1 X n p ·x
i i
nu divide q1 q2 . . . qn . Dacă = , după eliminarea numitorilor avem
p i=1 qi
q1 q2 . . . qn = px cu x ∈ Z. Rezultă contradicţia p|q1 q2 . . . qn .
6. Aducând la acelaşi numitor, avem (b +ac+ a+bc)(a+b) = ab+c(a2+b2 )+
abc2 , de unde (a + b)2 c + (a + b)2 = ab(c2 + 1) + c(a2 + b2 ). Rezultă că (a + b)2 =
ab(c2 + 1) + c(a2 + b2 − (a + b)2 ) = ab(c2 + 1) − 2abc = ab(c − 1)2 . Dacă c = 1,
 
1 1 1 a+b 2
atunci + = , fals. Deci ab = şi (c − 3)(c + 1) =
a+b b+a a+b c−1
 
(a + b)2 (a − b)2 (a − b)(c − 1) 2
(c − 1)2 − 4 = −4= = . În concluzie,
ab ab a+b
Ê È
c−3 (c − 3)(c + 1) |a − b||c − 1|
= = ∈ Q.
c+1 c+1 (c + 1)|a + b|
(L. Panaitopol )

145
√ √ √
7. Avem 2n + 1 = m2 , m ∈ N. Pentru ca 2n + 2, 2n + 3, . . . , 3n + 3
să fie toate iraţionale, este necesar şi suficient ca 3n + 3 < (m + 1)2 . Ultima
m2 − 1
condiţie este echivalentă cu 3 + 3 < (m + 1)2 . De aici deducem că
2
m2 − 4m + 1 < 0. Cum m este număr natural impar, din ultima inegalitate
rezultă că m = 1, 3. Soluţiile problemei sunt n = 0, 4.

2n2 + 1 a ± a2 + 24a − 8
8. = a ∈ Z şi deci n = . Din a2 +24a −8 = k 2 ,
n+3 4
5 13
k ∈ N, rezultă (a + 12 + k)(a + 12 − k) = 152. Rezultă n = − , 16, , −2,
2 2
7 25
−22, − , −4, − .
2 2
2m2 + 1 3m2 − 1
9. Punem = a, = b, a, b ∈ Q şi se arată că 3m(2b − a) =
m−3 3m − 5
10b − 9a − 5. Din m ∈ R\Q rezultă 2b − a = 10b − 9a − 5 = 0 şi apoi

− 5 ± 377
m= .
16
a ab na2 b2
10. Avem = x ∈ Q şi c = . Rezultă a2 + b2 = şi cum
b a+b (a + b)2
a = bx, avem (x2 + 1)(x + 1)2 = nx2 , adică x4 + 2x3 + (2 − n)x2 + 2x + 1 = 0.
Rădăcinile raţionale pot fi numai ±1. Pentru x = −1 obţinem contradicţia
n = 0, iar pentru x = 1 rezultă n = 8.
√ √
11. Dacă 3 pn + 1 ∈ Q, rezultă 3 pn + 1 = x ∈ N şi din pn = x3 − 1 =
n
(x−1)(x2 +x+1) rezultă x−1 = pi . Cum x2 +x+1 > x−1, rezultă 0 ≤ i < .
2
Avem pn = (pi + 1)3 − 1 = pi (p2i + 3pi + 3). Dacă i = 0, avem pn = 7 şi deci
n = 1 şi p = 7. Dacă i ≥ 1, avem pn−i = p2i + 3pi + 3 şi deci pi |3 deoarece
n − i > i. Rezultă i = 1, p = 3 şi contradicţia 3n−1 = 21. Aşadar, singura
posibilitate este n = 1 şi p = 7.
√ √
12. Fie 3 − x2 = a, 3 2 − x3 = b, a, b ∈ Q. Avem x3 = 2−b3 , x2 = 3−a2
 
2 − b3 2
2 − b3 2
şi deci x = ∈ Q. Apoi 3−a = , adică a2 +c2 = 3, a, c ∈ Q, şi
3 − a2 3 − a2
deci am obţinut o contradicţie. Să justificăm ultima afirmaţie. Fie m, p, n ∈ Z,
m p
n 6= 0, (m, p, n) = 1, astfel ı̂ncât a = , c = . Presupunn̂d că a2 + c2 = 3,
n n
obţinem că m2 +p2 = 3n2 . Din 3|m2 +p2 , deducem 3|m şi 3|p. De aici rezultă că
9|m2 + p2 = 3n2 , 9|3n2 , 3|n2 , 3|n. Am obţinut ı̂n acest moment o contradicţie,
căci 3|(m, n, p) = 1.

146
√ √ √
13. Dacă n + 3 n + 5 = q ∈ Q şi n ∈ / Q, ajungem la contradicţie ı̂n
√ √ √ √
felul următor: n + 5 = q − n, n + 5 = q − 3q 2 n + 3qn − n n. Deoarece
3 3

n∈ / Q, rezultă că 0 = 3q 2 + 3n. Cum 3q 2 ≥ 0, n ≥ 0, deducem că n = 0
√ √ √
şi 3 5 ∈ Q, ceea ce este evident imposibil. Presupunem că n + 3 n + 5 ∈ Q
√ √
şi rezultă n ∈ Q şi 3 n + 5 ∈ Q şi deci n = x2 , n + 5 = y 3 cu x, y ∈ N.
Avem deci y 3 = x2 + 5. Dacă x = 2k + 1, atunci avem y 3 = 4k(k + 1) + 6
şi deci 2|y. Rezultă contradicţia M8 = M8 + 6. Aşadar x = 2k şi y = 2h + 1
(k ∈ Z, h ∈ N). Avem y 3 − 1 = x2 + 4, adică 2h(4h2 + 6h + 3) = 4(k 2 + 1). Cum
4h2 + 6h + 3 este impar, rezultă h = 2m şi deci m(16m2 + 12m + 3) = k 2 + 1.
Deoarece numărul 16m2 + 12m + 3 ≥ 3 este de forma M4 + 3, el are un divizor
prim p de forma M4 + 3. Avem p|k 2 + 1, adică p|k şi p|1, contradicţie.
y
14. Pentru x = 0 rezultă y = 0 sau y = 1. Pentru x 6= 0 notăm =t∈Q
x
şi ı̂nlocuind y = tx, ecuaţia devine x(t3 + 1) = t2 + 1. Rezultă t 6= −1 şi
t2 + 1 t(t2 + 1)
x= 3 ,y= 3 cu t ∈ Q\{−1}.
t +1 t +1
15. Presupunem că există a, b, c, d naturale (a, b) = 1, (c, d) = 1 astfel ca
a c a c b d
x = , y = şi + + + = n, adică
b d b d a c
cd(a2 + b2 ) + ab(c2 + d2 ) = nabcd. (1)
Rezultă a|b2 cd şi b|a2 cd. Cum (a, b) = 1, avem a|cd şi b|cd, adică [a, b]|cd
şi deci ab|cd. Analog se arată că cd|ab. Aşadar ab = cd şi relaţia (1) devine
a2 +b2 +c2 +d2 = nab. Rezultă (a+b)2 +(c−d)2 = nab şi (a−b)2 +(c+d)2 = nab.
Deoarece 3|n, rezultă 3|(a + b)2 + (c − d)2 şi 3|(a − b)2 + (c + d)2 . Se ştie că
dacă U, V ∈ Z şi 3|U 2 + V 2 , atunci 3|U şi 3|V . În cazul nostru rezultă 3|a + b şi
apoi 3|a − b, adică 3|a şi 3|b. Aceasta contrazice condiţia (a, b) = 1 şi justifică
enunţul.
16. Arătăm ı̂ntâi că ecuaţia A2 + B 2 + C 2 = 7D2 are ı̂n numere ı̂ntregi
numai soluţia A = B = C = D = 0. Dacă D este impar, rezultă fie că A, B, C
sunt toate impare, fie două dintre ele sunt pare şi al treilea impar.
În primul caz A2 +B 2 +C 2 ≡ 3 (mod 8) şi ı̂n cel de-al doilea caz A2 +B 2 +
C 2 ≡ 1 (mod 4). Cum 7D2 ≡ 7 (mod 8), obţinem o contradicţie ı̂n ambele
cazuri. Dacă D este par, rezultă că unul din numerele A, B, C este par şi
celelalte două sunt impare, sau toate cele trei numere sunt pare. În primul caz
ajungem la contradicţia 2 ≡ A2 + B 2 + C 2 ≡ 7D2 ≡ 0 (mod 4). Deci singura
posibilitate este ca A, B, C, D să fie toate pare. Repetând raţionamentul pentru

147
       
A 2 B 2 C 2 D 2
ecuaţia + + =7 , deducem că numerele A, B, C.D
2 2 2 2
se divid toate cu 4. Continuând, concluzionăm că A, B, C, D se divid cu toate
puterile lui 2 şi deci A = B = C = D = 0. Să presupunem acum că ecuaţia
a b c
din enunţ ar avea soluţii: x = , y = , z = , a, b, c, d ∈ Z, d 6= 0. Atunci
d d d
ecuaţia din enunţ se rescrie a2 + b2 + c2 + ad + bd + cd = d2 şi (2a + d)2 +
(2b + d)2 + (2c + d)2 = 7d2 . Conform celor demonstrate anterior, rezultă că
d = 2a + d = 2b + d = 2c + d = 0. Am obţinut o contradicţie căci d 6= 0.
17. Evident x = y ∈ Q∗+ verifică ambele ecuaţii. Pentru x 6= y avem:
 
x 1
a) Fie de exemplu y > x. Punem z = şi obţinem y = 1 + x şi de
y−x z
 
1
aici xy = x(1+ z )x şi deci x(1+ z )x = y x şi de aici y = x(1+ z ) , adică 1 + x =
1 1 1

z
 z  z+1
1 1 s
x(1+ z ) şi de aici x = 1 +
1
şi apoi y = 1 + . Fie x = , (r, s) = 1,
z z r
 n
n r n m
z = , (m, n) = 1. Rezultă = şi deci rm (m + n)n = sm nn . Cum
m s m+n
(rm , sm ) = 1 şi ((m + n)n , nn ) = 1, rezultă (m + n)n = ksm şi nn = krm şi
apoi k = 1. Din (m + n)n = sm şi nn = rm cu (m, n) = 1 rezultă n = am ,
r = an şi apoi m + n = bm şi s = bn . Avem am + m = bm şi deci b ≥ a + 1.
Avem deci m = bm − am ≥ (a + 1)m − am . Dacă m ≥ 2, rezultă m > am şi
 

1 z
contradicţia a < 1. Aşadar m = 1, adică z ∈ N şi soluţiile sunt x = 1 + ,
z
 
1 z+1
y = 1+ cu z ∈ N∗ .
z
 z
1 1 z
b) Facem x → şi y → şi obţinem ecuaţia de la a) şi deci x = ,
y x z+1
 z+1
z
y= , cu z ∈ N∗ .
z+1
m 1 m
18. Fie q = , m, n ∈ N∗ , (m, n) = 1. Dacă < q < 2, atunci q = =
n 2 n
9m(m2 − mn + n2 ) (m + n)3 + (2m − n)3 1
2 2
= 3 3
. Deoarece < q < 2, avem că
9n(m − mn + n ) (m + n) + (2n − m) 2
2m − n ∈ N∗ şi 2n − m ∈ N∗ . Dacă q ∈ Q∗+ este oarecare, fie (rn )n∈N un
1
şir de numere raţionale strict pozitive astfel ı̂ncât lim rn = √
3 q
. Deoarece
n→∞

148
1
lim rn3 · q = 1, există un n0 ∈ N astfel ı̂ncât < rn3 0 q < 2. Conform pasului
n→∞ 2
∗ 3
a3 + b3 e
anterior, există a, b, c, d ∈ N astfel ı̂ncât rn0 q = 3 . Notând rn = ,
c + d3 0
f
(af )3 + (bf )3
e, f ∈ N∗ , avem că q = , unde af, bf, ec, ed ∈ N∗ .
(ec)3 + (ed)3
19. Avem identitatea (a−b)3 +(b−c)3 +c3 = 3b2 (a−c)+(a3 −3b(a2 −c2 )).
Punem a = 12x(x + 1), b = (x + 1)3 , c = 12x(x − 1) cu x ∈ Q şi obţinem
r
72x(x + 1)6 = (a − b)3 + (b − c)3 + c3 . Fie r ∈ Q, r 6= −72. Punem x = şi
72
     3
a−b 3 b−c 3 c
avem r = + + . Pentru r = −72 avem
(x + 1)2 (x + 1)2 (x + 1)2
r = (−4)3 + (−2)3 + 03 .

20. Fie xy = a (1), x2 +y = b (2), y 2 +x = c (3). Înmulţind (2) cu y şi (3)


cu x obţinem (1 − a)x + by = c (4), cx + (1 − a)y = b (5). Dacă (1 − a)2 6= bc,
c(1 − a) − b2
rezultă contradicţia x = ∈ Q. Înmulţind relaţiile x2 = b − y,
(1 − a)2 − bc
y 2 = c − x, rezultă bx + cy = bc + a − a2 (6). Dacă c(1 − a) 6= b2 , din (4) şi (6)
rezultă 2
8 x ∈ Q. Dacă b(1 − a) 6= c , din (5) şi (6) rezultă x ∈ Q. Rămâne deci
> 2
< (1 − a) = bc (7)
cazul c(1 − a) = b2 (8). Scăzând (8) şi (9) rezultă (c−b)(b+c−a+1) = 0
>
:
b(1 − a) = c2 (9)
(10). Dacă b = c, rezultă x + y = 1 sau x = y. Dacă x = y, atunci x2 = a şi
x2 +x = b şi contradicţia x = b−a ∈ Q. Dacă b 6= c, din (10) avem a−1 = b+c
 
2
c 2 3c2 c
şi (7) devine (b + c) = bc ⇔ b + + = 0 şi deci b = − şi c = 0, adică
2 4 2
din nou b = c. Rezultă deci x + y = √ 1. Un exemplu
√ de numere care ı̂ndeplinesc
1+ 5 1− 5
condiţiile din enunţ este x = ,y= .
2 2
(L. Panaitopol )

21. Presupunem că cos 1◦ ∈ Q. Deoarece cos 2α = 2 cos2 α − 1, deducem


imediat că cos 2k ∈ Q, ∀ k ∈ N. Căutăm un k ∈ N∗ astfel ı̂ncât 2k ≡ 1(45). Este
.
bun k = ϕ(45) = 24 dar observăm că este bun şi k = 12; 212 − 1 = 4095..45.
Din presupunerea făcută ştim că cos 212 ∈ Q. Dar cos 212 = cos 4096 = cos 136,
deoarece 4096 ≡ 136(360).

2
cos 136 = cos 1 cos 135 − sin 1 sin 135 = − (cos 1 + sin 1). Deci cos 1 + sin 1 =
2

149
q
√ , unde q ∈ Q (q = −2 cos 136 ∈ Q conform presupunerilor făcute).
2
q q2 2 cos 1q
Avem sin 1 = √ − cos 1, sin2 1 = + cos2 1 − √ . Cum sin2 1 = 1 −
2 2 2
cos2 1 ∈ Q, cos 1 ∈ Q, q ∈ Q, cos 1 6= 0, q 6= 0, deducem din identitatea
q2 2 cos 1q √
sin2 1 = +cos2 1− √ că 2 ∈ Q. Acest lucru este absurd. Demonstraţia
2 2
este ı̂ncheiată.
Observaţie. O altă soluţie se bazează pe formula pentru cos nx (se scrie pentru
cos 45x).

π 2π π
22. Să presupunem că cos ∈ Q. Atunci cos = 2 cos2 − 1 este, de
7 7 7
2π 2π
asemenea, un număr raţional. Notăm ε = cos + i sin . Avem ε7 = 1 şi
7 7
X6 2π
εj = 0. Notăm α = ε+ε6 = 2 cos . α ∈ Q conform presupunerilor făcute.
j=0
7
Avem α2 = ε2 +ε12 +2 = ε2 +ε5 +2, α3 = ε3 +3ε8 +3ε13 +ε18 = ε3 +ε4 +3(ε+ε6 )
6
X
şi α3 + α2 − 2α = εj + 2 = 1. α verifică ecuaţia x3 + x2 − 2x − 1. Singurele
j=1
rădăcini raţionale ale ecuaţiei ar putea fi ±1. Se observă uşor că ±1 nu sunt
soluţii ale acestei ecuaţii şi deci α nu poate fi raţional. Demonstraţia este
ı̂ncheiată.

p 1 1
23. Presupunem că = arccos √ , p, q ∈ N∗ , (p, q) = 1, q ≥ 2.
q π s 2003
 
πp 1 πp 2002 p
πp πp q
Avem cos =√ şi sin = . Din (−1) = cos + i sin ,
q 2003 q 2003 q q
deducem că
 q  q−2 ‚r Œ2  q−4 ‚r Œ4
1 1 2002 1 2002
√ −Cq2 √ +Cq4 √ . . . = (−1)p .
2003 2003 2003 2003 2003

Dacă q = 2k, atunci p este impar (deoarece (p, q) = 1) şi termenul drept este
egal cu −1. Egalitatea de mai sus se scrie

2 4
1 − C2k · 2002 + C2k · 20022 + · · · + (−1)k C2k
2k
(2002)k = −2003k .

Trecând la o congruenţă modulo 2002, obţinem că 1 ≡ −2003k (2002),


1 ≡ −1(2002); contradicţia obţinută ne arată că acest caz este imposibil.

150
Dacă q = 2k + 1, egalitatea anterioară se rescrie sub forma

1 € Š
2 k 2k k
√ 1 − C2k+1 2002 + · · · + (−1) C 2k+1 2002 = ±1.
2003 · 2003k

De aici rezultă că 2003 este raţional, ceea ce este absurd. Demonstraţia este
ı̂ncheiată.

24. Evident că n = 1, 2, 3 sunt soluţii ale problemei deoarece cos π = −1,
π π 1
cos = 0, cos = . Presupunem că există n ∈ N, n ≥ 4 astfel ı̂ncât
2 3 2
π
q = cos ∈ Q. Notăm Pn (x) = cos(n arccos x), Qn (x) = sin(n arccos x). Avem
n
Pn (x) + iQn (x) = (cos arccos x + i sin(arccos x))n = (cos y + i sin y)n , unde y =
arccos x. Deducem că Pn (x) = cosn y − Cn2 cosn−2 y sin2 y + Cn4 cosn−4 y sin4 y +
· · · = xn − Cn2 xn−2 (1 − x)2 + . . . Cn4 xn−4 (1 − x2 )2 − Cn6 xn−6 (1 − x2 )3 . . . şi,
 
π
punând x = q, rezultă că −1 = cos n = cos(n arccos q) = q n − Cn2 q n−2 (1 −
n
q 2 ) + Cn4 q n−4 (1 − q 2 )2 − Cn6 q n−6 (1 − q 2 )3 . . . . Coeficientul lui xn ı̂n Pn (x) este
1 + Cn2 + Cn4 + Cn6 + · · · = 2n−1 . q ∈ Q este rădăcină a polinomului Pn (x) + 1
care are coeficienţi ı̂n Z şi coeficientul lui xn este 2n−1 . Dacă n este impar,
deducem că termenul liber al lui Pn (x) + 1 este 1 şi atunci rădăcina q ∈ Q a
π a
polinomului Pn (x) + 1, 0 < q = cos < 1, este de forma q = , a, b ∈ N∗ ,
n b
1
unde a este un divizor al lui 1 şi b un divizor al lui 2n−1 . Deci q = m , m ∈ N,
2
1 π π 1
0 ≤ m ≤ n − 1. Deoarece n ≥ 4, avem că m = q = cos > cos = .
2 n 3 2
Obţinem că 2 > 2m , m = 0, q = 1, ceea ce constituie o contradicţie. Analizăm
acum cazul n = 2k, k ∈ N, k ≥ 2. În această situaţie, termenul liber al
polinomului Pn (x) + 1 este (−1)k + 1. Vom considera două subcazuri.
Primul subcaz este acela ı̂n care k este par. În acest subcaz deducem că
π a
0 < q = cos < 1 este de forma q = , a, b ∈ N∗ , a|2, b|2n−1 . De aici rezultă
2k b
1
că q = m , m ∈ N, 0 ≤ m ≤ n − 1 şi obţinem aceeaşi contradicţie ca mai sus.
2
π
Subcazul k impar se analizează ţinând cont de faptul că cos =
k
   
π π π π
cos 2 = cos 2 = 2 cos2 − 1 ∈ Q, deoarece cos ∈ Q.
2k n n n

151
Ţinând cont de faptul că k ≥ 3 şi de primul caz analizat, rezultă √
că k = 3,
π 3
n = 2k = 6. Am obţinut o contradicţie şi ı̂n acest caz, căci cos = ∈/ Q.
6 2

25. Alegem a = 2, b = log√2 3. Se arată uşor că a, b ∈ R|Q, ı̂nsă
b
√ log√2 3 √ √2
a = 2 = 3 ∈ Q. Altfel: fie x = 2 . Dacă x ∈ Q, problema este
√ €√ Š2
rezolvată. Dacă x ∈
/ Q, calculăm x =2 2 = 2 ∈ Q.

√ m
26. Deoarece 3− > 0, rezultă 3n2 − m2 > 0. Dacă 3n2 − m2 = 1,
n

rezultă m2 + 1 = 3n2 . Luând √ constată că m2+1..3
m = 3k + r, cu r ∈ {0, 1, 2}, se √ ¦
√ m 2+2 √ m m2+2 m
şi deci 3n2 − m2 ≥ 2; 3 ≥ . Rezultă 3 − ≥ − . Este
√ n n n n
m2 + 2 − m 1 €√ Š
suficient să arătăm că > ⇔ (m+1) m2 + 2 − m >
n√ n(m + 1) √
2 2
1 ⇔ (m + 1)(m + 2 − m ) > m + 2 + m ⇔ m + 2 > m2 + 2, ceea ce este
2

adevărat.
√ √ √ √
27. Notând A = a + b 2 + €c 3,√BŠ= a + b 2 − c 3, D = a2 + 2b2 −
√ 2 € √ Š
3c2 − 2ab 2, avem că ABD = a+b 2 −3c2 a2 +2b2 −3c2 −2ab 2 =
√ √
(a2 +2b2 −3c2 )2 −8a2 b2 . Deoarece 1, 2 şi 3 sunt liniar independente peste
Q şi (a, b, c) 6= (0, 0, 0), se deduce uşor că ABD 6= 0. Din cele de mai sus
€ √ √ Š
deducem că |ABD| ≥ 1. Pe de altă parte, |B| ≤ 106 1 + 2 + 3 < 107 ,
€ √ Š
|D| ≤ 1012 1 + 2 + 3 + 2 2 < 1013 .
Avem deci că 1 ≤ |ABD| < |A| · 107 · 1013 = |A| · 1020 . De aici rezultă că
1
|A| > 20 , exact ceea ce trebuia arătat.
10
28. Este o observaţie imediată că, dacă numerele a1 , a2 , . . . , a2n+1 au
proprietatea din enunţ şi α ∈ R, atunci numerele αa1 , αa2 , . . . , αa2n+1 au
proprietatea din enunţ şi de asemenea, numerele a1 + α, a2 + α, . . . , a2n+1 + α
au aceeaşi proprietate.
Cazul I : ai ∈ Q, ∀ i = 1, 2n + 1. Folosind observaţiile de mai sus, putem
presupune că ai ∈ Z, ∀ i = 1, 2n + 1 şi unul dintre numere, să zicem a1 ,
2n+1
X
este egal cu 0. Fie s = ai . Aplicând proprietatea din enunţ pentru un
i=1
i = 1, 2n + 1, deducem că s − ai este par ∀ i = 1, 2n + 1.
În particular, pentru i = 1, obţinem că s este par (deoarece a1 = 0). Deci
ai par ∀ i = 1, 2n + 1. Folosind observaţiile de mai sus, deducem că numerele

152
a1 a2 a2n+1
ı̂ntregi 0 = , ,..., au proprietatea din enunţ. Deducem ı̂n acelaşi
2 2 2
fel că numerele obţinute sunt pare şi deci 4|ai , ∀ i = 1, 2n + 1. Analog, 2k |ai ,
∀ i = 1, 2n + 1 şi ∀ k ∈ N. De aici deducem că a1 = a2 = · · · = a2n+1 = 0.
Cazul II : ai ∈ R. R este spaţiu vectorial peste Q şi există o bază {ej }j∈I
a lui R peste Q. Există {j1 , j2 , . . . , js } ⊆ I astfel ı̂ncât ∀ i = 1, 2n + 1,
s
X
ai = mi,k ejk , mi,k ∈ Q ∀ k = 1, s şi i = 1, 2n + 1. Deoarece ej1 , ej2 , . . . , ejs
k=1
sunt liniar independente peste Q şi a1 , . . . , a2n+1 au proprietatea din enunţ,
atunci şi numerele m1,k , m2,k , . . . , m2n+1,k au aceeaşi proprietatea pentru
∀ k = 1, s. Cum mj,k ∈ Q ∀ j = 1, 2n + 1, din cazul I deducem că m1,k =
m2,k = · · · = m2n+1,k ∀ k = 1, s. De aici rezultă că a1 = a2 = · · · = a2n+1 .

29. a) Fie V spaţiul vectorial peste Q generat de mulţimea S.


Fie {v1 = 1, v2 , . . . , vr } ⊆ S o bază a lui V peste Q. Dacă r = 1, demonstraţia
este ı̂ncheiată. Să presupunem că r ≥ 2. Alegem un q ∈ Q∗+ astfel ı̂ncât qv2 > 1.
Pentru fiecare i = 0, n există numerele ai,j ∈ Q (j = 1, r) astfel ı̂ncât xi =
r
X
aij vj . Pentru 0 ≤ i < j ≤ n, (i, j) 6= (0, n), ştim că există 0 ≤ k < l ≤ n,
j=1
(i, j) 6= (k, l), astfel ı̂ncât xj − xi = xl − xk . Deoarece {v1 , . . . , vr } este o bază a
lui V peste Q, deducem din ultima egalitate că aj,s −ai,s = al,s −ak,s ∀ s = 1, r.
Dacă {u1 , u2 , . . . , ur } este o altă bază a lui V peste Q şi dacă notăm yi =
r
X
aij uj , deducem că mulţimea S 0 = {y0 , y1 , . . . , yn } are aceeaşi proprietate
j=1
ca mulţimea S (deoarece aj,s − ai,s = al,s − ak,s ∀ s = 1, r). Vom considera
u1 = v1 = 1, u2 = qv2 > 1, ut = vt , ∀ t = 3, r şi y0 , y1 , . . . , yn numerele
definite ca mai sus. Am văzut mai sus că mulţimea S 0 = {y0 , y1 , . . . , yn } ar
trebui să aibă aceeaşi proprietatea ca mulţimea S. Este evident că y0 = x0 = 0
şi y1 = x1 = 1. Ar trebui ca, cu excepţia distanţei 1, celelalte distanţe dintre
punctele lui S 0 să apară cel puţin de două ori. Fie d distanţa maximă ı̂ntre
punctele lui S 0 . Ea apare evident o dată şi nu coincide cu distanţa dintre 0 şi
1, căci această distanţă maximă este mai mare sau egală cu distanţa dintre
y0 = 0 şi u2 > 1. Cum v2 = xi , pentru un i ∈ N, 1 ≤ i ≤ n − 1, avem că
u2 = yi şi distanţa dintre y0 şi yi este u2 = qv2 > 1; deci d > 1. Am ajuns la
o contradicţie. Deci r = 1 şi problema este demonstrată.
b) Problema este caz particular al problemei de la punctul a).
(Soluţie N. Beli )

153
∞ 1
X a 2
30. Presupunem că n2
= , a, b ∈ N∗ şi ı̂nmuţim egalitatea cu b2n .
n=1
2 b
" #
€ Š ∞
X 1
2 2 2 2
Obţinem a2n −b 2n −1 + 2n −2 + ··· + 1 = b . Deoarece
k=1
2(n+k)2 −n2
" #
1 1 ∞
X 1
< , rezultă că numărul natural b este mai
2(n+k)2 −n2 2k(2n+1) k=1
2(n+k)2 −n2
b
mic decât pentru orice n şi deci contradicţie.
22n+1 −1

X 1 a
31. Presupunem că = , a, b ∈ N∗ . Fie n astfel ca an ≥ b.
n=1
(an )! b
!
n
X 1 a X∞ 1 a ∞
X 1
= − şi deci (an )! − este un număr
k=1
(ak )! b k=n+1 (ak )! b k=n+1 (ak )!

X (an )! 1 1
natural şi apoi x = este natural. Dar 0 < x ≤ + +
k=n+1
(ak )! an+1 an+1 · an+2
1 1 1
··· < + ≤ 1 şi deci x ∈ (0, 1) ∩ N = ∅.
+ ··· =
an+1 (an+1 )2an+1 − 1
La fel se obţine o contradicţie ı̂n cea de-a doua problemă, ţinând cont că

X (an )!(−1)k
0< < 1.
k=n+1
(ak )!
∞ 1
X
Consecinţă. e este iraţional deoarece e = .
n=0
n!

32. Un exemplu de astfel de număr este 0, a1 a2 . . . unde


(
0, dacă n = 2k − 1, k ∈ N∗
an =
1, dacă n 6= 2k − 1, k ∈ N∗ .
Dacă n = 2k − 1 (k ∈ N∗ ), atunci 2n + 1 = 2k+1 − 1, 2n + 2 = 2k+2 , an =
a2n+1 = 0, a2n+2 = 1 şi evident că a2n+1 +a2n+2 = 1+an = 1. Dacă n 6= 2k −1,
atunci an = 1 şi trebuie arătat (pentru a demonstra că a2n+1 +a2n+2 = 1+an )
că 2n + 1 6= 2k − 1, 2n + 2 6= 2h − 1. Dacă 2n + 1 = 2k − 1, atunci n = 2k−1 − 1
şi obţinem o contradicţie; 2n + 2 6= 2h − 1 deoarece 2n + 2 este număr par
iar 2h − 1 este impar. Vom arăta ı̂n continuare că orice număr 0, a1 a2 . . . ,
pentru care a2n+1 + a2n+2 = 1 + an , ∀ n ≥ 1 şi a1 + a2 6= 2 este iraţional.
În particular, numărul de mai sus este iraţional fiindcă a1 + a2 = 0 + 1 6= 2.
Să presupunem că numărul 0, a1 a2 . . . este raţional. Există atunci h, k ∈ N∗
astfel ı̂ncât am+k = am , ∀ m ≥ h.

154
8
>
> a3 + a4 = 1 + a1
>
>
> a + a6 = 1 + a2
< 5
Adunând relaţiile a7 + a8 = 1 + a3 deducem că an+1 + an+2 + · · · +
>
> ..
>
> .
>
:
a2n+1 + a2n+2 = 1 + an
a2n+2 = n + a1 + a2 pentru orice n ≥ 1. Punem ı̂n această relaţie n = lk − 2,
unde l ∈ N∗ este ales astfel ı̂ncât lk − 1 ≥ h. Obţinem că ls = a1 + a2 + lk − 2,
unde s = ah + ah−1 + · · · + ah+k−1 . Deducem că l|a1 + a2 − 2, ∀ l ∈ N∗ , astfel
ı̂ncât lk − 1 ≥ h şi de aici rezultă că a1 + a2 − 2 = 0, a1 + a2 = 2. Am obţinut
o contradicţie.
(L. Panaitopol )

33. Este suficient să arătăm că şirul an este periodic de la un rang ı̂ncolo.
Dacă a1 , a2 sunt ambele pare, atunci an = 2, ∀ n ≥ 3. Să presupunem că a1 şi
a2 nu sunt ambele pare. Dacă a1 şi a2 sunt impare, atunci a3 = 2. Dacă a1 este
par şi a2 impar, atunci a4 = 2, iar dacă a1 este impar şi a2 este par, atunci
a5 = 2. De aici rezultă că putem presupune că a1 = 2 şi a2 = p, p prim impar
(scopul nostru este de a arăta că şirul este periodic de la un rang ı̂ncolo). Fie
q cel mai mic număr prim impar astfel ı̂ncât aj = 2 şi aj+1 = q. Arătăm că
q = 3. Să presupunem că q > 3. Dacă q = 6k + 1 (k ∈ N∗ ), atunci aj+2 = 3,
aj+3 = 2, aj+4 = 5, aj+5 = 7, aj+6 = 2, aj+7 = 3 şi se contrazice definiţia lui
q. Să presupunem acum că q = 6k − 1, k ∈ N∗ . Dacă 6k + 1 este prim, atunci
aj+2 = 6k + 1, aj+3 = 2, aj+4 = 3 şi se contrazice din nou minimalitatea lui q.
6k + 1
Dacă 6k + 1 nu este prim, atunci aj+2 ≤ , aj+2 = r este prim diferit de
5
6k + 1
2 şi 3, aj+3 = 2, aj+4 ≤ 2 + r ≤ 2 + < 6k + 1 = q şi iarăşi se contrazice
5
minimalitatea lui q. Deci q = 3 şi se arată uşor că ak+9 = ak , ∀ k ≥ j (avem
aj = 2, aj+1 = 3, aj+2 = 5, aj+3 = 2, aj+4 = 7, aj+5 = 3, aj+6 = 2, aj+7 = 5,
aj+8 = 7, aj+9 = 2, aj+10 = 3). Enunţul este demonstrat.
(L. Panaitopol )

34. Folosind criteriul lui Eisenstein pentru numărul prim 2, deducem


imediat că polinomul xn − 2 din Q[x] este ireductibil. Să presupunem că
n−1
X €√ Šj n−1
X
aj xj + a0 − q şi h = (xn − 2, g). Cum
n
aj 2 = q ∈ Q. Notăm g(x) =
j=0 j=1
xn − 2 este ireductibil, rezultă h = 1 sau h = xn − 2. Nu putem avea h = 1

deoarece xn − 2 şi g au rădăcina comună n 2. Deci avem (xn − 2, g) = xn − 2.

155
Cum grad g ≤ n − 1, deducem g = 0 şi aj = 0 ∀ j ∈ 1, n − 1. Cealaltă
implicaţie este banală.

35. Notăm cu f (x) = ar xr + ar−1 xr−1 + · · · + a1 x + a0 , f (x) ∈ Z[x],


ar 6= 0, polinomul ireductibil care-l are ca rădăcină pe α, şi cu α1 =
α, α2 , . . . , αr rădăcinile complexe ale acestui polinom. Alegem 0 < c =
1
r < 1. Vom arăta că acest c are proprietăţile din enunţ
Y
|ar | (1 + |α| + |αj |)
j=2
(este evident că el nu depinde de alegerea lui p şi q). Fie p ∈ Z, q ∈ N∗ , alese
p p 1 c
arbitrar. Să presupunem că α − ≥ 1. Atunci α − ≥ 1 ≥ r > r.
q q q q
p
Analizăm acum cazul α − < 1. Atunci
q
  r r  
p p Y p p Y p
f = |ar | α − αj − ≤ |ar | α − |α| + |αj | + α − <
q q j=2 q q j=2 q
r
p Y
< |ar | α − (|α| + |αj | + 1) .
q j=2
!
p
Avem qr f = ar pr + ar−1 pr−1 q + · · · + a1 pq r−1 + a0 q r ≥ 1 (ultima ine-
q
!
p
galitate are loc deoarece qr f este un număr ı̂ntreg nenul). Deci
q

   
p p 1 p c
α− >f r =c f ≥
q q Y q qr
|ar | (|α| + |αj | + 1)
j=2

şi enunţul este demonstrat.


∞ 1
X 1 ∞ 1
X
36. Suma din enunţ este convergentă căci α = <
. Să =
n=1 n=1
4n!
3 4n
presupunem că α este algebric şi fie f ∈ Z[x], grad f = r, polinomul minimal
care-l are ca rădăcină pe α. Să arătăm ı̂ntâi că r ≥ 2. Dacă r = 1, atunci α
p X∞ 1 1
ar fi număr raţional, α = , p, q ∈ N∗ . Fie k ∈ N∗ astfel ı̂ncât j!
<
q j=k
4 q
(k există fiindcă restul unei serii convergente tinde la zero). Înmulţim egalitatea

156
p X∞ 1
=α= m!
cu q · 4k! şi obţinem
q m=1
4
 ‹ ∞
X
1 1 1 1
p · 4k! − q · 4k! 1!
+ 2! + · · · + k! =q .
4 4 4 m=k+1
4m!−k!

Numărul din stânga este un număr natural nenul şi de aici rezultă că
X∞ 1 1 1 1
≥ . Pe de altă parte avem inegalitatea ≤
m=k+1
4m!−k! q 4m!−k! 4(m−1)!
∀ m ∈ N, m ≥ k + 1 şi de aici rezultă contradicţia

X ∞
X X∞
1 1 1 1 1
≤ m!−k!
≤ (m−1)!
= j!
< .
q m=k+1 4 m=k+1
4 j=k
4 q

Deci r ≥ 2 şi putem alege constanta c din Criteriul Liouville asociată lui
X∞ 1 X∞ 1
α= . Fie k ∈ N, k ≥ r astfel ı̂ncât < c (k există deoarece, după
j=1
4j! j=k
4j!
cum am menţionat mai sus, restul unei serii convergente tinde la zero).
k!
Avem că pentru ∀ n ≥ k + 1 ≥ r + 1, r ≤ r ≤ n − 1. De aici deducem
(n − 1)!
k! 1 1
că 1 ≤ n − r , (n − 1)! ≤ n! − rk!, n!−rk! ≤ (n−1)! . Alegem q = 4k! ,
(n − 1)! 4 4
p = 4k!−1! + 4k!−2! + · · · + 4k!−(k−1)! + 1 şi din Criteriul lui Liouville rezultă că
p p X∞ 1 X∞ 1
qr α − > c. Pe de altă parte, q r α − = 4rk! n!
= n!−rk!

q q n=k+1
4 n=k+1
4

X 1 ∞ 1
X
= < c, conform alegerii lui k. Am obţinut o contradicţie
n=k+1
4(n−1)! j=k 4j!
şi am arătat că α este transcendent.

37. Este suficient să arătăm că mulţimea {{mα}|m ∈ Z} este densă ı̂n
[0, 1). Fie I ⊆ [0, 1) un interval de lungime ε < 1 şi fie n ∈ N∗ astfel ı̂ncât
1
< ε. Dacă k, h ∈ Z, k 6= h, atunci {kα} 6= {hα}. Dacă {kα} = {hα},
n
[kα] − [hα]
deducem că kα − [kα] = hα − [hα], α = ∈ Q, ceea ce constituie
k−h
o contradicţie. Considerăm cele n + 1 numere distincte {kα}, k = 1, n + 1
din intervalul [0, 1). Conform principiului cutiei, aplicat celor n + 1 numere
" !
j j+1
şi celor n intervale , , j = 0, n − 1, deducem existenţa numerelor
n n
naturale 1 ≤ h < k ≤ n + 1, 0 ≤ i ≤ n − 1, astfel ı̂ncât {hα} şi {kα} aparţin

157
" !
i i+1 1
intervalului , . De aici rezultă că |{kα} − {hα}| < . Deducem că
n n n
1 1
{α(k − h)} < sau {α(h − k)} < . Am arătat că există a ∈ Z, a 6= 0
n n
1 1
astfel ı̂ncât 0 < {aα} < . Deoarece < ε şi lungimea lui I este ε, deducem
n n
existenţa unui b ∈ N∗ astfel ı̂ncât b{aα} ∈ I. Deoarece I ⊆ [0, 1), este evident
că {baα} = b{aα} ∈ I şi problema este demonstrată.

38. Notăm cu inf{{nα}|n ∈ N∗ }. Arătăm că β = 0. Dacă β > 0 atunci


1 1
există n ∈ N∗ astfel ı̂ncât ≤ β < . Din definiţia infimumului de-
n+1 n
1 β+1 1
ducem existenţa unui m ∈ N∗ astfel ı̂ncât ≤ β ≤ γ < < ,
! n+1 n+1 n
β+1 1 1
γ = {mα} β < < deoarece β < . De aici rezultă că {(n+1)mα} =
n+1 n n
!
β+1
{(n + 1)γ} = (n + 1)γ − 1 < β deoarece γ < , ceea ce constituie o
n+1
contradicţie cu definiţia lui β. Deci β = 0. Fie acum I ⊆ [0, 1) un interval de
ε
lungime ε. Deoarece β = 0, rezultă că ∃ m ∈ N∗ astfel ı̂ncât 0 ≤ {mα} < . Nu
2 !
[mα]
putem avea {mα} = 0 căci ar rezulta că α este număr raţional α = ,
m
ε
ceea ce nu este adevărat. Deci 0 < {mα} < . Deoarece lungimea intervalului
2
ε
I este ε şi 0 < {mα} < , deducem că există n ∈ N∗ astfel ı̂ncât n{mα} ∈ I.
2
Deoarece n{mα} < 1, rezultă că {nmα} = n{mα} ∈ I. În acest moment am
demonstrat că mulţimea {{nα}|n ∈ N∗ } este densă ı̂n [0, 1).

158
CAPITOLUL 6

Baze de numere. Suma cifrelor

1. 2358(10) , 4466(8) .
3 3 3 1 1
2. 13, (03)(4) = 4 + 3 + 2
+ 4 +··· = 7+ 2 · = 7+ = 12, 1(5) .
4 4 4 1 5
1−
42
3. c = 10011(2) , r = 1000(2) .
4. a) 11; b) 9.
5. 19.
6. Numărul din enunţ este (b2 + b + 2)2 .
7. 11211(b) = (b2 + b + 1)(b2 + 1).
8. Avem (2b + 3)(b + 1) şi deci numărul este compus. Deoarece
(2b + 3, b + 1) = (2b + 3 − 2b − 2, b + 1) = (1, b + 1) = 1, rezultă 2b + 3 = x2 şi
b + 1 = y 2 şi de aici x2 − 2y 2 = 1. Soluţia minimală este x = 3, y = 2. Avem
2b + 3 = 32 şi deci b = 3. Este necesar ca b ≥ 6. Următoarele soluţii se obţin
√ € √ Šn √ √
din xn +yn 2 = 3 + 2 2 , x2 +y2 2 = 17+12 2 şi deci (x2 , y2 ) = (17, 12).
Pentru x = 17 avem 2b + 3 = 172 = 289 şi b = 143.
9. Avem b4 + 3b3 + 5b2 + 5b + 2 = (b2 + b + 2)(b + 1)2 şi deci
b4+ 3b3 + 5b2 + 5b + 2
= (b + 1)2 .
b2 + b + 2
10. Avem 49x + 7y + z = 121z + 11y + x cu x, z ∈ 1, 6 şi y ∈ 0, 6.
12x − y − 30z = 0 şi deci y = 6k. Convin tripletele (5, 0, 2), (3, 6, 1).
11. Avem ax + y = (a − 1)2 , adică y = a(a − 2 − x) +1 şi deci a|y − 1. Cum
y < a, rezultă y = 1 şi de aici x = a − 2 şi apoi yx(a) = ay + x = a + a − 2 =
2(a − 1).
c a
12. Avem 10a + b + = 20c + b + cu a, c ∈ 1, 9 şi b ∈ 0, 9; rezultă
10 20
a = 2c. Avem (a, c) ∈ {(2, 1), (4, 2), (6, 3), (8, 4)} şi b ∈ 0, 9. Numărul de soluţii
este 4 · 10 = 40.

159
13. Şirul are perioada 20. Trebuie să arătăm că 10 divide (n+20)n+20 −n.
Ţinând cont de formula binomială a lui Newton, este suficient să arătăm că
nn+20 −nn este multiplu de 10. Deoarece nn+20 −nn = nn (n20 −1) este multiplu
de n(n4 − 1) (dacă n ∈ N∗ ), enunţul rezultă imediat.

14. Avem condiţia 10n−1 ≤ nn < 10n . Rezultă imediat n < 10. După
verificări găsim n ∈ {1, 8, 9}.

15. Pentru b = 10 pătratele sunt 102 = 100, 112 = 121, . . . , 312 = 961.
Unele dintre acestea nu sunt pătrate pentru toate bazele b > 10. De exemplu,
152 = 225 scris ı̂n bază 11 este 2 · 112 + 2 · 11 + 5 = 269 care nu este pătrat
perfect. În final rămân 100, 121, 144, 169, 400, 441, 484, 900, 961.

16. Dacă raţia progresiei este 0, enunţul este evident. Fie a ∈ N, b ∈ N∗


şi k ∈ N astfel ı̂ncât 10k > a. Atunci s(a + 10k · b) = s(a) + s(b) şi enunţul este
demonstrat. Generalizarea acestei probleme face obiectul problemei 39.

17. Avem x2 = 99 = (10−1)9 = M100+90−1 = M100+89 şi deci ultimele


două cifre sunt 8 şi 9. Raţionăm prin inducţie. Avem xn+1 = (10 − 1)xn =
M100 + 10xn − 1 deoarece xn este impar. Avem

xn+1 = M100 + 10(M100 + 89) − 1 = M100 + 889 = M100 + 89.

18. Avem n · 9a = 10an−an = a(1234567890−123456789) = aa . . . a0a(10) ,


numărul cifrelor a fiind 9.

1
19. Fie n = 0, 11 . . . 1. Avem 9n = 0, 99 . . . 9 = 1 − şi deci
s 102001
√ 1 1 1 √ 1 1
1 > 9n = 1− > 1− , adică > n > − =
102001 102001 3 3 3 · 102001
0, 33 . . . 3} . . . . Aşadar primele 2001 zecimale sunt 3.
| {z
2001 cifre
(D. Popovici )

20. Numărul are 9+90·2+3 = 192 de cifre. După eliminare rămân 172 de
cifre. Numărul rămas va fi maxim dacă primele cifre vor fi mari. Eliminăm de
la ı̂nceputul numărului 1, 2, . . . , 8 pentru ca prima cifră să fie 9. Apoi eliminăm
cifrele situate după 9 şi anume 1, 0, 1, 1, 1, 2, . . . , 1, 4, 1 şi ar urma cifra 5. În
sfârşit eliminăm cifra 1 situată după 5. Numărul maxim este 9561718 . . . 99100.

160
n −1
2X
21. Numărul se scrie N = a 10k . Deoarece (1+x)(1+x2 )(1+x4 ) . . .
k=0
€ Š n−1
Y k
1+x 2n−1 = 1 + x + x2 + · · · + x2
n −1
, avem N = a (102 + 1). Deoarece
k=0
k h
(102 + 1, 102 + 1) = 1 pentru k 6= h, enunţul se verifică.
√ √
22. Fie A = xn , 102n xn = 10n · A. Avem 9xn < 102n şi deci 9x2n <
102n xn = B 2 . Rezultă [B] ≥ 3xn . Se arată că B < 3xn + 1 şi deci [B] = 3xn =
B
33 . .
| {z } . 3 . Deci A = = 33 . . . 3, 33 . . . 3} . . . .
| {z
10n
2n n

23. Pătratele perfecte sunt de forma M4 sau M4 + 1 şi deci numerele


n = aa . . . a nu sunt pătrate pentru a ∈ {1, 2, 5, 6, 9}. Pentru a = 3, a = 7 sau
a = 8, numărul nu este pătrat pentru că pătratele nu se termină ı̂n 3, 7 sau 8.
De asemenea, 44 . . . 4 = 4 · 11 . . . 1 nu este pătrat.

24. Suma cifrelor s(n) = 35 = 3k +2. Deoarece n ≡ s(n) (mod 9), rezultă
n ≡ s(n) (mod 3), adică n = 3h + 2. Pătratele perfecte sunt ı̂nsă de forma 3h,
3h + 1. Deci n nu este pătrat perfect.
k
X k
X
25. Presupunem că 2m = ai 10i şi 2n = aσ(i) 10i . Dacă m > n,
i=0 i=0
k
X k
X
atunci 2m < 10 · 2n şi deci m − n ≤ 3. Avem 2m = ai (9 + 1)i = M9 + ai
i=0 i=0
k
X k
X k
X
şi 2n = M9 + aσ(i) . Cum ai = aσ(i) , rezultă 2m − 2n = M9 adică
i=1 i=0 i=0
2n (2m−n − 1) = M9, ceea ce este ı̂n contradicţie cu 1 ≤ m − n ≤ 3.
k
X € Š
26. n = ai pi . ep (n) = ak pk−1 + ak−1 pk−2 + · · · + a2 p + a1 +
€ i=0 Š
ak pk−2 + ak−1 pk−3 + · · · + a3 p + a2 + · · · + (ak p + ak−1 ) + ak =
ak (pk − 1) + ak−1 (pk−1 − 1) + · · · + a1 (p − 1) n − sp (n)
= .
p−1 p−1
27. Avem s(n) = s(1000n) = s(125 · 8n) ≤ s(125)s(8n) ≤ 8s(8n). Am
folosit s(mn) ≤ s(m)s(n) (curs, pagina 39).

10k − 1
28. Fie mk = 10nk +k + 1 unde nk = (k ∈ N∗ ). Alegem
9
x = (10nk − 1)10k + 1 şi y = 10nk +k . Se arată că mk = x + s(x) = y + s(y).

161
29. Pentru x = 10n − 1 avem x2 = |99 {z
. . . 9} 8 00 . . . 0} 1 şi deci s(x2 ) = 9n.
| {z
n−1 n−1
Alegem x = 10n − 1, y = 10m − 1, z = 10m+n − 1.
m
X m
X
30. Pentru k = ai 10i avem k − s(k) = ai (10i − 1) = M9. În cazul
i=0 i=0
. . .
nostru a − n..9, b − n..9, a + b − n..9. Rezultă 9|(a + b − n) − (a − n) − (b − n) = n.
Deci n, a, b = M9. Arătăm că pentru n = 9k putem alege a = b cu proprietatea
cerută. Pentru n = 9 alegem a = b = 54 şi avem s(a) = s(b) = s(108) = 9.
Pentru n = 18 alegem a = b = 954 şi avem s(a) = s(b) = s(1908) = 18.
Pentru n = 9k alegem a = b = 999 | {z . . . 9} 54. Avem a + b = 1 9| . .{z
. 999} 08 şi deci
k−1 k−1
s(a) = s(b) = s(a + b) = 9k.
(M. Lascu, V. Zidaru)

31. Deoarece n ≡ s(n) (mod 9), deducem că a = s(s(s(20052005 ))) ≡


s(s(20052005 )) ≡ s(20052005 ) ≡ 20052005 (mod 9). Avem deci a ≡ 20052005 ≡
72005 = (76 )334 · 7 ≡ 7 (mod 9), deoarece 76 ≡ 1 (mod 9). Avem inegalitatea
20052005 < 104·2005 şi deci s(20052005 ) ≤ 9 · 4 · 2005 = 72180. De aici rezultă
că s(s(20052005 )) ≤ s(69999) = 42, a = s(s(s(20052005 ))) ≤ s(39) = 12. Din
1 ≤ a ≤ 12, a ≡ 7 (mod 9), deducem că a = 7.

32. N = b2n + bn + 1. Fie ε ∈ C, astfel ı̂ncât ε2 + ε + 1 = 0 şi f (x) =


.
x2n + xn + 1. Deoarece 3 - n, avem că f (ε) = 0 şi deci f ..x2 + x + 1. Cum
.
N ..b2 + b + 1 şi N > b2 + b + 1 (b ≥ 2, n ≥ 2), rezultă că N este compus.
Observaţie. Enunţul se păstrează dacă n ∈ N, n ≥ 2, n 6= 3k .
Soluţie. n = 3k · m, m ∈ N, 3 - m, m ≥ 2. Avem că b2n + bn + 1 =
.
c2m + cm + 1 .. c2 + c + 1, c2m + cm + 1 > c2 + c + 1. În cele de mai sus
k
c = b3 ≥ 2 şi am ţinut cont de cazul precedent (3 - m).
!
k−1
Y
33. n = ak ak−1 . . . a1 a0 , p(n) = ai ak ≤ 9k ·ak ≤ ak ·10k ≤ n. Avem
i=0
deci că n2 − 15n + 5 = p(n) ≤ n, n2 − 16n + 5 ≤ 0 şi cum rădăcinile ecuaţiei

16 ± 236 √
de grad 2, x2 − 16x + 5 = 0, sunt x1;2 = = 8 ± 59, deducem că
2
n ∈ {1, 2, . . . , 15}. Cum n2 −15n +5 < 0 pentru n = 1, 14, observăm că n = 15
este singura soluţie a problemei.

34. Fie n ∈ M , 10k ≤ n < 10k+1 . Numărăm câte astfel de numere există.
n are k + 1 cifre. Prima cifră poate fi oricare ı̂n intervalul [1, 8], iar celelalte

162
cifre pot fi oricare ı̂n intervalul [0, 8]. Există 8 · 9k astfel de numere. Deci
!k !k
X 1 1 9 X 1 ∞
X 9 1
≤ k ·8·9k = 8 şi ≤8 =8 = 80.
n 10 10 n∈M
n k=0
10 9
10k ≤n<10k+1
1−
10
Am arătat că suma din enunţ este convergentă şi este cel mult 80.

35. Se arată imediat formula s(2k) = 2s(k) − 9L(k), valabilă pentru


! !
Xm L(2k ) 1X m 2s(2k ) − s(2k+1 ) 1 s(2m+1 )
∀ k ∈ N. = = 2− . De aici
k=0
2k 9 k=0 2k 9 2m
∞ L(2n )
X 2
rezultă că = . Am ţinut cont ı̂n cele de mai sus de faptul că
n=0
2n 9
s(n)
lim = 0. Pentru n ∈ N∗ , fie k ∈ N astfel ı̂ncât 10k ≤ n < 10k+1 .
n→∞ n
s(n) 9(k + 1)
Avem că ≤ . Când n tinde la infinit, k tinde la infinit. Cum
n 10k
9(k + 1) s(n)
lim k
= 0, deducem că lim = 0 şi demonstraţia problemei este
k→∞ 10 n→∞ n
ı̂ncheiată.

36. Cazul I : (n, 10) = 1. Fie h ∈ N∗ astfel ı̂ncât 10h ≡ 1 (mod n) (alegem
de exemplu h = ϕ(n)) şi B ∈ N, 0 ≤ B ≤ n − 1, astfel ı̂ncât k + 9B ≡ 0(n)
(am folosit aici că 3 - n). Alegem A = k − B ≥ n − B ≥ 1 şi N = (10h + 102h +
103h + · · · + 10Ah ) + (10h+1 + 102h+1 + · · · + 10Bh+1 ). Suma cifrelor lui N este
A + B = k iar N ≡ A + 10B = k + 9B ≡ 0(n). Aceasta trebuia demonstrat.
Cazul II : Fie n = 2s 5t · m, (m, 30) = 1, s, t ∈ N; r = max{s, t}. Din cazul
precedent ştim că ∃ N ∈ N astfel ı̂ncât m|N şi suma cifrelor lui N este k.
Alegem numărul 10r · N care are evident proprietăţile căutate.
(Soluţie: John Scholes)

37. Fie n scris ı̂n baza 10 sub forma n = ak . . . a1 a0 , unde k ≥ 8. Arătăm


4
10m
că n > s(n) şi deci nu avem soluţii cu k ≥ 8. Considerăm şirul bm = ,
(m + 1)4
!4 !4
bm+1 m+1 2 160
m ∈ N∗ . = 10 ≥ 10 = > 1. Deci şirul bm este
bm m+2 3 81
108
crescător. Cum b8 = > 15241 > 94 = 6561, rezultă că bm > 94 ∀ m ≥ 8.
94
Avem că n ≥ 10k > 94 (k + 1)4 ≥ s(n)4 , deoarece k ≥ 8 şi s(n) ≤ 9(k + 1). Deci
nu avem soluţii pentru k ≥ 8. Arătăm că nu avem soluţii nici pentru k = 7 şi
a7 ≥ 2. Dacă a7 ≥ 3, atunci n ≥ 3 · 107 > 724 = 26873856 = (8 · 9)4 ≥ s(n)4 .

163
Dacă a7 = 2, atunci n ≥ 2 · 107 > 654 = 17850625 ≥ s(n)4 . Deci k ≤ 7
şi dacă k = 7, atunci a7 = 1. Cum n = m4 < 2 · 107 , rezultă că m ≤ 66.
Verificând valorile de la m = 0 la 66, găsim următoarele soluţii ale problemei
noastre: n = 0, n = 1, n = 74 = 2401, n = 224 = 234256, n = 254 = 390625,
n = 284 = 614656 şi cea mai mare soluţie n = 364 = 1679616.
(A. Gica)

38. Fie n = 99 . . . 9, cifra 9 apărând de 2k + 1 ori. Presupunem că


n = a + b, s(a) = s(b). Scriem a = a2k a2k−1 . . . a1 a0 , b = b2k b2k−1 . . . b1 b0 ,
aj , bj ∈ {0, 1, . . . , 9}, ∀ j = 0, 2k. Din a + b = n = 99 . . . 9, deducem că
2k
X
aj + bj = 9, ∀ j = 0, 2k. Rezultă contradicţia (2k + 1)9 = (aj + bj ) =
j=0
s(a) + s(b) = 2s(a). Enunţul este demonstrat.

39. Dacă raţia este zero, enunţul este evident. Analizăm cazul când raţia
a este un număr strict pozitiv. Notăm cu b primul termen al şirului. Fie
" k #
∗ k
10 + a − b
k ∈ N astfel ı̂ncât 10 ≥ max{a, b}. Considerăm nk = ∈ N.
a
" #
10k + a − b
Avem că ank = a + b ≤ (10k + a − b) + b = 10k + a şi că
a
!
10k + a − b
ank > a − 1 + b = 10k . Deci 10k ≤ [ank ] ≤ 10k + a şi de aici
a
deducem că există o infinitate de indici (nk )k astfel ı̂ncât s ([ank ]) = 1 + j,
0 ≤ j ≤ [a].
(L. Panaitopol )

40. Vom arăta că problema nu are soluţii pentru b ≥ 1396. Presupunem
că am avea b ≥ 1396 care să satisfacă proprietatea din enunţ.
”√ √ Š
Fie x, x + 1, . . . , x + k − 1, toate numerele naturale y din intervalul b, 2b .
√ √
Deci x − 1 < b ≤ x < x + 1 < · · · < x + k − 1 < 2b ≤ x + k. Fie y ca mai
sus. Ultima cifră a lui y 2 ı̂n baza b este y 2 − b deoarece b ≤ y 2 < 2b. Folosind

proprietatea din enunţ a lui b, deducem existenţa lui c ∈ N, 0 ≤ c < b, astfel
” √ Š
ı̂ncât y 2 − b = c2 . Pentru fiecare j = 0, k − 1 avem un cj ∈ N ∩ 0, b astfel
ı̂ncât (x + j)2 − c2j = b. Evident că cj < cj+1 ∀ j = 0, k − 2. Arătăm că are loc
inegalitatea mai bună cj+1 ≥ cj + 3. Să presupunem că cj+1 = cj + 1. Atunci
(x + j + 1)2 − (x + j)2 = (cj + 1)2 − c2j = 2cj + 1 = 2(x + j) + 1, cj = x + j,
b = 0; am obţinut o contradicţie. Dacă cj+1 = cj + 2, rezultă contradicţia
(x + j + 1)2 ≡ (x + j)2 (mod 2). Deci cj+1 ≥ cj + 3, ∀ j = 0, k − 2. Adunând

164
toate aceste inegalităţi obţinem că ck−1 ≥ c0 + 3(k − 1) ≥ 3k − 3. Avem că
”√ — ”√ — √
k= 2b − b + ε, unde ε ∈ {−1, 0, 1} şi deci inegalităţile b > ck−1 ≥
”√ — ”√ — €√ Š √ √ √
3k − 3 ≥ 3 2b − 3 b − 6 > 3 2b − 1 − 3 b − 6 = 3 2b − 3 b − 9.
€ √ Š
√ 9 9 3 2+4
Rezultă că b < √ = < 37, 35, b < 1396, ceea ce con-
3 2−4 2
stituie o contradicţie. Se fac verificările şi se obţine că singurele soluţii sunt
b = 2, 3, 4, 5, 8, 12, 16.

41. Fie k ∈ N, n = 2k − k ∈ N∗ , a = 22 . . . 211 . . . 1, unde cifra 2 apare


de k ori şi cifra 1 de 2k −2k ori. Avem că p(a) = 2k şi s(a) = 2k +(2k −2k) =
2k = p(a). Am arătat că există o infinitate de numere n pentru care propoziţia
este adevărată. Pentru partea a doua a enunţului considerăm k0 ∈ N astfel
2k 2m
ı̂ncât 2 6 > 9(210)k , ∀ k ∈ N, k ≥ k0 (k0 există deoarece lim = ∞).
m→∞ m
Alegem n = 210k = (2 · 3 · 5 · 7)k . Vom arăta că nu există a ∈ N cu n cifre
astfel ı̂ncât s(a) = p(a). Să presupunem că ar exista un astfel de a. Se arată
imediat că a nu conţine cifra 0. Pentru fiecare i ∈ N, 1 ≤ i ≤ 9, notăm cu
9
X
xi numărul de cifre i din scrierea lui a ı̂n baza 10. Avem că xi = n. Din
i=1
9
Y 9
X 9
X
s(a) = p(a), deducem că ix i = ixi = n + (i − 1)xi ≥ n = (2 · 3 · 5 · 7)k
i=2 i=1 i=2
şi că 2x2 +x6 +2x4 +3x8 · 3x3 +x6 +2x9 · 5x5 · 7x7 > 2k · 3k · 5k · 7k . De aici deducem
că cel puţin unul din exponenţii din stânga este mai mare strict decât k.
Notăm cu b baza exponentului respectiv (deci b ∈ {2, 3, 5, 7}). Avem că bk |n
9
Y 9
Y 9
X 9
X
şi bk ixi şi din relaţia ix i = n + (i − 1)xi rezultă că bk (i − 1)xi .
i=2 i=2 i=2 i=2
Ultima sumă nu poate fi 0 (căci ı̂n acest caz a ar fi alcătuit doar din cifra 1 şi
9
X
1 = p(a) = s(a) = n > 1, contradicţie) şi deci (i − 1)xi ≥ bk ≥ 2k . Avem
i=2
P
9
(i−1)xi
i=2
că 9(210)k = 9n ≥ s(a) = p(a) ≥ 2x2 +x3 +2x4 +x5 +x6 +x7 +3x8 +2x9 ≥ 2 6 ≥
2k 2k
2 şi am obţinut contradicţia
6 9·210k ≥2 6 . Enunţul este demonstrat ı̂n acest
moment.

42. A rezolva exerciţiul este acelaşi lucru cu a arăta existenţa numerelor


k, n ∈ N astfel ı̂ncât M · 10k ≤ 2n < 10k (M + 1). Acest lucru este echivalent
cu lg M + k ≤ n lg 2 < lg(M + 1) + k. Notăm k0 = [lg M ].

165
M +1
Cazul I : {lg M } + lg < 1. În acest caz lg(M + 1) = lg M +
M
M +1 M +1
lg = [lg M ] + {lg M } + lg , [lg(M + 1)] = k0 , {lg(M + 1)} =
M M
M +1
{lg M } + lg > {lg M }. Deoarece lg 2 nu este raţional, există o infinitate
M
de n ∈ N astfel ı̂ncât {lg M } < {n lg 2} < {lg(M + 1)} (aceasta deoarece dacă
α 6∈ Q, mulţimea {{nα}|n ∈ N∗ } este densă ı̂n [0, 1); problema 38, Capitolul
5). Întrucât există o infinitate de n-uri cu proprietatea de mai sus, va exista
şi unul pentru care [n lg 2] ≥ k0 . Inegalităţile {lg M } < {n lg 2} < {lg(M + 1)}
ne conduc la lg M + [n lg 2] − k0 < n lg 2 < lg(M + 1) + [n lg 2] − k0 . Alegem
k = [n lg 2] − k0 ∈ N şi problema este demonstrată.
M +1
Cazul II : {lg M }+lg ≥ 1. În acest caz lg(M +1) = [lg M ]+{lg M }+
M
M +1
lg = [lg(M + 1)] + {lg(M + 1)}, [lg(M + 1)] = k0 + 1, {lg(M + 1)} =
M
M +1
{lg M } + lg − 1 < {lg M } (deoarece M + 1 < 10M ). Există o infinitate
M
de n ∈ N astfel ı̂ncât {lg M } < {n lg 2} < 1 ≤ 1 + {lg(M + 1)}. Alegem n astfel
ı̂ncât să satisfacă şi inegalitatea [n lg 2] ≥ k0 . Avem că {lg M } = lg M − k0 <
{n lg 2} = n lg 2 − [n lg 2] < 1 + {lg(M + 1)} = lg(M + 1) − k0 . Alegem
k = [n lg 2] − k0 şi din nou avem inagalităţile lg M + k < n lg 2 < lg(M + 1) + k.
Cazul II este demonstrat şi cu el ı̂ntreaga problemă.

43. Fie a numărul format din primele două cifre ale lui 2n şi 5n . Rezultă
prin verificări că n ≥ 4. Avem a10k < 2n < (a+1)10k şi a10h < 5n < (a+1)10h .
După ı̂nmulţirea acestor relaţii avem a2 10k+h < 10n < (a + 1)2 10k+h . Cum
10 ≤ a < 100, rezultă că 102 < a2 < (a + 1)2 ≤ 104 , 10k+h+2 < 10n < 10k+h+4

şi deci n = k + h + 3, a2 < 103 < (a + 1)2 . Rezultă a < 1000 < a + 1 şi
deci a < 31, 6 · · · < a + 1, adică a = 31. Vom arăta că există numere n cu
proprietatea din enunţ. Deci 31 · 10k < 2n < 32 · 10k şi 31 · 10h < 5n < 32 · 10h .
Rezultă de aici lg 31 < n lg 2 − k < lg 32 (1) şi lg 31 < n lg 5 − h < lg 32 (2).
Relaţia (2) se scrie lg 31 < n−h−n lg 2 < lg 32 (20 ). Cum h = n−k −3, (20 ) se
scrie lg 31 < k + 3 − n lg 2 < lg 32 (200 ). Avem 0, 49 < 2 − lg 32 < n lg 2 − k − 1 <
2 − lg 31 < 0, 51. Folosind problema 38 de la Capitolul 5, deducem existenţa
lui n ∈ N∗ , n ≥ 4 pentru care 2 − lg 32 < {n lg 2} < 2 − lg 31. Alegem
k + 1 = [n lg 2] ≥ 1 şi problema este rezolvată.

166
CAPITOLUL 7

Funcţii aritmetice
k
Y
1. Pentru n = pαi i , τ (n) este impar ⇔ αi + 1 impar ⇔ αi = 2βi ,
i=1
!2
k
Y
∀ i ∈ 1, k, deci n = pβi i .
i=1

n n n
2. Divizorii sunt d1 , d2 , . . . , dk sau , , . . . , (k = τ (n)). P 2 =
d1 d2 dk
k
Y k n
Y Y
di = nτ (n) , unde P = d.
i=1 i=1
di d|n

Y τ (n)
3. a) Dacă P = d, ştim din problema precedentă că P = n 2 . Me-
d|n
È
√ τ (n) τ (n) √
dia geometrică a divizorilor lui n este τ (n) P = n 2 = n. Deci n are
proprietatea din enunţ dacă şi numai dacă este pătrat perfect.
τ (n)
b) Dacă n are proprietatea din enunţ, atunci P = n2 . Dar P = n 2 şi deci
τ (n) = 4. De aici rezultă că n = p3 , p prim sau n = pq, p, q prime distincte.
k
Y
4. Pentru n = pαi i fie A = {1, 2, . . . , k}. Dacă M ⊂ A, M =
i=1
αi αi α n
{i1 , i2 , . . . , ir } se consideră factorii d = pi1 1 pi2 2 . . . pirir şi . Avem Ckr posi-
d
k
X
bilităţi şi ı̂n total Ckr = 2k descompuneri.
r=0

1
5. Avem x, y > n şi deci x = n + r, y = n + s. Ecuaţia devine +
n+r
1 1
= şi deci sr = n2 . Deducem că r este orice divizor al lui n2 . Numărul
n+s n
de soluţii este deci τ (n2 ).
Observaţie. Problema mai apare cu o altă soluţie la Capitolul 3, problema 21.

167
k
Y k
Y k
Y
6. Fie n = pαi i . Avem τ (ns ) − τ (nt ) = (sαi + 1) − (tαi + 1).
i=1 i=1 i=1
Fie s = t + y. Avem
k
Y k
Y
τ (ns ) − τ (nt ) = ((tαi + 1) + yαi ) − (tαi + 1) =
i=1 i=1
k
Y k
Y
= (tαi + 1) + My − (tαi + 1) = My = M(s − t).
i=1 i=1
k
Y h
Y βi
7. Fie m = (pαi i ), n = ri . Avem
i=1 i=1
k
Y k
Y
s s
τ (m ) − τ (n ) = (sαi + 1) − (sβi + 1) = Ms + 1 − (Ms + 1) = Ms.
i=1 i=1
α1 α2 αi
8. a) Fie n = p1 p2 . . . pi , (α1 + 1)(α2 + 1) . . . (αk + 1) = 18. Pentru
k= 1 ⇒ n = p17 . Pentru k = 2 ⇒ n = p8 q, sau n = p5 q 2 . Pentru k = 3 ⇒
n = p2 q 2 r. n minim este 22 · 32 · 5 = 180. Nu putem avea k ≥ 4 deoarece 18
nu se scrie ca produs de cel puţin 4 divizori supraunitari.
b) n = 360.
r
Y r
Y
9. a) Fie n = pαi i . Rezultă că σ(n) = σ (pαi i ).
i=1 i=1
Deoarece σ (pαi i ) > 1 şi σ(n) este prim, deducem că r = 1, adică n = pα ,
α
pab − 1
p prim. Dacă α + 1 = a · b, a > 1, b > 1, atunci σ(p ) = =
€ Š p−1
(pa )b−1 + (pa )b−2 + · · · + (pa )1 + 1 (pa−1 + pa−2 + · · · + p + 1) este număr
compus.
b) Se aplică punctul a) şi se obţin soluţiile n = 16 şi n = 25.
k
Y k
Y
10. n = pαi i , ϕ(n) = pαi i −1 (pi −1). k ≥ 3 ⇒ ϕ(n) ≥ (2−1)(3−1)(5−1) = 8.
i=1 i=1
a) Avem k ≤ 2. Dacă n = pα , avem pα−1 (p − 1) = 4 şi n ∈ {5, 8}. Dacă
n = pα q β , avem pα−1 q β−1 (p − 1)(q − 1) = 4 şi n ∈ {10, 12}.
b) n ∈ {7, 9, 14, 18}.
c) 3(p1−1)(p2−1) . . . (pk −1) = p1 p2 . . . pk ⇒ p1 = 3 şi apoi 2(p2−1)...(pk−1) =
p2 ...pk ⇒ p2 = 2 şi contradicţia (p3 − 1) . . . (pk − 1) = p3 . . . pk pentru k ≥ 3.
Deci n = 2α 3β , α, β ≥ 1.
11. a) Dacă a = 1, atunci m = 1. Să presupunem că n ≥ 2. n = pα1 1 . . . pαr r ,
p1 < p2 < · · · < pr , pi prim, αi ∈ N∗ , ∀ i = 1, r. Ecuaţia noastră se rescrie

168
m(p1 − 1)(p2 − 1) . . . (pr − 1) = p1 p2 . . . pr . Dacă r ≥ 3, atunci membrul stâng
se divide cu 4, pe când cel drept nu se divide. Deci r = 1 sau r = 2. Dacă
m
r = 1, avem că m(p1 − 1) = p1 , p1 |m, (p1 − 1) = 1, p1 − 1 = 1, p1 = 2,
p1
m = 2. Deci, ı̂n acest caz, m = 2, n = 2α , α ∈ N∗ . Dacă r = 2, atunci
m(p1 − 1)(p2 − 1) = p1 p2 . Dacă p1 ≥ 3, atunci obţinem aceeaşi contradicţie ca
mai sus (4 divide membrul stâng, pe când cel drept nu are această calitate).
m
Deci p1 = 2, m(p2 − 1) = 2p2 , p2 |m, (p2 − 1) = 2. De aici rezultă că
p2
p2 = 3 = m şi soluţia este n = 2α · 3β , α, β ∈ N∗ .
b) Nu putem avea că n = 1, n = pα1 1 . . . pαr r , p1 < p2 < · · · < pr , pi prim,
αi ∈ N∗ , ∀ i = 1, r. Ecuaţia noastră se rescrie sub forma m(p1 −1) . . . (pr −1) =
(m−1)p1 p2 . . . pr . Cum pr |m(p1 −1) . . . (pr −1) şi pr - (pi −1, ∀ i = 1, r (deoarece
1
pi − 1 ≤ pr − 1), deducem că pr |m. Avem că m = pr · k, k ∈ N∗ şi 1 − ≥
! ! ! ! m!
1 1 1 1 1 1 1
1− ≥ 1− . . . 1− . Cum 1 − = 1 − 1− . . . 1− ,
pr p1 pr m p1 p2 pr
1 1
inegalităţile de mai sus sunt chiar egalităţi. Din 1 − =1− deducem că
m pr
! !
1 1 1
m = pr , iar din 1 − = 1− . . . 1− rezultă că r = 1. Deci neapărat
pr p1 pr
m este număr prim şi n = mk , k ∈ N∗ .

n n(m − 1)
12. Dacă = m, rezultă ϕ(n) = şi se foloseşte pro-
n − ϕ(n) m
blema 11.

€ Š
m
Y m
Y σ pαk k s
13. Pentru n = pαi i se alege s = 1 + (1 + αi ) şi avem € Š =
i=1 i=1 σ pαk k
pkαk s+1 − 1 .
∈ N deoarece αk s+1 .. αk +1. Se ţine seama că σ este multiplicativă.
pαk k +1 −1

σ(n) 1 + p + p2 + p3
14. Alegem n = p3 cu p prim p ≥ 3 şi avem = =
τ (n) 4
(p + 1)(p2 + 1)
∈ N, deoarece p + 1 şi p2 + 1 sunt pare.
4

169
ϕ(m)
15. Fie p prim. Alegem m = pα k, n = pβ k, (k, p) = 1 şi avem =
! ! m
ϕ(pα )ϕ(k) 1 ϕ(k) ϕ(n) 1 ϕ(k) ϕ(m)
= 1− şi = 1− = .
pα k p k n p k n

k
Y h
Y k
Y j
Y
βi yi
16. Fie m = pαi i qixi , n = pi ri cu q1 < q2 < · · · < qh şi
i=1 i=1 i=1 i=1
Yh
r1 < r2 < · · · < rj . Se ajunge la concluzia qh | (qi − 1), care este evident o
i=1
contradicţie.

17. Avem {a1 , a2 , . . . , aϕ(n) } = {n − a1 , n − a2 , . . . , n − aϕ(n) } şi deci


ϕ(n) ϕ(n) ϕ(n)
X X X
ai = (n − ai ), adică 2 ai = n · ϕ(n).
i=1 i=1 i=1
" # " #
1 1
18. (n, k) > 1, atunci = 0. Dacă (n, k) = 1 avem = 1.
(n, k) (n, k)

19. a) Luăm n prim şi demonstraţia este analogă cu cea de la punctul b).

b) Dacă n + 1 este prim, n ≥ 3, atunci n este compus σ(n) ≥ n + n

(curs, pagina 57) şi σ(n + 1) = n + 2. Rezultă σ(n + 1) − σ(n) ≤ 2 − n şi
deci lim inf(σ(n + 1) − σ(n)) = −∞.
n→∞

X 1 X 1 X 1 1 X 1
20. a) ≥ = ≥ = ∞.
n≥1
τ (n)σ(n) p prim τ (p)σ(p) p prim 2(p + 1) 3 p prim p

b) τ (n) = O(nα ) (curs, pagina 53) şi deci τ (n) < M 4 n. Cum σ(n) ≥ n,
!2
τ (n) M2
rezultă < √ şi enunţul este imediat.
σ(n) n n

21. Avem σ(n) ≥ n şi ϕ(n) ≥ n pentru n ≥ 7 (curs, pagina 64). Rezultă
1 1 X∞ 1
≤ 3 pentru n ≥ 1 şi cum 3 converge, rezultă enunţul.
σ(n)ϕ(n) n 2 n=1 n 2

k
Y h
Y k
Y j
Y
βi γi
22. Fie m = pαi i qi , n = δi
pi ri . Avem ϕ(m) · ϕ(n) =
i=1 i=1 i=1 i=1
!2 ! j !
k
Y 1 h
Y 1 Y 1
mn 1− 1− 1− . Factorii primi ai lui d sunt p1 , . . . , pk ,
i=1
pi i=1
qi i=1
ri

170
iar cei ai lui M sunt p1 , . . . , pk , q1 , . . . , qh , r1 , . . . , rj .
k  2 r   j  ‹
Y 1 Y 1 Y 1
ϕ(M )ϕ(d) = M d 1− 1− 1− .
i=1
pi i=1
qi i=1
ri
Enunţul rezultă din mn = M · d.

23. a) Cu notaţiile din problema precedentă


€ Š
σ(mn) k
Y pαi i +γi +1 − 1 (pi − 1)
= € Š€ Š ≤ 1,
σ(m)σ(n) i=1 pαi i +1 − 1 pγi i +1 − 1
egalitatea obţinându-se doar pentru (m, n) = 1.
b) Se tratează analog cu problema 22.

24. Fie n = pα1 1 . . . pαk k q1γ1 . . . qhγh , m = pβ1 1 . . . pβk k r1δ1 . . . rtδt , descompune-
rile standard ı̂n factori primi ale numerelor m şi n. Avem că
k
Y h
Y t
Y
τ (mn) = (1 + αi + βi ) (γi + 1) (δi + 1) şi
i=1 i=1 i=1
!
k
Y h
Y t
Y
τ (m)τ (n) = (1 + αi )(1 + βi ) (γi + 1) (δi + 1).
i=1 i=1 i=1

Enunţul rezultă imediat din inegalitatea evidentă 1+αi +βi ≤ (1+αi )(1+βi ),
valabilă pentru ∀ i = 1, k.

25. Dacă n este compus avem σ(n) > n + n (curs, pagina 57). Cum
τ (n) ≥ 2, rezultă τ (n)σ(n) > 2n + 2. Dacă n este prim, avem egalitate.
k
Y h
Y k
Y l
Y
βi γi
26. Fie m = pαi i qi , n = σi
pi ri ,
i=1 i=1 i=1 i=1
!2 ! !
k
Y 1 h
Y 1 Y l 1
ϕ(m)ϕ(n) = mn 1− 1− 1− ≤
i=1
pi i=1
qi i=1 ri
! ! !
k
Y h
Y 1 Y l 1
1
≤ mn 1− 1− 1− = ϕ(mn).
i=1
pi i=1
qi i=1 ri
! !
k
Y 1 Y h 1
Avem nϕ(m) = mn 1− 1− ≥ ϕ(mn).
i=1
pi i=1 qi

27. Avem ϕ(m)+ϕ(n) = ϕ(mn) ≥ ϕ(m)ϕ(n). Deci (ϕ(m)−1)(ϕ(n)−1) ≤ 1.


Avem că ϕ(n) sau ϕ(m) ∈ {1, 2}. Este imediat că m 6= 1 şi n 6= 1. Dacă
ϕ(n) = 1, rezultă că n = m = 2. Dacă ϕ(m) = 1, rezultă analog că n = m = 2.

171
Dacă ϕ(n) > 1, ϕ(m) > 1, deducem că ϕ(m) = ϕ(n) = 2, m, n ∈ {3, 4, 6}.
Singurele soluţii ı̂n acest caz sunt (m, n) = (3, 4), (4, 3).
k
Y h
Y k
Y l
Y
βi γi
28. Fie m = pαi i qi , n = δi
pi ri , pi 6= qj , pi 6= rj , qi 6= rj .
i=1 i=1 i=1 i=1
Avem
!2 !2 !2
k
Y h
Y l
Y
2 2 2 1 1 1
ϕ (mn) = m n 1− 1− 1− ≤
i=1
pi i=1
qi i=1
ri
!2 ! !
k
Y h
Y l
Y
2 2 1 1 1
≤m n 1− 1− 1− = ϕ(m2 )ϕ(n2 ).
i=1
pi i=1
qi i=1
ri

29. a) Deoarece funcţiile τ , ϕ şi σ sunt multiplicative, este suficient să


demonstrăm inegalitatea pentru n = pα , p prim, α ∈ N∗ . τ 2 (pα )ϕ(pα ) =
α+1
!2 α+1
!2
p 1 p 1
(α + 1)2 pα−1 (p − 1) ≥ 4pα−1 (p − 1) = 4 1− ≥ 4 1− =
p−1 p p−1 2
pα+1 pα+1 −1
> = σ(pα ). În mod analog se demonstrează punctele b), c), d).
p−1 p−1

30. Ştim că τ (n) ≤ 2 n pentru n ∈ N∗ (curs, pagina 52) şi că
σ(n) < n ln n pentru n ≥ 7 (curs, pagina 57). pentru a demonstra enunţul,

este suficient să demonstrăm că 2 n n ln n < n2 pentru n ≥ 81 (pentru
13 ≤ n ≤ 80, inegalitatea din enunţ se verifică uşor). Deci trebuie arătat că

√ 0
1 2 x−4
f (n) = n−2 ln n > 0 pentru n ≥ 81. Avem că f (x) = √ − = >0
2 x x 2x
pentru x > 16. Deci f este strict crescătoare pe intervalul [16, +∞). Cal-
culăm f (81) = 9 − 8 ln 3 şi observăm că ultimul număr este strict pozitiv.
Aceasta este echivalent cu a arăta că e9 > 38 . Aceasta este adevărat deoarece
327
e9 > (2, 7)9 = 9 > 38 (aceasta este adevărat deoarece 319 > 109 ). Din cele
10 √
de mai sus rezultă că n > 2 ln n pentru n ≥ 81 şi enunţul este demonstrat.
(Soluţie: M. Pârvulescu)

31. Fie A = {k ∈ N | k ≤ n−1, (k, n) = 1} şi B = {d ∈ N∗ | d|n}, unde n ∈ N,


n ≥ 2. Avem că A ∩ B = {1}, |A| = ϕ(n), |B| = τ (n) şi A ∪ B ⊆ {1, 2, . . . , n},
|A ∪ B| ≤ n. Pe de altă parte, |A ∪ B| = |A| + |B| − |A ∩ B| = ϕ(n) + τ (n) − 1 şi
de aici rezultă imediat că ϕ(n) + τ (n) − 1 ≤ n. Egalitate avem dacă şi numai
dacă A ∪ B = {1, 2, . . . , n}. Arătăm că acest lucru se ı̂ntâmplă doar pentru n
prim şi n = 4. De asemenea, n = 1 verifică egalitatea ϕ(n) + τ (n) = n + 1.
Să presupunem că ∃ p < q, p şi q prime, astfel ı̂ncât pq|n. Avem n = pα q β m,

172
m, α, β ∈ N∗ , p - m, q - m. Avem că pα+1 - n, pα+1 < pα · q ≤ n, ceea ce arată
că pα+1 ∈ {1, 2, . . . , n}\{A ∪ B}. Deci n este de forma pα , p prim, α ∈ N∗ . Să
presupunem că α ≥ 2. Dacă p 6= 2, atunci 2p < pα , (2p, pα ) 6= 1, 2p - pα şi
am ajuns la o contradicţie. Deci p = 2. Dacă α ≥ 3, avem că 6 - 2α , 6 < 2α ,
(6, 2α ) 6= 1; imposibil. Deci singurele soluţii sunt n = 1, n = 4 şi n este prim.

32. Dacă n este prim, avem egalităţile σ(n) = n + 1, ϕ(n) = n − 1,


τ (n) = 2 şi este evident că σ(n) = ϕ(n) + τ (n). Fie acum n ∈ N∗ astfel

ı̂ncât σ(n) = ϕ(n) + τ (n). Evident că n 6= 1. Ştim că τ (n) ≤ 2 n (curs,

pagina 52). Presupunem prin absurd că n este compus. Atunci σ(n) > n + n

(curs, pagina 57) şi ϕ(n) ≤ n − n (curs, pagina 64). Obţinem contradicţia
√ √ √ √
n + n < σ(n) = ϕ(n) + τ (n) ≤ (n − n) + 2 n = n + n. Deci n este prim
şi enunţul este demonstrat.
(Soluţie: M. Pârvulescu)

33. Dacă n este prim, atunci nτ (n) = 2n şi σ(n)+ϕ(n) = (n+1)+(n−1) =


2n = nτ (n). Reciproc, fie n ∈ N∗ astfel ı̂ncât σ(n) + ϕ(n) = nτ (n); evident
că n ≥ 2. Dacă n = pα1 1 pα2 2 . . . pαr r , p1 < p2 < · · · < pr , pi prim, αi ∈ N∗ ,
!
σ(n) Y r 1
∀ i = 1, r, ı̂mpărţind relaţia anterioară la n, obţinem că + 1− =
n i=1
pi
σ(n) X1 X
τ (n). Însă = şi τ (n) = 1, ceea ce ne conduce la concluzia că
n d|n
d d|n
! !
r
Y 1 X 1
1− = 1− . Pe de altă parte, avem inegalităţile evidente
i=1
pi d|n
d
! ! ! !
X 1 1 r
Y 1 X 1 r
Y 1
1− ≥1− ≥ 1− . Deoarece 1− = 1− ,
d|n
d p1 i=1 pi d|n
d i=1
pi
!
X 1 1
din inegalităţile precedente deducem că r = 1 şi că 1− =1− .
d|pα
d p1
1
De aici obţinem că α = 1 şi deci n = p1 este număr prim.
k
Y
34. a) Avem n = pαi i , p1 < p2 < · · · < pk (pj prim, αj ∈ N∗ , ∀ j = 1, k).
i=1
! ! ! !
ϕ(n) Y k 1 1 1 1 1
Avem că = 1− ≥ 1− 1− ... 1− = şi
n j=1
pj 2 3 k+1 k+1
ln 2n ϕ(n) 1 ln n
n ≥ 2k . De aici deducem că k + 1 ≤ şi ≥ ≥ .
ln 2 n k + 1 ln 2n

173
τ (n)ϕ(n)
b) Notăm f (n) = . Deoarece f este multiplicativă, este suficient
σ(n)
2(p − 1)
să arătăm că f (ps ) ≥ 1, ∀ p prim, p ≥ 3 şi s ∈ N∗ . f (p) = ≥ 1. Pentru
p+1
(s + 1)ps−1 (p − 1)2 3(p − 1)2 ps−1 p2 · ps−1
s ≥ 2 avem f (ps ) = ≥ > > 1.
ps+1 − 1 ps+1 − 1 ps+1 − 1

σ(n)ϕ(n) s
ps+1 −1 ps−1 (p −1) 1
c) f (n) = . Avem că f (p ) = · = 1 − <1
n2 p−1 p2s ps+1
1 1
şi f (ps ) = 1− s+1 ≥ 1− 2 , ∀ p prim şi s ∈ N∗ . Deoarece f este multiplicativă,
p p
! !
Y 1 Y 1 6
rezultă imediat că f (n) < 1 şi f (n) > 1− 2 > 1− 2 = 2.
p prim
p p prim
p π
p|n

σ(n)ϕ(n)
d) Notăm f (n) = care este o funcţie multiplicativă. Avem
nτ (n)
ps+1 − 1 2s+1 − 1
f (ps ) = > 1 pentru p prim, p ≥ 3 şi f (2s ) = > 1 pen-
p(s + 1) 2(s + 1)
3
tru s ≥ 2; f (2) = . Avem probleme doar ı̂n cazul n = 2m, m ∈ N, m impar,
4
3 8
m ≥ 3. În acest caz totul este ı̂n regulă căci f (n) ≥ f (2) · f (3) = · = 1.
4 6
! !
X Xn
d ! !
σ 2 (n) d|n d|n
d X X1 sY
e) = = d ≥ τ (n) · τ (n) d · τ (n) ·
n n d|n d|n
d d|n
Í
Y1
τ (n)
= τ 2 (n). Am folosit ı̂n inegalităţile precedente de două ori inegali-
d|n
d
tatea mediilor.

X Xk X1 σ(k) X 1 X n σ(k) Xn X1
35. σ(k) = d= =k , = · = =
d|k d|k
d d|k
d k d|k
d k=1 k k=1 d|k
d
" #
n
n
X d n 1
X
≤n < 2n.
d=1
d d=1
d2

174
π2 ∞ 1
X
Observaţie. Deoarece = , raţionamentul de mai sus ne arată că
k=1
k2 6
X σ(k) π2
< n.
k 6
k
Y
36. Se tratează separat cazurile n = pα şi n = pα q β . Pentru n = pαi i ,
i=1
p1 < p2 < · · · < pk , k ≥ 3, avem
k k € k k ‚ Œ
Y Y Š Y Y 1
τ (n) = τ (pαi i ) ≤ pαi i −1 +1 = pαi i −1 1+ ≤
i=1 i=1 i=1 i=1 pαi i −1
k
Y k
Y n
≤ pαi i −1 · 2k ≤ pαi i −1 · 3k−1 ≤ p1α1 −1 pα2 2 . . . pαk k = .
i=1 i=1
d

37. Pentru prima parte a problemei alegem n = 2k şi σ(n) = 1 + 2 +


· · · + 2k = 2k+1 − 1 < 2n = 2k+1 . Pentru a doua parte, alegem n = 945m,
m ∈ N, (m, 210) = 1. Avem că σ(n) = (1 + 3 + 9 + 27)(1 + 5)(1 + 7)σ(m) =
1920σ(m) > 2 · 945m = 2n.

38. Dacă afirmaţia din enunţ nu ar fi adevărată, atunci ar rezulta că şirul
!
σ(m) σ(m)
m ∈ N∗ ar fi mărginit. Acest lucru contrazice faptul că lim =
m m→∞ m

+∞ (curs, pagina 58).


4
X
39. Dacă n ar fi impar, atunci d1 , d2 , d3 , d4 ar fi impare şi n = d2i ar
i=1
fi par; contradicţie. Deci n este par şi d2 = 2. Dacă cumva 4|n, atunci d3 sau
4
X
d4 ar fi egale cu 4, şi trecând ı̂n egalitatea d2i = n la o congruenţă mod 4,
i=1
deducem contradicţia 1 + d2 ≡ 0 (mod 4) (d = d3 sau d = d4 ). Deci neapărat
4 - n. Aceasta ne arată că d3 este un număr prim impar p. d4 poate fi ori
2p ori un număr prim impar q. Această din urmă alternativă este imposibilă,
căci ı̂n acest caz ar rezulta că n = 1 + 4 + p2 + q 2 este impar. Deci d4 = 2p
şi n = 1 + 4 + p2 + 4p2 = 5 + 5p2 . Cum p|n, rezultă că p|n − 5p2 = 5, p = 5,
n = 130. Se verifică imediat că 130 este soluţie şi din cele de mai sus rezultă
că este singura.

40. Funcţiile τ (k) şi τ 3 (k) sunt multiplicative şi deci funcţiile lor suma-
torii sunt multiplicative (curs, pagina 49). Aceasta arată că este suficient

175
!2
X
să probăm enunţul pentru n = pα , p prim, α ∈ N. Avem τ (d) =
d|n
!2
α 2 2
(α + 1)(α + 2)
(τ (1) + τ (p) + · · · + τ (p )) = (1 + 2 + 3 + · · · + (α + 1)) =
2
!2
X (α + 1)(α + 2)
3 3 3 3
şi τ (d) = 1 + 2 + · · · + (α + 1) = şi egalitatea din
d|n
2
enunţ este demonstrată.

41. Fie a = p pα1 1 . . . pαr r , b = pq1β1 . . . qsβs , pi , qj prime ∀ i = 1, r, ∀ j = 1, s,


αi ∈ N∗ , βj ∈ N∗ , pi 6= qj , p 6= pi , p 6= qj , ∀ i = 1, r, ∀ j = 1, s. Avem că
r
Y s
Y r
Y s
Y
3τ (a)τ (b) = 3·2 (1+αi )·2 (1+βi ) şi 4τ (ab) = 4·3 (1+αi ) (1+βi ); este
i=1 i=1 i=1 i=1
chiar egalitatea din enunţ. Pentru afirmaţia reciprocă, fie a = pα1 1 . . . pαr r a1 ,
b = pβ1 1 . . . pβr r a2 , a1 , a2 ∈ N∗ , (a1 , a2 ) = 1, pi - a1 a2 , pi prim, αi ∈ N∗ , βj ∈ N∗ ,
∀ i = 1, r, astfel ı̂ncât 3τ (a)τ (b) = 4τ (ab). Din această egalitate rezultă imediat
că
r
Y r
Y
3 (1 + αi )(1 + βi ) = 4 (1 + αi + βi ). (1)
i=1 i=1

β+1
Avem 3(1 + α)(1 + β) ≥ 4(1 + α + β) ⇔ 3αβ ≥ 1 + α + β ⇔ α ≥ .
3β − 1
β+1
Această ultimă inegalitate este adevărată deoarece ≤ 1 ≤ α. Egalitatea
3β − 1
(1) are loc dacă şi numai dacă r = 1, α1 = β1 = 1.
(L. Panaitopol )

42. Trebuie rezolvată ecuaţia τ (n) = 3 4n. Evident că n 6= 1 şi 2|n. Fie
n = 2a pa11 . . . par r , a ∈ N∗ , ai ∈ N∗ , pi prim ∀ i = 1, r, pi 6= pj , pentru i 6= j,

pi 6= 2, ∀ i = 1, r. Deoarece 3 4n = τ (n) ∈ N∗ , rezultă că a = 3b + 1, ai = 3bi
∀ i = 1, r (b ∈ N, bi ∈ N∗ ). Însă ştim că τ (n) = (a + 1)(a1 + 1) . . . (ar + 1) şi

obţinem ecuaţia (3b + 2)(3b1 + 1) . . . (3br + 1) = 3 4n = 2b+1 pb11 . . . pbrr ,

3b + 2 pb11 pbrr
= . . . . (*)
2b+1 3b1 + 1 3br + 1

pbi i 5bi 1 + 4bi


Dacă pi ≥ 5 ⇒ ≥ ≥ > 1 (bi ∈ N∗ ). Se arată imediat
3bi + 1 3bi + 1 1 + 3bi
3c 3
prin inducţie că ≥ ∀ c ∈ N∗ . Să presupunem că b ≥ 3. Se arată
3c + 1 4

176
3b + 2 11
imediat prin inducţie că ≤ ∀ b ∈ N, b ≥ 3. Deci, dacă b ≥ 3, avem că
2b+1 16
b1 b
11 3b + 2 p1 prr 3 11 3
≥ b+1 = ·. . .· ≥ . Am obţinut contradicţia ≥ . Deci
16 2 3b1 + 1 3br + 1 4 16 4
pb11 pbrr
b ∈ {0, 1, 2}. Dacă b = 0 sau b = 2, din (*) rezultă că 1 = ·. . .· .
3b1 + 1 3br + 1
Avem soluţia r = 0 şi deci n = 2, n = 27 = 128. Dacă r ≥ 1, atunci neapărat
pbi i 3c 4
p1 = 3, b1 = 1 (căci > 1 pentru pi ≥ 5 şi > 1 ∀ c ≥ 2), =
3bi + 1 3c + 1 3
b2 b bi
p2 pr r pi 7 4
... . Dacă pi ≥ 7, atunci ≥ > şi nu se poate. Deci
3b2 + 1 3br + 1 3bi + 1 4 3
5b2 25 4
r = 2 şi p2 = 5. Dacă b2 ≥ 2, atunci ≥ > . Dacă b2 = 1, rezultă
3b2 + 1 7 3
4 5 5 pb11 pbrr
contradicţia = . Dacă b = 1, avem egalitatea = ... .
3 4 4 3b1 + 1 3br + 1
Se arată imediat că r = 1, p1 = 5, b1 = 1, n = 2000.

43. Fie τ (k) numărul divizorilor naturali ai lui k. Avem identitatea


" # " #
n
X Xn n n
τ (i) = deoarece d ≤ n divide dintre numerele 1, 2, . . . , n (curs,
i=1 i=1
i d
pagina 53). Pentru i = pα1 1 pα2 2 . . . pαk k avem τ (i) = (α1 + 1)(α2 + 1) . . . (αk + 1)
(curs, pagina 52) şi deci τ (i) este impar dacă şi numai dacă α1 , α2 , . . . , αk
sunt pare. Aşadar τ (i) este impar doar pentru i = j 2 . Numărul numerelor τ (i)
√ √
impare este s dacă s2 ≤ n < (s + 1)2 , adică s ≤ n < s + 1, adică s = [ n].
" #
Xn n ”√ —
Aşadar ≡ n (mod 2).
i=1
i

44. Pentru n = pα1 1 pα2 2 . . . pαk k cu pi , i ∈ 1, k numere prime distincte şi


αi ≥ 1, numărul divizorilor este (α1 + 1)(α2 + 1) . . . (αk + 1).
.
Notăm a = 103n+1 + 4(−1)n şi avem a .. 4. Rezultă a = 10(1000)n +
n n n
4(−1)n = 10(7 · 11 · 13 − 1)n
+ 4(−1)n
= 10(M7 + (−1)n )
+ 4(−1)n = M7 +
.
14(−1)n ..7. Avem an = 4 · 7 · bn , unde bn este impar şi bn ≥ b1 = 357.
Dacă bn are un divizor prim diferit de 7, atunci an are cel puţin 3·2·2 = 12
divizori.
Dacă bn = 7k , atunci k > 3 şi bn are mai mult de 3·4 = 12 divizori. Aşadar
bn are cel puţin 11 divizori supraunitari.
(L. Panaitopol )

177
45. Numărul divizorilor este (a + 1)(b + 1) iar media aritmetică este
(1 + p + p2 + · · · + pa )(1 + q + q 2 + · · · + q b )
m= . Dacă p şi q sunt impare,
(a + 1)(b + 1)
p+1 q+1
atunci putem alege a = b = 1 şi avem m = · . Dacă p = 2 şi
2 2
2
q este impar, alegem b = q şi a = q + q + · · · + q 4 q−1 şi avem m =
2 a 2
(1 + 2 + 2 + · · · + 2 )(1 + q + q + · · · + q ) q
. Cum 1 + q + q 2 + · · · + q q =
(1 + q 2 + q 4 + · · · + q q−1 )(q + 1)
(1 + q 2 + q 4 + · · · + q q−1 )(q + 1), rezultă m = 1 + 2 + 22 + · · · + 2a = 2a+1 − 1.
Dacă p este impar şi q = 2, alegem b = p2 + p4 + · · · + pp−1 şi a = p.
(L. Panaitopol )

46. Numărul căutat n se divide cu 23 · 32 · 5 · 7 şi deci n = 2α1 · 3α2 · 5α3 ·


7α4 · 11α5 · 13α6 . . . cu α1 ≥ 4, α2 ≥ 2, α3 ≥ 1, α4 ≥ 1, α5 ≥ 0 . . . . Numărul
divizorilor este τ (n), τ (n) = (α1 +1)(α2 +1)(α3 +1)(α4 +1) . . . (αk +1). Pentru
k ≥ 6 avem contradicţia τ (n) ≥ 4·3·24 = 192, deci k ≤ 5. Scrierile convenabile
ale lui 144 sunt 4 · 3 · 3 · 2 · 2 = 6 · 3 · 2 · 2 · 2 = 12 · 3 · 2 · 2 = 9 · 4 · 2 · 2 = 6 · 6 · 2 · 2.
Numărul minim este n = 25 · 32 · 5 · 7 · 11 = 110880 şi se divide cu 1, 2, 3 . . . , 10.

47. Dacă N este impar, atunci d2 + d4 este par şi se obţine contradicţia
..
dd5 .2. Aşadar N este par şi deci d2 = 2. Deoarece dd5 > d6 rezultă d5 ≥ 7.
Cum d4 + 2 divide pe dd5 , rezultă că d4 + 2 este un divizor al lui n şi deci
d5 ≤ d4 + 2. Dacă d5 = d4 + 1, rezultă că d4 + 2 = d6 . d4 , d4 + 1, d4 + 2 sunt
.
divizori ai lui n şi sunt numere consecutive, deci n..3, adică d = 3. rezultă 3
d4 ≤ 2 · 3 = 6 şi deci d5 = d4 + 1 ≤ 7. Am arătat că d5 ≥ 7 şi deci d5 = 7,
d6 = 8. Rezultă 8|N şi deci se obţine contradicţia d4 = 22 = 4. Rămâne deci
numai cazul d5 = d4 + 2. Numerele N cu 16 divizori au formele: a) p15 ; b)
p7 q; c) p3 q 3 ; d) p3 qr; e) pqrs, unde p, q, r, s sunt numere prime distincte.
În cazul nostru, unul dintre factori este 2. Arătăm că 4 - N . Să presupunem că
4|N . Atunci d4 = 4 sau d3 = 4. Dacă d4 = 4, atunci d3 = 3, d5 = d4 + 2 = 6;
contradicţie (căci d5 ≥ 7). Să presupunem că d3 = 4. Dacă d4 = 8, atunci
d5 = 10; contradicţie. Deci d4 este un număr prim impar şi d5 = d4 + 2 este,
de asemenea, număr prim impar. Suntem ı̂n cazul N = 23 · d4 · d5 . Deci d6 = 8,
d4 + 2 < 8, d4 = 5, d5 = 7. Avem 10 = d7 = dd5 = (d4 + d2 )d6 = 7 · 8;
contradicţie. Deci 4 - N şi d4 este impar (dacă 2|d4 atunci unul din numerele
d4 şi d5 = d4 + 2 este divizibil cu 4; am văzut că acest lucru nu este adevărat).
Din cele 5 cazuri posibile pentru N este evident că a) şi c) nu pot avea loc
(deoarece 2|N şi 4 - N ). Cazul b) poate avea loc doar dacă N = 2 · q 7 , q prim,

178
q ≥ 3. Dar ı̂n acest caz d3 = q, d4 = 2q; ultima relaţie este imposibilă căci am
observat că d4 trebuie să fie impar. Dacă are loc cazul d), atunci N = 2p3 q,
unde p şi q sunt numere prime impare distincte. Dacă q < p, atunci d3 = q,
d4 = p (deoarece d4 este impar), d5 = p + 2 = p2 ; contradicţie. Dacă p < q,
atunci d3 = p, d4 = p2 sau d4 = q (deoarece d4 este impar). Dacă d4 = p2 ,
atunci d5 = p2 + 2 = q. Pentru p 6= 3 obţinem contradicţia q = p2 + 2 ≡ 0
(mod 3), q > 3, q prim. Deci p = 3, q = 11. Am obţinut contradicţia d4 = 6.
Dacă d4 = q, atunci d5 = q + 2 = p2 . Dar 2p < p2 este divizor al lui N şi
am obţinut o contradicţie. Deci singurul caz posibil este e), N = 2pqr, unde
3 ≤ p < q < r sunt numere prime distincte. Avem d3 = p, d4 = q (deoarece
d4 este impar) şi d5 = q + 2. Cum q - d5 , 2 - d5 , deducem că r = q + 2.
Avem d5 ≤ 16 şi deci q ≤ 14, q şi q + 2 fiind prime (q ≥ p + 2 ≥ 5). Dacă
q = 5, atunci p = 3, d4 = 5, d5 = 6; contradicţie (căci d5 ≥ 7). Dacă q = 11,
11
atunci r = q + 2 = 13. Deoarece 11 = d4 < 2p, rezultă p > = 5, 5. Cum
2
p < q = 11, deducem că p = 7. Se verifică imediat că N = 2 · 7 · 11 · 13 = 2002
are proprietăţile din enunţ. Din cele de mai sus rezultă că este singurul număr
cu proprietăţile din enunţ.
48. Plecăm de la observaţia că d2 ≡ 1(24), ∀ d ∈ Z, (d, 24) = 1. Este
!
X X n X d2 + n
2
imediat că n 6= t ∀ t ∈ Z. Avem că σ(n) = d= d+ = .
d|n d|n
d d|n
d

d< n

Însă d2 + n ≡ 1 + n ≡ 0 (mod 24), conform observaţiei precedente şi faptului


că (d, 24) = 1, care rezultă din d|n şi n = 24k − 1. Enunţul este deci justificat.
49. Analizăm ı̂ncâi cazul qi 6= 2 ∀ i = 1, k. Alegem αi = qi ∀ i = 1, k.
σ(n) Yk 1 + q + q 2 + · · · + q qi
i i i
Media aritmetică a divizorilor lui n este = =
τ (n) i=1
1 + qi
k €
Y Š
1 + qi2 + qi4 + · · · + qiqi −1 ∈ N. Să presupunem acum că qk = 2. Alegem
i=1
1 + q1 + · · · + q1q1
αi = qi ∀ i = 1, k − 1 şi αk = − 1 = q12 + q14 + · · · + q1q1 −1 .
1 + q1
Avem că
! α +1
σ(n) k−1
Y€ Š 2 k −1
= 1+ qi2 + qi4 + ··· + qiqi −1 =
τ (n) i=1
αk + 1
!
k−1
Y€ Š
= 1+ qi2 + qi4 + ··· + qiqi −1 (2αk +1 − 1) ∈ N.
i=2

179
Observaţie. Pentru k = 1 şi q1 6= 2 se alege α1 = q1 şi enunţul este adevărat.
2n − 1
Însă pentru k = 1, q1 = 2 enunţul nu mai este adevărat, deoarece 6∈ N
n
∀ n ∈ N, n ≥ 2 (acest lucru a fost demonstrat ı̂n Capitolul 11, problema 13).

50. Dacă n = 1, 6, 2k sau număr prim, se verifică imediat că


a1 , a2 , . . . , aϕ(n) formează progresie aritmetică. Dacă ϕ(n) = 1, atunci n = 1, 2.
Dacă ϕ(n) = 2, atunci n = 3, 4, 6. Putem presupune deci că ϕ(n) ≥ 3
şi că a1 , a2 , . . . , aϕ(n) formează progresie aritmetică. Dacă a2 = 2 se arată
imediat că n este prim. Dacă a2 = 3 atunci n = 2k . Presupunem că
a2 ≥ 4 (de fapt a2 ≥ 5). Deoarece a2 > 3, rezultă că 3|n şi că 3¤|¤a2 .
a3 = 2a2 − a1 = 2a2 − 1. Cum (a3 , n) = 1, 3|n, rezultă că 3¤|¤a3 şi deci a2 ≡ 1
(mod 3). Dar n − 1 = aϕ(n) = a1 + (ϕ(n) − 1)(a2 − a1 ) = 1 + (ϕ(n) − 1)(a2 − 1),
n = 2 + (ϕ(n) − 1)(a2 − 1). Deoarece a2 ≡ 1 (mod 3), din ultima relaţie rezultă
că n ≡ 2 (mod 3). Contradicţie, căci 3|n.
(L. Panaitopol )

51. Din formula de inversiune a lui Möbius (curs, pagina 50), rezultă
X n
că an = µ(d)2 d . Evident că a1 = 2 şi 1|a1 . Putem presupune că n > 1,
d|n
n = pα1 1 . . . pαr r , pi 6= pj pentru i 6= j, pi prim, αi ∈ N∗ pentru i = 1, r. Este
suficient să arătăm că pαi i |an ∀ i = 1, r. Arătăm pentru i = 1. n = pα1 1 · n1 ,
p1 ¤|¤n1 .
X X n X  α n1 α −1 n1
‹
n
p1 1 p1 1
an = µ(d)2 + d µ(p1 d)2 p1 d = µ(d) 2 d −2 d .
d|n1 d|n1 d|n1
α1 n1 α1 −1 n1
Arătăm că pentru ∀ d|n1 , pα1 1 |E = 2p1 ‚d −€2p1 d şiŠ din Œ
formula precedentă
n1 α α −1
n1 α1 −1 p1 1 −p1 1
rezultă că pα1 1 |an . Dar E = 2 p
d 1 2 d
− 1 . Dacă p1 6= 2 ⇒
€ Š
n1 α α −1
α1 1α −1 1α p1 1 −p1 1
2p1 −p1 = 2ϕ(p1 ) ≡ 1 (mod pα1 1 ) ≡ 1 (mod pα1 1 ).
şi deci 2 d

n1 α1 −1 . .
n1
= 2 2 1 .. 22 1 .. 2α1 deoarece 2α1 −1 ≥ α1
p α −1 α −1
Dacă p1 = 2, atunci 2 d 1 d

∀ α1 ∈ N∗ (această ultimă inegalitate rezultă imediat prin inducţie după α1 ).


Deci şi ı̂n acest caz pα1 1 |E. Problema este ı̂n acest moment rezolvată.
γm , b = pβ1 . . . pβk r δ1 . . . r δh , descompunerile
52. Fie a = pα1 1 . . . pαk k q1γ1 . . . qm 1 k 1 h
standard ı̂n factori primi ale numerelor a, b. Se arată uşor prin inducţie că
k
Y k
Y k
Y
(1 + αj + βj ) > (1 + αj ) + (1 + βj ), ∀ k ≥ 2. De aici deducem că
j=1 j=1 j=1
ecuaţia nu are soluţii pentru k ≥ 2.

180
Notăm x = (1 + γ1 ) . . . (1 + γm ), y = (1 + δ1 ) . . . (1 + δh ) şi pentru k ≥ 2
avem că
! !
k
Y k
Y k
Y
τ (ab) = (1 + αj + βj )xy > (1 + αj ) xy + (1 + βj ) xy ≥
j=1 j=1 j=1
! !
k
Y k
Y
≥ (1 + αj ) x + (1 + βj ) y = τ (a) + τ (b).
j=1 j=1

Analizăm acum cazul k = 1, x ≥ 3. Avem că

1 + α1 1 + β1 1 + β1
+ 1 + α1 +
τ (a) + τ (b) y x 3 < 1.
= ≤
τ (ab) 1 + α1 + β1 1 + α1 + β1

Analog se arată că nu avem soluţii pentru k = 1, y ≥ 3, pentru k = 1,


x = y = 2 şi pentru k = x = y = 1. Deci pentru k = 1 singurele soluţii
posibile sunt x = 2, y = 1 şi x = 1, y = 2. Analizăm prima situaţie (cealaltă
1 + β1
τ (a) + τ (b) 1 + α1 + 2
se tratează similar). Avem 1 = = , 1 + α1 + β1 =
τ (ab) 1 + α1 + β1
1 + β1
1 + α1 + , β1 = 1 şi avem soluţia a = pα1 1 q1 , b = p1 (p1 , q1 sunt prime
2
distincte, α1 ∈ N∗ ). Dacă k = 0, ecuaţia din enunţ se transcrie x + y = xy,
(x − 1)(y − 1) = 1, x = y = 2, a = q1 , b = r1 (q1 şi r1 sunt prime distincte).

53. Evident că k = 1 este soluţie. Dacă n = pα1 1 pα2 2 . . . pαr r , p1 < p2 <
Y r 1 + 2α
i
· · · < pr , pi prim, αi ∈ N∗ , ∀ i = 1, r, ecuaţia noastră se scrie = k.
i=1
1 + αi
Din această relaţie este evident că k trebuie să fie impar. Vom arăta că această
condiţie este şi suficientă. Vom face un raţionament prin inducţie după k.
Pentru k = 1 am văzut că enunţul este adevărat. Pentru k = 3, luăm n = 24 ·32
1 + 2α1 1 + 2α2 9 5
şi · = · = 3. Fie k ≥ 5, număr impar cu proprietatea că
1 + α1 1 + α2 5 3
enunţul este adevărat pentru orice număr impar mai mic sau egal cu k − 2.
Dacă k = 4t+1, aplicăm ipoteza de inducţie pentru 2t+1 şi deducem existenţa
Y r 2α + 1
i
numerelor ai ∈ N∗ , ∀ i = 1, r, astfel ı̂ncât = 2t + 1.
i=1
αi + 1
r+1
Y 2αi + 1 4t + 1
Alegem ar+1 = 2t şi = (2t + 1) = 4t + 1.
i=1
αi + 1 2t + 1

181
Dacă k = 4t − 1, scriem k = 2i α − 1, (α, 2) = 1, i ≥ 2 şi considerăm
numerele a1 = 2(3 · 2i−2 · α − 1), . . . , ai−1 = 2(3i−1 α − 1) şi ai = 3i−1 α − 1.
Pentru 2 ≤ j ≤ i − 1 avem că 2aj + 1 = 3(aj−1 + 1).
Avem şi 2ai +1 = ai−1 +1, 2a1 +1 = 3(α ·2i −1) = 3k. Calculând, obţinem
Yi 2a + 1
j 2a1 + 1 i−2 3i−1 · k k
că = 3 = i−1 = . Cum α este impar şi α < k,
a +1
j=1 j
ai + 1 3 ·α α
din ipoteza de inducţie deducem existenţa numerelor bj (j = 1, t) astfel ı̂ncât
! !
Yt 2b + 1 Y r 2a + 1 Y t 2b + 1 k
j j j
= α. Avem că · = α = k. Pasul de
j=1 j
b +1 j=1 j
a +1 j=1 j
b +1 α
inducţie a fost demonstrat şi problema noastră este rezolvată.

54. Probabilitatea de mai sus trebuie ı̂ntâi definită (deoarece N∗ este o


mulţime infinită). Pentru n ∈ N∗ notăm cu pn probabilitatea ca alegând pe
x şi y independent din mulţimea {1, . . . , n}, ele să fie prime ı̂ntre ele. Vom
arăta că (pn )n≥1 este un şir convergent; limita sa o vom numi probabilitatea
ca luând x, y din N∗ , ele să fie prime ı̂ntre ele.
Soluţie. Cazurile favorabile corespunzătoare mulţimii {1, . . . , n} sunt ı̂n
număr de pn · n2 . Cazurile favorabile corespunzătoare mulţimii {1, 2, . . . , n +
1} sunt cele favorabile de la mulţimea {1, 2, . . . , n}, plus numărul de fracţii
a n+1
ireductibile de forma şi , unde 1 ≤ a, b ≤ n + 1. Cum ultimul
n+1 b
n2 · pn + 2ϕ(n + 1)
număr este evident 2ϕ(n+1), am obţinut formula pn+1 = ,
(n + 1)2
n2 · pn = (n − 1)2 pn−1 + 2ϕ(n)
..
valabilă pentru ∀ n ∈ N∗ . Avem că . .
22 · p2 = 12 · p1 + 2ϕ(2)
p1 = 1
n
X
Adunând aceste egalităţi, obţinem că n2 pn = 1 + 2 ϕ(j), pn =
j=2
n
X
1+2 ϕ(j)
j=2 X 3 2
. Ştim (curs, pagina 65) că ϕ(n) = x + O(x ln x).
n2 n≤x
π2
Ţinând cont de această formulă asimptotică, rezultă imediat că pn este un
6
şir convergent şi că limita sa este 2 .
π

182
X
55. Fie f = 1. Funcţia sumatorie este F = 1 = τ (n). Din formula de
k|n
 
X n
inversiune a lui Möbius avem f (n) = µ(k)F (curs, pagina 50) şi deci
k|n
k
 
X n
1= µ(k)τ .
k|n
k

X
56. Dacă f este multiplicativă şi n = pα1 1 pα2 2 . . . pαi i , avem µ(k)f (k) =
k|n
(1 − f (p1 ))(1 − f (p2 )) . . . (1 − f (pi )) (curs, pagina 47). Pentru f (k) = τ (k)
X i
Y i
Y
avem µ(k)f (k) = (1 − τ (pj )) = (1 − 2) = (−1)i .
k|n j=1 j=1

57. Fie f (k) = µ(k)τ (k). Funcţia f este multiplicativă şi deci
X Y
µ(k)f (k) = (1 − f (p)) (curs, pagina 47), adică
k|n p|n

X Y Y
µ2 (k)τ (k) = (1 − µ(p)2) = 3 = 3ω(n) .
k|n p|n p|n

Cum µ2 (k) = |µ(k)|, se obţine enunţul.

γ(k) X µ(k)γ(k)
58. Funcţia f (k) = este multiplicativă. Avem =
k k|n
k
!
Y γ(p)
1− = 0 deoarece γ(p) = p. Pentru funcţia multiplicativă f (k) =
p|n
p
µ(k) X µ(k)
, funcţia sumatorie F (n) = este multiplicativă.
γ(k) k|n
γ(k)
!
X µ(k) 1 Y 1 ϕ(n)
F (pα ) = = 1 − şi deci f (n) = 1− = .
k|pα
γ(k) p p|n
p n
Observaţie. Formulele precedente puteau fi demonstrate ţinând cont că
X X µ(k)
n·µ(n) = µ(n)·γ(n) şi µ(k) = 0 pentru n > 1 (curs, pagina 50), =
k|n k|n
k
ϕ(n)
(curs, pagina 65).
n

183
" #
1
59. Pentru n = 1, µ(1) = 1. Presupunem relaţia adevărată pentru
1
n ≥ 1 şi o demonstrăm pentru n + 1. Avem

n+1
X •n+1
X ˜ • ˜ • ˜‹
n+1 n+1 n
Sn+1 = µ(k) = µ(k) − +
k=1
k k=1
k k
n+1
X • ˜ n+1
X • ˜ • ˜‹
n n+1 n
+ µ(k) = µ(k) − + 1.
k=1
k k=1
k k

" # " # (
n+1 n 0, pentru k¤|¤n + 1 X
− = , deci Sn+1 = µ(k) + 1 = 1,
k k 1, pentru k|n + 1 k|n+1
X
deoarece µ(k) = 0 (curs, pagina 50). O altă demonstraţie a acestui enunţ
k|n+1
se găseşte ı̂n problema 63 a).

60. Pentru n = 1 identitatea se verifică. Presupunând identitatea valabilă


pentru n, avem

n+1
X • ˜
n+1
X • ˜ • ˜‹ n
X • ˜
n+1 n+1 n n
Sn+1 = ϕ(k) = ϕ(k) − + ϕ(k) .
k=1
k k=1
k k k=1
k

" # " # " # (


n n+1 n 0, pentru k¤|¤n + 1
Avem = 0 şi − = şi deci Sn+1 =
n+1 k k 1, pentru k | n + 1
X n(n + 1) X
ϕ(k) + . Deoarece ϕ(k) = n + 1 (curs, pagina 61), avem
k|n+1
2 k|n+1
n(n + 1) (n + 1)(n + 2)
Sn+1 = n + 1 + = , ceea ce ı̂nseamnă că identitatea
2 2
din enunţ este adevărată (M. O. Drimbe). O altă demonstraţie a acestui enunţ
se găseşte ı̂n problema 63 c).

61. Pentru n = 1 identitatea se verifică. Presupunem relaţia adevărată


pentru n şi o demonstrăm pentru n + 1.

n+1
X n
X • ˜2
n
Sn+1 = 2 ϕ(k) = Sn + 2ϕ(n + 1) = 1 + µ(k) + 2ϕ(n + 1) =
k=1 k=1
k
n+1 • ˜2 n+1 ‚• ˜2 • ˜2 Œ
X n+1 X n+1 n
=1+ µ(k) + 2ϕ(n + 1) − µ(k) − .
k=1
k k=1
k k

184
Avem 8
2 32 2 32
>
< 0, pentru k¤|¤n + 1
n+1 n    2
4 5 −4 5 = n+1 2 n+1 2(n + 1)
k k >
: − −1 = − 1 pentru k|n + 1
k k k
" #2 !
n+1
X
n+1 X 2(n+1)
şi de aici Sn+1 = 1 + µ(k) +2ϕ(n+1)− µ(k) −1 =
k=1
k k|n+1
k
" #2
n+1
X n+1 X µ(k) X
1 + µ(k) + 2ϕ(n + 1) − 2(n + 1) + µ(k) =
k=1
k k|n+1
k k|n+1
" #2
n+1
X n+1 X µ(k) ϕ(n + 1)
1+ µ(k) , deoarece avem = (curs, pagina 65)
k=1
k k|n+1
k n+1
X
şi µ(k) = 0 (curs, pagina 50).
k|n+1

 
n
62. f (k) apare pentru fiecare multiplu al lui k, deci de ori.
k
63. Se aplică problema precedentă.
a) Pentru f = µ şi n > 1 rezultă F (n) = 0 (curs, pagina 50).
µ(k) ϕ(k)
b) Pentru f (k) = şi k ≥ 1 rezultă F (k) = (curs, pagina 65).
k k
c) Pentru f (k) = ϕ(k) rezultă F (k) = k (curs, pagina 61).
1 √
64. Fie f (n) = . Avem ϕ(n) ≥ n pentru n ≥ 7 (curs, pa-
ϕ(n)σ(n)
1 X∞
gina 64). Cum σ(n) > n, rezultă f (n) ≤ 3 şi deci seria f (n) este abso-
n2 n=1

X Y
lut convergentă. Cum f este multiplicativă, avem |µ(n)|f (n) = (1 +
n=1 p prim
!
|µ(n)| ∞
X Y 1
f (p)) (curs, pagina 47) şi deci = 1+ =
n=1
ϕ(n)σ(n) (p − 1)(p + 1)
Y 1 π2
 = 6
(curs, paginile 47, 48).
p prim 1
1−
p2
1 n +x2n +x3n +. . . .
xn
65. Avem = 1+x = xn +x2n +x3n +. . .
1 − xn 1 − xn
∞ xn ϕ(n) ∞ ∞  
X X X X
n 2n 3n n
şi S = = (x + x + x + . . . )ϕ(n) = x ϕ(d) .
n=1
1 − xn n=1 n=1 d|n

185
X
Avem ϕ(d) = n (curs, pagina 61) şi deci
d|n


X nxn+1 − (n + 1)xn + 1 x
S= nxn = lim x 2
=
n=1
n→∞ (x − 1) (x − 1)2

n x
deoarece lim nxn = lim   = 0. Avem = 1, x2 − 3x + 1 = 0 şi
n→∞ n→∞ 1 n (x − 1)2
x

3− 5
deci x = .
2

186
CAPITOLUL 8

Distribuţia numerelor prime

1. Cazul n = 1 este imediat. Pentru n ≥ 2, ı̂n fiecare din intervalele


(10n−1 , 2 · 10n−1 ), (2 · 10n−1 , 4 · 10n−1 ), (4 · 10n−1 , 8 · 10n−1 ), există cel puţin
un număr prim (conform postulatului Bertrand) care are n cifre.

n
2. a) Am arătat că dacă n ∈ N, n ≥ 2, ∃ q prim astfel ı̂ncât <q≤n
2
4
(problema 4). Deoarece n ≥ 4, q > = 2, q ≥ 3; alegem p = 2.
2
b) Dacă 15 ≤ n ≤ 30, alegem m = 30 = 2 · 3 · 5. presupunem că n ≥ 31.
 
n
Avem că ≥ 5 şi din teorema lui Cebı̂şev ştim că ∃ r prim astfel ı̂ncât
6
       
n n n n n n n
<r<2 ≤ 2 = . Din r > obţinem că r ≥ + 1 > > 5.
6 6 6 3 6 6 6
n n
Deci < r < şi n < 6r < 2n. Cum r > 5, rezultă că r 6= 2, r 6= 3. Alegem
6 3
p = 2, q = 3 şi problema este rezolvată.

3. Fie p1 < p2 < · · · < ps−1 < ps primele s numere prime. Fie
n > p1 p2 . . . ps şi n = p1 p2 . . . ps−1 c + r cu 0 ≤ r < p1 p2 . . . ps−1 . Rezultă
c ≥ ps . Fie p prim, p ∈ (c, 2c). Avem că p > ps . Pentru m = p1 p2 . . . ps−1 p
avem m < 2n şi m ≥ p1 p2 . . . ps−1 (c + 1) = p1 p2 . . . ps−1 c + p1 p2 . . . ps−1 > n.

4. a) Conform teoremei lui Bertrand-Cebı̂şev, pentru ∀ s ∈ N, s ≥ 2,


∃ p prim astfel ı̂ncât s < p < 2s. Dacă n = 2s, s ≥ 2, enunţul rezultă din
observaţia de mai sus. Dacă n = 2, luăm p = 2. Dacă n = 2s + 1, s ≥ 2,
există p prim astfel ı̂ncât s < p < 2s < 2s + 1 = n. Din p > s rezultă că
1 n
p ≥ s + 1 > s + = . Dacă s = 1, n = 3 şi alegem p = 2.
2 2
b) Presupunem că ∃ x, k, n ∈ N, k ≥ 2, n ≥ 2, astfel ı̂ncât n! = xk . Din
n
punctul a) ştim că ∃ p prim astfel ı̂ncât < p ≤ n. Deoarece p ≤ n şi 2p > n,
2

187
deducem că ep (n) = 1. Din n! = xk rezultă 1 = ep (n) = kα (deoarece n! = xk )
şi contradicţia k|1 (k ∈ N, k ≥ 2).
n
X
n a
bi
X i i=2 n!
5. = , bi = ai , ∀ i = 2, n. Există, conform problemei 4,
i=2
i n! i
n n!
p prim astfel ı̂ncât < p ≤ n. Dacă i 6= p, atunci p şi p|bi . Din ipoteză
2 i
ştim că p¤|¤ap . Deoarece 2p > n ≥ p, rezultă că p¤|¤bp , p|bi , ∀ i = 2, n, i 6= p. Deci
Xn Xn a
i
p¤|¤ bi şi cum p|n!, obţinem că ∈
/ Z.
i=2 i=2
i

1 1 1
6. Presupunem că S = + + ··· + ∈ N∗ . Rezultă că
n n+1 n+k
k+1
≥ S ≥ 1, n ≤ k + 1, 2n − 1 ≤ n + k. Pentru n = 1, enunţul rezultă
n
din problema precedentă. Dacă n ≥ 2, conform postulatului lui Bertrand,
există un număr prim q astfel ı̂ncât n < q ≤ 2n − 1 ≤ n + k. Fie p cel
mai mare număr prim din intervalul (n, n + k]. Dacă 2p ≤ n + k, ı̂ntre p şi
2p există un număr prim (postulatul lui Bertrand), ceea ce contrazice maxi-
malitatea lui p. Aşadar 2p > n + k. Deci p divide un singur numitor din
1 1 1 1 1
fracţiile ce definesc S. Avem S = + +···+ + +···+ =
n n+1 p p+1 n+k
Mp + n(n + 1) . . . (p − 1)(p + 1) . . . (n + k)
6∈ Z, deoarece p¤|¤n(n + 1) . . .
mp
(p − 1)(p + 1) . . . (n + k).

7. Soluţia 1. Fie pk+1 cel mai mic prim care nu divide pe n. Avem că
(pk+1 , n) = 1 şi din ipoteză deducem că p2k+1 ≡ 1 (mod p1 p2 . . . pk ) şi inega-
litatea p2k+1 − 1 ≥ p1 p2 . . . pk , care se mai scrie şi ca p1 p2 . . . pk < p2k+1 . Dacă
k ≥ 5, avem că 2pk > pk+1 , 2pk−1 > pk+1 şi deci p1 p2 . . . pk ≥ (2pk )(3pk−1 ) >
(2pk )(2pk−1 ) > p2k+1 . Pentru k = 4, avem că p1 p2 p3 p4 = 210 > 112 . Din toate
aceste consideraţii deducem că k ≤ 3. Pentru k = 0 avem că n este impar.
Din 22 ≡ 1(n) deducem că n = 3. Dacă k = 1, avem n = 2α · m, 3¤|¤m, α ∈ N∗ .
.
Avem 32 − 1..n şi deci n ∈ {2, 4, 8}. Dacă k = 2, avem că n = 2α · 3β · m,
.
(30, m) = 1, α, β ∈ N∗ . Din 52 − 1..n deducem că n ∈ {6, 12, 24}. Pentru
.
k = 3 avem n = 2α · 3β · 5γ · m, (210, m) = 1, α, β, γ ∈ N∗ . Din 72 − 1..n şi
5|n, observăm că nu avem nici o soluţie ı̂n acest caz. Deci soluţiile problemei

188
constituie o submulţime a mulţimii {2, 3, 4, 6, 8, 12, 24}. Se observă imediat că
toate elementele mulţimii satisfac proprietatea din enunţ.
Soluţia 2. Fie n cu proprietatea din enunţ. Arătăm că 5¤|¤n.
Dacă 5|n, fie x ∈ Z astfel ı̂ncât x ≡ 2(5) şi x ≡ 1(p), ∀ p prim p 6= 5,
p|n (existenţa lui x rezultă din lema chineză a resturilor). Este evident că
(x, n) = 1 şi deci x2 ≡ 1(n). Rezultă că 5|n|x2 − 1. Aceasta este fals ı̂nsă,
deoarece x2 −1 ≡ 4−1 ≡ 3 (mod 5). Deci am arătat că 5¤|¤n. Avem că (5, n) = 1
şi conform enunţului, avem 52 ≡ 1 (mod n). De aici rezultă că n|24. Un calcul
uşor arată că toţi divizorii lui 24 au calitatea cerută. Deci soluţia problemei
este mulţimea {2, 3, 4, 6, 8, 12, 24}.
8. Fie (pk )k≥1 şirul numerelor prime. Pentru n ≥ 2 există k, astfel ı̂ncât
p1 p2 . . . pk ≤ n < p1 p2 . . . pk pk+1 . Dacă qn > pk+1 , atunci p1 , p2 , . . . , pk+1 sunt
divizori ai lui n şi se obţine contradicţia n ≥ p1 p2 . . . pk+1 .
qn 2pk 1
Deci qn ≤ pk+1 < 2pk şi de aici 0 < < ≤ k−2 . Pentru
n p1 p2 . . . pk−1 pk 2
qn
n → ∞ rezultă k → ∞ şi deci lim = 0.
n→∞ n
9. Demonstrăm prin inducţie. Pentru k = 1 şi k = 2 proprietatea se
verifică. Presupunem proprietatea adevărată pentru orice 1 ≤ j ≤ k−1.
Adunând 2k cu 1, 2, . . . , 2k − 1, obţinem toate numerele naturale din
[2k + 1, 4k − 1]. Din postulatul lui Bertrand rezultă că pentru n ≥ 3, ı̂n inter-
valul [n, 2n − 3] se află cel puţin un număr prim. Aşadar există a ∈ 1, 2k − 1
astfel ca p = 2k + a să fie prim. Rezultă că a este impar. Perechile (a, 2k),
 
a−1 a+1
(a + 1, 2k − 1), . . . , k + ,k + au suma elementelor p.
2 2
a−1
Rămân elementele {1, 2, . . . , 2j} unde j = . Din ipoteza de inducţie,
2
rezultă că elementele acestei mulţimi pot fi dispuse ı̂n perechi cu suma ele-
mentelor număr prim.
10. Avem a2 = a1 + 1, a4 = a2 + 2 = a1 + 3. Cum a2 < a3 < a4 ,
rezultă a3 = a1 + 2. Arătăm prin inducţie că an = a1 + n − 1. Presupunem
proprietatea adevărată pentru orice i ≤ n. Dacă n + 1 este par, avem an+1 =
n+1 n+1 n+1
a n+1 + = a1 + −1+ = a1 + n. Dacă n + 1 este impar, avem
2 2 2 2
an < an+1 < an+2 şi an = a1 + n − 1 şi an+2 = a1 + n + 1, deoarece n + 2 este
par. Din a1 + n − 1 < an+1 < a1 + n + 1 rezultă an+1 = a1 + n. Arătăm că
a1 = 1. Dacă a1 = 0, rezultă a4 = 3.

189
Cum a4 este prim, rezultă contradicţia 4 este prim. Presupunem a1 ≥ 2. Fie n
astfel ı̂ncât an = pk = a1 + n − 1. Rezultă că n este prim, adică n = pk − a1 + 1
este prim şi deci ∀ h ≥ k, ph −a1 +1 este prim. Rezultă de aici ph −a1 +1 ≤ ph−1 ,
adică ph − ph−1 ≤ a1 − 1, ceea ce contrazice lim sup(ph − ph−1 ) = ∞ (curs,
h→∞
pagina 72).
(L. Panaitopol )

11. Presupunem că x ∈ Q; ∃ h ∈ N∗ astfel ı̂ncât an+h = an , ∀ n ≥ n0 .


Fie pn > n0 astfel ı̂ncât pn+1 − pn > 2h (curs, pagina 72). Deci există mai
mult de 2h zerouri consecutive. Deducem că perioada este formată doar din
zerouri; contradicţie.
(L. Panaitopol )

π(n) π(n−1) π(n−1)


12. Dacă n este compus π(n) = π(n−1) şi deci = < .
n n n−1
π(n) 1+π(n−1) π(n−1)
Dacă n este prim, atunci π(n) = π(n−1)+1 şi deci = >
n n n−1
(ultima inegalitate se reduce la π(n − 1) ≤ n − 1).

13. Fie q număr prim q ∈ (n! + n, n! + 2n). Se arată că q − n! ∈ (n, 2n)
şi este prim. Deci π(n! + 2n) − π(n! + n) ≤ π(2n) − π(n).

14. Şirul an = pn+1 − pn este nemărginit (curs, pagina 72) şi deci există o
infinitate de indici pentru care an ≥ ak , ∀ k = 1, n. Avem pn+1 −pn ≥ pk −pk−1
şi deci pn+1 ≥ pn + pk − pk−1 ≥ 2pk − pk−1 .

15. Fie k = π(n), 1 < a1 < a2 . . . am ≤ n şi (pt )t≥1 şirul numerelor prime.
k
Y αj (i)
Avem ai = pj , αj (i) ≥ 0. Deoarece a1 ¤|¤a2 a3 . . . ak , există j astfel ı̂ncât
j=1
k
X
αj (1) > αj (i), deci αj (1) > αj (i) pentru i ∈ 2, k. În acest mod, fiecăruia
i=2
dintre numerele a1 , a2 , . . . , am ı̂i corespunde unul singur dintre numerele prime
p1 , p2 , . . . , pk şi deci enunţul este demonstrat.

pn
16. Punem x = pn şi avem n ∼ , adică pn ∼ n ln pn (1). Prin
ln pn
ln n ln ln pn
logaritmare avem ln pn ∼ ln n + ln ln pn şi deci + ∼ 1. Deoarece
ln pn ln pn
ln ln x
lim = 0, rezultă ln pn ∼ ln n şi ţinând seama de (1), avem pn ∼ n ln n.
x→∞ ln x

190
n
X
pk
k=1 1
17. Din pn ∼ n ln n rezultă că trebuie arătat că lim = .
n→∞ n2 ln n 2
Aplicând lema lui Stolz avem
n+1
X n
X
pk − pk
k=1 k=1 pn+1
lim = lim  n =
n→∞ (n + 1)2 ln(n+1)−n2 ln n n→∞ 1
n ln 1+ +2n ln(n+1)+ln(n+1)
n
n
X
pk
pn+1 1 1 k=1 1
= lim ·  n = şi deci lim = .
n→∞ n ln(n + 1) 1 2 n→∞ n2 ln n 2
ln 1 +
n 1
+2+
ln(n + 1) n
18. Vom demonstra mai mult decât se cere ı̂n enunţ, şi anume
n 1
X
∼ ln ln n. Vom folosi lema lui Stolz, ţinând seama că
k=1
pk
n+1
X 1 Xn 1

p
k=1 k
p
k=1 k 1
pn ∼ n ln n : lim = lim =
n→∞ ln(ln(n + 1)) − ln ln n n→∞ ln(n + 1)
pn+1 ln
ln n
n ln n 1 1
= lim lim 0 „ Ž1 = lim „ Ž =
n→∞ pn+1 n→∞ 1 n→∞
1
B ln 1+ C ln 1+
B n C n
B C
n ln n ln B
B1+
C
C n ln n ln e
B ln n C ln n
@ A

n 1
X

1 k=1
pk
= lim „ Žn = 1 şi deci lim = 1.
n→∞
1 n→∞ ln ln n
ln 1+
n
n p
X n 1
X
k+1 − pk
19. Notăm Sn = . Avem lim Sn = 0 (deoarece Sn ≥
k=1
ln pk n→∞
k=1
pk
∞ 1
X
iar seria este divergentă). Deoarece pk ∼ k ln k, avem
k=1
pk

191
n p n  
X k+1 − pk pn+1 X 1 1
Sn ∼ ∼ + pk − ∼ n+
k=3
ln k ln n k=3 ln(k − 1) ln k
„ Ž „ Žk
1 1
ln 1+ ln 1+
n
X k−1 n
X k−1 n
X 1
+ k ln k ∼n+ ∼n+ .
k=3
ln(k − 1) ln k k=3
ln(k − 1) k=3
ln(k − 1)
n
X 1
k=2
ln(k − 1)
Cu ajutorul lemei lui Stolz, rezultă că lim = 0 şi deci Sn ∼ n.
n→∞ n
√ n √ √ n  
X
pk+1 − pk pn+1 X √ 1 1
20. Avem Sn = = + pk − +
k=1
k n k=2
k−1 k
√ n √
pn+1 X pk
O(1). Deoarece pn ∼ n ln n, rezultă = O(1) şi Sn ∼ +
n k=2
(k − 1)k

Xn ln k ln n
O(1) ∼ √ . Deoarece lim 1 = 0, rezultă că există ε > 0 astfel
n→∞
k=2 k k n3

√ ln k ε X ∞ 1
1
ı̂ncât ln n < εn 3 , ∀ n ≥ 1 şi deci √ < 7 . Deoarece seria 7 este
k k k6 n=1 n6

X∞ ln n
convergentă, rezultă că şi seria √ este convergentă şi deci Sn = O(1),
n=1
n n
adică şirul (Sn )n≥1 este mărginit. Cum acest şir este şi crescător, rezultă că
∞ √p √
X n+1 − pn
este convergent şi deci seria este convergentă.
n=1
n

an
21. Fie = kn . Deoarece pn ∼ n ln n, avem
bn
 
pa n an ln an ln kn
∼ = kn 1 + . (1)
pbn bn ln bn ln bn
an pan
Dacă lim = lim kn = 1, rezultă imediat din (1) că lim = 1.
n→∞ bn n→∞ n→∞ pb
n
pa n
Fie acum lim = 1. Fie L un punct limită al şirului (kn )n≥1 . Există un
n→∞ pb
n
subşir (kni )i≥1 cu lim kni = L. Dacă L = ∞, se contrazice (1) şi deci L ∈ R.
i→∞
 
ln kni
Din (1) rezultă 1 = lim kni 1+ şi deci lim kni = 1.
i→∞ ln bni i→∞

192
an
Cum singurul punct limită este 1, rezultă că lim kn = 1, adică lim = 1.
n→∞ n→∞ bn
(L. Panaitopol )

22. Fie C1 < C2 < · · · < Ck toate numerele compuse din 1, n. Dacă n nu
se poate scrie ca sumă a două numere compuse, atunci n−C1 , n−C2 , . . . , n−Ck
sunt prime (eventual n − Ck = 1). Aşadar π(n) ≥ k − 1. Pe de altă parte,
n
k = n − 1 − π(n) şi deci π(n) ≥ n − 2 − π(n), adică π(n) ≥ − 1. Se arată
2
n
uşor că pentru n ≥ 15 avem π(n) < − 1. Aşadar numerele n care nu se
2
scriu ca sumă de două numere compuse sunt mai mici ca 15. Avem 8 = 4 + 4,
10 = 4 + 6, 12 = 4 + 8, 13 = 4 + 9, 14 = 4 + 10. Numerele care se scriu ca
sumă a două numere compuse sunt 8, 10 şi n ≥ 12.
Observaţie. O altă soluţie a acestei probleme se găseşte ı̂n Capitolul I,
problema 10.
23. Dacă notăm cu pj al j-lea număr prim, ştim că pk+1 < 2pk ∀ k ∈ N∗ ,
pk+2 < 2pk ∀ k ∈ N, k ≥ 4 şi pk+3 < 2pk ∀ k ∈ N, k ≥ 5 (curs, pagina 81).
a) Evident că n ≥ 5. Fie k ∈ N, k ≥ 2, astfel ı̂ncât pk < n ≤ pk+1 . Avem
că n = p1 + p2 + · · · + pk ≤ pk+1 . Însă pentru k ≥ 5 avem p1 + p2 + · · · + pk ≥
pk−1 + pk > 2pk−1 > pk+1 şi rezultă o contradicţie. Deci 2 ≤ k ≤ 4 şi se
observă imediat că doar k = 2, n = 5 este soluţie a problemei noastre.
b) Să demonstrăm ı̂ntâi prin inducţie că p1 + p2 + · · · + pk > 2pk+1 ∀ k ∈ N,
k ≥ 5. Presupunem enunţul adevărat pentru k ≥ 5 şi ı̂l demonstrăm pentru
k + 2. (p1 + p2 + · · · + pk ) + pk+1 + pk+2 > 2pk+1 + pk+1 + pk+2 > 4pk+1 > 2pk+3 .
5
X 6
X
Deoarece pi = 28 > 26 = 2p6 şi pi = 41 > 34 = 2p7 , rezultă că enunţul
i=1 i=1 X
este adevărat pentru ∀ k ∈ N, k ≥ 5. Fie n ∈ N∗ astfel ı̂ncât n = p.
p prim
p≤ n
2

n
Evident că n ≥ 4 şi ∃ k ∈ N∗ astfel ı̂ncât pk ≤ < pk+1 . Avem că n =
2
p1 + p2 + · · · + pk < 2pk+1 . Însă pentru k ≥ 5 avem că p1 + · · · + pk > 2pk+1 şi
obţinem o contradicţie. Deci k ≤ 4 şi se arată imediat că soluţiile sunt k = 3,
n = 10 şi k = 4, n = 17.
c) Arătăm ı̂ntâi prin inducţie că p1 + p2 + · · · + pk > 3pk+1 pentru k ∈ N,
k ≥ 7. Presupunem enunţul adevărat pentru k ≥ 7 şi ı̂l demonstrăm pentru
k + 3.
(p1 +· · ·+pk )+pk+1 +pk+2 +pk+3 > 3pk+1 +pk+1 +pk+2 +pk+3 > 6pk+1 > 3pk+4 .

193
7
X 8
X 9
X
Deoarece pi = 58 > 57 = 3p8 , pi = 77 > 69 = 3p9 , pi = 100 > 87 =
i=1 i=1 i=1
3p10 , rezultă că enunţul este adevărat pentru ∀ k ∈ N, k ≥ 7.
X
Fie n ∈ N∗ astfel ı̂ncât n = p. Evident că n ≥ 6 şi ∃ k ∈ N∗ astfel
p prim
p≤ n
3

n
ı̂ncât pk ≤ < pk+1 . Avem că n = p1 + p2 + · · · + pk < 3pk+1 . Însă pentru
3
k ≥ 7 avem că p1 + · · · + pk > 3pk+1 şi obţinem o contradicţie. Deci k ≤ 6 şi
se arată imediat că soluţia este k = 6 şi n = 41.

24. Deoarece pentru n > 1, n2 nu este prim, condiţia se scrie π(n2 ) −


3 n 2 n
π(n) = n. Avem > π(n) > , n ≥ 3 (curs, pagina 79). Avem n >
2 ln n 3 ln n
2 n 2 3n
− , adică 6 ln n > 2n−9. Fie f (x) = 6 ln x−2x+9, f : (3, ∞) → R.
3 2 ln n 2 ln n
6
Avem f 0 (x) = − 2 < 0 şi deci f este descrescătoare. Avem f (12) < 0 şi deci
x
pentru n ≥ 12, f (n) < 0. Căutăm numerele n ı̂n mulţimea 2, 11. Convine
numai n = 4.

25. Este suficient să arătăm că π(10n ) − π(10n−1 ) ≥ n, ∀ n ∈ N∗ . Pentru


n = 1 inegalitatea este evidentă, aşa că putem presupune că n ≥ 2. Pentru
2 k 3 k
∀ k ∈ N, k ≥ 3, au loc inegalităţile < π(k) < (curs, pagina 79).
3 ln k 2 ln k
 
n n−1
2 10n 3 10n−1 10n−1 20 3
Avem că π(10 ) − π(10 )≥ − = − .
3 ln 10n 2 ln 10n−1 ln 10 3n 2(n − 1)
1 1 1 3 10
Deoarece > 3
= şi − > − (pentru că n ≥ 2) avem
ln 10 ln e 3 2(n − 1) 3n
 
10n−1 20 10
(continuând inegalităţile de mai sus) că π(10n )−π(10n−1 ) > − =
3 3n 3n
10n
> n şi problema este rezolvată (faptul că 10n > 9n2 ∀ n ∈ N∗ rezultă
9n
imediat prin inducţie). Problema 1 este un caz particular al acestei probleme.

26. Fie k numărul de cifre ale lui n. 10k−1 ≤ n < 10k ; deci ln n ≥
 
ln n 2n
(k −1) ln 10. Avem s(n) ≤ 9k ≤ 9 +1 . Cum π(n) > pentru n ≥ 3
ln 10 3 ln n
ln n 2n
(curs, pagina 79), rezultă +1 > . Se arată că n < 1000. Dacă n
ln 10 27 ln n
are 3 cifre, s(n) ≤ 27 = π(103) şi deci n ≤ 103. Rezultă s(n) ≤ 18 = π(61),

194
deci n ≤ 61. Prin verificări rezultă că n ∈ {15, 27, 39}.
(L. Panaitopol )

27. Avem f (1) = 1, f (2) = f (p1 ) = 1 + 1 = 2, f (3) = f (p2 ) = 2 + 1 = 3,


f (4) = f (2) + f (2) = 4, f (5) = f (p3 ) = 3 + 1 = 4, f (6) = f (2) · f (3) = 6.
2
Arătăm prin inducţie că P (n) : f (n) ≥ n 3 . Presupunem P (k) adevărat pentru
k ≤ n − 1. Dacă n este prim n = pk , avem f (n) = f (pk ) = k + 1 > k. Arătăm
2 2 n
că k > pk3 pentru k ≥ 1. Avem π(n) > , n ≥ 3 (curs, pagina 79). Pentru
3 ln n
2 pk 3 x
n = pk avem k > . Presupunem că pk ≥ k 2 . Cum funcţia f (x) =
3 ln pk ln x
3 3
2 pk 2 k2 4 k2
pentru x ≥ e este crescătoare, avem k ≥ ≥ = . Se arată
3 ln pk 33 9 ln k
ln k
2
9 √
că inegalitatea ln x > x este falsă pentru x ≥ 114. Aşadar pentru k ≥ 114
2
4
avem k > pk . După verificări rezultă că inegalitatea este adevărată pentru
3

2 2
k ≥ 3. Aşadar pentru k ≥ 3 şi n = pk prim, f (n) > k > pk3 = n 3 . Pentru
2 3
k = 1, f (p1 ) = 1 + 1 = 2 > 2 3 iar pentru k = 2, f (p2 ) = 2 + 1 > 2 2 şi deci
3 r
Y r
Y r
Y 2
αi 2
f (pk ) > pk2 ∀ k ≥ 1. Dacă n = pαi i avem f (n) = f (pi )αi ≥ pi3 = n3
i=1 i=1 i=1
1 1 ∞
X 1
şi deci 2 < 4 şi deci seria este convergentă. Rezultă (curs,
f (n) n3 n=1
f 2 (n)
∞  
X 1 Y 1
pagina 46) că avem 1 − = 1 şi deci
n=1
f 2 (n) p prim f 2 (p)


X 1 1 1
2 (n)
=   = ∞  .
n=1
f ∞
Y 1 Y 1
1− 2
1−
n=1
f (pn ) n=1
(n + 1)2

∞  
∞ n(n + 2)
Y 1 Y
Dar 1− = . Fie
n=1
(n + 1)2 n=1
(n + 1)2

n k(k + 2)
Y 1·3 2·4 3·5 (n − 1)(n + 1) n(n + 2) n+2
xn = = · 2 · 2 ... · =
k=1
(k + 1)2 22 3 4 n 2 (n + 1)2 2(n + 1)

195
∞ n(n + 2)
Y 1 X∞ 1 1
şi deci = lim x n = şi de aici = = 2.
n=1
(n + 1)2 n→∞ 2 n=1
f 2 (n) 1
2
(L. Panaitopol )

π(x)
28. Avem lim = 1 (curs, pagina 73). Rezultă de aici că
x→∞ x
ˆ ’

π(ax) − π(bx) π(ax) a log x π(bx) b log x


lim = lim · − · =a−b
x→∞ x x→∞ xa log ax bx log bx
log x log ax log bx
π(ax) log x
deoarece lim = 1 şi lim = 1. Rezultă că pentru ∀ ε > 0,
x→∞ ax x→∞ log ax

log ax
x
ε < a − b, ∃ x0 astfel că pentru x > x0 avem π(ax) − π(bx) > (a − b − ε)
log x
şi deci lim (π(ax) − π(bx)) = ∞.
x→∞

29. Trebuie arătat că pentru orice a > b > 0, ı̂n intervalul [b, a] există cel
puţin un element al lui A. Ţinem seama de problema precedentă şi alegem un
număr prim q astfel că π(aq) − π(bq) ≥ 1 şi deci ∃ p astfel că bq ≤ p ≤ aq şi
p
deci a ≤ ≤ b.
q

X  1

30. Ştim că ψ(x) = θ x i (curs, pagina 85, Teorema 2) şi
√ i=1
ψ(x) − 2ψ ( x) ≤ θ(x) (curs, pagina 85, Consecinţa 3). De aici deducem
√ √
că θ ( x) ≤ ψ(x) − θ(x) ≤ 2ψ ( x). Ştim că există constantele 0 < b < c
astfel ı̂ncât bn ≤ θ(n) ≤ ψ(n) ≤ cn, ∀ n ≥ 2 (curs, pagina 88). Pentru fiecare
n ∈ N, n ≥ 2 aplicăm teorema lui Lagrange funcţiei f (y) = y a pe inter-
valul [θ(n), ψ(n)] şi deducem existenţa unui dn ∈ [θ(n), ψ(n)] astfel ı̂ncât an =
√ √
(ψ(n))a − (θ(n))a = a(ψ(n) − θ(n))da−1n . Avem că b n ≤ ψ(n) − θ(n) ≤ 2c n,
bn ≤ dn ≤ c · n şi din toate consideraţiile precedente, deducem că seria din
X 1 X 1
enunţ are aceeaşi natură cu seria 1 . Ţinând cont că seria este
n 2 −a nl
convergentă dacă şi numai dacă l > 1, rezultă că seria din enunţ este conver-
1 1
gentă dacă şi numai dacă − a > 1, adică dacă şi numai dacă a < − .
2 2
(L. Panaitopol )

196
CAPITOLUL 9

Congruenţe

1. Pentru orice x ∈ Z avem x3 ≡ 0, 1, −1 (mod 9) şi de aici rezultă că


x3 + y 3 + z 3 ≡ 0, ±1, ±2, ±3 (mod 9) şi de aici rezultă enunţul.

2. Folosim ı̂n continuare următoarele congruenţe: 73 ≡ −1(43), 76 ≡ 63 ≡


1(43). Fie p prim, p > 3. Avem că p = 6t + 1 sau p = 6t + 5, t ∈ N. În primul
caz avem că 7p − 6p − 1 ≡ 7 − 6 − 1 ≡ 0(43). În cel de-al doilea, avem că
7p − 6p − 1 ≡ 75 − 65 − 1 ≡ −49 − 36 − 1 = −86 ≡ 0(43). Enunţul este
demonstrat.

3. 1000027 = 106 + 33 = (102 )3 + 33 = (102 + 3)(104 − 3 · 102 + 32 ) =


103 · 9709. Avem că 106 ≡ 1 (mod 7) şi 33 ≡ −1 (mod 7); deci 7|106 + 33 ,
1000027 = 103 · 7 · 1387. Trebuie verificaţi eventualii factori primi ai lui 1387

mai mici decât 1387 < 38. Se obţine că 19|1387, 1387 = 19 · 73 şi deci
1000027 = 103 · 7 · 19 · 73 este descompunerea ı̂n factori primi căutată.
nn n
4. Cum 1989 = 32 · 13 · 17, trebuie arătat că nn ≡ nn (mod a), unde
a ∈ {9, 13, 17}. Pentru a = 9 şi 3|n congruenţa este clară. Dacă a = 9 şi
n n
3¤|¤n, trebuie arătat că nn ≡ nn (mod 6). Dacă n ≡ 1(3), este clar că nn ≡
n
nn (3). Dacă n ≡ −1(3) rezultă că nn ≡ nn (3) căci nn ≡ n(2). Faptul că
n
nn ≡ nn (2) e banal. Pentru a = 13 şi 13|n, congruenţa iniţială este evidentă.
n
Dacă a = 13 şi 13¤|¤n, trebuie arătat că nn ≡ nn (12). Am arătat anterior
n n
că nn ≡ nn (3). Arătăm că nn ≡ nn (16) (avem nevoie doar de congruenţa
modulo 4, dar aceasta ne ajută pentru cazul următor). Dacă n este par, n ≥ 4
n
şi nn ≡ nn ≡ 0(16). Dacă n este impar, n2 ≡ 1(8), n4 ≡ 1(16) şi pentru a
n
arăta că nn ≡ nn (16) este suficient să arătăm că nn ≡ n(4) pentru n impar.
Dacă n ≡ 1(4) congruenţa este evidentă; la fel şi pentru n ≡ −1 (mod 4).
Dacă a = 17 şi 17|n, congruenţa iniţială este evidentă. Pentru a = 17 şi 17¤|¤n,
n
ea este echivalentă cu nn ≡ nn (16), lucru demonstrat anterior. Am folosit ı̂n
cele de mai sus că m6 ≡ 1(9) dacă 3¤|¤m, m12 ≡ 1(13) dacă 13¤|¤m şi m16 ≡ 1(17)
dacă 17¤|¤m.

197
5. Presupunem că ∃ p prim astfel ı̂ncât p|2m − 1 şi p|2n + 1. Cum 2m − 1
este impar (m ≥ 1), rezultă că p 6= 2. Avem 2m ≡ 1 (mod p) şi deci 2mn ≡ 1
(mod p). Dar 2n ≡ −1 (mod p) şi deci 2mn ≡ (−1)m ≡ −1 (mod p). Din cele
două relaţii de mai sus rezultă că 1 ≡ −1 (mod p) şi deci p = 2. Contradicţie.

6. Deoarece n este impar, din Teorema lui Euler ştim că 2ϕ(n) ≡ 1
(mod n). Dar ϕ(n) ≤ n şi deci ϕ(n)|n!, n! = k · ϕ(n), k ∈ N, 2n! = (2ϕ(n) )k ≡
1k = 1 (mod n).

7. Scriem algoritmul lui Euclid pentru 1000 şi 237:

1000 = 237 · 4 + 52
237 = 52 · 4 + 29
52 = 29 · 1 + 23
29 = 23 · 1 + 6
23 = 6·3 + 5
6= 5·1 + 1
5= 1·5

1000 1
Avem că =4 . Calculăm
237 1
4+
1
1+
1
1+
1
3+
1
1+
5
1 173 1000 173 (−1)7 1
4+ = . Avem că − = =−
1 41 237 41 41 · 237 41 · 237
4+
1
1+
1
1+
1
3+
1
(7 este numărul de operaţii din algoritmul lui Euclid). Deci 41·1000−237·173 =
−1 şi 237 · 173 ≡ 1 (mod 1000).

8. a) 237 = 3 · 79. 79 · 13 = 1027 ≡ 27 (mod 1000) şi ecuaţia nostră se


rescrie (ı̂nmulţind congruenţa iniţială cu 13) 81x ≡ 13 (mod 1000).

198
Deoarece 1 ≡ 8001 = 9 · 889 (mod 1000), deducem că 1 ≡ 81 · 8892 ≡
81 · 1112 = 81 · 12321 ≡ 81 · 321 (mod 1000). Înmulţind congruenţa 81x ≡ 13
(mod 1000) cu 321 obţinem că x ≡ 13 · 321 = 4173 ≡ 173 (mod 1000).
b) Notăm x = 12371199 . Deoarece (237, 1000) = 1, rezultă că 237ϕ(1000) ≡
1 (mod 1000). Cum ϕ(1000) = 400, din cele de mai sus rezultă că 237400 ≡ 1
(mod 1000). 237 · x ≡ 2371200 = (237400 )3 ≡ 13 ≡ 1 (mod 1000). Ţinând cont
de punctul a) x ≡ 173 (mod 1000), ceea ce ı̂nseamnă că 173 sunt ultimele trei
cifre ale numărului nostru.
c) x = 12361199 . Evident că x ≡ 0 (mod 8). Cum ϕ(125) = 100,
(−14)100 ≡ 1 (mod 125), deducem că −14x ≡ (−14)1200 ≡ 1(125). Înmulţim
ultima congruenţă cu −9 şi obţinem (9 · 14 = 126 ≡ 1(125)) că x ≡ −9 ≡ 116
(mod 125). Din această congruenţă rezultă că x = 116 + 125k, k ∈ N. Cum
8|x, rezultă k ≡ 4 (mod 8) şi x ≡ 616 (mod 1000). Ultimele trei cifre sunt
616.
d) Notăm x = 12351199 . Evident că x ≡ 0 (mod 125) şi x ≡ 31199 ≡ 3
(mod 8) (căci 32 ≡ 1 (mod 8)). Din x = 125k, x ≡ 3 (mod 8), rezultă că
5k ≡ 3(8), k ≡ 7 (mod 8), x ≡ 875 (mod 1000). Ultimele trei cifre sunt 875.

9. (a, 11) = 1 şi a10 ≡ 1 (mod 11). Din 3a7 ≡ 5 (mod 11) rezultă a7 ≡ 9
(mod 11) şi deci a21 ≡ 3 (mod 11), a ≡ 3 (mod 11).

10. Avem a7 ≡ 10 (mod 19) şi a18 ≡ 1 (mod 19). Deci a91 ≡ (a7 )13 ≡
1013 ≡ 13 (mod 19) şi a91 ≡ a(a18 )5 ≡ a (mod 19).

11. Evident că 17¤|¤a şi a16 ≡ 1 (mod 17). Ecuaţia din enunţ se rescrie ca
a5 ≡ 3 (mod 17). Ridicăm la puterea a 13-a ultima congruenţă şi obţinem că
a65 ≡ 313 ≡ 12 (mod 17). Dar a65 ≡ a(a16 )4 ≡ a (mod 17) şi am obţinut că
a ≡ 12 (mod 17).

12. 103 ≡ 1 (mod 37). Dacă n = 3k, k ∈ N, atunci 10n −1 = (103 )k −1 ≡ 0


(mod 37). Dacă n = 3k + 1, atunci 10n − 1 ≡ 10 − 1 = 9 (mod 37), iar dacă
n = 3k + 2, atunci 10n − 1 ≡ 102 − 1 = 99 ≡ 25 (mod 37).

13. a) Deoarece 109 este prim, avem că 108! ≡ −1 (mod 109). Notând x =
100!, avem că x(−8)(−7) . . . (−1) ≡ −1(109). Deci 40320x ≡ −1 (mod 109),
99x ≡ −1(109), −10x ≡ −1(109), 10x ≡ 1 ≡ 110(109), x ≡ 11 (mod 109).
k k+2
b) Pentru orice număr prim p avem că Cp−1 ≡ Cp−1 (mod p) ∀ k ∈ N,
130 0
k + 2 ≤ p − 1. De aici deducem că C210 ≡ C210 ≡ 1(211) (am ţinut cont că
211 e număr prim).

199
210! 1
Deci ≡ 1(211) şi 130! · 75! ≡ − (mod 211); am
130! · 80! 80 · 79 · 78 · 77 · 76
ţinut cont de teorema lui Wilson (210! ≡ −1 (mod 211)). După câteva calcule
1
obţinem că 130! · 75! ≡ − ≡ 70 (mod 211).
3

14. Notăm cu a cifra din enunţ. Avem congruenţa x2 ≡ a·1111 (mod 104 ).
De aici deducem că x2 ≡ 7a (mod 16). Avem a ∈ {0, 1, 4, 5, 6, 9}. Dacă a este
impar, atunci şi x este impar şi avem că 1 ≡ x2 ≡ 7a (mod 8), a ≡ 7 (mod 8).
Am ajuns la o contradicţie căci 1, 5, 9 nu au aceste proprietăţi. Dacă a este
par, a 6= 0, rezultă că şi x este par şi 0 ≡ x2 ≡ 7a (mod 4). De aici deducem
că a = 4. Avem x = 2k, k ∈ N, 4k 2 ≡ 28 (mod 16), k 2 ≡ 7 ≡ 3 (mod 4);
ultima congruenţă este imposibilă. De aici a = 0 şi enunţul este demonstrat.

15. Avem (a, 7) = 1 şi deci a6 ≡ 1 (mod 7). Din a5 ≡ 2 (mod 7) de-
ducem 2a ≡ 1 (mod 7) şi deci a ≡ 4 (mod 7). Avem (a, 13) = 1 şi deci
a12 ≡ 1 (mod 13). Rezultă succesiv a36 ≡ 1 (mod 13), a41 ≡ a5 (mod 13) ≡ 2
(mod 13). Avem 1 ≡ a12 (mod 13) ≡ a2 · a10 (mod 13) ≡ 4a2 (mod 13) şi de
aici a2 ≡ −3 (mod 13) şi apoi a4 ≡ 9 (mod 13). Aşadar 2 ≡ a5 (mod 13) ≡ 9a
(mod 13) şi deci a ≡ 6 (mod 13). Avem a ≡ 4 (mod 7) şi a ≡ 6 (mod 13),
adică a = 7x + 4 şi a = 13y + 6 şi deci 7x = 13y + 2 = 14y − y + 2.
y−2 y−2
x = 2y− ∈ Z. Rezultă = λ ∈ Z şi ı̂n final y = 7λ+2 şi a = 91λ+32.
7 7
2
16. a) Din Mica Teoremă a lui Fermat rezultă imediat că 3p = (3p )p ≡
2 2 2
3p ≡ 3 (mod p) şi 11p ≡ 11 (mod p). Deci 0 ≡ 3p + 11p ≡ 3 + 11 = 14
(mod p) şi deci p = 2 sau p = 7. Se observă imediat că doar p = 7 este soluţia
problemei.
2 2
b) 42p + 32p ≡ 16 + 9 = 25 (mod p). Rezultă că p = 5 şi se verifică
imediat că 5 este soluţie a problemei.

4(2p−1 − 1)(2p−1 + 1) .
17. Avem n − 1 = . Deoarece 2p−1 − 1..p, rezultă că
3
2p|n − 1, adică n − 1 = 2kp. Cum 4p − 1 = 3n, rezultă că 2n−1 − 1 = 22kp − 1 =
(4p )k − 1 = (4p − 1)(4p(k−1) + 4p(k−2) + · · · + 4p + 1) = 3n(4p(k−1) + 4p(k−2) +
.
· · · + 4p + 1) şi deci 2n−1 − 1..n.

18. Din Mica Teoremă a lui Fermat rezultă 2p ≡ 2 (mod p) şi deci 2p +1 ≡ 3
(mod p). Cum p|2p + 1, adică 2p + 1 ≡ p (mod p), rezultă p = 3.

200
19. Deoarece ap ≡ a(p) şi bp ≡ b(p), condiţia din enunţ ne arată că
p
X
a = b+pc, c ∈ Z. Avem că ap = (b+pc)p = bp +Cp1 bp−1 ·pc+ Cpk bp−k (pc)k ≡
k=2
bp + Cp1 bp−1 pc ≡ bp (mod p2 ).

20. Avem E = apq − aq = aq (aq(p−1) − 1) = a(ap−1 − 1)b, unde b =


. .
aq−1 (a(p−1)(q−1) + a(p−1)(q−2) + · · · + a(p−1) + 1) şi deci E ..ap − a..p. De aici
rezultă că p|apq − ap − aq + a şi la fel se arată că q|apq − ap − aq + a.

21. Din Mica Teoremă a lui Fermat ştim că (k 2 )p ≡ k 2 (mod p) şi deci
!2 !2 !2
Xn Xn n(n + 1) Xn X n
2p+1 3 p
k ≡ k = = k ≡ k (mod p).
k=1 k=1
2 k=1 k=1
p−1
€ √ Šp € √ Šp X 2
22. Avem 2 + 3 + 2 − 3 = 2p+1 +2 Cp2k 2p−2k 3k = 2p+1 +Mp.
€ √ Šp ” √ k=1— √ √
Deoarece 0 < 2− 3 < 1, deducem că (2 + 3)p = (2+ 3)p +(2− 3)p −1,
” √ —
(2 + 3)p = 2p+1 − 1 + Mp = 2(2p − 2) + 3 + Mp ≡ 3 (mod p). Să observăm
” √ — ” √ —
că enunţul este adevărat şi pentru p = 2 căci (2 + 3)2 = 7 + 4 3 =
”√ — ” √ —
7 + 48 = 13 şi (2 + 3)2 − 3 = 10 este evident multiplu de 2.

23. Presupunem n compus. Fie q prim şi q|n şi deci n = sq cu s > 1.
Avem 3sq − 2sq = (3s )q − (2s )q = pm şi deci 3s − 2s |pm , adică 3s − 2s = pt cu
q
X
m > t. Avem 3s = 2s + pt şi pm = (2s + pt )q − 2sq = Cqk 2s(q−k) ptk . Rezultă
k=1
q
X
pm−t = q2s(q−1) + Cqk 2s(q−k) p(k−1)t . Rezultă p|q2s(q−1) şi deci p = 2 sau
k=2
p = q. Din enunţ rezultă că p este impar, deci p = q şi din 3sq − 2sq = pm
rezultă (3p )s − (2p )s = pm şi de aici 3p − 2p = pu cu u ≥ 1. Din teorema lui
. . . .
Fermat rezultă 3p − 3..p şi 2p − 2..p. Deci 3p − 2p − 1..p, adică pu − 1..p, ceea ce
constituie o contradicţie. Deci n este prim.

24. Avem 2ϕ(m) ≡ 1 (mod m) deoarece (2, m) = 1. Notând x = 2ϕ(m)−1 ,


m+1
avem 2x ≡ 1 (mod m) şi deci x ≡ (mod m).
2
25. Deoarece (a, b) = 1, avem că aϕ(b) ≡ 1 (mod b) şi bϕ(a) ≡ 1 (mod a).
Deci aϕ(b) = 1 + bh şi bϕ(a) = 1 + ak, k ∈ N. Avem 0 ≡ aϕ(b) bϕ(a) (mod ab) şi
deci (1 + bh)(1 + ak) ≡ 0 (mod ab). De aici rezultă imediat că 1 + ak + bh ≡ 0
(mod ab) şi aϕ(b) + bϕ(a) = 1 + (1 + bh + ak) ≡ 1 (mod ab).

201
26. Este suficient să arătăm că ∀ p prim, m = pb · m1 , p¤|¤m1 , b ≥ 1, atunci
pp |am − am−ϕ(m) (se arată imediat că m ≥ ϕ(m) ∀ m ∈ N∗ ). Dacă p¤|¤a, atunci
b
aϕ(p ) ≡ 1 (mod pb ) şi deci aϕ(m) ≡ 1 (mod (pb )) căci ϕ(m) = ϕ(m1 )ϕ(pb ).
am − am−ϕ(m) = am−ϕ(m) (aϕ(m) − 1) va fi deci un multiplu de pb . Rămâne de
analizat cazul p|a. Este suficient să arătăm ı̂n acest caz că m − ϕ(m) ≥ b. Dar
 
ϕ(m1 ) Y 1
b−1
m − ϕ(m) = p (pm1 − (p − 1)ϕ(m1 )). Deoarece = 1− ≤
m1 p prim
q
q|m1

p
1< , rezultă imediat că pm1 − (p − 1)ϕ(m1 ) ≥ 1 şi pentru a demonstra
p−1
că m − ϕ(m) ≥ b este suficient să arătăm că pb−1 ≥ b ∀ b ∈ N∗ . Acest lucru
ı̂nsă este evident deoarece pb−1 ≥ 2b−1 ≥ b (ultima inegalitate se arată prin
inducţie după b ∈ N∗ ).

27. a) Înmulţind congruenţele cu 4, 3 şi respectiv 5, obţinem x ≡ −2


(mod 7), x ≡ −1 (mod 8), x ≡ −2 (mod 9).
Urmărind etapele din demonstraţia lemei chineze (curs, pagina 99), m1 = 7,
m2 = 8, m3 = 9 şi M = 7 · 8 · 9 = 504, M1 = 72, M2 = 63, M3 = 56. Din
M1 y1 ≡ 1 (mod m1 ), adică 72y1 ≡ 1 (mod 7), rezultă y1 = 4. Analog obţinem
y2 = −1, y3 = 5. Avem apoi e1 = M1 y1 = 4 · 72 = 288, e2 = −63, e3 = 280 şi
x
e = a1 e1 +a2 e2 +a3 e3 = −2 · 288−63(−1)+280(−2) = −1073 ≡ −65 (mod 504)
şi deci x = 504λ − 65, λ ∈ Z.
b) Se obţine x = 273λ + 11.

28. 343 = 73 . Rezolvăm congruenţa x3 + x + 4 ≡ 0(7) şi observăm că


singura soluţie este x = 2. Căutăm acum x = 2 + 7α astfel ı̂ncât x3 + x + 4 ≡
0(49). Înlocuind pe x obţinem că 8 + 3 · 22 · 7α + (2 + 7α) + 4 ≡ 0(49). Aceasta
se scrie 7(2 + 13α) ≡ 0(72 ), 2 + 13α ≡ 0(7), α = 2. Se observă că x = 16 este
soluţie şi pentru x3 + x + 4 ≡ 0(343).

29. 990 = 2 · 32 · 5 · 11. Avem că x ≡ 0, 1(2), x ≡ 2, 6(9), x ≡ 2(5),


x ≡ 2, 8(11). Avem opt soluţii modulo 990 care se obţin din Lema chineză a
resturilor. După calcule rezultă că x ≡ 2, 222, 272, 492, 497, 717, 767, 987(990).

30. a) Avem f (x) = 7x4 + 19x + 25. Congruenţa f (x) ≡ 0 (mod 3) are
numai soluţia x0 = 1. Avem f 0 (x) = 28x3 + 19 şi deci f 0 (1) 6≡ 0 (mod 3).
Aşadar congruenţa f (x) ≡ 0 (mod 32 ) are soluţie unică x1 = 3k + x0 , adică
f (x0 ) f (1)
x1 = 3k +1. Cum = = 17 şi f 0 (x0 ) = 47, avem 47k ≡ −17 (mod 3)
p 3
şi deci k = −1. Aşadar x1 = −2. Căutăm x2 de forma x2 = 9h + x1 = 9h − 2.

202
f (x1 ) f (−2)
Cum = = 11 şi f 0 (−2) = −205, avem −205h ≡ −11 (mod 3) şi
p2 9
h = 2. Rezultă x2 = 16. Am folosit Teorema 1, pagina 104, curs.
b) Fie f (x) = x3 − x + 2. Congruenţa f (x) ≡ 0 (mod 2) are soluţiile
x0 = 0 şi x00 = 1. Avem f 0 (x0 ) = 3x2 − 1. Avem f 0 (1) ≡ 0 (mod 2) şi
f (1) 2
= = 1 6≡ 0 (mod 2) şi deci x00 = 1 nu generează soluţii pentru congruenţa
2 2
f (0)
f (x) ≡ 0 (mod 4). Pentru x0 avem = 1 şi f 0 (0) ≡ 1 (mod 2). Căutăm
2
x1 = 2k + x0 = 2k cu k = −1 (mod 2), deci k = 1 şi x1 = 2. Căutăm
f (2) 8
x2 = 4h + 2. Cum 2 = = 2 şi f 0 (2) = 11, avem 11h ≡ −2 (mod 2) şi
2 4
f (2)
deci h = 0, adică x2 = 2. Căutăm x3 = 8l + 2. Cum 3 = 1 şi f 0 (2) = 11,
2
avem 11l ≡ −1 (mod 2) şi deci l = 1 şi x3 = 10. Analog se determină x4 = 10.
Aşadar f (x) ≡ 0 (mod 32) dacă şi numai dacă x ≡ 10 (mod 32).

 
1
31. Avem ϕ(121) = 112 1− = 110. Dacă x este soluţie, atunci
11
(x, 11) = 1 şi deci xϕ(121) ≡ 1 (mod 121), adică x112 ≡ x2 (mod 121). Este
suficient să considerăm congruenţa f (x) ≡ 0 (mod 121) cu f (x) = x2 + 3x + 5.
f (x0 ) 33
Congruenţa f (x) ≡ 0 (mod 11) are numai soluţia x0 = 4. Avem = =3
11 11
f (x 0 )
şi f 0 (x0 ) = 11. Cum 6≡ 0 (mod 11), rezultă că nu există soluţii pentru
11
congruenţa f (x) ≡ 0 (mod 112 ).

32. Pentru n = 1 congruenţa are soluţiile 1, 2, . . . , p−1. Fie f (x) = xp−1−1.


Avem f 0 (x) = (p − 1)xp−2 şi pentru orice a ∈ {1, 2, . . . , p − 1} avem
f 0 (a) = (p − 1)ap−2 6≡ 0 (mod p) şi deci fiecare soluţie a generează câte o
soluţie pentru congruenţa xp−1 ≡ 1 (mod p2 ). Aşadar această congruenţă are
p − 1 soluţii. Aplicând ı̂n continuare acest procedeu, enunţul este justificat.

33. a) Fie a = M8 + α. Dacă α este impar şi x2 ≡ a (mod 23 ), rezultă


că x este impar şi deci M8 + 1 ≡ M8 + α (mod 8) şi deci α ≡ 1 (mod 8),
adică pentru α ∈ {3, 5, 7} nu avem soluţii. Dacă α = 1, presupunem că există
k maxim, pentru care există x astfel că x2 ≡ a (mod 2k ). Avem k ≥ 3 şi
x2 − a = 2k b, cu b impar şi x impar. Fie x1 = 2k−1 + x. Avem x21 − a =
(2k−1 + x)2 − a = 22k−2 + 2k x + x2 − a = 22k−2 + 2k x + 2k b = 2k (2k−2 + x + b).

203
.
Cum x + b este par, rezultă x21 − a .. 2k+1 ; contradicţie. Aşadar pentru orice k
şi a = M8 + 1 ecuaţia x2 ≡ a (mod 2k ) are soluţii.
b) Fie a par. Avem a = 2s A cu s ≥ 1, A impar. Dacă s este impar, arătăm
că x2 ≡ a (mod 2s+1 ) nu are soluţii. Din 2s+1 |x2 − 2s A rezultă 2s |x2 . Cum
s este impar, avem 2s+1 |x2 şi deci 2s+1 |2s A. Rezultă contradicţia 2|A. Dacă
s
s este par, din 2s+3 |x2 − a = x2 − 2s A rezultă 2s |x2 şi deci x = 2 2 B. Avem
2s+3 |2s B 2 − 2s A, adică 8|B 2 − A şi deci B este impar. Rezultă 8|M8 + 1 − A,
s
adică 8|A−1, A = 8d+1 şi a = 2s (8d+1). Pentru orice k ≥ s+3 avem x = 2 2 B
şi din x2 ≡ a (mod 2k ) avem 2s (B 2 − A) ≡ 0 (mod 2k ), adică B 2 − A ≡ 0
(mod 2k−s ). Cum A = 8d + 1, din cazul a) rezultă că există B care verifică
condiţia. Aşadar pentru a par forma este a = 4t (8d + 1). Soluţia finală este
a = 4t (8d + 1) cu t, d ∈ N.

34. Fie p un divizor prim al lui 2n2 + 6n + 5. Evident că p 6= 2. Să


presupunem că p = 4k +3 (k ∈ N). Avem că p|2n2 +6n+5 = (n+1)2 +(n+2)2
şi de aici deducem că p|n + 1, p|n + 2, p|1 = (n + 2) − (n + 1); contradicţie. Deci
orice divizor prim al lui 2n2 + 6n + 5 este de forma 4k + 1 şi de aici rezultă
imediat enunţul.

35. Condiţia necesară şi suficientă, ca un număr să se scrie ca sumă de


două pătrate, este ca factorii săi primi de forma 4k + 3 să se afle la putere
pară (eventual 0). Cum ab + bc + ca este prim, rezultă că (a, b) = 1, (b, c) = 1,
(c, a) = 1. Cum ı̂n descompunerea lui abc factorii primi de forma 4k + 3 se
găsesc la putere pară, acelaşi lucru se ı̂ntâmplă pentru fiecare dintre numerele
a, b, c.
(L. Panaitopol )

36. ab are factorii primi de forma 4k + 3 numai la putere pară (eventual


0). Cum (a, b) = 1, rezultă că a şi b au de asemenea factorii primi de forma
4k + 3 la putere pară şi deci a şi b se scriu ca sume de două pătrate.

37. Avem a + b = c prim, ab = a(c − a). Dacă d este un număr prim astfel
ı̂ncât d|a şi d|c − a, atunci d|c şi d = c. Cum 0 ≤ c − a ≤ c şi c|c − a, rezultă
imediat că a = 0 sau b = 0, ceea ce este imposibil. Deci (a, b) = 1 şi se aplică
exerciţiul precedent.

38. Dacă n este par, n ≥ 2, atunci 7n2 − 1 ≥ 3 şi 7n2 − 1 ≡ 3 (mod 4).
Dacă n este impar, 7n2 − 1 ≡ 6 (mod 8). În ambele cazuri deducem existenţa
unui număr prim p, p ≡ 3 (mod 4), astfel ı̂ncât p|7n2 − 1.

204
m2 + 1
Dacă ar fi ı̂ntreg, ar rezulta că p|m2 + 1; obţinem contradicţia p|1.
7n2 − 1

2a2 − 1
39. Presupunem că este număr ı̂ntreg. Dacă b este par, se obţine
b2 + 2
contradicţia 2|2a2 −1. Rezultă că b este impar; b = 2k+1. Avem b2 +2 = M4+3
şi deci b2 +2 are un factor prim p = 4s+3 care divide şi pe 2a2 −1. Din p|b2 +2
şi p|2a2 − 1 rezultă p|b2 + 2 + 2(2a2 − 1) = (2a)2 + b2 . Deoarece p = 4s + 3,
rezultă p|2a şi p|b. Cum p|b2 + 2, rezultă contradicţia p|2.
(L. Panaitopol )

40. Dacă x = y = 0, enunţul este evident. Să presupunem că (x, y) 6= (0, 0).
Există atunci cel mai mare divizor comun È
al numerelor x, y : d = (x, y),
x = dx1 , y = dy1 , (x1 , y1 ) = 1. Deoarece ax2 + by 2 ∈ Q, rezultă imediat că
1È È
ax2 + by 2 = ax21 + by12 ∈ Q şi că ax21 + by12 = z 2 , z ∈ N. Dacă x1 şi y1
d
sunt impare, rezultă că z este par şi obţinem contradicţia 0 ≡ z 2 = ax21 +by12 ≡
a + b ≡ 3 + 3 ≡ 2 (mod 4). Dacă x1 6≡ y1 (mod 2), rezultă că z este impar şi
contradicţia 1 ≡ z 2 = ax21 + by12 ≡ 3 (mod 4). Deci x1 şi y1 sunt ambele pare,
ceea ce contrazice faptul că x1 şi y1 sunt prime ı̂ntre ele.

41. Să presupunem că există numerele x, y, z cu proprietatea din enunţ.


Dacă z este impar, atunci z 2 −c2 ≡ 1−1 ≡ 0 (mod 8) şi obţinem o contradicţie,
căci x2 + a2 ≡ 1, 2 (mod 4), y 2 + b2 ≡ 1, 2 (mod 4) şi (x2 + a2 )(y 2 + b2 ) nu
este multiplu de 8. Deci z este par şi z 2 − c2 ≡ −1 (mod 4), z 2 − c2 ≥ 3
(este număr pozitiv congruent cu 3 modulo 4). Există deci un număr prim
p ≡ 3 (mod 4), p|z 2 − c2 şi exponentul lui p ı̂n z 2 − c2 este impar. Acest lucru
este ı̂nsă imposibil căci z 2 − c2 se scrie ca sumă a două pătrate: z 2 − c2 =
(x2 + a2 )(y 2 + b2È ) = (xy + ab)2 + (xb − ya)2 .
Consecinţă: x2 y 2 + x2 + y 2 + 2 ∈ / Q.
Soluţie. Dacă numărul din enunţ ar fi raţional, rezultă că x2 y 2 +x2 +y 2 +2 =
z 2 , z ∈ Z. Luăm a = b = c = 1 ı̂n problema precedentă şi rezultă o contradicţie.
n
42. Fie p|n, p = 4k +3, p număr prim. Notăm y p = z şi avem x2 +3 = z p .
Dacă x este impar, atunci z este par şi avem contradicţia M8 + 4 = M8.
Pentru x = 2a rezultă z = 2b − 1 şi ecuaţia se scrie 4(a2 + 1) = (2b − 1)p + 1 =
2b(2b−1)p−1 −(2b−1)p−2 +· · ·+1) şi deci b = 2c. Rezultă a2 +1 = c(M4+p) =
c(M4 + 3) şi există q prim, q ≡ 3 (mod 4), q|a2 + 1; contradicţie.

205
43. a) Dacă x este par, din (x2 −1)(x2 +1) = py 4 rezultă (x2 −1, x2 +1) = 1
şi cum p¤|¤x2 + 1, rezultă x2 − 1 = pa4 şi x2 + 1 = b4 . Din x2 + 1 = b4 rezultă
x = 0 şi contradicţia pa4 = −1.
b) Dacă x este impar, considerăm soluţia minimală (x0 , y0 ), x0 , y0 ∈ N.
€ Š€ Š € Š
Presupunem x0 > 1 şi y0 > 0. Din x20 −1 x20 +1 = py04 , x20 − 1, x20 + 1 = 2
şi 4¤|¤x20 +1, rezultă x20 +1 = 2a4 şi x20 −1 = 8pb4 . Prin scădere avem a4 −1 = 4pb4 ,
adică a2 + 1 = 2c4 şi a2 − 1 = 2pd4 . De aici avem c4 − pd4 = 1 cu soluţia
(c, d) ∈ N2 . Cum c < x0 şi d < y0 , se contrazice minimalitatea soluţiei (x0 , y0 ).
Aşadar rezultă x = ±1 şi y = 0.

44. Fie 1 ≤ k ≤ p − 1, k p = p2 · qk + rk , 1 ≤ rk ≤ p2 − 1. Avem că


(p − k)p ≡ Cpp−1 ·p · k p−1 −k p ≡ −k p (mod p2 ) şi de aici rezultă că rk + rp−k ≡ 0
p−1
X
(mod p2 ). Avem rk + rp−k = p2 şi deci (rk + rp−k ) = p2 (p − 1); adică
k=1
p−1
X p2 (p − 1)
rk = .
k=1
2

45. Condiţia din enunţ se rescrie an − a ≡ 0 (mod n2 ) ∀ n ∈ N∗ . Să


presupunem că |a| ≥ 2. Alegem n = |a| şi deducem că a|a| ≡ a (mod a2 ).
Deoarece |a| ≥ 2, rezultă din ultima congruenţă că 0 ≡ a (mod a2 ) şi de aici
contradicţia a2 |a. Deci |a| = 1. Dacă a = −1, luăm n = 2 şi obţinem contradicţia
(−1)2 −(−1) ≡ 0 (mod 4). Deci a = 1 este singura soluţie a problemei.

46. Dacă c = 1, a şi b pot fi orice numere naturale. Dacă c = 2, se arată


imediat că a şi b trebuie să fie pare. În cele ce urmează presupunem că c ≥ 3.
Scriem a = cq1 + n, b = cq2 + m, q1 , q2 , m, n ∈ N, 0 ≤ m, n ≤ c − 1. Avem
că 0 ≡ 2a + 2b + 1 ≡ 2n + 2m + 1 (mod 2c − 1). Deci 2c − 1|2n + 2m + 1 şi
0 < 2n + 2m + 1 ≤ 2c−1 + 2c−1 + 1 = 2c + 1 < 2(2c − 1). De aici deducem
că 2n + 2m + 1 = 2c − 1, 2n + 2m = 2c − 2. Dacă n = 0, atunci m = 0
şi c = 2, ceea ce este imposibil. Deci n ≥ 1 şi la fel se arată că m ≥ 1.
Avem 2m−1 + 2n−1 = 2c−1 − 1. Cum termenul drept este impar, rezultă că
m = 1 sau n = 1. Să presupunem că n = 1. Avem 2m−1 = 2c−1 − 2, m ≥ 2,
2m−2 = 2c−2 −1. Ultimul număr fiind impar, deducem că m = 2, c = 3. Soluţia
generală este ı̂n acest caz a ≡ 1(3), b ≡ 2(3).

47. Căutăm n sub forma n = 2pq, p 6= q, p, q numere prime impare.


22pq+ 2 ≡ 22q + 2(p), 22pq + 2 ≡ 22p + 2(q). Trebuie găsite numerele prime
impare distincte p şi q astfel ı̂ncât 22q−1 ≡ −1(p) şi 22p−1 ≡ −1(q).

206
Găsim numerele p = 3, q = 11. Însă numărul 2 · 3 · 11 = 66 < 100. Observăm
că p = 11, q = 43 sunt convenabile şi am obţinut numărul n = 2 · 11 · 43 = 946.
Avem 22·43−1 = 285 ≡ 25 (mod 11) şi 22·11−1 = 221 = (27 )3 = 1283 ≡
(−1)3 (mod 43) = −1 (mod 43).

48. Dacă p = 2 luăm n = 2α. Pentru p ≥ 3 fie n = (2α + α)(p − 1) + α.


α
Avem 2n+n = (2p−1 )2 +α · 2α+p(2α+α) − 2α ≡ 1 · 2α − 2α (mod p) ≡ 0 (mod p)
şi deci p|2n + n pentru orice α natural.

49. Cazul (a, b) = d > 1. ∃ n0 ∈ N∗ a.ı̂. ∀ n ≥ n0 , an +b > d. Deci d|an +b


şi an + b > d, ceea ce ı̂nseamnă că an + b este compus. Cazul (a, b) = 1. Fie
n0 ∈ N∗ astfel ı̂ncât an0 + b ≥ 2 şi fie p prim astfel ı̂ncât p|an0 + b. Evident că
p¤|¤a (deoarece (a, b) = 1). Fie k ∈ N∗ . Atunci ak(p−1)+n0 +b ≡ an0 +b ≡ 0 (mod p).
Punem n = k(p − 1) + n0 şi p|an + b, an + b > an0 + b ≥ p. Deci an + b este
compus pentru ∀ k ∈ N∗ .

50. Vom arăta că singura soluţie pară este n = 0. Dacă n este
(2a + 1)n + (2b + 1)n
par, atunci (2a + 1)n + (2b + 1)n ≡ 2 (mod 4) şi
2n
poate fi ı̂ntreg doar dacă 2n 6≡ 0 (mod 4); cum n este par, deducem
că n = 0. Pentru n impar, avem că (2a + 1)n + (2b + 1)n = (2a +
€ Š
2b+2) (2a + 1)n−1 − (2a + 1)n−2 (2b + 1) + · · · + (2b + 1)n−1 . Cea de-a doua
(2a + 1)n + (2b + 1)n
paranteză este număr impar şi deci din ∈ Z, deducem
n
2n
că 2 |2a + 2b + 2. Cum a, b ∈ N, avem 2 ≤ 2a + 2b + 2 şi deci există doar un
număr finit de numere n ∈ N cu 2n |2a + 2b + 2.

51. Pentru n = 8, luăm numerele 0, 20, 40, 1, 2, 4, 7, 12, care nu au cali-


tatea din enunţ. Fie acum n ≥ 9 şi n numere ı̂ntregi distincte. Dacă cel puţin
7 dintre ele au clase modulo 20 distincte, atunci aceste 7 numere formează
6·7
C72 = = 21 perechi. Conform principiului cutiei, există a, b, c, d astfel
2
ı̂ncât (a, b) 6= (c, d), a 6= b, c 6= d şi a + b = c + d (mod 20). E clar că a 6= c,
a 6= d, b 6= c, b 6= d. Dacă există cel mult 6 clase distincte, atunci din cele
9 numere găsim a, b, c, d, astfel ı̂ncât a ≡ c (mod 20), b ≡ d (mod 20) sau
a ≡ b ≡ c ≡ d (mod 20). În ambele situaţii problema este rezolvată. Minimul
căutat este 9.

52. Avem a1 = 2, a2 = 3, a3 = 7, a4 = 43. Să presupunem că an+1 = 5.


Evident că n ≥ 2 şi (a1 a2 . . . an + 1, 6) = 1. Rezultă că a1 a2 . . . an + 1 = 5k şi
obţinem contradicţia 3 ≡ a1 a2 . . . an + 1 = 5k ≡ 1 (mod 4). Să presupunem

207
acum că an+1 = 11. Evident că n ≥ 3 şi că (a1 a2 . . . an + 1, 42) = 1. Rezultă că
a1 a2 . . . an + 1 = 5a · 11b . Deoarece a1 a2 . . . an + 1 ≡ 3 (mod 4), deducem că b
este impar. Pe de altă parte, 1 ≡ a1 a2 . . . an + 1 = 5a · 11b ≡ (−1)a+b (mod 3)
şi de aici obţinem că a + b este număr par. Cum b este impar, rezultă că şi
a este impar. Deci a = 2c + 1, b = 2d + 1, b, d ∈ N. Avem a1 a2 . . . an + 1 =
55 · 25c · 121d . Trecem ı̂n această egalitate la o congruenţă modulo 7 şi obţinem
că 1 ≡ −1 · 4c · 2d ≡ −22c+d (mod 7), 22c+d ≡ −1 (mod 7). Aceasta este o
contradicţie, căci 2m ≡ 1, 2, 4 (mod 7), ∀ m ∈ N.
(L. Panaitopol )
 
n
53. Avem = k, (n − p + 1) ≤ pk ≤ n,
p
 
n(n − 1) . . . (pk + 1)(pk − 1) . . . (n − p + 1) − (p − 1)!
Cnp − k = k .
(p − 1)!

Deoarece n(n − 1)(n − 2) . . . (pk + 1)(pk − 1) . . . (n − p + 1) ≡ (p − 1)! (mod p),


enunţul este imediat.

54. Dacă n este prim ştim că Cnk ≡ 0 (mod n) ∀ k = 1, n − 1. Demon-


străm reciproca. Să presupunem că n = pα · m, p prim, α ∈ N∗ , m ∈ N∗ , p¤|¤m,
are proprietatea din enunţ. Este imediat că n¤|¤Cnp (deoarece exponentul lui p ı̂n
(n − p + 1) . . . (n − 1)n
Cnp = este egal cu α − 1 şi de aici deducem că n = p,
p!
ceea ce trebuia demonstrat.

55. Fie n ∈ N∗ cu proprietăţile cerute. n = Cn1 trebuie să fie impar.


n−k n−k
Cnk+1 = Cnk (k + 1 ≤ n). De aici rezultă că exponentul lui 2 ı̂n
k+1 k+1
este 0, ∀ k = 0, n − 1. Dacă n ≥ 5, punând k = 3 ı̂n relaţia precedentă,
rezultă că n ≡ 3 (mod 4). Dacă n ≥ 9, punem k = 7 şi obţinem că n ≡ 7
(mod 8). Fie t ∈ N astfel ı̂ncât 2t ≤ n < 2t+1 . Alegem k = 2t − 1 şi obţinem că
n ≡ −1(2t ). Din această congruenţă şi din inegalităţile 2t < n < 2t+1 deducem
că n = 2t+1 − 1. Pentru a demonstra reciproca, folosim un raţionament prin
recurenţă. Cn0 = 1 este impar, Cn1 = 2t+1 − 1 este impar. Presupunem că Cnk
n−k
este impar şi k + 1 ≤ n. Cnk+1 = Cnk . Dacă k e par, evident că Cnk+1 este
k+1
impar. Dacă k este impar, k+1 = 2α ·m, m impar, 2α ≤ 2α m ≤ 2t+1 −1 < 2t+1 ,
n−k 2t+1 − 1 − 2α m + 1 2α (2t+1−α − m) 2t+1−α − m
α < t + 1 şi = = = .
k+1 2α m 2α m m

208
n−k
Cum Cnk+1 = Cnk , deducem că Cnk+1 este impar, ceea ce trebuia demon-
k+1
strat.

56. Să presupunem că are loc congruenţa din enunţ şi p n-ar fi prim.
p
Atunci p = a · b, cu a, b ∈ N∗ şi 2 ≤ a ≤ . Nu putem avea simultan a ≥ n şi
2
a ≥ p − n + 1, căci ı̂n acest caz ar rezulta că 2a ≥ n + (p − n + 1) = p + 1,
p+1
a≥ , ceea ce este o contradicţie. Deci a ≤ n − 1 sau a ≤ p − n. În orice
2
caz (n − 1)!(p − n)! ≡ 0 (mod a). Congruenţa din enunţ implică faptul că
0 ≡ (−1)n (mod a), ceea ce constituie evident o contradicţie. Să presupunem
acum că p este prim şi să demonstrăm congruenţa din enunţ. Se arată imediat
k k+2
că Cp−1 ≡ Cp−1 (p), ∀ k ∈ N, k + 2 ≤ p − 1 (p fiind număr prim). Cum
0 1
Cp−1 ≡ 1(p) şi Cp−1 = p−1 ≡ −1 (mod p), deducem că Cp−1 m ≡ (−1)m (mod p),
n−1
∀ m ∈ N, m ≤ p − 1. Avem că Cp−1 (n − 1)!(p − n)! = (p − 1)! ≡ (−1) (mod p).
n−1 n−1
Cum Cp−1 ≡ (−1) (mod p), din cele de mai sus rezultă că (n−1)!(p−n)! ≡
n
(−1) (mod p), ceea ce trebuia demonstrat.

(p − j + 1)(p − j + 2) . . . (p − 1)p
57. Pentru 1 ≤ j ≤ p−1 avem că Cpj = ,
j!
 
Cpj j−1
(j − 1)! (−1)j−1 2p
≡ (−1) ≡ (mod p). Dacă p = 6t + 1, k = = 4t
p j! j 3
Cp1 + Cp2 + · · · + Cp4t 1 1 1 1 1 1 1
şi ≡ 1 − + − + ··· + − ≡ 1+ + +
p 2 3 4 4t − 1 4t 2 3
    
1 1 1 1 1 1 1 1 1
··· + −2 + + + + ··· + ≡ 1 + + ··· + − 1+ +
4t 2 4 6 8 4t 2 4t 2

1 1 1 1 1
+···+ ≡ + + · · · + (p). Totul rezultă acum ţinând cont
3 2t 2t + 1 2t + 2 4t
1 1 6t + 1 p
că + = = ≡ 0(p),
2t + j 4t − j + 1 (2t + j)(4t − j + 1) (2t + j)(4t − j + 1)
 
2p Cp1 + Cp2 + · · · + Cp4t+3
∀ 1 ≤ j ≤ t. Dacă p = 6t + 5, k = = 4t + 3 şi ≡
3 p
  
1 1 1 1 1 1 1
1 − + − + ··· + ≡ 1 + + ··· + − 1 + + ··· +
2 3 4 4t + 3 2 4t + 3 2

1 1 1 1
≡ + + ··· + (mod p). Enunţul rezultă ţinând
2t + 1 2t + 2 2t + 3 4t + 3

209
1 1 p
seama de + = ≡ 0 (mod p).
2t + 2 + j 4t + 3 − j (2t + 2 + j)(4t + 3 − j)
(Bogdan Enescu)
.
58. Deoarece C93 − 3 = 84 − 3 = 81 .. 33 , putem presupune că p ≥ 5.
 
p (2p + 1) . . . (3p) (2p+1)(2p+2) . . . (2p + (p − 1)−(p−1)!
C3p − 3 = −3=3 .
p! (p−1)!
Este suficient să arătăm că p3 |E
„ = (2p + 1)(2p Ž + 2) . . . (2p +„
(p − 1))Ž− (p − 1)!.
X p−1
X1
3 2
1
Dar E = Mp + (2p) (p − 1)! + 2p(p − 1)! .
1≤i<j≤p−1
ij i=1
i
Deoarece xp−1 − 1 = (x − 1)(x − 2) . . . (x − (p − 1)) ı̂n Zp [x], rezultă
X 1
din relaţiile lui Viète că (p − 1)! este egal cu (−1)p−3 ı̂nmulţit
1≤i<j≤p−1
ij
cu coeficientul lui x2 din polinomul xp−1 − 1. Deoarece p ≥ 5, p − 1 ≥ 4,
rezultă că„x2 nu apare,Ždeci coeficientul căutat este zero. Am arătat că
X 1
p|(p − 1)! şi ţinând cont de formula lui E, mai trebuie să
1≤i<j≤p−1
ij
„ Ž
p−1
X p−1
X
1 1 a
arătăm că p2 |(p − 1)! sau că = , (a, b) = 1, p2 |a. Însă
i=1
i i=1
i b
p−1 p−1
p−1
X 2  
1 X 1 1 X2
1 c
= + =p =p ,
i=1
i i=1
i p−i i=1
i(p − i) d
(c, d) = 1, p|c (vezi exerciţiul următor).

p−1 p−1
59. Pentru fiecare i = 1, , există un unic j astfel ı̂ncât 1 ≤ j ≤
2 2
1 1 + i(p − i)j 2 c
şi i2 · j 2 ≡ 1 (mod p). + j2 = = , (c, d) = 1, p|c,
i(p − i) i(p − i) d
deoarece 1 + i(p − i)j 2 ≡ 1 − i2 j 2 ≡ 1 − 1 ≡ 0 (mod p). Din cele de mai
p−1
X
2
sus rezultă că este suficient să arătăm că p j 2 . Acest lucru este imediat
j=1
m
X m(m + 1)(2m + 1)
deoarece are loc formula j2 = . Deci
j=1
6
 
p−1 p+1 p−1 p−1 p+1
p−1
X
· 2 +1 p ·
2 ... p.
2
2 2 2 2
j2 = =
j=1
6 6

210
60. Avem
‡ ‘

1 1 1 1 1 1
+ + ··· + =p + + ··· +
1 2 p−1 1(p − 1) 2(p − 2) p−1 p+1
·
2 2
1 1 1 a
şi deci + + ··· + = , (a, b) = 1, p|a. În problema 58, am arătat
1 2 p−1 b
X 1 a1 1 1 1
că = , (a1 , b1 ) = 1, p|a1 . Deoarece 2 + 2 + · · · + =
1≤i<j≤p−1
ij b1 1 2 (p − 1)2
p−1
!2
X 1 X 1
−2 , din cele de mai sus rezultă enunţul.
j=1
j 1≤i<j≤p−1
ij

2m−1
X
61. Avem −2S2m+1 = (−1)k Sk S2m−k C2m
k
şi totul rezultă din această
k=1
egalitate. Egalitatea de mai sus se demonstrează folosind funcţia f (x) =
X∞ S
k k
x = ex + 22x + · · · + enx . Calculăm coeficientul lui x2m din seria de
k=0
k!
2m (−1)k S
X S2m−kk
puteri f (x)·f (−x). Acest coeficient este pe de o parte ,·
k=0
k! (2m − k)!
iar pe de altă parte, deoarece f (x)f (−x) = n + (n−1)(ex +e−x )+(n−2)(e2x +
€ Š X (n − k)k 2m
n−1 2nS2m 2S2m+1
e−2x )+· · ·+ e(n−1)x + e−(n−1)x , el este 2 = − .
k=1
(2m)! (2m)! (2m)!
X (−1)k Sk
2S2m+1 2m−1 S2m−k
De aici rezultă imediat că − = · şi, ı̂nmulţind
(2m)! k=1
k! (2m − k)!
2m−1
X
k
cu (2m)!, −2S2m+1 = C2m (−1)k Sk S2m−k , adică exact identitatea de care
k=1
aveam nevoie.

62. a) Putem presupune că 1 ≤ x ≤ y ≤ z. Este imediat că (x, y) =


(x, z) = (y, z) = 1. Dacă x = 1, atunci y ≡ 1 (mod z), z|y − 1. Cum 0 ≤
y−1 < z, deducem că y = 1 (z poate fi orice număr natural nenul). Dacă x ≥ 2,
xy − 1 yz − 1 zx − 1
atunci x < y < z. Cum , , sunt numere ı̂ntregi, deducem
z x y
(xy − 1)(yz − 1)(zx − 1) (xy − 1)(yz − 1)(zx − 1)
că ∈ Z. Deoarece = xyz −
xyz xyz
1 1 1 1 1 1 1 1
x−y−z+ + + − , rezultă că + + − ∈ Z.
x y z xyz x y z xyz

211
1 1 1 1 1 1 1 1 1 1 13
Cum 0 < + + − < + + ≤ + + = , deducem că
x y z xyz x y z 2 3 4 12
1 1 1 1 1 1 1
+ + − = 1. Dacă x ≥ 3, rezultă că y ≥ 4, z ≥ 5 şi + + ≤
x y z xyz x y z
1 1 1 1 1 1 1
+ + < 1, deci nu există soluţii. Deci x = 2, + − = . Dacă
3 4 5 y z 2yz 2
1 1 1 1 1
y ≥ 4 obţinem contradicţia + ≤ + < . Deci y = 3 şi z = 5. În
y z 4 5 2
mod analog se arată că (1, 1, 1), (1, 1, 2), (1, 2, 3), (2, 3, 7) sunt singurele soluţii
pentru punctul b).

63. a) y 5 ≡ 0, 1, 10 (mod 11) şi y 5 − 4 ≡ 7, 8, 6 (mod 11). Pe de altă parte,


x2 ≡ 0, 1, 4, 9, 5, 3 (mod 11) şi deci ecuaţia x2 = y 5 − 4 nu are soluţii ı̂n Z.
Observaţie. Se putea rezolva ecuaţia (x + 2i)(x − 2i) = y 5 , folosind arit-
metica inelului Z[i] şi am fi ajuns la aceeaşi concluzie. Calea ar fi fost ı̂nsă ceva
mai lungă.
b) 2891 = 72 · 59. Dacă 7|y, atunci neapărat 7|x şi ajungem la contradicţia
73 |x3 − 3xy 2 + y 3 = 72 · 59. Deci 7¤|¤y. Fie z ∈ Z astfel ı̂ncât yz ≡ 1 (mod 7).
Înmulţim ecuaţia cu z 3 şi notând t = xz, deducem că t3 − 3t + 1 ≡ 0 (mod 7).
Evident că t 6≡ 0(7). Atunci t3 ≡ ±1 (mod 7). Dacă t3 ≡ 1(7), atunci 3t ≡ 2(7)
şi t ≡ 3(7); obţinem o contradicţie căci 33 ≡ −1 (mod 7). Dacă t3 ≡ −1(7),
rezultă contradicţia 3t ≡ 0(7), t ≡ 0(7). Deci congruenţa t3 − 3t + 1 ≡ 0
(mod 7) nu are soluţii, ceea ce arată că ecuaţia iniţială nu are soluţii.
Observaţie. Exerciţiul de la OIM (1982) mai avea o parte: dacă pentru
n ∈ N∗ ecuaţia x3 − 3xy 2 + y 3 = n are o soluţie, atunci ecuaţia are cel puţin 3
soluţii. Ecuaţia se mai scrie şi sub forma (y − x)3 − 3(y − x)(−x)2 + (−x)3 = n.
De aici deducem imediat că dacă (x1 , y1 ) este soluţie a ecuaţiei din enunţ,
atunci o soluţie este şi (x2 , y2 ), unde y2 − x2 = x1 , −x2 = y1 . Deci x2 = −y1 ,
y2 = x1 − y1 . De aici rezultă şi că x3 = −y2 = y1 − x1 , y3 = x2 − y2 = −x1 este
soluţie a ecuaţiei din enunţ. Se arată imediat că dacă 1 ≤ i < j ≤ 3 sunt astfel
ı̂ncât (xi , yi ) = (xj , yj ), atunci x1 = y1 = 0 şi n = 0; contradicţie. Enunţul
este deci demonstrat.

64. Fie n cu proprietatea din enunţ. Arătăm că n = 2t , t ∈ N. Să


presupunem că n-ar fi aşa; atunci ar exista un k ≥ 3, k impar, astfel ı̂ncât
k|n. Atunci 2k − 1|2n − 1|m2 + 9. Cum k ≥ 2, 2k − 1 ≡ 3 (mod 4), există
un divizor prim p al lui 2k − 1 de forma p ≡ 3 (mod 4). Cum 2k − 1 ≡
(−1)k − 1 = −2 ≡ 1(3), avem că p 6= 3. Deci p|2k − 1|2n − 1|m2 + 9 şi

212
p−1
m2 ≡ −9 (mod p). Ridicăm această congruenţă la puterea şi obţinem
p−1 p−1
2
că 1 ≡ mp−1 ≡ (−1) 2 3p−1 ≡ (−1) 2 ≡ −1 (mod p), ceea ce constituie o
contradicţie (am folosit mai sus Mica Teoremă a lui Fermat: mp−1 ≡ 3p−1 ≡ 1
(mod p); p¤|¤3m deoarece p 6= 3). Am arătat că n = 2t , t ∈ N.
Fie acum n = 2t , t ∈ N şi vom arăta că ∃ m ∈ N astfel ı̂ncât 2n − 1|m2 + 9.
Pentru t = 0 sau t = 1, luăm m = 0. Presupunem ı̂n continuare că t ≥ 2.
t
€ t−1 Š
Avem că 2n − 1 = 22 − 1 = (22 − 1)(22 + 1)(24 + 1)(28 + 1) . . . 22 + 1 . Ştim
€ a b
Š
(curs, pagina 149) că 22 + 1, 22 + 1 = 1 ∀ a < b şi de aici rezultă, folosind
j−1
€ j
Š
Lema Chineză a Resturilor, că există c ∈ Z astfel ı̂ncât c ≡ 22 22 + 1
j
€ j
Š
∀ j = 1, t − 1. De aici deducem imediat că c2 + 1 ≡ 22 + 1 ≡ 0 22 + 1
€ t−1
Š
∀ j = 1, t − 1 şi deci (22 + 1)(24 + 1)(28 + 1) . . . 22 + 1 c2 + 1.
t
Ţinând cont de o identitate anterioară, rezultă că 2n − 1 = 22 − 1 =
€ t−1 Š
3(22 + 1)(24 + 1) . . . 22 + 1 |3c2 + 3|9c2 + 9. Alegem m = 3c şi problema
este rezolvată.

65. ”Traducând” problema ı̂ntr-un alt limbaj, avem de găsit numerele


m(m + 1)
n ∈ N pentru care funcţia f : Z → Zn , f (m) = este surjectivă.
2
Cum f (m + 2nk) = f (m), ∀ m, k ∈ Z şi f (m) = f (−1 − m) ∀ m ∈ Z,
j(j + 1)
rezultă că funcţia g : {0, 1, 2, . . . , n − 1} → Zn , g(j) = , ∀ j = 0, n − 1
2
este şi ea surjectivă. Cum |{0, 1, 2, . . . , n − 1}| = |Zn |, deducem că g este şi
n−1
injectivă. Dacă n este impar, atunci g(n − 1) = · n = 0 = g(0); cum
2
g este injectivă, rezultă că n − 1 = 0, n = 1. Deci singurul număr impar
care satisface condiţia din enunţ este n = 1. Fie acum n = 2k · t, k ∈ N∗ ,
t ∈ N, t ≡ 1(2), cu proprietatea din enunţ. Fie 0 ≤ α ≤ 2k − 1 astfel ı̂ncât
αt + 1 ≡ 0 (mod 2k ). Evident că α este impar şi deci 1 ≤ α ≤ 2k − 1,
αt(αt + 1)
1 ≤ αt < 2k t = n, g(αt) = = αt · β · 2k−1 (αt + 1 = β · 2k , β ∈ Z).
2
n
Avem că g(αt) = β · α · t2k−1 = β · t2k−1 = 0 sau , după cum β este par
2
sau impar. Nu se poate să avem că g(αt) = 0 = g(0) căci g este injectivă şi
n
1 ≤ αt ≤ n − 1. Deci g(αt) = = g(n − 1). Cum g este injectivă, avem că
2
αt = n − 1 = 2k · t − 1. Din ultima egalitate rezultă că t = 1. Să arătăm acum
că orice număr n = 2k , k ∈ N, satisface condiţiile din enunţ. Este suficient să
arătăm că funcţia g de mai sus este injectivă.

213
x(x + 1) y(y + 1)
Să presupunem că 0 ≤ x, y ≤ 2k − 1 sunt astfel ı̂ncât ≡
2 2
(mod 2k ). De aici rezultă că x2 +x ≡ y 2 +y (mod 2k+1 ) şi că (x−y)(x+y+1) ≡
0(2k+1 ). Cum x − y 6≡ x + y + 1 (mod 2), deducem că x − y ≡ 0(2k+1 ) sau
x + y + 1 ≡ 0(2k+1 ). Însă 1 ≤ x + y + 1 ≤ 2(2k − 1) + 1 = 2k+1 − 1 < 2k+1 şi
deci x + y + 1 6≡ 0(2k+1 ). Nu ne rămâne decât că x ≡ y(2k+1 ) şi rezultă că g
este injectivă, ceea ce trebuia demonstrat.

66. Pentru p = 2, gradul maxim este 1 (luăm f (x) = x). Pentru p ≥ 3


vom arăta că gradul maxim este p − 2. Fie f (x) = xp−2 . Presupunem că
np−2 ≡ mp−2 (p). Dacă n ≡ 0(p), atunci evident că m ≡ 0(p) şi n ≡ m ≡
0(p). La fel se raţionează ı̂n cazul m ≡ 0(p). Rămâne de analizat situaţia
m · n 6≡ 0(p). În acest caz ı̂nmulţim congruenţa np−2 ≡ mp−2 (p) cu nm şi
obţinem, ţinând cont că np−1 ≡ mp−1 ≡ 1(p), că n ≡ m(p). Mai trebuie
arătat că nici un polinom de grad p − 1 nu are proprietatea din enunţ. Să
presupunem că există f de grad p − 1, cu proprietăţile din enunţ. Notăm
f (x) = ap−1 xp−1 + ap−2 xp−2 + · · · + a1 x + a0 , ap−1 ∈ {1, 2, . . . , p − 1}. Arătăm
p−1
X
ı̂ntâi că ∀ k ∈ N, 1 ≤ k ≤ p − 2, avem că Sk = nk ≡ 0 (mod p). Fie α ∈ N,
n=1
2 ≤ α ≤ p−1, un generator pentru (Z∗p , ·) (despre care ştim că este grup ciclic).
p−1
X p−1
X
k
Avem că Sk = n ≡ (αn)k ≡ αk Sk (mod p) şi că Sk (1 − αk ) ≡ 0(p).
n=1 n=1
Cum 1 6≡ αk (p), deoarece 1 ≤ k ≤ p − 2 şi ordinul lui α ı̂n (Z∗p , ·) este p − 1,
deducem că Sk ≡ 0(p), ∀ k = 1, p − 2. Ţinând cont de injectivitatea lui f ca
p−1
X p−1
X (p − 1)p
funcţie de la Zp ı̂n Zp , deducem că f (j) ≡ t= ≡ 0(p). Pe de
j=0 t=0
2
p−1
X p−2
X
altă parte, f (j) ≡ ap−1 Sp−1 + ak Sk + pa0 ≡ ap−1 Sp−1 ≡ ap−1 (p − 1) ≡
j=0 k=1
−ap−1 (p). Comparând cele două expresii, obţinem că ap−1 ≡ 0(p), ceea ce este
o contradicţie, căci ap−1 ∈ {1, 2, . . . , p − 1}. Enunţul este demonstrat.

67. Din teorema lui Wilson ştim că (p − 1)! + 1 ≡ 0 (mod p). Atunci
(p − 1)! ≡ pr − 1 (mod p2 ). Din Mica Teoremă a lui Fermat ştim că j p−1 ≡ 1
(mod p) ∀ j = 1, p − 1. Deci j p−1 ≡ 1 + prj (mod p2 ). Calculăm ((p − 1)!)p−1
modulo p2 ı̂n două moduri. Întâi ((p − 1)!)p−1 ≡ (pr − 1)p−1 ≡ −Cp−1
1 pr + 1 =

1 − pr(p − 1) ≡ 1 + pr (mod p ).2

214
p−1
Y p−1
Y p−1
X
p−1
Pe de altă parte, ((p−1)!)p−1 = j ≡ (1+prj ) ≡ 1+p rj (mod p2 ).
j=1 j=1 j=1
p−1
X p−1
X
Din cele două congruenţe rezultă că pr ≡ p rj (mod p2 ). j p−1 ≡ p −
j=1 j=1
p−1
X
1+p rj ≡ p − 1 + pr ≡ p + (p − 1)! (mod p2 ). Aceasta era ceea ce trebuia
j=1
demonstrat.

68. a) ⇒ b). Deoarece n este prim, din teorema lui Wilson rezultă că
(n − 1)! + 1 ≡ 0 (mod n) şi deci 4((n − 1)! + 1) + n ≡ 0 (mod n). Deoarece
n + 2 este prim, din aceeaşi teoremă rezultă că (n + 1)! ≡ −1 (mod n + 2) şi
n+1
(n−1)!(−2)(−1) ≡ n+1 (mod n+2), (n−1)! ≡ . Deci 4((n−1)!+1)+n ≡
2
 
n+1
4 + 1 + n = 3n + 6 ≡ 0 (mod n + 2). Demonstrăm acum cealaltă
2
implicaţie. Fie n ∈ N∗ cu proprietatea că 4((n−1)!+1)+n ≡ 0 (mod n(n+2)).
Să presupunem că n ar fi par. Este imediat că n ≥ 6. Deoarece (n, n + 2) = 2,
există un număr prim impar q astfel ı̂ncât q|n(n + 2). Arătăm că q ≤ n − 1.
n 1 1
Dacă q|n, atunci n = qα, α ≥ 2. Avem că = 1+ ≤ 1+ <
n−1 n−1 5
n
2 ≤ α = şi deci q ≤ n − 1. Dacă q|n + 2, atunci n + 2 = qα, α ≥ 2 şi
q
n+2 3 3 n+2
= 1+ ≤ 1+ < 2 ≤ α = , q ≤ n − 1. În ambele cazuri
n−1 n−1 5 q
q ≤ n − 1 şi deci (n − 1)! ≡ 0 (mod q). Dar ştim că 4((n − 1)! + 1) + n ≡ 0
(mod n(n + 2)). Combinând aceste două congruenţe (folosind şi că q|n(n + 2),
rezultă că n + 4 ≡ 0(q). Dacă q|n, obţinem contradicţia 4 ≡ 0 (mod q), iar
dacă q|n + 2 obţinem contradicţia 2 ≡ 0 (mod q). Nu ne rămâne decât că n
este impar, n ≥ 3. Din n impar şi 4((n − 1)! + 1) + n ≡ 0 (mod n(n + 2)),
rezultă că (n − 1)! + 1 ≡ 0 (mod n) şi reciproca teoremei lui Wilson ne asigură
că n este prim. Avem că E = n [4((n − 1)! + 1) + 1) + n] ≡ 0 (mod n + 2). Dar
E = 4n! + 4n + n2 = 4(n! − 1) + (2 + n)2 ≡ 4(n! − 1) (mod n + 2). Din cele de
mai sus rezultă că n! ≡ 1 (mod n + 2), (n + 1)! ≡ −1 (mod n + 2) şi reciproca
teoremei lui Wilson ne asigură că n + 2 este şi el număr prim.

69. a) Fie p prim, p|m, m = pk · n, p¤|¤n. Trebuie să arătăm că pk |(m − 1)!.
m m
Dacă m 6= pk , atunci m = pk · n, n ≥ 2 şi pk = ≤ ≤ m − 1, pk |(m − 1)!.
n 2

215
m m
Dacă m = pk , k > 2, atunci p 6= pk−1 , pk−1 = ≤ ≤ m − 1 şi deducem
p 2
că pk = p · pk−1 |(m − 1)!. Dacă m = p2 , p ≥ 3, atunci 2p ≤ p2 − 1 = m − 1 şi
p · 2p = 2p2 |(m − 1)!. În toate cazurile am arătat că pk |(m − 1)!.
b) Se verifică imediat că p = 2, 3, 5 sunt soluţii ale problemei. Vom arăta că
nu există o altă soluţie. Să presupunem că ∃ p prim, p ≥ 7 şi k ∈ N∗ astfel ı̂ncât
(p − 1)! + 1 = pk . Avem că (p − 1)! = pk − 1, (p − 2)! = pk−1 + pk−2 + · · · + p + 1.
Deoarece p − 1 ≥ 6 şi p − 1 este număr compus, afirmaţia de la punctul a) ne
asigură că (p − 1)|(p − 2)!. Trecând ı̂n egalitatea de mai sus la o congruenţă
modulo p − 1, deducem că 0 ≡ k (mod p − 1). Cum k ∈ N∗ şi p − 1|k, rezultă
că k ≥ p − 1. Evident că (p − 1)! < pp−2 (p − 1) şi obţinem contradicţia
pk ≥ pp−1 > pp−1 − pp−2 + 1 = pp−2 (p − 1) + 1 > (p − 1)! + 1.

70. Secvenţa 33, 34, 35, 36 are calităţile cerute ı̂n enunţ şi putem
presupune că n ≥ 5. Considerăm secvenţa (n + 1)! + 2, (n + 1)! + 3,
(n + 1)! + 4, . . . , (n + 1)! + (n + 1) de n numere consecutive. Arătăm că nici
unul nu e de forma pα , p prim. Presupunem că (n + 1)! + j = pα , unde
!

(n + 1)!
2 ≤ j ≤ n + 1 şi α ∈ N . Cum j + 1 = pα , deducem că j = pβ ,
j
(n + 1)! (n + 1)!
β ∈ N∗ , β ≤ α. Dacă β ≥ 2, atunci p şi deci + 1 = pα−β ≡ 1
j j
(mod p). De aici deducem că β = α, ceea ce constituie o contradicţie, căci
(n + 1)!
+ 1 > 1. Deci β = 1 şi n + 1 < 2p (dacă am avea inegalitatea
j
(n + 1)!
n + 1 ≥ 2p, ar rezulta că + 1 ≡ 1(p) şi se obţine o contradicţie ca mai
p
(n + 1)!
sus). Ecuaţia noastră se rescrie sub forma + 1 ≡ pα−1 , unde p este
p
prim, p ≤ n + 1 < 2p. Dacă n + 1 = p, atunci (p − 1)! + 1 = pα−1 şi p ≥ 6
(deoarece n ≥ 5). Însă teorema lui Liouville ne arată că p = 2, 3 sau 5 şi am
(n + 1)!
obţinut o contradicţie. Deci p + 1 ≤ n + 1 < 2p. Avem că ≡ 0((p + 1)2 )
p
p+1 (n + 1)!
deoarece 2 < < p + 1 apar ı̂n . Deci pα−1 ≡ 1((p + 1)2 ).
2 p
De aici rezultă imediat că α − 1 este par şi că p + 1|α − 1 (deoarece
pα−1 = (p+1−1)α−1 ≡ 1−(α−1)(p+1) (mod (p+1)2 ). Pe de altă parte avem
pα−1 − 1
că 1 · 2 · 3 . . . (p − 2)(p + 1) · . . . · (n + 1) = = pα−2 + pα−3 + · · · + p + 1.
p−1

216
Cum p − 1 este număr compus, p − 1 ≥ 6, ştim dintr-un exerciţiu precedent că
(p − 1)|(p − 2)!. Din egalitatea precedentă rezultă că 0 ≡ (α − 1) (mod p − 1)
 
p−1 p+1
şi p − 1|α − 1. Cum , = 1, p − 1|α − 1, p + 1|α − 1, rezultă că
2 2
p2 − 1 (n + 1)!
α − 1. Avem că + 1 ≤ 1 · 2 · . . . · (p − 1)(p + 1) . . . (2p − 1) + 1 <
2 p
p2 −1
(1(2p − 1))(2(2p − 2)) . . . ((p − 1)(p + 1)) + 1 < p2(p−1) + 1 < p2p < p 2 ≤ pα ,
(n + 1)!
+ 1 < pα şi am obţinut o contradicţie.
p
71. Avem 37 = 2187 = 2p + 1, 314 ≡ 4p + 1 (mod p2 ). De asemenea
avem 214 = 16384 = 15p − 11 şi deci 228 = −330p + 121 (mod p2 ). Rezultă
de aici că 32 · 228 = −2970p + 1089 (mod p2 ). Cum −2970 = −p − 1877 şi
1089 = p − 4, rezultă 32 228 ≡ −1876p − 4 (mod p2 ) şi deci 32 · 226 ≡ −469p − 1.
Ridicând la puterea 7, obţinem 314 · 3182 ≡ −(7 · 469p + 1) (mod p2 ). Deoarece
7 · 469 = 3283 = 3p + 4, rezultă 314 · 2182 ≡ −4p − 1 (mod p2 ) ≡ −314
(mod p2 ) şi apoi 2182 ≡ −1 (mod p2 ). Deoarece 182·6 = p−1, rezultă 2p−1 ≡ 1
(mod p2 ). Numerele prime cu această proprietate se numesc numere Wieferich.
Ele sunt destul de rare. Pentru p < 3 · 107 există numai numerele p = 1093 şi
p = 3511. În 1909 s-a arătat că dacă p nu este număr Wieferich, ecuaţia lui
Fermat xp + y p = z p nu are soluţii pentru (xyz, p) = 1.

217
CAPITOLUL 10

Resturi pătratice. Simbolul lui Legendre


     
30 5 2 3
1. Avem = . Deoarece 211 ≡ 3 (mod 8),
211 211211 211
   
2 3
avem că = −1. Pentru calculul lui aplicăm legea de re-
211 211
     
3 211 3−1 211−1
· 105
211
ciprocitate pătratică: = (−1) 2 2 = (−1) =
211 3 3
       
211 1 5 211 211−1 5−1
− =− = −1. Avem şi = (−1) 2 · 2 = (−1)105·2 .
3 3 211 5
       
5 211 1 30
= = = 1. Deci = (−1)(−1)(+1) = +1.
211 5 5 211
2. Din Mica Teoremă a lui Fermat ştim că 3p ≡ 3 (mod p) şi deci 3p +1 ≡
 p   
3 +1 4
4 (mod p). Avem că = = 1.
p p
! ! !
a2 c2 − b2 d2 a2 c2 − (p − a2 )(q − c2 ) a2 c2 + a2 (q − c2 )
3. = = =
p p p
! ! !
a2 q q p
= = . Am folosit mai sus că p¤|¤a şi că p ≡ q ≡ 1 (mod 4).
p p q
p−1 p−1
X
2 X
2
4. rj ≡ k 2 (mod p). Dar
j=1 k=1
„ Ž„ Ž
p−1 p−1 p−1
p−1 +1 2 +1
X
2 2 2 2 (p2 − 1)
k2 = =p ≡ (mod p)
k=21
6 24
(căci p 6= 2, 3). Enunţul este demonstrat.
5. Avem a ≡ −b (mod p) şi
          
a −b −1 b p−1 b b
= = = (−1) 2 = deoarece p ≡ 1 (mod 4).
p p p p p p

219
6. Dacă p > 2 este număr prim şi a este un număr ı̂ntreg care nu este
   
a 4a
multiplu de p, atunci = .
p p „ Ž1
0
‡ ‘ p−1
p−1 B4 − n(n + 1) C
− n(n + 1) B 4 C
B C
a) 4 =B B C=
p p C
B C
@ A

„ Ž „ Ž„ Ž
p − (2n + 1)2 −1 (2n + 1)2 p−1
= = = (−1) 2 = 1.
p p p
0 „ Ž1
ˆ  ’ p+1
p+1 B4 + n(n + 1) C
4 + n(n + 1) B 4 C
4 B C
b) =B
B
C=
C
p B 4 C
@ A

„ Ž „ Ž
p + (2n + 1)2 (2n + 1)2
= = = 1.
p p

7. Presupunem că ∃ n, m ∈ N astfel ı̂ncât 167|2n + 3m . 167 este număr


       
2 3 167 2
prim şi = 1, =− =− = 1. Din 3m ≡ −2n (mod 167),
167 167 3 3
   m       
3 m 3 − 2n −1 2 n −1
rezultă că 1 = = = = = = −1,
167 167 167 167 167 167
ceea ce ı̂nseamnă că am obţinut o contradicţie.
Observaţie. Se arată uşor, folosind aceeaşi schemă de raţionament, că
p¤|¤2 +3m , ∀ n, m ∈ N, dacă p este un număr prim astfel ı̂ncât p ≡ 23 (mod 24).
n

8. Folosind faptul că ak ak−1 . . . a1 a0 ≡ a0 − a1 + a2 − a3 + · · · + (−1)k ak


   
7 11
(mod 11), rezultă că 111 . . . 161 ≡ 6, 7 (mod 11). Dar = − =
11 7
            
4 6 2 3 3 11 2
− = −1 şi = =− = = = −1 şi deci
7 11 11 11 11 3 3
111 . . . 161 6≡ k 2 (mod 11). Aceasta arată că numărul 11 . . . 161 nu poate fi
pătrat.
(A. Gica)

15k 2 + 16l2 16k 2 − 15l2


9. Avem 15a+16b = k 2 , 16a−15b = l2 , a = ,b= ,
481 481
∗ 2 2 2 2
k, l ∈ N . Cum 481 = 13 · 37, deducem că 15k + 16l ≡ 0(13), 2k ≡ −3l (13),

220
 
5
k2 ≡ 5l2 (13).
Cum = −1, rezultă că 13|l şi 13|k. Avem şi 15k 2 + 16l2 ≡
13
0(37), 32l2 ≡ −30k 2 (37), −5l2 ≡ −30k 2 (37), l2 ≡ 6k 2 (37), ceea ce coroborat
 
6
cu faptul că = −1, ne conduce la concluzia că 37|k, 37|l. Cel mai mic
37

l ∈ N cu proprietatea că 13|l, 37|l, este 13 · 37 = 481. Luăm k = l = 481 şi
avem că a = 31 · 481, b = 481.

10. Fie 1 ≤ y ≤ p − 1 astfel ı̂ncât xy ≡ 1 (mod p). Atunci


! ! ! ! !
x(x + k) y2 x(x + k) xy(xy + yk) 1 + yk
= = =
p p p p p
p−1
! p−1
!
X x(x + k) X 1 + yk
şi = . Deoarece p¤|¤k, avem că 1 + yk 6≡ 1 + zk
x=1
p y=1
p
(mod p) ∀ 1 ≤ y < z ≤ p − 1. Cum 1 + yk 6≡ 1 (mod p) ∀ y = 1, p − 1,
p−1   p−1  !  
X 1 + yk X a 1
din observaţiile precedente deducem că = − =
y=1
p a=1
p p
 
1
0− = −1. Aceasta demonstrează enunţul.
p
11. p 6= 2 căci 4 · 2+1 = 9 nu este prim. Deci p este impar şi q ≡ 5 (mod 8).
 
2 q−1
Deducem din criteriul lui Euler că −1 = ≡ 2 2 = 22p = 4p (mod q),
q
adică enunţul.
       
p 3
3−1 p−1 p −1
12. Calculăm = (−1) 2 · 2 = (−1)6k−1 = − =
3 p 3 3
           
2 k 3 4 k 12k 1
− = 1. Avem = = = = 1.
3 p p p p p p
(A. Gica)

13. Presupunem că ∃ k ∈ Z astfel ı̂ncât (p − 2)! + 1 = k 2 . Din teorema


lui Wilson ştim că (p − 1)! ≡ −1 (mod p) şi deci (p − 2)! ≡ 1 (mod p). Din
egalitatea (p − 2)! + 1 = k 2 deducem că 2 ≡ k 2 (mod p). Aceasta este o
 
2
contradicţie, căci = −1, deoarece p ≡ 3, 5 (mod 8).
p
Observaţie. Rezolvarea ı̂n N a ecuaţiei x! + 1 = y 2 este o problemă de-
schisă. Conjectura este că (x, y) = (4, 5), (5, 11), (7, 71) sunt singurele soluţii
ale ecuaţiei.
(Brocard, 1876)

221
14. Din teorema lui Wilson deducem succesiv că (p − 1)! ≡ −1 (mod p),
p−1
(p − 2)! ≡ 1 (mod p), (p − 3)!(−2) ≡ 1 (mod p), (p − 3)! ≡ (mod p).
„ Ž „ 2 Ž
k 2 (p − 3)! + 1
Dacă (p − 3)! + 1 = k 2 , k ∈ Z, atunci 1 = = =
p p
‡ ‘ ‡ ‘ ‡ ‘
p−1 p+1 p+1 „ Ž „ Ž
+1 4 2p + 2 2
2 = 2 = 2 = = = −1
p p p p p

(ultima egalitate are loc deoarece p ≡ 3, 5 (mod 8)).

15. Dacă p = 2, rezultă imediat că a şi b sunt pare. Presupunem ı̂n
continuare că p 6= 2. Fie d = (a, b). Avem a = dα, b = dβ, (α, β) = 1. Dacă p|d,
rezultă p|a şi p|b. Dacă p¤|¤d, rezultă p|α2 +αβ+β 2 şi deci p|(2α+β)2 +3β 2 . Avem
(p, 2α + β) = 1 şi (p, β) = 1. Avem (2α + β)2 ≡ −3β 2 (mod p). Cum (p, β) = 1,
rezultă că există γ astfel ca βγ ≡ 1 (mod p) şi deci (γ(2α + β))2 ≡ −3 (mod p),
   
−3 3 p−1
adică = 1 şi deci = (−1) 2 . Din legea de reciprocitate a lui Gauss,
p p
      
3 p p−1 p 3k + 2
rezultă că = (−1) 2 . Aşadar = 1 şi apoi = 1, adică
p 3 3 3
 
2
= 1; contradicţie.
3
16. Dacă p ≡ 2 (mod 3) atunci, conform problemei 15, deducem că p|n,
p|1, ceea ce constituie o contradicţie.

17. Să presupunem că p 6= 5. Atunci 4n2 +4n−4 ≡ 0 (mod p), (2n+1)2 ≡
     
5 p 5
5 (mod p), = 1 şi = = 1. Deci p ≡ ±1 (mod 5).
p 5 p
18. Să presupunem că p 6= 3. Deoarece x2 ≡ 3y 2 (mod p), (x, y) = 1,
p 6= 3, deducem că p¤|¤xy. Fie z ∈ Z astfel ı̂ncât yz ≡ 1 (mod p). Deducem că
 
3
(xz)2 ≡ 3 (mod p) şi că = 1.
p
 
−2
19. Dacă p 6= 2, se arată la fel ca ı̂n problema precedentă că = 1.
p
De aici se deduce imediat că p ≡ 1, 3 (mod 8).

20. Dacă p = 2 enunţul este clar. Fie p > 2 şi k maxim a.ı̂. pk |a şi pk |b.
Avem a = pk A, b = pk B şi rezultă pn−2k = A2 + 2B 2 . Evident că n − 2k ≥ 0.

222
Dacă n = 2k ajungem la contradicţia 1 = A2 + 2B 2 ≥ 2B 2 ≥ 2 (B 6= 0 căci
b 6= 0). Deci n − 2k ≥ 1. Evident că (AB, p) = 1 şi din A2 ≡ −2B 2 (mod p)
şi existenţa unui B 0 ∈ Z cu BB 0 ≡ 1 (mod p), rezultă (AB 0 )2 ≡ −2 (mod p),
 
−2 p−1 p2 −1 (p−1)(p+5)
adică = 1. Deci (−1) 2 · (−1) 8 = 1 ⇔ (−1) 8 =1⇔p=
p
8k + 1 sau p = 8k + 3. În aceste cazuri se arată că p = x2 + 2y 2 (curs, pagina
127, Teorema 1).
(M. Ţena)

21. Avem xp−3 ≡ −3 (mod p) şi deci x 6≡ 0 (mod p) căci p 6= 3.


Înmulţim congruenţa cu x2 şi obţinem că −3x2 ≡ xp−1 ≡ 1 (mod p), (−3x)2 ≡
−3 (mod p). De aici deducem că
     ‹  ‹
−3 p−1 3 p−1 p p−1 3−1 p
1= = (−1) 2 = (−1) 2 · · (−1) 2 · 2 =
p p 3 3

şi p ≡ 1 (mod 3). Această condiţie este necesară. Demonstrăm că este şi sufi-
 
−3
cientă. Dacă p ≡ 1(3), din = 1 deducem că există un număr ı̂ntreg a
p
astfel ı̂ncât a2 ≡ −3 (mod p). Există de asemenea x1 ∈ Z astfel ı̂ncât 3x1 ≡
a (mod p) şi x2 ∈ Z astfel ı̂ncât −3x2 ≡ a (mod p). Atunci (−3xi )2 ≡ a2 ≡
−3 (mod p) şi −3x2i ≡ 1 (mod p) ∀ i = 1, 2. Acestea sunt cele două soluţii
modulo p ale ecuaţiei.

3x2 + 1
22. Să presupunem că ∃ x, k ∈ Z, k 6= 0, astfel ı̂ncât să fie număr
6k − 1
ı̂ntreg. Există atunci p prim p ≡ −1 (mod 6) astfel ı̂ncât p|6k − 1|3x2 + 1.
 
2 2
−3
De aici deducem că 3x ≡ −1 (mod p), (3x) ≡ −3 (mod p), 1 = =
p
       
p−1 3 p−1 p p−1 p p
(−1) 2 = (−1) 2 (−1) 2 = . Egalitatea 1 = este imposi-
p 3 3 3
bilă căci p ≡ −1 (mod 3).
(L. Panaitopol )

23. Dacă n 6= 0, 1, −1, atunci n3 − n − 1 6= −1 şi n3 − n − 1 ≡ −1 (mod 6).


Există deci p prim, p = 6k − 1, p|n3 − n − 1. Dacă fracţia din enunţ ar fi număr
ı̂ntreg, atunci p|m2 + m + 1. Conform problemei 15, deducem p|m şi p|1, ceea
ce este o contradicţie.
(L. Panaitopol )

223
24. Pentru y 6= 0, ±1 avem că y 4 − y 2 = 6k, k ∈ Z, k 6= 0, şi conform
3x2 + 1
problemei 22, 4 nu poate fi număr ı̂ntreg.
y − y2 − 1

3x2 + 1
25. Să presupunem că 2 este ı̂ntreg. Dacă y este par, atunci 3y 2 +
3y + 5
5 = 6k − 1, k 6= 0, şi se obţine o contradicţie conform problemei 22. Deci y
este impar, ceea ce implică faptul că x este impar. Dar 3y 2 + 5 ≡ 3 + 5 ≡
0 (mod 8) şi 3x2 + 1 ≡ 3 + 1 ≡ 4 (mod 8) şi deci 3y 2 + 5¤|¤3x2 + 1.
(L. Panaitopol )

3x2 + 1
26. Nu putem avea n = 2t + 1, căci ar rezulta că ∈ Z, x = 3t ,
6k − 1
k = 6m−1 , ceea ce contrazice problema 22. Deci n = 2t. Dacă m ≥ 2, deducem
că 6m − 1 ≡ −1 (mod 4), 6m − 1 > −1, şi deci există p prim, p ≡ −1 (mod 4),
3n + 1
p|6m − 1. Dacă m ∈ Z, obţinem contradicţia p|6m − 1|3n + 1, 32t ≡
6 −1
 
−1
−1 (mod p), =1 (ultima egalitate este contradictorie căci p ≡ 3 (mod 4).
p
Deci m = 1 şi se deduce imediat că n ≡ 2 (mod 4).

27. 2a2 +2a−1 ≡ 3 (mod 4) şi 2a2 +2a−1 ≥ 3. De aici deducem existenţa
unui număr prim p, p ≡ 3 (mod 4) care divide pe 2a2 + 2a − 1. Evident că p
divide şi b2 + b + 1. Să presupunem că p 6= 3. Atunci 4a2 + 4a − 2 ≡ 0 (mod p),
 
2
3
(2a + 1) ≡ 3 (mod p), = 1 şi 4b2 + 4b + 4 ≡ 0 (mod p), (2b + 1)2 ≡
p
      
−3 −1 −3 3
−3 (mod p), = 1. De aici deducem că = = 1,
p p p p
p ≡ 1 (mod 4), ceea ce constituie o contradicţie. Deducem că 2a2 +2a−1 = 3k ,
k ∈ N∗ . Dar 2 · 3k = (2a + 1)2 − 3, 3|2a + 1, 2 · 3k ≡ −3 (mod 9) şi de aici
deducem k = 1. Deci 2a2 + 2a = 4, a2 + a = 2, a = 1 şi b2 + b + 1 ≡ 0(3).
De aici rezultă imediat că b ≡ 1 (mod 3). Deci soluţiile problemei sunt toate
perechile (1, b), unde b este un număr natural de forma 3t + 1 (t ∈ N).

28. Să presupunem că p ≡ 3 (mod 4). Atunci q = 2p + 1 ≡ 7 (mod 8) şi


   
2 2 q−1
= 1. Conform Criteriului lui Euler, avem că 1 = ≡ 2 2 = 2p (mod q)
q q
q−1
şi deci q|2 −1 = Mp . Să presupunem acum că q|2 −1. Atunci 1 ≡ 2p = 2 2 ≡
p p
   
2 2
(mod q) şi de aici deducem că = 1. Dar q = 2p + 1 ≡ 3 (mod 4).
q q

224
 
2
Cum = −1, dacă q ≡ 3 (mod 8), deducem că q ≡ 7 (mod 8) şi aceasta
q
implică faptul că p ≡ 3 (mod 4) (dacă p ≡ 1 (mod 4), atunci q = 2p + 1 ≡
3 (mod 8)).

p−1
29. Dacă p = 4k + 1, considerăm q prim, q , deci p = 4hq + 1. Dacă
4
    
2 p q
q = 2, avem p = 8h + 1 şi = 1. Dacă q ≥ 3, avem = 1 şi deci
p q p
       
q p 4hq + 1 1 p+1
= = = = 1. Dacă p = 4k − 1, considerăm q ,
p q q q 4
 
2
deci p = 4hq − 1. Dacă q = 2, avem p = 8h − 1 şi = 1.
p
    
p q p−1 q−1 q−1 p
Dacă q ≥ 3, avem = (−1) 2 2 = (−1) 2 şi apoi =
q p q
       
4hq − 1 −1 q−1 q p+1
= = (−1) . Deci
2 = 1. Dacă d , avem
q q p 4
k   k  q αi
Y d Y i
d= qiαi şi = = 1.
i=1
p i=1
p

30. Dacă p ≡ 1, 7 (mod 8), alegem q = 2. Dacă p = 8k + 5, p > 5,


p−1
scriem descompunerea canonică a lui = q1α1 . . . qrαr , qi prim, αi ∈ N∗ ,
4
∀ i = 1, r, r ∈ N∗ (deoarece p > 5). Alegem q = q1 . Evident că q 6= 2, q < p şi
       
q q−1 p−1 p p 1
= (−1) 2 · 2 = = = 1. Pentru p = 8k + 3, p > 3, scriem
p q q q
p+1
= q1α1 . . . qrαr , qi prim, αi ∈ N∗ , r ∈ N∗ (deoarece p > 3) şi qi impar
4
∀ i = 1, r. Alegem q = q1 . Evident că q1 < p şi
       
q q−1 p−1
· 2 p q−1 p q−1 −1
= (−1) 2 = (−1) 2 = (−1) 2 = 1.
p q q q
(A. Gica)

31. Presupunem prin absurd că ecuaţia din enunţ ar avea soluţii. Notăm
a = x1 + x2 + · · · + xn . Din Mica Teoremă a lui Fermat ştim că xpi ≡ xi (mod p)
n
X n
X
p
∀ i = 1, n. Avem deci 2 + a2 = xi ≡ xi = a (mod p). Deci a2 − a + 2 ≡
i=1 i=1
 
−7
0 (mod p). 4(a2 − a + 2) ≡ 0 (mod p), (2a − 1)2 ≡ −7 (mod p), = 1.
p

225
Am obţinut o contradicţie căci
          
−7 −1 7 p−1 p p−1 7−1 p −1
1= = = (−1) 2 (−1) 2 · 2 = = = −1.
p p p 7 7 7
32. Fie p număr prim p|3n + 2. Avem 3n ≡ −2 (mod p). Dacă n
 
−2
este par, atunci −2 este rest pătratic (mod p) şi deci = 1, adică
p
p−1 p2 −1 (p−1)(p+5)
(−1) 2 (−1) 8 = 1 ⇔ (−1) = −1. Am obţinut o contradicţie,
8

(p − 1)(p + 5)
deoarece pentru p = 24k + 13 şi p = 24k + 23, numărul este
8
 
−6
impar. Dacă n este impar, avem 3n+1 ≡ −6 (mod p) şi deci = 1, adică
p
 
p−1 p2 −1 3
(−1) 2 (−1) 8 = 1. (1)
p
      
3 p 3−1 p−1 3 p
Dacă p = 24k + 13, avem = (−1) 2 · 2 = 1 şi deci = =
p 3 p 3
   
24k + 13 1 (p−1)(p+5)
= = 1. Aşadar din (1) avem (−1) 8 = 1, ceea ce
3 3
constituie o contradicţie.
    
3 p 3−1 p−1 3
Dacă p = 24k + 23, avem = (−1) 2 · 2 = −1 şi deci =
p 3 p
     
p 24k + 23 2 (p − 1)(p + 5)
− =− =− = (−1)(−1) = 1, şi cum este
3 3 3 8
impar, se contrazice (1).
(L. Panaitopol )

33. Să presupunem că ∃ k, n ∈ N astfel ı̂ncât 4qk +r|q n +1. 4qk +r = pα1 1 ·
pα2 2 . . . pαs s , p1 < p2 < · · · < ps , pi prim, αi ∈ N∗ , ∀ i = 1, s (4qk + r ≥ r ≥ 3).
Avem că pi 6= 2 (deoarece 4qk + r este impar) şi pi = 6 q (deoarece q¤|¤4qk + r)
n n n
∀ i = 1, s. Din 4qk+r|q +1 deducem că pi |q +1, q ≡ −1 (mod pi ), ∀ i = 1, s.
 
−1
Cazul n par. În acest caz q n este pătrat perfect şi deci = 1. De aici
pi
s
Y
obţinem că pi ≡ 1 (mod 4) ∀ i = 1, s. 4qk + r = pαi i ≡ 1 (mod 4). Acest
i=1
lucru este fals pentru că 4qk + r ≡ r ≡ 3 (mod 4).
Cazul n impar. q n+1 ≡ −q (mod pi ). Cum q n+1 este pătrat, deducem că
      
−q −1 q pi −1 pi pi −1 q−1
1= = = (−1) 2 (−1) 2 · 2 .
pi pi pi q

226
Subcazul q ≡ 3 (mod 4). Din egalitatea de mai sus rezultă că
   
pi pi −1 pi −1 pi
1= (−1) 2 (−1) 2 = , ∀ i = 1, s.
q q
De aici deducem imediat că
ˆ s ’
Y
s  αi pαi i       
Y pi i=1 4kq + r r −r −1
1= = = = = =
i=1
q q q q q q

 
−1 q−1
= = (−1) 2 = −1, ceea ce constituie o contradicţie.
q
   
pi pi −1 −1
Subcazul q ≡ 1 (mod 4). În acest caz =(−1) 2 = , ∀ i = 1, s.
q pi
 αi   αi    
pi pi − 1 αi −1
De aici = = = , unde ultimul simbol este sim-
q q pi pαi i
Ys  pαi 
i
bolul lui Jacobi. Obţinem o contradicţie calculând ı̂n două feluri .
i=1
q
ˆ s ’
Y
s  αi  pαi i       
Y pi i=1 4qk + r r −r −1
O dată = = = = =
i=1
q q q q q q

  s  αi  s    
−1 Y pi Y −1 −1
= 1. Pe de altă parte = = =
q i=1
q i=1
pαi i 4qk + r
4qk+r−1
(−1) 2 = −1, deoarece r ≡ 3 (mod 4).
Observaţie. Dacă alegem q = 3, r = 11, se obţine teorema lui Fermat
12k + 11¤|¤3n + 1, ∀ k, n ∈ N.
(A. Gica)

34. Se arată imediat prin inducţie că an = 2n−2 (n2 −n+4). Să presupunem
că există un număr prim p = 15k + 7 şi un n ∈ N astfel ı̂ncât p|an . Din formula
lui an de mai sus, rezultă că n2 − n + 4 ≡ 0 (mod p), 4n2 − 4n + 16 ≡ 0 (mod p),
       
− 15 − 15 −1 3 5
(2n − 1)2 ≡ −15 (mod p), = 1. Dar = =
p p p p p
         
p−1 p p−1 p p−1 p p 1 2
(−1) 2 (−1) 2 1 (−1) 2 2 = = = −1 şi am
3 5 3 5 3 5
obţinut o contradicţie.

227
( )
p−1 p−1
35. Fie A = j2 − ip | j = 1, , i = 1, . Să arătăm că
2 4
p−1 p−1 p−1 p−1
|A| = · . Fie 1 ≤ j, k ≤ , 1 ≤ i, t ≤ astfel ı̂ncât
2 4 2 4
2 2 2 2
j − ip = k − tp. De aici rezultă că p|j − k şi j = k (deoarece 1 ≤ j,
p−1 p−1 p−1
k≤ ), i = t. Aceasta justifică faptul că |A| = · .
2 2 4
p−1
0 6∈ A căci egalitatea j 2 = ip este imposibilă pentru ∀ j = 1, . Dacă
2
notăm B = {a ∈ A | a < 0}, C = {a ∈ A | a > 0}, evident că A = B ∪ C,
B ∩ C = ∅. Pentru a calcula cardinalul lui B, trebuie calculat numărul de
p−1 p−1
perechi (j, i), 1 ≤ j ≤ ,1≤i≤ , pentru care j 2 < ip. Pentru un i
2 4
p−1 ”√ —
fixat ı̂ntre 1 şi , numărul de j-uri cu proprietatea cerută este ip . Deci
4
p−1 p−1
X 4 hÈ i X 2  j2
|B| = ip = S. Analog se arată că |C| = . S = |B| = |A| − |C|.
i=1 j=1
p
p−1
X2  j2
Pentru a-l calcula pe S trebuie să socotim |C| = . Pentru fiecare
j=1
p
 
p−1 j2
1≤j ≤ , avem că j 2 = p · qj + rj , qj , rj ∈ N, rj ≤ p − 1, = qj ,
2 p
p−1 p−1 p−1 p−1
X
2 X
2 j 2 − rj
1X 2 1X 2 n p − 1o
|C| = qj = = j2 − rj . Mulţimea rj | j = 1,
j=1 j=1
p p j=1 p j=1 2
este formată din toate resturile pătratice modulo p.
     
p − rj − rj rj
Însă, deoarece p ≡ 1 (mod 4), avem că = = =1
p p p
p−1
8 9 8 9 p−1
< p − 1= < p − 1= X 2 X2
şi deci p − rj j = 1, = rj j = 1, . rj = (p − rj ) =
: 2 ; : 2 ; j=1 j=1

p−1 p−1
p−1 2
X 2
X p−1
p − rj , rj = p . Revenind ı̂n formulele de mai sus: |C| =
2 j=1 j=1
4
p−1 p−1
1 X2 1 X2 p2 − 1 p − 1 p − 1 p − 1 p2 − 1
j2 − rj = − şi S = |A| − |C| = · − +
p j=1 p j=1 24 4 2 4 24

228
     
p−1 p−1 p−1 p+1 p−1 p+1 p+1 p2 − 1 1
= − +1 = − = 1− =
4 4 2 6 4 2 6 8 3
p2 − 1 2 p2 − 1
· = .
8 3 12
36. Dacă p = 2, atunci f (x) = x4 sau f (x) = (x2 + b 1)2 . Dacă p|a,
atunci f (x) = x4 . Presupunem ı̂n continuare că p¤|¤a, p 6= 2. Din identitatea
    
− 2a − 1 2a
= , deducem că unul din cele trei simboluri Legendre
p p p
 
−1
este +1. Dacă = 1, există b ∈ Z astfel ı̂ncât b2 ≡ −1 (mod p) şi
p
 
2a
f (x) = x4 − a2 b2 = (x2 − ab)(x2 + ab). Dacă = 1, atunci ∃ b ∈ Z astfel
p
2
ı̂ncât b2 ≡ 2a (mod p) şi f (x) = (x2 + a)2 − b x2 = (x2 + bx + a)(x2 − bx + a).
 
− 2a
Dacă = 1, atunci ∃ b ∈ Z astfel ı̂ncât b2 ≡ −2a (mod p) şi f (x) =
p
2
(x2 − a)2 − b x2 = (x2 + bx − a)(x2 − bx − a).

37. Fie q cel mai mic nerest pătratic modulo p. Să presupunem că q >
√ √
p + 1, adică q ≥ [ p] + 2. Există un j ∈ N∗ astfel ı̂ncât jq < p < (j + 1)q.
€√ Š€ √ Š €√ Š√ √
Deoarece q(q − 1) ≥ [ p] + 2 [ p] + 1 > p + 1 p = p + p > p,
deducem că j ≤ q − 2. Avem că 0 < (j + 1)q − p < q (conform inegalităţilor
de mai sus). Pe de altă parte, deoarece j + 1 ≤ q − 1, rezultă (ţinând seama
      
j+1 (j + 1)q − p j+1 q
de definiţia lui q) că = 1. Avem că = =
p p q p
 
q
= −1, 0 < (j + 1)q − p < q şi se contrazice definiţia lui q. Demonstraţia
p
este ı̂ncheiată.

38. Fie m1 , m2 ∈ N∗ , (m1 , m2 ) = 1 şi n1 , n2 ∈ Z astfel ı̂ncât f (n1 ) ≡


0 (mod m1 ) şi f (n2 ) ≡ 0 (mod m2 ). Deoarece (m1 , m2 ) = 1, ∃ k, l ∈ Z astfel
ı̂ncât n1 +km1 = n2 +lm2 = n. Avem că f (n) ≡ f (n1 ) ≡ 0 (mod m1 ) şi f (n) ≡
f (n2 ) ≡ 0 (mod m2 ). Cum (m1 , m2 ) = 1, avem că f (n) ≡ 0 (mod m1 m2 ).
Această observaţie ne arată că este suficient să demonstrăm enunţul pentru
m = pk , p prim, k ∈ N∗ . Fie p prim, p 6= 2, 3, 13. Dacă cumva p¤|¤f (n) ∀ n ∈
     
−3 13 − 39
N, atunci ar rezulta că = = = −1. Acest lucru nu
p p p
    
− 39 − 3 13
este posibil căci ar rezulta contradicţia −1 = = =
p p p

229
(−1)(−1) = 1. Există deci α ∈ {3, −13, 39} şi x0 ∈ Z astfel ı̂ncât p|x20 + a.
Este evident că (p, x0 ) = 1 (deoarece p 6= 3, 13).
Aplicăm Teorema 1, punctul a), pagina 104 din curs, pentru polinomul
g(x) = x2 + a, p, x0 (g 0 (x0 ) = 2x0 6≡ 0 (mod p) deoarece p 6= 2 şi p¤|¤x0 ) şi
obţinem un n ∈ Z astfel ı̂ncât g(n) ≡ 0 (mod pk ) (k ∈ N∗ era fixat). Evident
că f (n) ≡ 0 (mod pk ).
Dacă p = 3, aplicăm acelaşi raţionament ca mai sus, unde g(x) = x2 − 13,
x0 = 1. Pentru p = 13, alegem g(x) = x2 + 3, x0 = 6 şi aplicăm procedura
precedentă.
Pentru p = 2, alegem g(x) = x2 + 39. Avem că 23 |g(1). Presupunem că
2k |g(n), k ∈ N, k ≥ 3. Avem n2 + 39 = 2k · b. Alegem t ∈ Z, t ≡ b (mod 2).
Atunci g(n+2k−1 t) ≡ 0(2k+1 ). Aceasta se ı̂ntâmplă deoarece (n+2k−1 t)2 +39 =
n2 +39+2k ·nt+22k−2 t2 = 2k (b+nt)+22k−2 t2 , b+nt este număr par (fiindcă n
este impar şi b şi t au aceeaşi paritate), 2k − 2 ≥ k + 1 (k ≥ 3). Un raţionament
prin inducţie după k ı̂ncheie demonstraţia.

230
CAPITOLUL 11

Ordinul unui element (Gaussian)

2. Trebuie arătat că pentru p ∈ {11, 31, 61} avem γp (20)|15. Pentru p = 11
avem r1 = 9.
i 1 2 3 4 5 6 7 8 9 10
ri 9 7 5 3 1 10 8 6 4 2
bi 9 4 3 5 1

Aşadar γ11 (20) = 5. Avem 205 − 1 = 11 · 61 · 4769. Deci γ61 (20) = 5. Pentru
p = 31 avem 202 ≡ −3 (mod 31), 204 ≡ 9 (mod 31), 205 ≡ 180 (mod 31) şi deci
205 ≡ −6 (mod 31), 2015 ≡ 1 (mod 31). Aşadar 31|2015 − 1.

3. Trebuie arătat că γ8 (a), γ9 (a) şi γ25 (a) sunt divizori ai lui 60. Cum
a este impar a2 ≡ 1 (mod 8) şi deci γ8 (a) | 2 | 60. Deoarece (a, 9) = 1, rezultă
aϕ(9) ≡ 1 (mod 9) şi deci a6 ≡ 1 (mod 9). Din (a, 25) = 1 rezultă a20 ≡ 1 (mod 25)
şi deci γ9 (a)|6, şi 6|60, şi γ25 (a)|20, şi 20|60.

4. Ecuaţia este echivalentă cu x8 ≡ 8 (mod 13). Ştim (curs, pagina 115)


12
că ultima ecuaţie are soluţii dacă şi numai dacă 8 (12,8) = 83 ≡ 1 (mod 13). Dar
83 = 29 = 512 ≡ 5 (mod 13) şi deci ecuaţia nu are soluţii.

5. Evident că ecuaţia are soluţii pentru a ≡ 0 (mod 37).


37−1
Dacă a 6≡ 0 (mod 37), ecuaţia are soluţii dacă şi numai dacă a d ≡ 1 (mod 37),
unde d = (27, 37 − 1) = 9 (curs, pagina 115). Congruenţa a4 ≡ 1 (mod 37) are
doar soluţiile a ≡ 1, 6, 31, 36 (mod 37) şi răspunsul la problema din enunţ este:
a ≡ 0, 1, 6, 31, 36 (mod 37).

6. Avem că n = γan −1 (a). Pe de altă parte, deoarece (a, an − 1) = 1,


n
rezultă că aϕ(a −1) ≡ 1 (mod an − 1). De aici deducem imediat că n =
γan −1 (a) | ϕ(an − 1) (curs, pagina 112).

7. Dacă a ≡ 0 (mod p), rezultă imediat că x ≡ 0 (mod p). Să presupunem
că a 6≡ 0 (mod p). Atunci ax ≡ xn+1 ≡ a2 (mod p) şi x ≡ a (mod p). Deci
an ≡ a (mod p), an−1 ≡ 1 (mod p). De aici deducem că γp (a) | n − 1. Cum

231
1. Arătăm că γ13 (17) = γ7 (17) = γ27 (17) = 6 şi aceasta rezolvă problema.
m = 13; a = 17
i 1 2 3 4 5 6 7 8 9 10 11 12
ri 4 8 12 3 7 11 2 6 10 1 5 9
bi 4 3 12 9 10 1
Tablul precedent ne arată că γ13 (17) = 6
m = 7; a = 17
i 1 2 3 4 5 6

232
ri 3 6 2 5 1 4
bi 3 2 6 4 5 1
şi deci γ7 (17) = 6
m = 27; a = 17
i 1 2 3 4 5 6 7 8 9 10 11 12 13 14 15 16 17 18 19 20 21 22 23 24 25 26
ri 17 7 24 14 4 21 11 1 18 8 25 15 5 22 12 2 19 9 26 16 6 23 13 3 20 10
bi 17 19 26 10 8 1
Tabelul precedent ne arată că γ27 (17) = 6
γp (a) | (p − 1) (curs, pagina 112), rezultă că γp (a) | (n − 1, p − 1), γp (a) = 1,
a ≡ 1 (mod p).

8. Avem a2 ≡ axq−1 ≡ xq−2 (mod p), ax ≡ 1 (mod p). Deci aq ≡ a2 ·


aq−2 ≡ xq−2 · aq−2 ≡ (ax)q−2 ≡ 1q−2 = 1 (mod p). Deducem că γp (a) | q. Cum
q este prim, rezultă că γp (a) = 1 sau γp (a) = q. Dacă γp (a) = 1, rezultă
contradicţia a ≡ 1 (mod p). Deci γp (a) = q|p − 1 (curs, pagina 112), exact ce
trebuia demonstrat.

9. a) Înmulţind congruenţa −5n ≡ 7n (mod 11) cu 9n , obţinem 8n ≡


63n ≡ −1 (mod 11). Deci 82n ≡ 1 (mod 11). Cum γ11 (8) = 10, deducem că
10|2n, 5|n, n = 5t, t ∈ N. Deoarece 85 ≡ −1 (mod 11) şi 8n ≡ −1 (mod 11),
rezultă imediat că t este impar şi deci n = 10s + 5, s ∈ N.
b) Înmulţim congruenţa −5n ≡ 7n (mod 37) cu 15n şi obţinem 31n ≡
−1 (mod 37). Cum γ37 (31) = 4, obţinem la fel ca mai sus că n = 2t, t impar.
Deci n = 4s + 2, s ∈ N.
n n+1
10. Fie g = γp (2). Deoarece p|Fn , rezultă că 22 ≡ −1 (mod p), 22 ≡
n+1 n 2 n g h
1 (mod p), g|2 . Dacă g|2 , atunci −1 ≡ 2 = (2 ) ≡ 1 (mod p) şi am
obţinut o contradicţie căci p 6= 2. Deci g = 2n+1 |ϕ(p) = p − 1 (curs, pagina
 
n+1
2
112). Avem că p = 1+t·2 şi p ≡ 1 (mod 8) deoarece n ≥ 2. Deci 1 = ≡
p
p−1 p−1
2 2 (mod p) (conform Criteriului lui Euler) şi de aici deducem că g =
2
n
t · 2 . Cum g = 2 n+1 , rezultă că t este par şi că p ≡ 1 (mod 2n+2 ).

11. a) Fie q un divizor prim al lui F4 = 216 + 1. Conform problemei


precedente, q ≡ 1 (mod 24+2 ), q ≡ 1 (mod 64), q = 64t+1, t ∈ N∗ . Dacă 216 +1
”√ —
nu ar fi prim, ar admite un divizor prim mai mic decât 216 + 1 = 28 = 256.
Cum q = 64t + 1, singurul candidat posibil ar fi 193. Se arată imediat că restul
ı̂mpărţirii lui 65537 la 193 este 110 şi deci 216 + 1 este prim.
15
b) Fie q un divizor prim al lui 122 + 1. Evident că q 6= 2, 3. Avem că
15 16
122 ≡ −1 (mod q), 122 ≡ 1 (mod q) şi d, ordinul lui 12 ı̂n (Z∗q , ·), este un
15
divizor al lui 216 . Dacă d|215 , deducem contradicţia −1 ≡ 122 ≡ 1 (mod q).
Deci d = 216 |q − 1 = |Z∗q |. q = 216 · k + 1, k ∈ N∗ , q ≥ 216 + 1.
15
Mai rămâne să arătăm că q = 216 + 1 | 122 + 1. Din Mica Teoremă a
16
lui Fermat ştim că 22 = 2q−1 ≡ 1(q), iar din Criteriul lui Euler ştim că
 
215 q−1 3
3 =3 2 ≡ (mod q).
q

233
 
15 16 15 15 3
Cu aceste observaţii deducem că 122 +1 ≡ 22 · 32 +1 ≡ 32 +1 ≡
+
q
     16 !  16 
3 q q−1 2 +1 15 2 +1
1 (mod q). Însă = (−1) 2 = (−1)2 = =
q 3 3 3
   
2 15 3
= −1 şi deci 122 +1 ≡ +1 = 0 (mod q), ceea ce trebuia demonstrat.
3 q
12. 233 − 1 = (211 − 1)(222 + 211 + 1) = 23 · 89 · 4196353. Deoarece
7 = 23 − 1|233 − 1, deducem că 233 − 1 = 23 · 89 · 7 · 599479. Fie q un divizor
prim al lui 599479. Evident că q|233 − 1 şi 233 ≡ 1 (mod q).
Notând cu d ordinul lui 2 ı̂n (Z∗q , ·), avem că d|33, d ∈ {1, 3, 11, 33}. Dacă
d ∈ {1, 3, 11}, ar rezulta că q ∈ {7, 23, 89}. Deoarece 7, 23 şi 89 nu divid
5999479, obţinem că d = 33. Cum d|q − 1, deducem că q = 33k + 1. q fiind
număr prim impar, rezultă că k este par şi q = 66t+1 (t ∈ N∗ ). Dacă 599479 nu
”√ —
ar fi prim, ar trebui să aibă un divizor prim mai mic decât 599479 = 774.
Singurele prime q de forma 66t + 1 mai mici decât 774 sunt 67, 199, 331, 397,
463, 661, 727. Cum nici unul din aceste numere nu-l divide pe 599479, deducem
că 599479 este prim şi 23 · 89 · 7 · 599479 este descompunerea ı̂n factori primi
a lui 233 − 1.

13. Presupunem că ∃ n ∈ N, n ≥ 2, astfel ı̂ncât n|2n −1. Evident că n este
impar. n = pα1 1 . . . pαr r , 3 ≤ p1 < p2 < · · · < pr , pi prim, αi ∈ N∗ , ∀ i = 1, r. Fie
k ordinul lui 2 ı̂n (Z∗p1 , ·). Evident că 2 ≤ k ≤ p1 − 1. Cum 2n ≡ 1 (mod p1 ),
rezultă că k|n. ∃ q prim, q|k. Obţinem contradicţia q|k|n, q ≤ k ≤ p1 − 1 < p1 .

14. Fie p divizorul prim minim al lui n. Dacă n|3n − 2n , rezultă p > 3. Fie
a astfel că 2a ≡ 1 (mod p). Din 3n ≡ 2n (mod p), rezultă (3a)n ≡ 1 (mod p).
Fie γp (3a) = i. Rezultă i|p − 1 şi i|n. Cum i < p şi i|n, rezultă i = 1 (conform
minimalităţii lui p). Aşadar 3a ≡ 1 (mod p) şi cum 2a ≡ 1 (mod p), rezultă
a ≡ 0 (mod p) şi apoi 0 ≡ 1 (mod p); contradicţie.

15. Pentru k = 1 avem n = 1. Arătăm că pentru k ≥ 2, n = 2k−2 . Pentru


m impar avem că 4k5m − 1 deoarece 5m − 1 = 4(5m−1 + 5m−2 + · · · + 5 + 1) şi
numărul 5m−1 +5m−2 +· · ·+5+1 este impar. Pentru orice m avem că 2k5m +1
deoarece 5m + 1 ≡ 2 (mod 4). Fie n = 2i (2a + 1), a, i ∈ N. Avem identitatea
€ Š€ Š€ Š€ 2 Š € i−1 Š
5n −1 = 52a+1 −1 52a+1 +1 52(2a+1) +1 52 (2a+1) +1 . . . 52 (2a+1) +1 .
i
Din observaţiile precedente deducem că 2i+2 k52 (2a+1) −1. Pentru k ≥ 2 alegem
i ∈ N astfel ı̂ncât i + 2 = k şi din cele de mai sus deducem că numărul căutat
este n = 2k−2 .

234
16. Pentru n = 2i (2a + 1), a, i ∈ N, avem identitatea 3n − 1 =
€ Š€ Š€ Š€ 2 Š € i−1 Š
32a+1 − 1 32a+1 + 1 32(2a+1) + 1 32 (2a+1) + 1 . . . 32 (2a+1) + 1 .
j
Avem că 2k32a+1 − 1, 4k32a+1 + 1, 2k32 (2a+1) + 1, ∀ j ≥ 1, căci 32a+1 − 1 ≡
j
2 (mod 4), 32a+1 + 1 ≡ 4 (mod 8), 32 (2a+1) + 1 ≡ 2 (mod 4). Deci
i
2i+2 k32 (2a+1) − 1 pentru i ∈ N∗ . Deci numărul căutat ı̂n enunţ este n = 22003 .

17. Presupunem că q este număr prim. În acest caz simbolul Legendre
 
2 q 2 −1 (2p+1)2 −1 p(p+1)
= (−1) 8 = (−1) 8 = (−1) 2 = (−1)2(k+1)(4k+3) = 1
q
q−1
şi deci 2 este rest pătratic mod (q). Din Criteriul lui Euler rezultă că 2 2 ≡
1 (mod q), adică 2p ≡ 1 (mod q). Presupunem acum că 2p ≡ 1 (mod q). Dacă q

este compus, există q1 |q, q1 prim şi q1 ≤ q. În acest caz 2q1 −1 ≡ 1 (mod q1 ).
Deoarece 2p ≡ 1 (mod q1 ) şi p este prim, rezultă că p este ordq1 (2) şi deci

p|q1 − 1. De aici rezultă că q1 ≥ p + 1 care intră ı̂n contradicţie cu q1 ≤ q =

2p + 1. Aşadar q este prim.
Comentariu. Această teoremă permite identificarea unor numere Mersenne
Mp compuse cu p număr prim. Pentru p = 11, q = 2p + 1 = 23. Cum 23 este
prim, rezultă 23|M11 şi deci M11 este compus. În mod analog se arată că M23 ,
M83 , M131 , M179 etc. sunt compuse, fără să fi indicat ı̂nsă toţi factorii primi
ai lor. În afară de aceasta nu ştim dacă există o infinitate de numere prime
q = 2p + 1 şi deci nu ştim dacă teorema se aplică unui număr infinit de cazuri.

18. Pentru p = 2 alegem q = 3 şi e clar n2 6≡ 2 (mod 3), ∀ n ∈ N.


În continuare presupunem că ar exista un număr prim p ≥ 3 care să nu
satisfacă condiţia din enunţ. Fie pp−1 + pp−2 + · · · + p2 + p + 1 = q1α1 . . . qrαr ,
descompunerea canonică a numărului pp−1 + pp−2 + · · · + 1. Evident că qi 6= 2,
∀ i = 1, r. Pentru fiecare i = 1, r, presupunerea făcută implică existenţa unui
ni ∈ N astfel ı̂ncât npi ≡ p (mod p). Este evident că qi 6= p, ∀ i = 1, r şi că
2
qi ¤|¤ni . Avem că npi ≡ pp ≡ 1(qi ) şi notând cu di ordinul lui ni ı̂n (Z∗qi , ·), avem
că di |p2 , di ∈ {1, p, p2 }. Dacă di = 1 sau di = p, am avea că npi ≡ 1 ≡ p(qi ) şi
0 ≡ pp−1 +pp−2 +· · ·+p+1 ≡ p (mod qi ), p = qi , p|pp−1 +pp−2 +· · ·+p+1; am
ajuns la o contradicţie. Avem deci neapărat că di = p2 şi p2 |qi − 1 (deoarece
ordinul unui element divide cardinalul grupului). Rezultă că qi ≡ 1 (mod p2 )
∀ i = 1, r, şi ı̂nmulţind aceste congruenţe obţinem că q1α1 . . . qrαr ≡ 1 (mod p2 ).
Însă q1α1 . . . qrαr = pp−1 + pp−2 + · · · + p2 + p + 1 ≡ p + 1 (mod p2 ) şi am obţinut
contradicţia 1 ≡ p+1 (mod p2 ). Demonstraţia problemei este ı̂ncheiată ı̂n acest
moment.

235
19. Evident că (1, p) este soluţie a problemei, ∀ p prim. Dacă n este par,
atunci 2|(p − 1)n + 1 şi p este neapărat 2 (căci altminteri (p − 1)n + 1 ar fi
impar). Din n|(2 − 1)n + 1 = 2, deducem că n = 2. Putem deci presupune
că n este un număr impar, n ≥ 3. Fie q cel mai mic divizor prim al lui n.
Avem că q|n|np−1 |(p − 1)n + 1 şi deci (p − 1)n ≡ −1(q). Notăm cu d ordinul
lui p − 1 ı̂n (Z∗q , ·). Este evident că d|q − 1. Din (p − 1)n ≡ −1(q), rezultă că
(p − 1)2n ≡ 1(q) şi d|2n. Deci d|(2n, q − 1). Însă (q − 1, n) = 1 pentru că q este
cel mai mic divizor prim al lui n. Din d|(2n, q − 1) şi (n, q − 1) = 1 deducem
că d|(2, q − 1) = 2, d ∈ {1, 2}.
Dacă d = 1 avem că p − 1 ≡ 1(q), −1 ≡ (p − 1)n ≡ 1(q), q = 2. Acest
lucru este imposibil căci n este impar. Deci d = 2 şi p − 1 ≡ −1(q), p ≡ 0(q),
p = q, n = pa, a ∈ N∗ . Cum n ≤ 2p şi n este impar, obţinem că n = p, p
fiind un număr prim impar. Avem că (p − 1)p + 1 ≡ −Cp2 p2 + Cp1 · p − 1 + 1 =
p(p − 1)
−p2 + p2 ≡ p2 (mod p3 ) şi deci p3 ¤|¤(p − 1)p + 1. Însă din ipoteză ştim
2
că pp−1 = np−1 |(p − 1)n + 1 = (p − 1)p + 1 şi din cele de mai sus rezultă că
p − 1 < 3, p < 4, p = 3. Am arătat deci că soluţiile problemei sunt (3, 3), (2, 2)
şi (1, p), ∀ p prim.

20. Să presupunem că i este impar. Avem aj ≡ −1 (mod p) şi a2j ≡
1 (mod p). Din definiţia ordinului rezultă că i|2j. Cum i este impar, rezultă
că i|j, j = ki, k ∈ N. Avem contradicţia −1 ≡ aj = (ai )k ≡ 1k = 1 (mod p).
Deci i este par: i = 2t, t ∈ N∗ . Avem (at )2 ≡ 1 (mod p), at 6≡ 1 (mod p) şi
deci at ≡ −1 (mod p). Din definiţia lui j deducem că t ≥ j. Deoarece aj ≡ −1
(mod p), a2j ≡ 1 (mod p), rezultă că 2t = i|2j, t|j, t ≤ j. Această din urmă
i
inegalitate ı̂mpreună cu t ≥ j ne asigură că j = t = .
2
21. n = 1 şi n = 3 sunt soluţii ale problemei şi arătăm că sunt singurele.
Fie n > 1 soluţie a problemei şi fie p cel mai mic factor prim al lui n; evident
că p > 2 (n este impar deoarece n|2n + 1). Avem p|n|2n + 1. Fie i > 0 minim
γp (2) p − 1
astfel ı̂ncât p|2i + 1. Din probema 20 ştim că i = ≤ < p.
2 2
Fie k, r ∈ N astfel ı̂ncât n = ki + r, 0 ≤ r ≤ i − 1. Avem că −1 ≡
2 = (2i )k · 2r ≡ (−1)k · 2r (mod p). Dacă k este par, rezultă că p|2r + 1. Din
n

minimalitatea lui i deducem că r = 0 şi contradicţia p = 2. Deci k este impar şi
2r ≡ 1 (mod p). Dacă r > 0, avem i = r +j, 1 ≤ j ≤ i−1, −1 ≡ 2i = 2r ·2j ≡ 2j
(mod p) şi se contrazice minimalitatea lui i. Deci n = ik, i|n, i < p. Din
definiţia lui p deducem că i = 1 şi p = 3.

236
Fie n = 3t · α, (3, α) = 1. Să presupunem că t ≥ 2. Avem 2n + 1 =
t
(3 − 1)3 ·α + 1 ≡ 3Cn1 − 32 Cn2 + · · · + Cn2t−1 32t−1 (mod 32t ). Avem e3 (h) =
X∞  h X∞ h h
j
< j
= şi exponentul lui 3 ı̂n Cnh 3h este mai mare strict decât
j=1
3 j=1
3 2
h h
t+h− = t + ≥ t + 1 pentru h ≥ 2. Pentru h = 1 avem 3Cn1 = 3t+1 α. Din
2 2
aceste consideraţii deducem că 3t+2 ¤|¤2n + 1. Aceasta este o contradicţie căci
3t+2 divide 32t (t ≥ 2) şi 32t |n2 |2n + 1. Deci t = 1, n = 3α, 3¤|¤α. Să presupunem
că n > 3 şi fie q cel mai mic factor prim al lui α şi j minim pentru care q|2j +1.
γq (2) q−1
Avem j = ≤ < q şi deoarece q|n|2n + 1, deducem la fel ca mai
2 2
sus că j|n. Din definiţia lui q rezultă că j ∈ {1, 3}. Cum q|2j + 1, deducem
contradicţia q = 3(21 + 1 = 3, 23 + 1 = 32 ). Deci n = 1 şi n = 3 sunt singurele
soluţii ale problemei.
(L. Panaitopol )

22. Lema 1. Dacă p > 2 este prim şi p|a + b, atunci p2 |ap + bp .
Soluţie. ap + bp = (a + b)(ap−1 − ap−2 b + ap−3 b2 · · · − abp−2 + bp−1 ). Cum
a ≡ −b (mod p), rezultă că ap−1 − ap−2 b · · · − abp−2 + bp−1 ≡ p · bp−1 ≡
0 (mod p) şi enunţul este demonstrat.
Lema 2. Fie q 6= 5, q prim, astfel ı̂ncât q|2p + 3p , p fiind număr prim. Să
se arate că q > p.
Soluţie. Fie p ≥ i > 0 minim astfel ı̂ncât q|2i + 3i . Dacă i = 1 rezultă
contradicţia q = 5; deci i > 1. Să presupunem că i < p. Fie k, r ∈ N astfel
ı̂ncât p = ki + r, 0 ≤ r ≤ i − 1. Dacă r = 0 atunci p = ki. Cum i > 1
şi p este prim, deducem că i = p, ceea ce contrazice ipoteza i < p. Deci
1 ≤ r ≤ i − 1. Deoarece 3i ≡ −2i (mod q) şi 0 ≡ 2p + 3p (mod q), deducem
că 0 ≡ 2r · 2ki + 3r · 3ki ≡ 2ki (2r + (−1)k 3r ) (mod q) şi q|2r + (−1)k 3r . Dacă
k este par se obţine o contradicţie cu minimalitatea lui i(1 ≤ r ≤ i − 1 şi
q|2r + 3r ). Dacă k este impar, deducem că 2r ≡ 3r (mod q) şi 0 ≡ 2i + 3i =
2r ·2i−r +3r ·3i−r ≡ 2r (2i−r +3i−r ) (mod q), q|2i−r +3i−r . Cum 0 < i−r < i, se
obţine din nou o contradicţie cu minimalitatea lui i. Am arătat deci că i = p.
Fie a ∈ Z astfel ı̂ncât 3a ≡ 1 (mod q) (evident că q 6= 3). Atunci i > 0 este
minim cu proprietatea că (2a)i ≡ −1 (mod p). Conform problemei 20 (q 6= 2),
γq (2a) q − 1
deducem că i = ≤ < q − 1 şi deci p = i < q − 1 < q.
2 2
Soluţia problemei iniţiale. Definim q1 = 5 şi qk este un divizor prim diferit
de 5 al lui 2qk−1 + 3qk−1 pentru k ≥ 2 (trebuie observat că pentru p 6= 2, 5, p

237
prim, 2p + 3p ≡ 5p2p−1 6≡ 0 (mod 52 ) şi deci 2p + 3p are un divizor prim diferit
de 5. Alegem q2 = 11 căci 25 + 35 = 52 · 11). Conform Lemei 2, deducem că
q1 < q2 < q3 < . . . . Fie n = q1 q2 . . . qk . Pentru j ≥ 2 avem qj |2qj−1 + 3qj−1 .
Conform Lemei 1 avem că qj2 |2qj−1 qj + 3qj−1 qj |2n + 3n . Avem 52 |25 + 35 |2n + 3n
şi de aici rezultă că n2 |2n + 3n .
(L. Panaitopol )

23. 23−1 = 2·11. Trebuie deci să găsim x minim cu proprietatea că x11 6≡ 1
       
1 2 3 4
(mod 23) şi x2 6≡ 1 (mod 23). Deoarece = = = = 1,
23 23 23 23
rezultă că 111 ≡ 211 ≡ 311 ≡ 411 ≡ 1 (mod 23) (conform Criteriului lui Euler).
 
5
Avem că 511 ≡ = −1 (mod 23) şi 52 ≡ 2 (mod 23) şi deci 5 este numărul
23
căutat.

24. Deci p = 2q + 1, q fiind număr prim. Notăm g = γp (2). Atunci


g|p − 1 = 2q. Deci g ∈ {1, 2, q, 2q}. Dacă g = 1, rezultă contradicţia 2 ≡ 1
(mod p). Dacă g = 2, deducem că 22 ≡ 1 (mod p), p = 3, q = 1; contradicţie.
 
2 p−1
Dacă g = q, rezultă că 2q ≡ 1 (mod p) şi deci ≡ 2 2 = 2q ≡ 1 (mod p),
p
 
2
= 1; acest lucru este imposibil, căci p ≡ 3, 5 (mod 8). Deci g = 2q şi
p
enunţul este demonstrat.

25. Notăm g = γp2 (a), h = γp2 (a + p). Deoarece ah ≡ (a + p)h ≡


ag ≡ 1 (mod p) şi γp (a) = p − 1, deducem că p − 1|h şi p − 1|g. Pe de altă
parte g|ϕ(p2 ) = p(p − 1) şi h|ϕ(p2 ) = p(p − 1) (curs, pagina 112). Dacă
enunţul nu este adevărat, atunci g = h = p − 1 şi ap−1 ≡ 1 (mod p2 ), respectiv
(a + p)p−1 ≡ 1 (mod p2 ). Din aceste două congruenţe deducem prin scădere
că 0 ≡ (a + p)p−1 − ap−1 ≡ Cp−11 ap−2 p (mod p2 ). De aici rezultă contradicţia
1 ap−2 .
p|Cp−1

238
CAPITOLUL 12

Numere speciale
n n
1. Fn = k 2 ⇔ 22 + 1 = k 2 ⇔ (k − 1)(k + 1) = 22 . Rezultă k − 1 = 2a
n
şi k + 1 = 2b , a < b. Avem că 2b−1 − 2a−1 = 1 ⇒ a = 1 ⇒ k = 3 ⇒ 22 = 8;
contradicţie.

2. Fn+1 = (Fn − 1)2 + 1, F0 = 3, F1 = 5. Se arată că Fn ≡ 3 (mod 7) sau


Fn ≡ 5 (mod 7), ı̂nsă k 3 ≡ 0, 1, −1 (mod 7); contradicţie.

.
3. Se alege n = kϕ(m) şi Mn .. m (m impar).

4. Din 2n = k m + 1 rezultă k impar, k ≥ 3. Pentru m par rezultă


contradicţia 2n = M8 + 2. Pentru m impar avem 2n = (k + 1)(k m−1 − k m−2 +
· · · + 1). Contradicţia provine din faptul că M = k m−1 − k m−2 + · · · + 1 este
impar şi M > 1.

5. Fie d = (a1 , r). Rezultă de aici an+1 = d(a01 + r0 n) cu (a01 , r0 n) = 1 şi


deci d|an+1 . Rezultă a0n+1 = a01 + r0 n. Fie p prim p|a01 + r0 . Rezultă (p, r0 ) = 1.
.
Arătăm că există n astfel ca a0 + r0 n .. p2 . Fie n = 1 + (a0 + r0 )n . Avem
1 1 1
a0n+1 = a01 + r0 + (a01 + r0 )n1 r0 = (a01 + r0 )(1 + r0 n1 ). Cum (p, r0 ) = 1, există o
. .
infinitate de numere n1 a.ı̂. 1 + r0 n1 .. p şi deci an+1 = da0n+1 .. p2 .
(L. Panaitopol )

k
Y
6. n = pαi i . Deoarece n este perfect, rezultă că σ(n) = 2n.
i=1
! !
σ(n) Yk1 1 1 k
Y 1
=2= 1 + + 2 . . . αi < 1+ şi ln(1 + x) < x.
n i=1
pi pi pi i=1
pi − 1
!
Y 1 x
Avem 2 ≥ 1+ şi ln(1 + x) > .
p|n
p 1+x

239
7. Dacă p1 ≥ k + 1, rezultă p2 > k + 2, p3 > k + 3, . . . pk > 2k şi cum
k  
Y 1
2< 1+ , deducem contradicţia
i=1
pi − 1
„ Ž„ Ž „ Ž
1 1 1
2< 1+ 1+ ... 1+ = 2.
k k+1 2k − 1

8. Trebuie arătat că 2n ≡ 2 (mod n). Evident 2n ≡ 2 (mod 2)2n − 2 =


2(2n−1 − 1), 73 este număr prim şi avem 2 · 1103 = 72k + 46. 2089 = 72h + 1,
73 = 72+1. Deci n−1 = 72l +45 şi 2n−1 −1 = (272l )245 −1. Din Mica Teoremă
a lui Fermat avem 272 ≡ 1 (mod 73) şi deci 2n−1 − 1 ≡ 245 − 1 (mod 73).
Cum 29 − 1 ≡ 0 (mod 73), rezultă 2n−1 ≡ 1 (mod 73). Numerele 1103 şi 2089
sunt de asemenea prime. 2089 ≡ −115 (mod 1102); 2 · 73 ≡ 146 (mod 1102)
şi n − 1 ≡ 841 (mod 1102). Avem 1103 · 2089 · 233 = 229 − 1(1). Deci 229 ≡
1 (mod 1103). Cum 841 = 292 , rezultă 2n−1 ≡ (229 )29 ≡ 1 (mod 1103). Avem
n − 1 = 261 (mod 2089) şi deci 2n−1 ≡ 2261 ≡ (229 )9 (mod 2089). Din (1)229 ≡
1 (mod 2089) rezultă 2n−1 ≡ 1 (mod 2089) şi demonstraţia este terminată.

9. Un număr n este număr Caramichael dacă şi numai dacă:


a) n este impar;
b) n = q1 q2 . . . qk , qi 6= qj ;
c) k ≥ 3 şi qi − 1|n − 1. Avem n = 15841 = 7 · 31 · 73 şi n − 1 = 25 · 32 · 5 · 11
şi deci qi − 1|n − 1. Pentru n = 101101 = 7 · 11 · 13 · 101 şi n − 1 = 22 · 52 · 3 · 337
şi qi − 1|n − 1 deoarece 6, 10, 12, 100 sunt divizori ai lui n − 1.

10. Se arată că p > 2. Dacă pq este pseudoprim, se arată imediat că pq
divide pe 2p − 2 şi 2q − 2 şi deci Mp ≡ 1 (mod pq), Mq ≡ 1 (mod pq) şi apoi
Mp Mq ≡ 1 (mod pq). Avem (2p − 1, 2q − 1) = (2(p,q) − 1) = 1. Cum (Mp , Mq ) = 1,
Mp |2pq − 1 şi Mq |2pq − 1, rezultă că Mp Mq |2pq − 1|2Mp Mq −1 − 1|2Mp Mq − 2.

n n n n
11. Avem p−1|n−1 ⇔ p−1 p−1 ⇔ p−1 (p−1)+ −1 ⇔ p−1 −1.
p p p p

12. Pentru fiecare a, (a, k) = 1, 1 ≤ a < k, notăm ia ordinul lui a mod


Y
k şi i = ia şi deci pentru orice (a, k) = 1 avem ai ≡ 1 (mod k). Fie p
a
prim, p > k, p ≡ 1 (mod i). Alegem n = pk. Faptul că există o infinitate de
numere prime p cu proprietatea de mai sus, rezultă din Teorema lui Dirichlet
privitoare la numere prime din progresii aritmetice.

240
m(2n + m + 1)
13. tn+m −tn = = p. Rezultă imediat că m = 1, n = p−1
2
p−3
sau m = 2, n = , p fiind număr prim (impar ı̂n al doilea caz).
2
9k − 1 3k − 1
14. M = 9k−1 +9k−2 +· · ·+9+1 = şi notăm n = ; M = tn .
8 2
15. M = 2 · 10n (10n−1 + 10n−2 + · · · + 1) + (10n−1 + 10n−2 + · · · + 1) =
(10n − 1)(2 · 10n + 1) 2(10n − 1)
= tk , unde k = .
9 3
n m(m + 1) n
16. 22 + 1 = ⇔ 22 +1 = (m−1)(m+2) ⇔ m − 1 = 1 şi n = 0.
2
17. Fie z = tn + 1, x = ttn +1 , y = ttm .
18. k = tm + tn + tp ⇔ 8k + 3 = (2m + 1)2 + (2n + 1)2 + (2p + 1)2 şi se
aplică teorema lui Gauss relativă la scrierea numerelor ca sumă a trei pătrate
(monografie, pagina 185).
n(n + 1)
19. Notăm tn = . Arătăm că dacă tn = a2 , atunci t4n2 +4n = b2 .
2
Ţinând cont că t1 = 1, rezultă imediat enunţul. Mai trebuie demonstrată
afirmaţia precedentă.
(4n2 + 4n)(4n2 + 4n + 1) n(n + 1)
t4n2 +4n = =4 (2n + 1)2 = (2a(2n + 1))2 .
2 2
x(x + 1)
20. Alegem n = 4t + 3, t ∈ N. Să presupunem că 24t+3 − 1 = ,
2
x ∈ N. Atunci (2x + 1)2 = 24t+6 − 7 ≡ 26 − 7 = 57 ≡ 7 (mod 10) şi am obţinut
o contradicţie.
(Adriana Bulacu)

21. Alegem n = 10k + 86, k ∈ N, 11¤|¤k. Să presupunem că 2n − 1 =


x(x + 1) y(y + 1)
+ , x, y ∈ N. Atunci 2n+2 − 3 = 2x2 + 2x + 2y 2 + 2y + 1 =
2 2
(x + y + 1)2 + (x − y)2 . Avem ı̂nsă 2n+2 − 3 ≡ 288 − 3 ≡ 28 − 3 = 253 ≡
0 (mod 11) şi 112 ¤|¤2n+2 − 3, ceea ce ne conduce la o contradicţie, căci 2n+2 − 3
se scrie ca sumă de două pătrate şi exponentul lui 11 ı̂n descompunerea lui
2n+2 − 3 este 1. Să presupunem că 112 |2n+2 − 3. Atunci 210k+88 ≡ 3(121),
210k ≡ 1(121) (deoarece 288 ≡ 3 (mod 21) şi 110|10k (deoarece ordinul lui 2 ı̂n
U (Z121 , ·) este egal cu 110). Obţinem contradicţia 11|k.
(Adriana Bulacu)

241
n
22. Să presupunem că n ∈ N∗ . Avem că 4(22 + 1) + 1 = 2x2 + 2x + 2y 2 +
n n n
2y +1 = (x+y +1)2 +(x−y)2 . Dar 4(22 +1) ≡ 22 +2 ≡ (−1)2 +2 ≡ 1+2 ≡
n n
0(3). Dacă 4(22 + 1) + 1 ≡ 0 (mod 9), ar rezulta că 4(22 − 1) ≡ 0 (mod 9),
n
22 ≡ 1 (mod 9) şi contradicţia 6|2n (ultima relaţie de divizibilitate se deduce
din faptul că ordinul lui 2 ı̂n U (Z9 , ·) este 6). Am ajuns ı̂n acest moment la o
contradicţie, căci 4Fn +1 se scrie ca sumă de două pătrate, 3|4Fn +1, 9¤|¤4Fn +1.
Deci n = 0 şi {x, y} = {0, 2}.
(Adriana Bulacu)

23. 1 = u1 , 2 = u3 , 3 = u2 + u3 . Presupunem proprietatea adevărată


pentru k ≤ n − 1. Fie um ≤ n < um+1 ⇒ n = um + k, unde k < um+1 − um =
um−1 ≤ n − 1, şi deci k = ui1 + ui2 + · · · + uih cu i1 < i2 < · · · < ih < m.
√ √ ∞
1 n n
1+ 5 1− 5 X un
24. un = √ (a − b ), a = , b = . =
5 2 2 n=0
10n+1
‡ ‘

1 X ∞  a n 1 X ∞  b n 1 1 1 1
√ − √ = √ − = .
10 5 n=0 10 10 5 n=0 10 10 5 a b 89
1− 1−
10 10
1
25. Relaţiile se demonstrează folosind formula un = √ (an − bn ),
√ √ 5
1+ 5 1− 5
a = , b = şi pentru punctele a) şi d) se utilizează un
2 2
raţionament prin inducţie.

26. Dacă pi desemnează al i-lea număr prim, fie x0 ∈ N astfel ı̂ncât


x0 ≡ −j (mod p2j ), ∀ j = 1, n; x0 există conform Lemei Chineze a resturilor.
Notăm xk = x0 + kp21 p22 . . . p2n , ∀ k ∈ N. Avem că xk + j ≡ 0(p2j ), ∀ k ∈ N şi
∀ j = 1, n. Deci xk + 1, xk + 2, . . . , xk + n sunt n numere naturale consecutive
care nu sunt libere de pătrate ∀ k ∈ N. Cum lim xk = ∞, din cele de mai sus
k→∞
deducem lim (lm+1 − lm ) ≥ n ∀ n ∈ N şi deci lim (ln+1 − ln ) = ∞.
m→∞ m→∞
(L. Panaitopol )

lm π 2
27. Arătăm ı̂ntâi că lim = , unde lm desemnează al m-lea număr
m→∞ m 6
6 √
liber de pătrate. Aplicăm formula Q(x) = 2 x + O( x) (curs, pagina 69)
π
pentru x = lm (Q(x) reprezintă numărul de numere libere de pătrate mai
6 €√ Š
mici sau egale cu x). Obţinem că m = Q(lm ) = 2 lm + O lm . Împărţind
π

242
lm π2
la lm şi făcând pe m să tindă la infinit, obţinem formula lim = .
m→∞ m 6
Să presupunem că enunţul nu este adevărat. De aici deducem existenţa unui
k ∈ N∗ astfel ı̂ncât lm+2 − lm ≥ 3, ∀ m ≥ k. De aici deducem că lm+6 − lm ≥
(lm+6 − lm+4 ) + (lm+4 − lm+2 ) + (lm+2 − lm ) ≥ 9, ∀ m ≥ k. Să presupunem
că lm+6 − lm = 9. Atunci, lm+6 − lm+4 = lm+4 − lm+2 = lm+2 − lm = 3. Să
presupunem că lm = 4t + r, t ∈ N, r ∈ {1, 2, 3} (r 6= 0 căci lm este liber de
pătrate). Dacă r = 1 ajungem la contradicţia lm+2 = lm + 3 = 4t + 4. Dacă
r = 2 găsim contradicţia lm+4 = lm + 6 = 4t + 8, iar dacă r = 3 ajungem la
contradicţia l
8 m+6
= lm + 9 = 4t + 12. Deci lm+6 − lm ≥ 10 ∀ m ≥ k.
>
>
l k+6 − lk ≥ 10
>
< lk+12 − lk+6 ≥ 10
Avem că . şi deducem, adunând aceste inegalităţi,
>> ..
>
:
lk+6n − lk+6(n−1) ≥ 10
că lk+6n − lk ≥ 10n. Împărţim ultima inegalitate la k + 6n şi-l facem pe
lk+6n π2 π2
n să tindă la infinit. Ţinând cont că lim = , deducem că ≥
n→∞ k + 6n 6 6
10n 10
lim = . Deci rezultă contradicţia π 2 ≥ 10. Enunţul este demonstrat
n→∞ k + 6n 6
ı̂n acest moment.
(L. Panaitopol )

243
CAPITOLUL 13

Teorema lui Dirichlet

1. Există o infinitate de numere prime p = 15k + 7 cu k ∈ N∗ . Avem


.. .
p−2 . 5, p+2 .. 3, p−2 > 5, p+2 > 3 şi deci p−2 = 5((3k+1) şi p+2 = 3(5k+3)
sunt compuse.

2. Fie a = 10n+2 , b = 10n+1 + 1. În progresia aritmetică ak + b, k ∈ N∗


există o infinitate de numere prime deoarece (a, b) = 1. Un astfel de număr
prim are forma p = 10n+2 k0 + 10n+1 + 1 şi are cel puţin n cifre 0.

3. Fie a = 10n+1 şi b = 10n − 1. În progresia aritmetică ak + b, k ∈ N∗


există o infinitate de numere prime care verifică condiţia impusă.

4. Fie a = 1010 , b = 1023456789. Avem (a, b) = 1 şi deci există o infinitate


de numere prime p = an + b. Aceste numere se termină cu 1023456789.

5. Fie a = 2m+1 , b = 2m −1 şi deci există o infinitate de numere prime p =


an + b. Pentru un astfel de număr p = 2m+1 n + 2m − 1 avem p + 1 = 2m (2n + 1)
şi deci p + 1 se divide cu 2m dar nu se divide cu 2m+1 , adică 2m kp + 1.

6. Din teorema lui Dirichlet ştim că există o infinitate de k ∈ N astfel


ı̂ncât 18k + 1 să fie prim. Pentru aceste numere k, avem egalitatea 18k + 1 =
(2k + 14)3 + (3k + 30)3 + (−2k − 23)3 + (−3k − 26)3 şi enunţul este demonstrat.

7. Avem 9(6n − 7) = (2n + 1)3 + (n − 4)3 + 9(−n)3 . Cum (6, 7) = 1,


există o infinitate de numere n pentru care 6n − 7 = p prim şi problema este
rezolvată.
(L. Panaitopol )

8. În identitatea din soluţia problemei precedente, punem n = 3k + 1


şi avem 9(18k − 1) = (6k + 3)3 + (3k − 3)3 + 9(−3k − 1)3 şi deci 18k − 1 =
3(2k + 1)3 + 3(k − 1)3 + (−3k − 1)3 . În progresia (18k − 1)k≥1 există o infinitate
de numere prime p şi avem deci p = 3x3 + 3y 3 + z 3 , unde x = 2k + 1, y = k − 1,
p+1
z = −3k − 1 şi k = .
18
(L. Panaitopol )

245
9. Analizăm ı̂ntâi cazul k impar. Deoarece (8, k + 2) = 1, deducem din
teorema lui Dirichlet că există o infinitate de numere prime p de forma p =
8n + (k + 2), unde n ∈ N. Din teorema lui Gauss 8n + 2 = a2 + b2 + c2 ,
a, b, c ∈ N şi rezultă că p = a2 + p2 + c2 + k. Analizăm acum cazul k = 4h + 2.
Deoarece (8, 4h + 5) = 1, din teorema lui Dirichlet rezultă că există o infinitate
de numere prime p de forma p = 8n + 4h + 5, n ∈ N. Din teorema lui Gauss
deducem că 8n + 3 = a2 + b2 + c2 şi deci p = a2 + b2 + c2 + k. Pentru k = 4h
folosim din nou teorema lui Dirichlet pentru (8, 4h + 1) = 1 şi obţinem o
infinitate de numere prime p = 8n + 4h + 1, n ∈ N. 8n + 1 = a2 + b2 + c2 şi
problema este demonstrată şi ı̂n acest caz.
(L. Panaitopol )

10. Dacă a este impar, numerele impare n nu se reprezintă sub această


formă. Deci a este par. Relaţia se scrie (a+1)n+4a3 = ((a+1)x+2a2 )((a+1)y+2a).
Deoarece a este par, avem (a + 1, 4a3 ) = 1 şi din teorema lui Dirichlet există
n0 pentru care (a + 1)n0 + 4a3 este prim. În acest caz (a + 1)x + 2a2 = ±1 sau
− 2a2 ± 1 − 2a ± 1 2a2 − 1
(a + 1)y + 2a = ±1, adică x = sau y = şi deci ∈Z
a+1 a+1 a+1
2a2 + 1 2a + 1 2a − 1 1
sau ∈ Z, sau ∈ Z, sau ∈ Z, adică 2(a − 1) + ∈ Z,
a+1 a+1 a+1 a+1
3 1 3
sau 2(a−1)+ ∈ Z, sau 2− ∈ Z, sau 2− ∈ Z, adică a+1 = ±1,
a+1 a+1 a+1
sau a + 1 = ±3, adică a ∈ {0, −2, 2, −4}.
Pentru a = 0 avem n = xy şi alegem x = n şi y = 1. Pentru a = −2,
n = −xy − 4x + 8y şi alegem x = −n − 24, y = −3. Pentru a = 2 avem
n = 3xy + 4x + 8y şi luăm x = n + 8, y = −1, iar pentru a = −4 avem
n = −3xy − 8x + 32y şi luăm x = n + 96 şi y = −3. Un enunţ mai general se
găseşte ı̂n curs la pagina 159.

11. Există o infinitate de numere prime de forma p = 144k + 13, deoarece


(144, 13) = 1. Aceste numere verifică condiţia p 6= 3x − 2y .
.
Presupunem p = 3x − 2y . Deoarece p ≥ 13, rezultă x ≥ 3 şi deci 3x .. 9.
Rezultă −2y ≡ 13 (mod 9) ⇔ −2y ≡ 4 (mod 9) ⇔ 2y−2 ≡ −1 (mod 9) şi de
aici rezultă y − 2 = 3(2a + 1), adică y = 6a + 5 şi deci y ≥ 5. Rezultă de
.
aici 2y .. 16 şi deci 3x ≡ 13 (mod 16), adică 3x ≡ −3 (mod 16) şi deci 3x−1 ≡ −1
(mod 16). Avem 31 ≡ 3 (mod 16), 32 ≡ 9 (mod 16), 33 ≡ 11 (mod 16) şi 34 ≡ 1
(mod 16). Aceste resturi se repetă şi deci 3x−1 6≡ −1 (mod 16); contradicţie.

246
12. Dacă n este impar, alegem p astfel ı̂ncât n − p ≡ 0 (mod 4) şi deci
n − p = 4l = (l + 1)2 − (l − 1)2 . Dacă n este par, considerăm p > 2 şi deci n − p
este impar şi avem n − p = 4l + 1 = (2l + 1)2 − (2l)2 sau n − p = 4l − 1 =
(2l)2 − (2l − 1)2 . Alegerile sunt posibile deoarece există o infinitate de numere
prime de forma 4k + 1 şi o infinitate de numere prime de forma 4k − 1.
(L. Panaitopol )
a+n b+n a+n
13. Determinăm n ∈ Z pentru care + ∈ Z, adică +1+
b a b
 
b+n 1 1
+ 1 ∈ Z şi deci E = (a + b + n) + ∈ Z. Fie a = dα, b = dβ. Rezultă
a a b
1 = (α, β) şi apoi (αβ, α + β) = 1 (1).
(a + b + n)(α + β)
Avem E = ∈ Z. Fie d1 = (d, α + β). Rezultă α + β =
dαβ
(α + β) (n + a + b)s
d1 s, d = d1 t, (s, t) = 1. Avem E = (n + a + b) = .
dαβ tαβ
Din (1) rezultă (s, αβ) = 1 şi cum (s, t) = 1, rezultă (s, tαβ) = 1, adică
tαβ|n + a + b. Aşadar n + a + b = tαβk şi deci n = t(kαβ − d1 (α + β)). Cum
t|n şi kαβ − d1 (α + β) = 1, pentru cel mult o valoare a lui k este necesar ca
t = 1, adică d1 = d ⇔ d|α + β ⇔ d2 |a + b. Reciproc, pentru d1 = d avem t = 1,
d|α + β şi n = kαβ − d1 (α + β). Avem (αβ, d1 (α + β)) = (αβ, d1 ) = (αβ, d).
Dacă există q prim cu q|αβ şi q|d, rezultă de exemplu q|α. Cum q|d|α + β,
rezultă contradicţia q|β şi deci (αβ, d1 (α + β)) = 1 şi din teorema lui Dirichlet
a progresiilor aritmetice, ı̂n şirul xk = αβk − d1 (α + β) există o infinitate de
numere prime.
(L. Panaitopol )

14. Din teorema lui Dirichlet ştim că există o infinitate de numere prime
p de forma 8k + 7. Să presupunem că pentru un astfel de număr prim p există
m, n ∈ N, astfel ı̂ncât p|2m + n2 . Evident că p¤|¤n şi n2 ≡ −2m (mod p). De aici
 2     
n − 2m −1 2 m
obţinem o contradicţie, căci 1 = = = = (−1)1m = −1.
p p p p
(L. Panaitopol )

15. Presupunem că există un număr finit de astfel de numere prime şi
notăm cu m cel mai mare prim de această formă. Fie a = 5 · 13 · 29 · 37 · . . . · m
şi n = a2 + 22 . Cum (a, 2) = 1 pentru p număr prim, p|n, rezultă p = 4h + 1.
Dacă toţi factorii lui n sunt de forma 8l + 1, atunci n = 8t + 1. Însă a este
impar şi deci a2 = 8s + 1 şi de aici contradicţia n = a2 + 4 = 8s + 5. Aşadar
n are cel puţin un factor prim de forma 8k + 5 şi 8k + 5 > m; contradicţie.

247
16. Raţionăm prin inducţie după s. Pentru s = 1 proprietatea este
adevărată, conform teoremei lui Dirichlet a progresiilor aritmetice. Din ipoteza
de inducţie, există un termen al progresiei de forma ak0 + b = q1 q2 . . . qs−1 , qj
prim, ∀ j = 1, s − 1, q1 < q2 < · · · < qs−1 . Din teorema lui Dirichlet, există
o infinitate de numere prime ı̂n progresia (an + 1)n≥1 . Fie q = ak + 1 un
astfel de număr cu q > qs−1 . Deoarece q1 q2 . . . qs−1 q = (ak0 + b)(ak + 1) =
a(ak0 k + bk + k0 ) + b, rezultă că q1 q2 . . . qs−1 q face parte din progresia (xn )n≥1 .
În acest mod se face trecerea de la s − 1 la s. Cum prin ipoteza de inducţie
există o infinitate de numere cu s − 1 factori primi distincţi, rezultă că propri-
etatea este adevărată şi pentru s.

17. Din teorema lui Dirichlet există o infinitate de numere prime p =


m−1
6m n + 2 · 32 − 1. Avem p + 1 = 3m k (deoarece 2m−1 ≥ m) şi deci p + 1 are
€ m−1 Š
factorii 1, 3, 32 , . . . , 3m . De asemenea, p−1 = 6m n+2 32 − 1 . Din teorema
m
lui Euler rezultă (folosind că (2m , 3) = 1) că, 3ϕ(2 ) ≡ 1 (mod 2m ). Cum
 
m m
1 m−1
ϕ(2 ) = 2 1 − = 2m−1 , rezultă 32 ≡ 1 (mod 2m ) şi deci p − 1 = 2m h.
2
Cum p − 1 are factorii 1, 2, 22 , . . . , 2m , demonstraţia este ı̂ncheiată.

18. Cazul m = 1 este evident. Fie m ≥ 2. Din teorema lui Dirichlet


există o infinitate de numere prime p de forma p = km + 1. Fie n = pm−1 .
Avem σ(n) = 1 + p + p2 + · · · + pm−1 . Cum p ≡ 1 (mod m), rezultă σ(n) ≡
.
1 + 1 + · · · + 1 ≡ 0 (mod m). ϕ(n) = pm−2 (p − 1) = pm−2 km .. m şi τ (n) =
| {z }
m
m − 1 + 1 = m.

19. Fie q1 = 3, q2 = 5, q3 = 7, . . . , qn , . . . şirul numerelor prime impare.


Din Lema Chineză a resturilor (curs, pagina 99), deducem existenţa numărului
natural b, astfel ı̂ncât

b ≡ 1 (mod q1 q2 . . . qn )
b ≡ −1 (mod qn+1 qn+2 . . . q2n )
b ≡ −2 (mod q2n+1 q2n+2 . . . q3n ).

Deoarece (b, q1 q2 . . . q3n ) = 1, din teorema lui Dirichlet deducem că există
k ∈ N astfel ı̂ncât p = q1 . . . q3n · k + b să fie număr prim. p − 1 se divide cu
q1 q2 . . . qn , p + 1 se divide cu qn+1 qn+2 . . . q2n şi p + 2 cu q2n+1 q2n+2 . . . q3n .

248
20. Fie m = p1 ·p2 ·. . .·pr , unde p1 , p2 , . . . , pr sunt numere prime distincte
impare. Din Lema Chineză a resturilor ştim că există n ∈ N astfel ı̂ncât n ≡ −2
(mod pj ) ∀ j = 1, r şi n ≡ 1 (mod 4). Evident că (n, 4m) = 1.
Din teorema lui Dirichlet rezultă că există o infinitate de numere prime
p = n + t · 4m (t ∈ N). Evident că p ≡ n ≡ −2 (mod pj ), ∀ j = 1, r, p ≡ n ≡
    r  − 2p
− 2p 4 Y
1 (mod 4). Avem că = = 1, j = 1, r şi deci 1 = =
pj pj j=1
pj
  r   r      
−2 Y p Y pj m −m
= = = (am folosit mai sus că m ≡
m j=1 pj j=1
p p p
3 (mod 8) şi că p ≡ 1 (mod 4), precum şi proprietăţile simbolurilor Legendre
şi Jacobi).

21. Fie k = q1a1 q2a2 . . . qrar descompunerea standard ı̂n factori primi a
lui k. Există, conform teoremei lui Dirichlet, o infinitate de numere prime
p = (q1 − 1)(q2 − 1) . . . (qr − 1)t + 1 cu p > k. Alegem n = kp. Evident că
n este compus. Avem n − k = k(p − 1) = tq1 q2 . . . qr ϕ(k). Deoarece (a, n) =
1, deducem că p¤|¤a, ap−1 ≡ 1 (mod p) şi an−k = (ap−1 )k ≡ 1 (mod p). De
asemenea, (a, k) = 1, aϕ(k) ≡ 1 (mod k) şi an−k = (aϕ(k) )tq1 q2 ...qr ≡ 1 (mod k).
Cum (p, k) = 1 (deoarece p este prim, p > k), rezultă că an−k ≡ 1 (mod kp).

a
22. Fie a, b ∈ N∗ , (a, b) = 1, astfel ı̂ncât r = b ≥ 2 deoarece r 6∈ Z.
b
Există x0 , y0 ∈ N astfel ı̂ncât ax0 = by0 + 1, 0 ≤ x0 < b, 0 ≤ y0 < a. Există,
de asemenea, o infinitate de numere m ∈ N astfel ı̂ncât y0 + am să fie prim (se
aplică teorema lui Dirichlet deoarece (y0 , a) = 1). Alegem n = x0 + bm. Avem
an ax0 + abm ax0 1
că rn = = = am + = am + y0 + şi deci [rn] = am + y0
b b b b
(deoarece b ≥ 2). Enunţul este demonstrat.

249
CAPITOLUL 14

Reprezentări aditive

1. Pentru n impar, n ≥ 5, alegem a = 2, b = n − 2. Pentru n = 4k, k ≥ 2,


alegem a = 2k + 1, b = 2k − 1, iar pentru n = 4k + 2, k ≥ 2, alegem a = 2k + 3,
b = 2k − 1.

2. Facem următoarele alegeri:


dacă n = 6k, k ≥ 2, a = 2, b = 3, c = 6k − 5;
dacă n = 6k + 2, k ≥ 2, a = 4, b = 3, c = 6k − 5;
dacă n = 6k + 4, k ≥ 1, a = 2, b = 3, c = 6k − 1;
dacă n = 12k + 1, k ≥ 2, a = 6k − 7, b = 6k − 1, c = 9;
dacă n = 12k + 3, k ≥ 1, a = 6k − 1, b = 6k + 1, c = 3;
dacă n = 12k + 5, k ≥ 2, a = 6k − 5, b = 6k + 1, c = 9;
dacă n = 12k + 7, k ≥ 1, a = 6k + 5, b = 6k − 1, c = 3;
dacă n = 12k + 9, k ≥ 1, a = 6k − 1, b = 6k + 1, c = 9;
dacă n = 12k + 11, k ≥ 1, a = 6k + 1, b = 6k + 7, c = 3.

k+1 k−1
3. Dacă k este impar alegem x = 0, y = ,z= . Dacă k este
2 2
k k
par alegem x = 1, y = , z = − 1.
2 2
4. Dacă a este par, numerele impare n nu se pot scrie sub forma cerută.
Pentru a impar alegem n = a(2k + 1)2 . Deoarece an + 1 = (ax + 1)2 − (ay)2 ,
avem (a(2k + 1))2 + 1 = (ax + 1)2 − (ay)2 . Contradicţia rezultă din faptul că
(a(2k + 1))2 + 1 = M4 + 2 şi (ax + 1)2 − (ay)2 6= M4 + 2.

5. Din teorema lui Lagrange (curs, pagina 123) ştim că există numerele
ı̂ntregi a, b, c, x cu proprietatea că n = x2 + a2 + b2 + c2 . Înlocuind numerele
eventual cu opusele lor şi renotând, putem presupune că 3|a + b + c, 2|a + b.
 2  
2 2 2 2
a+b a−b 2
Folosim identitatea 3(a +b +c ) = (a+b+c) +2 −c +6 , din
2 2
a+b a−b a+b
care deducem că 3 − c. Notăm y = , 3z = a + b + c, 3t = − c.
2 2 2

251
Din identitatea precedentă obţinem egalitatea a2 + b2 + c2 = 3z 2 + 6t2 + 2y 2
şi n = x2 + a2 + b2 + c2 = x2 + 2y 2 + 3z 2 + 6t2 .

6. Dacă n ≥ 2 deducem că 8|a2 + b2 + c2 + d2 şi deci a, b, c, d sunt toate


pare. Pentru n = 0, ecuaţia 7 = a2 + b2 + c2 + d2 are (mai puţin o permutare
a celor patru numere) doar soluţia a = 2, b = c = d = 1. Pentru n = 1,
ecuaţia 28 = a2 + b2 + c2 + d2 are doar soluţiile a = 5, b = c = d = 1 şi
a = 4, b = c = d = 2. Din toate consideraţiile precedente, deducem că soluţiile
problemei pentru n ≥ 1 sunt a = 5 · 2n−1 , b = c = d = 2n−1 şi a = 2n+1 ,
b = c = d = 2n , iar pentru n = 0 avem doar soluţia a = 2, b = c = d = 1.

7. Deoarece p este prim, p ≡ 1 (mod 3), deducem că există x, y ∈ N astfel


ı̂ncât p = x2 + 3y 2 (curs, pagina 128). Avem 2p2 = 2x4 + 12x2 y 2 + 18y 4 =
(x + y)4 + (x − y)4 + (2y)4 şi enunţul este demonstrat (faptul că x + y, x − y,
2y sunt numere nenule este trivial).

8. Punem x + y = 1, s = x + a, t = 2x + b şi avem k = 3x2 − 3x +


1 + x2 + 2ax + a2 − 4x2 − 4xb − b2 = x(2a − 4b − 3) + 1 + a2 − b2 . Luăm
a = 2b + 1 şi obţinem x = 1 + (2b + 1)2 − b2 − k = 3b2 + 4b + 2 − k şi de aici
y = k − 3b2 − 4b − 1, s = 3b2 + 6b + 3 − k, t = 6b2 + 9b + 4 − 2k cu b ∈ Z.
(L. Panaitopol )

9. Alegem x + y = −3 şi avem 2005 = −9x2 − 27x − 27 + s2 + t2 . Punem


s = 3x + a şi de aici 2005 = 3x(2a − 9) + a2 + t2 − 27. Pentru a = 5 avem
2007 − t2
x= . Luăm t = 3c şi avem x = 669 − 3c2 . Rezultă y = −672 + 3c2 ,
3
s = 2012 − 9c2 , t = 3c. Din identitatea 2005 = (669 − 3c2 )3 + (3c2 − 672)3 +
(2012 − 9c2 )2 + (3c)2 rezultă şi existenţa unei infinităţi de scriei a lui 2005 sub
forma x3 + y 3 + s2 + t2 .
(L. Panaitopol )

10. Fie k = 5n + 3(n ∈ Z). Să presupunem că există x, y, z ∈ Z astfel


ı̂ncât k = x4 + y 4 − z 4 . Deoarece m4 ≡ 0, 1 (mod 5), ∀ m ∈ Z, deducem că
x4 + y 4 ≡ 0, 1, 2 (mod 5) şi x4 + y 4 − z 4 ≡ 0, 1, 2, 4 (mod 5). Dar k ≡ 3 (mod 5)
şi am obţinut o contradicţie.

11. Este suficient să facem demonstraţia pentru k ∈ N, pentru că prin
ı̂nmulţirea cu −1 se obţine scrierea pentru −k ∈ Z\N∗ . Primele patru cazuri
se verifică:
0 = 12 + 22 − 32 + 42 − 52 − 62 + 72 ;

252
1 = 12 ;
2 = −12 − 22 − 32 + 42 ;
3 = −12 + 22 .
Deoarece 4 = (n + 1)2 − (n + 2)2 − (n + 3)2 + (n + 4)2 , putem da o
demonstraţie prin inducţie de tipul P (k) → P (k + 4), deoarece primele patru
cazuri au fost verificate şi din k = ±12 ± 22 ± · · · ± n2 obţinem k + 4 =
±12 ± 22 ± · · · ± n2 + (n + 1)2 − (n + 2)2 − (n + 3)2 + (n + 4)2 .
12. Să presupunem că n = ±12 ± 22 · · · ± n2 . Deoarece ±a2 ≡ a (mod 2),
∀ a ∈ Z, deducem că n ≡ 1 + 2 + · · · + n (mod n), 1 + 2 + · · · + (n − 1) ≡
(n − 1)n
0 (mod 2), ≡ 0 (mod 2), 4|n(n − 1). Din ultima relaţie deducem că
2
k−1

n = 4k sau n = 4k + 1. Dacă n = 4k, k ∈ N∗ , atunci n = (4j + 1)2
Š j=0
−(4j + 2)2 − (4j + 3)2 + (4j + 4)2 , iar dacă n = 4k + 1, k ∈ N, atunci
k−1
X€ Š
n = 12 + (4j + 2)2 − (4j + 3)2 − (4j + 4)2 + (4j + 5)2 şi enunţul este
j=0
demonstrat.
(L. Panaitopol )

13. Presupunem că n are proprietatea din enunţ. Deoarece ±a2 ≡


n(n + 1)
a (mod 2), deducem că 1 + 2 + · · · + n = 0 (mod 2), ≡ 0 (mod 2),
2
n(n+1) ≡ 0 (mod 4). Din ultima congruenţă deducem că n = 4k sau n = k+3.
Este evident că numerele 3 şi 4 nu au proprietatea din enunţ. Vom arăta acum
că dacă n = 4k sau n = 4k + 3 şi n ≥ 7, atunci n are proprietatea din
enunţ. Arătăm ı̂ntâi că dacă n are proprietatea din enunţ, atunci şi n + 8 are
aceeaşi proprietate. Într-adevăr, dacă există o alegere a semnelor ± astfel ı̂ncât
0 = ±12 ± 22 · · · ± n2 , atunci 0 = ±12 ± 22 · · · ± n2 + (n + 1)2 − (n + 2)2 −
(n + 3)2 + (n + 4)2 − (n + 5)2 + (n + 6)2 + (n + 7)2 − (n + 8)2 şi afirmaţia
este adevărată pentru n + 8. Trebuie făcute verificările pentru n = 7, 8, 11, 12.
Avem:
12 + 22 − 32 + 42 − 52 − 62 + 72 = 0
− 12 + 22 + 32 − 42 + 52 − 62 − 72 + 82 = 0
12 − 22 + 32 + 42 + 52 − 62 − 72 − 82 + 92 − 102 + 112 = 0
12 − 22 − 32 + 42 − 52 − 62 − 72 + 82 − 92 + 102 − 112 + 122 = 0
şi enunţul este demonstrat ı̂n acest moment.

253
14. Alegem xj = j(j + 1), ∀ j = 1, n − 1, xn = n. Avem că
„ Ž „ Ž
n n−1
X 1 X 1 1 n−1
X 1 1 1
= + = − + = 1.
j=1
xj j=1
j(j + 1) n j=1 j j+1 n

1 1 1
15. Pentru n = 3 avem + + = 1. Presupunem că există numere n > 3
2 3 6
pentru care ecuaţia nu are soluţii ı̂n numere naturale distincte. Fie n0 cel mai
mic dintre acestea. Rezultă n0 ≥ 4 şi soluţia 2 ≤ a1 < a2 < · · · < an0 −1 pentru
1 1 1 1 1 1
ecuaţia + +···+ = 1. Avem deci + +···+ = 1 şi de
x1 x2 xn0 −1 a1 a2 an0 −1
1 1 1
aici soluţia (2, 2a1 , 2a2 , . . . , 2an0 −1 ) pentru ecuaţia + + ··· + = 1;
x1 x2 xn0
contradicţie.

1 1 1
16. Fie n1 ∈ N∗ astfel ı̂ncât ≤r< . Dacă r = , enunţul
n1 + 1 n1 n1 + 1
1 1 1
este demonstrat. Dacă <r< , notăm r1 = r − şi avem 0 <
n1 + 1 n1 n1 + 1
1 1 1 1
r1 < − = < 1. Există n2 ∈ N∗ astfel ı̂ncât ≤ r1 <
n1 n1 + 1 n1 (n1 + 1) n2 + 1
1 1 1
. Avem ≤ r1 < , n2 +1 > n1 (n1 +1), n2 ≥ n1 (n+1) > n1
n2 n2 + 1 n1 (n1 + 1)
1 1 1
şi dacă r1 = , atunci r = + , n2 + 1 > n1 + 1 şi enunţul
n2 + 1 n1 + 1 n2 + 1
este demonstrat.
1
Dacă r1 > , continuăm procedeul de mai sus. Trebuie arătat că
n2 + 1
procedura precedentă se opreşte ı̂ntr-un număr finit de paşi. Să presupunem
că am determinat numerele n1 < n2 < n3 < · · · < nk astfel ı̂ncât rk =
1 1 1 1
r− − −···− satisface inegalităţile 0 < rk < .
n1 + 1 n2 + 1 nk + 1 nk (nk + 1)
1 1
Fie nk+1 ∈ N∗ astfel ı̂ncât ≤ rk < . La fel ca mai sus se
nk+1 + 1 nk+1
1
arată că nk+1 > nk . Dacă rk = , atunci enunţul este demonstrat.
nk+1 + 1
aj 1
Notăm rj = , aj , bj ∈ N∗ , ∀ j = 1, k + 1, unde rk+1 = rk − .
bj nk+1 + 1
ak+1 ak 1 ak (nk+1 + 1) − bk
Avem = − = , ak+1 = ak (nk+1 + 1) − bk ,
bk+1 bk nk+1 + 1 bk (nk+1 + 1)

254
ak 1
bk+1 = bk (nk+1 +1). Avem ak+1 < ak ⇔ ak (nk+1 +1)−bk < ak ⇔ < .
bk nk+1
1
Ultima inegalitate este adevărată deoarece rk < . Avem deci a1 > a2 >
nk+1
· · · > ak > ak+1 ≥ 0. Cum nu există un şir strict descrescător de numere
naturale, rezultă că există un rang la care se ı̂ntâmplă că ak+1 = 0 şi enunţul
este demonstrat.

x y z
17. Punem X = , Y = , Z = cu (x, y, z, a) = 1 şi avem x2 + y 2 +
a a a
z 2 + ax + ay + az = a2 şi deci (2x + a)2 + (2y + a)2 + (2z + a)2 = 7a2 . Scriem
a = 2n α, unde n ∈ N şi α impar. Avem a2 = 4n (M8 + 1) şi 7a2 = 4n (M8 + 7),
ceea ce constituie o contradicţie, căci 7a2 se scrie ca sumă de trei pătrate.

a b
18. Fie x, y, z ∈ Q astfel ı̂ncât n = x2 + y 2 + z 2 . Notăm x = ,y= ,
d d
c
z = , (a, b, c, d) = 1, d ∈ N∗ , a, b, c ∈ N şi avem că nd2 = a2 + b2 + c2 . Dacă
d
n nu se scrie ca sumă a trei pătrate, atunci din teorema lui Gauss ştim că n =
4k (8l+7), k, l ∈ N. Dacă d = 2t ·s, t, s ∈ N, s impar, atunci nd2 = 4k+t s2 (8l+7).
Avem s2 ≡ 1 (mod 8) şi deci s2 (8l + 7) = 8f + 7, f ∈ N, nd2 = 4k+1 (8f + 7).
De aici deducem că nd2 nu se scrie ca sumă de trei pătrate, fapt ce contrazice
identitatea nd2 = a2 + b2 + c2 .

19. Să presupunem că a · b = x2 + y 2 + z 2 , unde x, y, z ∈ Z. Atunci


 2  2  2
a a·b x y z a
= 2 = + + . Să presupunem acum că = r12 + r22 + r32 ,
b b b b b b
a 2
r1 , r2 , r3 ∈ Q. Atunci a·b = b = (br1 )2 +(br2 )2 +(br3 )2 şi aplicând problema
b
18, deducem că ab se scrie ca sumă a trei pătrate de numere naturale.
(L. Panaitopol )

20. Alegem n = 8k + a(7 − b), unde k ∈ N este suficient de mare pentru


ca n să fie natural. Deoarece a este impar, avem că a2 ≡ 1(8) şi an + b ≡
a2 (7 − b) + b ≡ 7 − b + b ≡ 7(8) şi deci an + b 6= x2 + y 2 + z 2 , ∀ x, y, z ∈ Z.

21. Alegem n = 128k + 111, k ∈ N. Avem n ≡ 7 (mod 8) şi deci


n 6= a2 + b2 + c2 , ∀ a, b, c ∈ Z. Avem n + 1 = 16(8k + 7) = 42 (8k + 7) şi
deci n + 1 6= a2 + b2 + c2 , ∀ a, b, c ∈ Z.

255
22. Din cele trei numere consecutive n, n + 1, n + 2, unul este de forma
4k +1 sau 4k +2. Cum 4k +1 şi 4k +2 se scriu ca sumă de trei pătrate (conform
teoremei lui Gauss), enunţul este demonstrat.

23. Dacă n = x2 +y 2 +z 2 , atunci 4n = a2 +b2 +c2 +d2 , unde a = x+y +z,


b = x−y−z, c = x+y−z, d = x−y+z, a+b = c+d = 2x. Să presupunem acum
că 4n = a2 + b2 + c2 + d2 , a, b, c, d fiind numere ı̂ntregi pentru care a + b = c + d.
Avem d = a+b−c şi 4n = a2 +b2 +c2 +(a+b−c)2 = (a+b)2 +(a−c)2 +(b−c)2 .
Din ultima egalitate deducem că a + b, a − c, b − c sunt toate pare (raţionând
     
a+b 2 a−c 2 b−c 2
modulo 4) şi deci n = + + .
2 2 2
(L. Panaitopol )

24. n = 2k + 1, k ∈ N, 2n = 4k + 2. Din teorema lui Gauss a celor trei


pătrate deducem că ∃ x, y, z ∈ Z astfel ı̂ncât 4k + 2 = x2 + y 2 + z 2 . Rezultă că
unul din numere este par şi celelalte două impare; să presupunem că z = 2c, x
şi y fiind impare. Atunci x + y = 2a, x − y = 2b, a, b ∈ Z şi x = a + b, y = a − b.
Avem 4k + 2 = x2 + y 2 + z 2 = (a + b)2 + (a − b)2 + 4c2 = 2a2 + 2b2 + 4c2 şi
n = 2k + 1 = a2 + b2 + 2c2 .

25. n = 2k + 1, k ∈ N. Avem 4k + 1 = x2 + y 2 + z 2 conform teoremei


lui Gauss, (x, y, z ∈ Z). Raţionând modulo 4, deducem că două numere sunt
pare şi al treilea impar; să presupunem că x = 2a, y = 2b, z = 2c + 1.
Atunci 4k + 1 = 4a2 + 4b2 + 4c2 + 4c + 1 = 4a2 + 4b2 + 2c2 + 2(c + 1)2 − 1,
4k + 2 = 4a2 + 4b2 + 2c2 + 2(c + 1)2 , n = 2k + 1 = 2a2 + 2b2 + c2 + (c + 1)2 =
(a + b)2 + (a − b)2 + c2 + (c + 1)2 şi enunţul este demonstrat.

26. Fie k numărul de cifre ale lui n. Dacă k = 1, atunci n = 7, iar


dacă k = 2, atunci n = 55 (se verifică printr-un calcul rapid). Presupunem
ı̂n continuare că k ≥ 3. Dacă a = 2 sau a = 6, atunci n = 4h + 2 se scrie
ca sumă de trei pătrate. Dacă a = 8, atunci n = 4 · 22 . . . 2 şi cum 2 . . . 2 se
scrie ca sumă de trei pătrate, acelaşi rezultat e valabil şi pentru n. Pentru
a = 3 avem n ≡ 333 ≡ 5 (mod 8), pentru a = 5 avem n ≡ 555 ≡ 3 (mod 8),
iar pentru a = 7 avem n ≡ 777 ≡ 1 (mod 8) şi teorema lui Gauss ne asigură
că n se scrie ca sumă de trei pătrate. Pentru a = 1 sau a = 9 avem că
n ≡ 7 (mod 8) şi deci n 6= b2 + c2 + d2 , ∀ b, c, d ∈ Z. Pentru a = 4 avem că
n = 4 · 11 . . . 1 = 4(8k + 111) = 4(8(k + 13) + 7) şi deci n nu se scrie ca sumă
de trei pătrate.

256
Deci soluţiile problemei sunt numerele 7, 55 şi a . . . a, unde a = 1, 4, 9 şi
ultimul număr are cel puţin trei cifre.
(L. Panaitopol )

27. Arătăm că numerele căutate sunt 3, 7, 9, 15 şi 2m , m ∈ N∗ . Verificarea


pentru n = 3, 7, 9, 15 este trivială. Să presupunem că numărul n = 2m , m ∈ N∗ ,
n
nu are proprietatea din enunţ. Există atunci 1 ≤ a ≤ = 2m−1 , a = b2 +c2 +d2
2
şi n + a = 4i (8t + 7). Scriem a = 2s · l, l impar. Deoarece a ≤ 2m−1 , rezultă
s ≤ m − 1. Din 2m + 2s · l = 22i (8t + 7) deducem că s = 2i şi 2m−2i + l = 8t + 7.
Dacă m − 2i ≥ 3, atunci l ≡ 7 (mod 8) şi obţinem contradicţie, căci a = 4i · l,
l ≡ 7 (mod 8) implică faptul că a 6= b2 + c2 + d2 . Dacă m − 2i = 2, atunci
l ≡ 3 (mod 8), l ≥ 3 şi 2a = 22i+1 · l ≥ 6 · 22i > 22i+2 = n, ceea ce constituie
o contradicţie. Dacă m − 2i = 1, atunci l ≡ 5 (mod 8), l ≥ 5 şi contradicţia
2a = 22i+1 · l ≥ 10 · 22i > 22i+1 = n, iar egalitatea m − 2i = 0 este imposibilă
căci 2i = s ≤ m − 1. Fie acum n = 4i (8k + α), α ∈ {1, 2, 3, 5, 6}, n 6= 3, 9, 2m
şi arătăm că n nu are proprietatea din enunţ. Alegem a = 4i (7 − α) ≥ 1,
n
a = b2 +c2 +d2 şi n+a = 4i (8k+7) 6= x2 +y 2 +z 2 . Mai trebuie verificat că a ≤ .
2
n 8k + α
Dacă a > , rezultă că 7 − α > , 14 > 8k + 3α. Pentru k ≥ 2 ultima
2 2
egalitate este imposibilă. Pentru k = 1 ea poate avea loc doar dacă α = 1. În
acest caz n = 4i · 9, i ≥ 1 (deoarece n 6= 9) şi alegem a = 4i−1 · 11. Evident că
n
1 ≤ a ≤ , a = b2 + c2 + d2 şi n + a = 4i−1 · 47 6= x2 + y 2 + z 2 . Dacă k = 0,
2
inegalitatea 14 > 8k + 3α este ı̂ndeplinită doar pentru α = 1, 2, 3. Dacă α = 1
sau α = 2, atunci obţinem că n = 22i sau n = 22i+1 , ceea ce este imposibil.
Pentru k = 0, α = 3, avem n = 4i · 3, i ≥ 1 (căci n 6= 3) şi alegem a = 4i−1 · 3.
n
Evident că 1 ≤ a ≤ , a = b2 + c2 + d2 , n + a = 4i−1 · 15 6= x2 + y 2 + z 2 .
2
n
Pentru n = 4i (8k + 7), i ≥ 1, alegem a = 3 · 4i−1 = b2 + c2 + d2 , 1 ≤ a ≤ şi
2
n + a = 4i−1 [8(4k + 3) + 7] 6= x2 + y 2 + z 2 . Pentru n = 8k + 7, n ≥ 23, alegem
n
a = 8 = 22 + 22 + 02 , 1 ≤ a ≤ şi n + a = 8(k + 1) + 7 6= x2 + y 2 + z 2 .
2
Am demonstrat ı̂n acest moment că 3, 7, 9, 15, 2m (m ∈ N∗ ) sunt singurele
numere cu proprietatea din enunţ.
(L. Panaitopol )

257
28. Arătăm că A are caracteristica zero, ceea ce ar demonstra enunţul.
Să presupunem că A nu are caracteristica zero.
Notăm cu n ∈ N∗ caracteristica lui A. Deoarece 1 6= 0, avem că n ≥ 2. Să
presupunem că n 6= 4k (8l + 7) ∀ k, l ∈ N. Atunci există a, b, c ∈ N astfel ı̂ncât
n = a2 +b2 +c2 . Trecând acum ı̂n inelul A, avem 0 = a2 +b2 +c2 şi a = b = c = 0
(ı̂n A) datorită proprietăţii din enunţ. Deducem că n|a, n|b, n|c (ı̂n N). Nu
putem avea a = b = c = 0 ı̂n Z. Să presupunem că a 6= 0. Cum n|a, a ∈ N∗ ,
rezultă că a ≥ n şi obţinem contradicţia n = a2 + b2 + c2 ≥ a2 ≥ n2 (avem că
n2 > n deoarece n ≥ 2). Dacă există k, l ∈ N astfel ı̂ncât n = 4k (8l + 7), atunci
2n = 4k (8(2l + 1) + 6) şi deducem din teorema lui Gauss că există a, b, c ∈ N
astfel ı̂ncât 2n = a2 + b2 + c2 . Considerând această egalitate ı̂n inelul A,
obţinem 0 = a2 + b2 + c2 şi a = b = c = 0 (ı̂n A), conform proprietăţilor
inelului A. Deci n|a, n|b, n|c. Numerele naturale a, b, c nu pot fi toate zero;
să presupunem că a 6= 0. Cum n|a şi a ∈ N∗ , deducem că a ≥ n. Obţinem
că 2n = a2 + b2 + c2 ≥ a2 ≥ n2 , 2 ≥ n. Din această ultimă inegalitate şi din
n ≥ 2, deducem că n = 2. Am obţinut o contradicţie căci 2 6= 4k (8l + 7).
(L. Panaitopol )

29. Vom arăta că rezultatul este adevărat pentru n0 = 12 + (1 + 1 · 58)2 +


(1 + 2 · 58)2 + (1 + 3 · 58)2 + · · · + (1 + 114 · 58)2 .
Primul pas. 116q are proprietatea din enunţ ∀ q ∈ N. Pentru q = 0 enunţul
este evident; putem presupune deci că q ∈ N∗ . Folosind scrierea ı̂n baza 2 a lui
q, deducem existenţa lui r şi s astfel ı̂ncât q = r + 2s, unde r = Σa2 şi s = Σb2 ,
pătratele din scrierea lui r (respectiv s) fiind distincte (q = 2at + 2at−1 + · · · +
X X
2a1 + 2a0 , 0 ≤ a0 < a1 < a2 < · · · < at , r = 2aj , s = 2aj −1 ).
aj par aj impar
116q = 116r + 232s = Σ(10a)2 Σ(4a)2
+ +Σ(14b)2 Σ(6b)2 . Ţinând
+ cont că
numerele a (respectiv b) sunt distincte (şi puteri ale lui 2), rezultă imediat că
pătratele din scrierea lui 116q sunt distincte.
Pasul doi. Fie n ∈ N, n ≥ n0 şi r restul ı̂mpărţirii lui n la 116. Avem că
r ≡ 12 + (1 + 58)2 + (1 + 2 · 58)2 + · · · + (1 + (r − 1)58)2 (mod 116) şi deci
n = 116q +12 +(1+58)2 +(1+2·58)2 +· · ·+(1+(r−1)58)2 , unde q ∈ N. Faptul
că q ∈ Z este evident, iar apartenenţa lui q la N este o consecinţă a inegalităţii
n ≥ n0 şi a definiţiei lui n0 (n0 = 12 +(1+58)2 +(1+2·58)2 +· · ·+(1+114·58)2 ≥
12 + (1 + 58)2 + · · · + (1 + (r − 1)58)2 ; 0 ≤ r ≤ 115, r − 1 ≤ 114). Conform
primului pas, 116q se scrie ca suma unor pătrate distincte. Aceste pătrate sunt
şi pare. Însă pătratele 12 , (1+58)2 , (1+2·58)2 , . . . , (1+(r−1)58)2 sunt pătrate

258
impare distincte, ceea ce ı̂ncheie demonstraţia.
(R. Todor )

30. a) Arătăm ı̂ntâi că ab − a − b 6= ma + nb, ∀ m, n ∈ N. Presupunem că


∃ m, n ∈ N astfel ı̂ncât ab − a − b = ma + nb. Avem că a(b − 1 − m) = b(1 + n)
şi deci b − 1 − m ≥ 1. Din a(b − 1 − m) = b(1 + n) şi (a, b) = 1, deducem că
b|b − 1 − m şi b − 1 − m ≥ b (deoarece b − 1 − m ∈ N∗ şi este multiplu de b).
Ultima inegalitate constituie o contradiţie căci m ∈ N. Pentru a demonstra
complet punctul a), mai trebuie să demonstrăm că ∀ x ∈ N, x ≥ ab − a − b + 1
se scrie sub forma x = ma + nb, cu m, n ∈ N. Deoarece (a, b) = 1, ∃ m0 , n0 ∈ Z
astfel ı̂ncât m0 a + n0 b = x. Atunci, pentru ∀ t ∈ Z, numerele m = m0 + bt
şi n = n0 − at satisfac ecuaţia x = ma + nb. Pot să-l aleg pe t astfel ı̂ncât
0 ≤ m ≤ b − 1. Trebuie arătat că n ≥ 0. Să presupunem că n ≤ −1. Atunci
x = am + bn ≤ a(b − 1) − b = ab − a − b şi am obţinut o contradicţie. Deci
n ≥ 0 şi punctul a) este demonstrat.
b) Fie A = {x ∈ N|x ≤ ab − a − b, x = ma + nb, m, n ∈ N}, B = {x ∈ N|x
≤ ab − a − b, x 6= ma + nb, m, n ∈ N}. Din observaţiile anterioare ∀ x ∈ Z se
scrie unic sub forma x = am + nb, unde 0 ≤ m ≤ b − 1. Se defineşte funcţia
f : Z → Z, f (x) = ab − a − b − x = a(b − 1 − m) − b(n + 1). Se arată imediat că
|A ∪ B| (a − 1)(b − 1)
f este injectivă, f (A) ⊆ B, f (B) ⊆ A, |A| = |B| = = ,
2 2
ceea ce trebuia demonstrat.
Observaţie. Fie k ∈ N∗ , a1 , . . . , ak ∈ N∗ , (ai , aj ) = 1, ∀ 1 ≤ i < j ≤ k.
Se arată relativ uşor că există un cel mai mic număr natural G(a1 , a2 , . . . , ak )
astfel ı̂ncât ∀ x ∈ N, x ≥ G(a1 , . . . ak ), ∃ n1 , n2 , . . . , nk ∈ N astfel ı̂ncât
k
X
x = ni ai . Exerciţiul de mai sus arată că G(a1 , a2 ) = (a1 − 1)(a2 − 1).
i=1
Determinarea lui G(a1 , . . . , ak ) pentru un k ∈ N∗ arbitrar este ı̂ncă o problemă
deschisă. Ea a fost formulată de Frobenius.

31. Pasul 1. Să presupunem că ∃ k, l, t ∈ N astfel ı̂ncât 2abc−ab−ac−bc =


kab + lac + tbc. Avem că a(2bc − kb − lc − b − c) = bc(t + 1). De aici rezultă
că 2bc − kb − lc − b − c > 0 şi bc|2bc − kb − lc − b − c (deoarece (a, bc) = 1).
Deoarece 2bc − kb − lc − b − c < 2bc, din cele de mai sus rezultă egalitatea
2bc − kb − lc − b − c = bc. De aici b(c − k − 1) = c(l + 1), b|l + 1 şi l + 1 ≥ b.
Obţinem contradicţia bc = b(k + 1) + c(l + 1) ≥ b + cb > bc.
Pasul 2. Trebuie arătat că ∀ x ∈ N, x ≥ 2abc − ab − ac − bc + 1, ∃ k, l, t ∈ N
astfel ı̂ncât x = kab + lac + tbc. Deoarece (a, bc) = 1, ∃ s, t ∈ Z astfel ı̂ncât

259
as + tbc = x. Am observat ı̂n exerciţiul precedent că pe t ı̂l pot alege astfel
ı̂ncât 0 ≤ t ≤ a − 1.
Avem că

x − tbc 2abc − ab − ac − bc + 1 − bc(a − 1)


s= ≥ =
a a
abc − ab − ac + 1 abc − ab − ac
= > = bc − b − c.
a a
Deci s ∈ N, s > bc − b − c. Din exerciţiul precedent (punctul a)) ştim că
∃ k, l ∈ N astfel ı̂ncât s = kb + lc. Înlocuind ı̂n formula lui x, obţinem x =
as + tbc = a(kb + lc) + tbc = kab + lac + tbc. Exerciţiul este demonstrat ı̂n
acest moment.

32. Pentru n = 1 şi n = 2 enunţul este evident. Pentru n ≥ 3 putem să-l


scriem pe n = 8k + α, k ∈ N, α ∈ N, 3 ≤ α ≤ 10. Avem n = 8k + 3 + (α − 3).
Din teorema lui Gauss, 8k + 3 se scrie ca sumă a trei pătrate impare. Cum
0 ≤ α − 3 ≤ 7, deducem că n este suma a cel mult zece pătrate impare. Să
j
X
presupunem acum că 72k + 42 = x2j , unde xi sunt numere ı̂ntregi impare
i=1
şi 1 ≤ j ≤ 9. Deoarece x2i ≡ 1 (mod 8) ∀ i = 1, j, rezultă că 2 ≡ 72k + 42 =
j
X
x2j ≡ j (mod 8). Cum 1 ≤ j ≤ 9 şi j ≡ 2 (mod 8), deducem că j = 2 şi
i=1
72k + 42 = x21 + x22 . În ultima egalitate trecem la o congruenţă modulo 9
şi obţinem că x21 + x22 ≡ 6 (mod 9). Această congruenţă este imposibilă căci
a2 ≡ 0, 1, 4, 7 (mod 9) ∀ a ∈ Z şi deci x21 + x22 ≡ 0, 1, 2, 4, 5, 7, 8 (mod 9). Am
obţinut o contradicţie şi enunţul este demonstrat.

33. Fie q1 = 3, q2 = 7, q3 = 11, . . . , şirul infinit al numerelor prime de


forma 4m + 3. Din Lema Chineză a resturilor deducem imediat că ∃ x ∈ Z,
x ≥ 3 astfel ı̂ncât x ≡ qj − (j − 1) (mod qj2 ) ∀ j = 1, k. Numerele x, x + 1, x +
2, . . . , x + k − 1 sunt k numere naturale consecutive care nu se scriu ca sumă
de două pătrate. Fie j ∈ N, 1 ≤ j ≤ k. Avem că x + (j − 1) ≡ qj (mod qj2 ).
Deci qj |x + j − 1, qj2 ¤|¤x + j − 1. Cum qj ≡ 3 (mod 4), rezultă că x + j − 1 nu se
scrie ca sumă de două pătrate, ∀ j = 1, k.
m
34. Fie n = 22 − 1, m ∈ N, m ≥ 3. Să presupunem că n = p + 2a + 2b ,
m
a, b ∈ N, a > b, p prim. Avem 22 = 1 + p + 2a + 2b > 2a şi deci 2m > a.
Avem a − b = 2r · c, r ∈ N, c ∈ N, unde c este impar. Deoarece c este impar,

260
€ r
Šc € r
Š € r
Šc−1 € r
Šc−2 
avem 2a−b + 1 = 22 + 1 = 22 + 1 22 − 22 + · · · + 1 . Din
r r m
2 ≤ 2 c = a − b ≤ a < 2 , rezultă că r < m.
m
Din această ultimă inegalitate şi din identitatea n = 22 − 1 = (2 − 1)(2 +
2 m−1 r
1)(22 +1)(22 +1) . . . (22 +1) deducem că 22 +1|n. Dar 2a +2b = 2b (2a−b +1)
r
este de asemenea multiplu de 22 + 1 şi din egalitatea n = p + 2a + 2b rezultă
r r r
că 22 + 1|p. Cum p este prim şi 22 + 1 > 1, deducem că p = 22 + 1 şi
m r r
22 = 2 + 22 + 2b + 2a . Se arată uşor că 32 nu divide 2 + 22 + 2b + 2a
r
(singurele valori pentru care 16|2 + 22 + 2b + 2a sunt r = 0, b = 2, a = 3
m
şi r = 1, b = 1, a = 3) şi am obţinut o contradicţie, căci 32|22 deoarece
2m ≥ 23 = 8 > 5.
 ∞  ∞  ∞ 
X X X
k 2k 3k
35. Fie f (x) = x x x , funcţie definită pentru
k=0 k=0 k=0

X
∀ x ∈ C, |x| < 1. Evident că f (x) = r(n)xn . Pe de altă parte, avem că
n=0
1 1 1 1 1 17
f (x) = · 2
· 3
= 3
+ 2
+ +
1−x 1−x 1−x 6(1 − x) 4(1 − x) 72(1 − x)
1 1 1
+ + + ,
8(1 + x) 9(1 − ωx) 9(1 − ω 2 x)
2π 2π 1 X∞
unde ω = cos + i sin . Din = xn pentru ∀ x ∈ C, |x| < 1,
3 3 1−x n=0
1 X∞ 1 ∞ (n+1)(n+2)
X
n
deducem imediat că = (n+1)x şi că = xn .
(1 − x)2 n=0 (1−x)3 n=0 2
Din toate observaţiile anterioare deducem că
(n + 1)(n + 2) n + 1 17 (−1)n 2 2nπ
r(n) = + + + + cos =
12 4 72 8 9 3
(n + 3)2 7 (−1)n 2 2nπ
= − + + cos .
12 72 8 9 3
7 (−1)n 2 2nπ 7 1 2 32 4 1
Deoarece − + + cos ≤ + + = = < şi
72 8 9 3 72 8 9 72 9 2
r(n) ∈ N, rezultă din consideraţiile anterioare că r(n) este cel mai apropiat
(n + 3)2
număr natural de .
12

261
CAPITOLUL 15

Ecuaţii algebrice ı̂n numere ı̂ntregi

a + a2 a2 − a
1. Sistemul x − y = a, x + y = a2
are soluţiile x = ,y=
2 2
care sunt numere ı̂ntregi, căci 2|a(a + 1) şi 2|a(a − 1).

2. Să presupunem că există x şi y cu proprietatea din enunţ. Atunci


4x2 +12x+20 = (22y)2 , (2x+3)2 +11 = (22y)2 . Din ultima egalitate deducem
că 11|2x + 3 şi obţinem contradicţia 112 |(22y)2 − (2x + 3)2 = 11. Deci ecuaţia
nu are soluţii.

3. Presupunând că ecuaţia ar avea soluţii nedivizibile cu 3, rezultă


(folosind Mica Teoremă a lui Fermat) că x ≡ y ≡ z ≡ 1 (mod 3) sau
x ≡ y ≡ z ≡ −1 (mod 3). În primul caz rezultă că x3 ≡ y 3 ≡ z 3 ≡ 1 (mod 9)
şi contradicţia 2 ≡ 5 (mod 9), iar ı̂n al doilea caz rezultă că x3 ≡ y 3 ≡ z 3 ≡
−1 (mod 9) şi contradicţia −2 ≡ −5 (mod 9).
x2 +x 2
4. Deoarece x2 +x este par, ecuaţia noastră se scrie şi (x +x+2) = y 2 .
2
 2 
x +x
Dacă d = , x2 + x + 2 , rezultă că d|2 şi d = 1 sau d = 2, ∀ x ∈ Z,
2
x2 + x 2
x2 + x
∈ N, x + x + 2 ∈ N. Dacă d = 1, atunci = k 2 şi x2 + x + 2 = t2
2 2
cu k, t ∈ N. Înmulţind cu 4 a doua ecuaţie, obţinem (2t)2 − (2x + 1)2 = 7,
t = 2, x = 1 sau x = −2. În ambele cazuri y = ±2. Dacă d = 2, atunci
x2 + 2
= 2k 2 , x2 + x + 2 = 2t2 , k, t ∈ N. Înmulţind prima ecuaţie cu 8 obţinem
2
(2x + 1)2 − (4k)2 = 1 şi deci (2x + 1 − 2k)(2x + 1 + 2k) = 1 şi de aici k = 0,
x = 0 sau x = −1, y = 0.

5. Pentru a = 0 şi a = 1 nu există soluţii nenule. Pentru a = 2 avem


soluţia x = y ∈ N∗ . Să presupunem că a ≥ 3. Considerăm ecuaţia de grad 2 ı̂n
x, x2 − ayx + y 2 = 0 cu discriminant ∆ = a2 y 2 − 4y 2 = y 2 (a2 − 4). Deoarece x
este număr ı̂ntreg, deducem că y 2 (a2 − 4) = t2 , t ∈ N∗ (deoarece y 6= 0, a ≥ 3).
Rezultă că t = sy, s ∈ N∗ , a2 − 4 = s2 , (a − s)(a + s) = 4.

263
Cum 1 ≤ a−s < a+s, deducem că a−s = a+s = 2, a = 2, s = 0; contradicţie.
Singura soluţie a ecuaţiei din enunţ este a = 2, x = y ∈ N∗ .

x4 − y 4
6. Fie d = (x, y), x = ad, y = bd, (a, b) = 1. Avem că p = =
x3 + y 3
(x2 + y 2 )(x − y) d(a2 + b2 )(a − b)
= . Să presupunem că există un număr prim
x2 − xy + y 2 a2 − ab + b2
q astfel ı̂ncât q|a2 − ab + b2 şi q|a2 + b2 . Deducem că q|(a, b) = 1, ceea ce
constituie o contradicţie. Deci (a2 − ab + b2 , a2 + b2 ) = 1 şi la fel se arată că
(a2 − ab + b2 , a − b) = 1. Deoarece a2 − ab + b2 |d(a2 + b2 )(a − b), deducem că
d
a2 − ab + b2 |d şi p = 2 (a2 + b2 )(a − b). Cum p este prim, rezultă că
a − ab + b2
d
= a − b = 1, a2 + b2 = p. Deci p = b2 + (b + 1)2 = 2b2 + 2b + 1,
a − ab + b2
2
2p − 1 = (2b + 1)2 . Avem a = b + 1, d = a2 − ab + b2 = (b + 1)2 − b(b + 1) + b2 =
b2 +b+1 şi x = (b+1)(b2 +b+1), y = b(b2 +b+1). Din consideraţiile precedente,
deducem că ecuaţia din enunţ are soluţii dacă şi numai dacă 2p − 1 = k 2 , caz
ı̂n care soluţia este unică.
(L. Panaitopol )

7. Înmulţim cu 4a2 ecuaţia precedentă şi avem că (2a2 n+b)2 +4a2 c−b2 =
(2ak)2 , 4a2 c − b2 = x2 − y 2 , unde x = 2ak, y = 2a2 n + b. Ultima ecuaţie ar
putea avea o infinitate de soluţii ı̂ntregi doar dacă 4a2 c−b2 = 0. Acest fapt din
urmă nu este ı̂nsă posibil, căci ar rezulta că 4a2 |b2 şi 2a|b, ceea ce contrazice
ipoteza.
!
a(n+1) a(n+1)
8. Fie n prim, impar. Din n +b = k 2 , deducem că n +b,
2 2
n|a(n + 1) + 2b, n|a + 2b. Cum n|a + 2b, ∀ n prim, impar, rezultă că a + 2b = 0.
 
n(n + 1) an + a + 2b an2
În acest caz a + bn = n = . Luăm n = 1 şi obţinem
2 2 2
a
că = t2 , a = 2t2 , 2b = −a = −2t2 , b = −t2 . Deci soluţia problemei este dată
2
de numerele a = 2t2 , b = −t2 cu t ∈ Z.

9. Ecuaţia se rescrie x3 = (2y−1)(2y+3). Dacă notăm d = (2y−1, 2y+3),


rezultă că d|2y+3−(2y−1) = 4. Dar d este impar, fiind divizor al unor numere
impare. Deci d = 1 şi 2y − 1 = a3 , 2y + 3 = b3 , a, b ∈ Z. Rezultă că b3 − a3 = 4,
(b − a)(b2 + ab + a2 ) = 4, b − a ∈ N∗ (deoarece b2 +ab+a2 ≥ 0).
Dacă b − a = 1, atunci (a + 1)2 + a(a + 1) + a2 = 4, 3a2 + 3a = 3, a2 + a = 1.

264
Ultima ecuaţie nu are soluţii ı̂ntregi. Dacă b − a = 2, atunci 2 = (a + 2)2 +
a(a + 2) + a2 = 3a2 + 6a + 4, 3(a2 + 2a) = −2; contradicţie. Dacă b − a = 4,
atunci 1 = (a + 4)2 + a(a + 4) + a2 = 3a2 + 12a + 16, a2 + 4a + 5 = 0. Ultima
ecuaţie nu are soluţii ı̂ntregi şi deci ecuaţia iniţială nu are soluţii.

10. Prima ecuaţie se rescrie 5 · 23 = (5x + 8y)2 + y 2 , iar a doua 13 · 23 =


(13x + 17y)2 − 3y 2 . Dar a2 + b2 ≡ 0, 1, 2 (mod 4) şi c2 − 3d2 ≡ c2 + d2 ≡
0, 1, 2 (mod 4). Cum 5 · 23 ≡ 3 (mod 4) şi 13 · 23 ≡ 3 (mod 4), deducem că cele
două ecuaţii nu au soluţii ı̂ntregi.

11. Avem 19(x3 − 100) = 84(y 2 + 1) şi 7|x3 − 100. Rezultă că x3 ≡ 100 ≡
2 (mod 7), ceea ce constituie o contradicţie, căci x3 ≡ 0, ±1 (mod 7).

12. Avem că x2 ≡ 0, 1, 4, 9, 5, 3 (mod 11) şi x2 +4 ≡ 4, 5, 8, 2, 9, 7 (mod 11).


Am obţinut o contradicţie căci x2 + 4 = y 5 ≡ 0, 1, 10 (mod 11). Ecuaţia nu are
soluţii.

13. Avem că x2 ≡ 0, 1, 4, 9, 16, 6, 17, 11, 7, 5 (mod 19). Am obţinut o


contradicţie căci x2 + 5 = y 9 ≡ 0, 1, 18 (mod 19), x2 ≡ 14, 15, 13 (mod 19)
 
9
y
(am folosit criteriul lui Euler: dacă 19¤|¤y, atunci y ≡ = ±1 (mod 19)).
19
14. Ecuaţia se scrie 7(x3 + 4) = 3(4y 2 + 9). Deducem că 7|4y 2 + 9 =
(2y)2 + 32 şi contradicţia 7|2y, 7|3.

15. Notăm s = x + y şi p = xy. Avem s = 3 − z şi s3 − 3sp + z 3 = 3.


Deducem că (3 − z)3 − 3(3 − z)p + z 3 = 3, 3(3 − z)p = 24 − 27z + 9z 2 .
Nu putem avea z = 3, căci ar rezulta din ultima egalitate că 0 = 24.
3z 2 − 9z + 8 8
Deci p = = −3z + . Din ultima egalitate rezultă că
3−z 3−z
3 − z este un divizor al lui 8 şi deci 3 − z ∈ {±1, ±2, ±4, ±8}. Deducem
că z ∈ {−5, −1, 1, 2, 4, 5, 7, 11}. x şi y sunt soluţii ale ecuaţiei de grad 2,
8
t2 − (3 − z)t − 3z + = 0. Determinantul ecuaţiei este ∆ = (3 − z)2 +
3−z
32 32
12z − = (3 + z)2 + . Pentru ca ecuaţia să aibă soluţii ı̂ntregi, este
3−z z−3
2
necesar ca ∆ = s , s ∈ Z. Introducând valorile lui z de mai sus, observăm că
nu convin decât valorile z = −5, 1, 4. Pentru z = −5, rezultă că x = y = 4,
pentru z = 1 avem x = y = 1, iar pentru z = 4 avem (x, y) = (4, −5), (−5, 4).

16. Considerând ecuaţia de grad 2 ı̂n x, x2 + x(3y + 4) + 7y 2 − 9y − 6 = 0,


deducem că discriminantul ei trebuie să fie pătratul unui număr ı̂ntreg.

265
Deci (3y+4)2 −4(7y 2 −9y−6) = k 2 , k ∈ Z. De aici rezultă că −19y 2 +60y+40 =
√ √
2
60 − 6640 60 + 6640
k ≥ 0 şi deci y ∈ [y1 , y2 ] ∩ Z, unde y1 = şi y2 = .
38 38
2
Obţinem că y ∈ {0, 1, 2, 3}. Dacă y = 0, ecuaţia x +4x−6 = 0 nu are rădăcini
ı̂ntregi. La fel, pentru y = 2, ecuaţia x2 + 10x + 4 = 0 nu are rădăcini ı̂ntregi.
Pentru y = 1 avem x2 + 7x − 8 = 0 şi soluţiile x = 1, x = −8. Pentru y = 3
avem x2 + 13x + 30 = 0 şi soluţiile x = −3 şi x = −10.

17. Evident că (x + y)2 < z 2 = (x + y)2 + 3x + y + 1 < (x + y + 2)2 =


(x + y)2 + 4x + 4y + 4 (am folosit că x, y ∈ N) şi deci z = x + y + 1. Introducând
această valoare ı̂n ecuaţia iniţială, obţinem că (x+y)2+ 3x+ y+1 = (x+ y+ 1)2 =
(x+y)2 +2x+2y+1 şi x = y. Deci soluţia problemei este x = y = a, z = 2a+1,
unde a ∈ N.

18. Pentru x ≥ 2 avem că (x+2)3 < m3 < (x+3)3 şi deci nu există soluţii
ı̂n acest interval (inegalitatea (x+2)3 < m3 este echivalentă cu 6x2 +12x+8 <
9x2 + 8x + 9, 3x2 − 4x + 1 > 0, (3x − 1)(x − 1) > 0, ultima inegalitate fiind
adevărată deoarece x ≥ 2). Pentru x ≤ −12 avem că (x + 4)3 > m3 > (x + 3)3
şi deci nu există soluţii ı̂n x ı̂n acest interval (inegalitatea (x + 4)3 > m3
este echivalentă cu x3 + 12x2 + 48x + 64 > x3 + 9x2 + 8x + 9, 3x2 +√40x +
− 40 − 940
55 > 0, ultima inegalitate fiind adevărată deoarece −12 < ).
6
Deci x ∈ [−11, 1] ∩ Z şi făcând calculele, observăm că doar x = 1, m = 3 este
soluţie a problemei.

19. Avem identitatea (a2 + 2ab − b2 )2 + (b2 + 2ab − a2 )2 = 2(a2 + b2 )2 .


Cum orice număr prim p ≡ 1 (mod 4) se scrie p = a2 + b2 , a, b ∈ N, identitatea
precedentă rezolvă problema.

20. Alegem x = 4c, y = 2c3 , z = c6 − 8, t = c6 + 8, unde c este un număr


natural impar, c ≥ 3. Avem (x, y, z, t) = 1 şi x6 + y 6 + z 4 = t4 .

21. Dacă x este impar, atunci y este par şi obţinem contradicţia 0 ≡ y 3 =
x2 + 3 ≡ 1 + 3 = 4 (mod 8). Deci x este par şi y impar. Avem x2 + 4 = y 3 + 1 =
(y + 1)(y 2 − y + 1), 4|x2 + 4 = (y + 1)(y 2 − y + 1). Cum y 2 − y + 1 este impar,
rezultă că 4|y + 1, y ≡ −1 (mod 4) şi y 2 − y + 1 ≡ 3 (mod 4).
Din ultima congruenţă şi din y 2 − y + 1 ≥ 1 rezultă că există un divizor prim
p ≡ 3 (mod 4) al lui y 2 − y + 1. Obţinem contradicţia p|x2 + 22 , p|2.

22. Dacă x este par, atunci şi y este par, x = 2x1 , y = 2y1 , x1 , y1 ∈ Z,
x21 + 3 = 2y13 , x1 impar, 0 ≡ x21 + 3 = 2y13 (mod 4), 2|y1 şi am obţinut

266
contradicţia 4 ≡ x21 + 3 = 2y13 ≡ 0 (mod 8). Deci x şi y sunt impare,
x2 + 4 = y 3 − 8 = (y − 2)(y 2 + 2y + 4). Avem y 2 + 2y + 4 ≡ 3 (mod 4) (deoarece
y este impar), y 2 + 2y + 4 ≥ 3 şi deducem existenţa unui divizor prim p ≡
3 (mod 4) al lui y 2 + 2y + 4. Obţinem contradicţia p|y 2 + 2y + 4|x2 + 22 , p|2.

23. Fie p prim, p|n. Din u2 + v 2 = 2n(u + v) deducem că p|u2 + v 2 şi p|u,
 2  2
u v
+
p p n
p|v, deoarece p ≡ 3 (mod 4). Avem de rezolvat ecuaţia =2 .
u v p
+
p p
Repetând acest raţionament de câte ori este nevoie, obţinem că u = nu1 ,
v = nv1 , u1 , v1 ∈ N şi satisfac ecuaţia u21 + v12 = 2(u1 + v1 ). Deci (u1 − 1)2 +
(v1 − 1)2 = 2 şi u1 , v1 ∈ {0, 2}. Obţinem soluţiile (u, v) = (0, 2n), (2n, 0),
(2n, 2n). Soluţia (u, v) = (0, 0) nu convine, căci ı̂n acest caz nu este definit
u2 + v 2
raportul .
u+v

24. Fie xm , ym numere naturale determinate de egalitatea xm + ym 6 =
€ √ Š€ √ Šm
3 + 6 5 + 2 6 . Rezultă că x2m − 6ym 2 = 3, x = 3z , 3z 2 − 2y 2 = 1,
m m m m
2 −1
zm z2 − 1
zm impar. Alegem n = 2 şi 3n + 1 = 3 m
. Avem 2n + 1 = zm +1 =
2 2
2 −1
3zm 2 şi problema este rezolvată.
= ym
2
25. Fie a − 1, a, a + 1 laturile triunghiului, a ∈ N. Din inegalitatea tri-
unghiului È
rezultă că a+1 < a+(a−1) = 2a−1, 2 < a, a ≥ 3. Aria triunghiului
este s = p(p − a + 1)(p − a)(p − a − 1), unde p este semiperimetrul triun-
(a − 1) + a + (a + 1) 3a
ghiului, adică p = = . Deci
2 2
Í „ Ž „ Ž Í „ Ž
3a a a a 3a2 a2
s= +1 −1 = −1 ∈ N.
2 2 2 2 4 4

 
3a2 a2
Avem că −1 = k 2 , k ∈ N. Deducem că a este par, a = 2b, 3b2 (b2 −1) =
4 4
k 2 . De aici rezultă că 3(b2 − 1) = h2 = 9t2 (h = 3t, t ∈ N), b2 − 1 = 3t2 ,
√ € √ Šn
b2 − 3t2 = 1. Soluţiile acestei ecuaţii Pell sunt bn + tn 3 = 2 + 3 , n ∈ N∗
(n = 0 nu convine căci b0 = 1, a0 = 2, dar a ≥ 3). Soluţiile problemei sunt
numerele an = 2bn .

267
26. Se observă că y = −x este soluţie a ecuaţiei. Presupunem ı̂n con-
tinuare că s = x + y 6= 0. Notăm p = xy şi ecuaţia din enunţ se rescrie
16(x2 − 1)(y 2 − 1) = [(y − x)2 − 4]2 , 16x2 y 2 − 16(x2 + y 2 ) = (y − x)4 − 8(y − x)2 .
Cum x2 +y 2 = s2 −2p şi (y−x)2 = s2 −4p, deducem că 16p2 −16(s2 −2p) =
(s2 −4p)2 −8(s2 −4p). Ecuaţia se rescrie 16p2 −16s2 +32p = s4 −8ps2 +16p2 −
8s2 + 32p, s4 − 8s2 p + 8s2 = 0, s2 − 8p + 8 = 0 (deoarece s 6= 0). Deducem
că x2 + y 2 − 6xy + 8 = 0, (y − 3x)2 = 8(x2 − 1), y − 3x = 4z, 2z 2 = x2 − 1,
√ € √ Šn
x2 − 2z 2 = 1. Ştim că x + z 2 = ± 3 + 2 2 , n ∈ Z şi y se găseşte din
formula y = 3x + 4z.

27. Căutăm soluţii cu x + y = 2 şi z = 3u, u impar. Avem că (x + y)3 −


3(1 − u2 )
3xy(x + y) + 1 = 9u2 , 6xy = 9 − 9u2 , xy = . x şi y sunt soluţii
2
3(1 − u2 )
ı̂ntregi ale ecuaţiei t2 − 2t + = 0. Pentru ca ecuaţia să aibă soluţii
2 „ Ž
3(1 − u2 )
ı̂ntregi, este necesar şi suficient ca ∆ = 4 1 − să fie pătrat. Deci
2
3(1 − u2 )
1− = s2 , 3u2 − 2s2 = 1. Ultima ecuaţie are o infinitate de soluţii
2
ı̂ntregi (curs, paginile 134–135) şi problema este rezolvată.
(L. Panaitopol )

28. Ecuaţia se scrie (x + yz)2 = (y 2 − 1)(z 2 − 1). Putem alege de exemplu


y 2 − 1 = 2a2 , z 2 − 1 = 2b2 . Deoarece (y, a), (z, b) sunt soluţii ale ecuaţiei Pell
u2 − 2v 2 = 1, aflăm y şi z şi apoi x = ±2ab − yz.

29. Avem 6x2 + 2 = 8y 3 şi deci (x + 1)3 = (2y)3 + (x − 1)3 . Din teorema
lui Fermat pentru cazul n = 3, rezultă x + 1 = 0 sau x − 1 = 0, sau y = 0.
Convin doar soluţiile x = ±1, y = 1.

30. Avem 24x2 + 128 = (2y)3 şi deci (x + 4)3 = (x − 4)3 + (2y)3 şi deci
x = −4 sau x = 4, sau y = 0. Avem soluţiile (±4, 4).

31. Ecuaţia se scrie (x + 2)4 = (x − 2)4 + (2y)4 şi deci x = −2 sau x = 2,


sau y = 0. Soluţiile sunt (2, ±2), (0, 0).

32. Ecuaţia se scrie (2y)4 + (x − 1)4 = (x + 1)4 şi deducem că y = 0 sau
x = ±1. Obţinem soluţiile x = 1, y = ±1 şi x = y = 0.

33. Să presupunem că xn + y n = z n . Dacă x = y, atunci 2xn = z n şi



rezultă contradicţia n 2 ∈ Q. Deci x 6= y şi să presupunem că x < y. Pe de

268
altă parte, z n = xn + y n > y n şi deci z > y, z ≥ y + 1. Avem xn = z n − y n ≥
(y + 1)n − y n > ny n−1 > nxn−1 şi deci x > n; de aici rezultă punctul a). Să
presupunem că n este impar şi x + y = p este prim. Atunci p|xn + y n = z n ,
p|z, z ≥ p şi obţinem contradicţia xn + y n = z n ≥ (x + y)n > xn + y n ; de
aici rezultă punctul b). Dacă z este prim şi n este impar, atunci x + y = z k
(deoarece x + y|z n şi z este prim), k ≥ 1. Dacă k = 1 obţinem contradicţia
xn + y n = z n = (x + y)n > xn + y n . Deci k ≥ 2 şi x + y ≥ z 2 . Deoarece x < y,
   2 n
x+y x+y n z z 2n
avem < y. Deducem inegalităţile z n > y n > ≥ = n ,
2 2 2 2
2n > z n , 2 > z ≥ 1. Rezultă că z = 1 şi obţinem contradicţia 1 = z n =
xn + y n ≥ 2. Punctul c) este rezolvat.

34. Demonstrăm prin inducţie după n că ecuaţia are soluţii. Pentru n = 0
luăm x = y = 0. Să presupunem că 2n = (x + y)2 + 3x + y. Dacă y = 0, atunci
2n = x2 + 3x şi luăm x1 = 0, y1 = x + 1 şi avem că (x1 + y1 )2 + 3x1 + y1 =
(x + 1)2 + (x + 1) = x2 + 3x + 2 = 2n + 2. Dacă y ≥ 1 alegem x1 = x + 1,
y1 = y − 1 şi avem că (x1 + y1 )2 + 3x1 + y1 = (x + y)2 + 3x + 3 + y − 1 = 2n + 2;
pasul de inducţie este demonstrat. Arătăm acum unicitatea.
Să presupunem că x, y, x1 , y1 ∈ N satisfac egalitatea (x + y)2 + 3x + y =
(x1 + y1 )2 + 3x1 + y1 . Dacă x + y > x1 + y1 , atunci x + y ≥ x1 + y1 + 1 şi
(x+y)2 +3x+y ≥ (x1 +y1 +1)2 +(x1 +y1 +1)+2x ≥ (x1 +y1 )2+3(x1 +y1 )+2+2x >
(x1 + y1 )2 + 3x1 + y1 . Am obţinut o contradicţie. La fel arătăm că nu putem
avea că x1 + y1 > x + y; rămâne că x + y = x1 + y1 . De aici rezultă imediat
că x = x1 şi y = y1 .

35. Să presupunem că k este par şi n ∈ N, n ≥ k. Alegem x1 = x2 = · · · =


x k = a, x k +1 = x k +2 = · · · = xk = a + 2, xk+1 = xk+2 = · · · = xn = a + 1
2 2 2
(x1 + x2 · · · + xn )2 k € 2 Š
şi observăm că x21 + x22 + · · · + x2n − = a + (a + 2)2 +
n 2
n2 (a + 1)2
(n − k)(a + 1)2 − = k(a2 + 2a + 2 − (a + 1)2 ) = k. Să presupunem
n
acum că k este impar, n este un număr prim, n ≥ k şi x1 , x2 , . . . , xn este
o soluţie a ecuaţiei din enunţ. Deducem că n|x1 + x2 + · · · + xn şi notăm
x1 + x2 · · · + xn Xn Xn
a= , yj = xj − a, ∀ j = 1, n. Avem că yj = xj − na = 0
n j=1 j=1
n
X n
X n
X
şi yj2 = x2j − 2a xj + na2 = na2 + k − 2a2 n + na2 = k.
j=1 j=1 j=1

269
n
X n
X
Din yj = 0 şi yj2 = k, deducem că k este par.
j=1 j=1
(L. Panaitopol )

36. Pentru n = 1 evident că ecuaţia nu are soluţii. Să presupunem că
X
n ≥ 2. Ecuaţia din enunţ se rescrie (xj − xi )2 = n (1).
1≤i<j≤n
)2
Pentru n = 2 avem (x2 − x1 = 2, care nu are soluţii ı̂ntregi, iar pentru
n = 3 avem (x2 −x1 )2 +(x3 − x1 )2 +(x3 − x2 )2 = 3, |x2 − x1 | = |x3 − x2 | =
|x3 − x1 | = 1, ceea ce este imposibil. Pentru n = 4 avem soluţia x1 = x2 = 1,
x3 = x4 = 0. Să presupunem că n ≥ 5 şi că există cel puţin trei valori distincte
X
ı̂n şirul x1 , x2 , x3 , . . . , xn ; fie acestea x1 , x2 , x3 . Atunci (xj − xi )2 ≥
1≤i<j≤n
n ”
X —
(x3 −x1 )2 +(x3 −x2 )2 +(x2 −x1 )2 + (xi − x1 )2 + (xi − x2 )2 + (xi − x3 )2 ≥
i=4
3 + (n − 3)2 = 2n − 3 > n, deoarece n ≥ 5.
Am folosit mai sus că (x − x1 )2 + (x − x2 )2 + (x − x3 )2 ≥ 2, ∀ x ∈ Z
(deoarece x1 6= x2 , x1 6= x3 , x2 6= x3 ). Evident că nu putem avea x1 = x2 =
· · · = xn şi atunci singura variantă posibilă este x1 = x2 = · · · = xk = a,
xk+1 = xk+2 = · · · = xn = b (după o renotare), unde 1 ≤ k ≤ n − 1 şi a 6= b.
Ecuaţia (1) se rescrie k(n−k)(a−b)2 = n. Dacă 2 ≤ k ≤ n−2, atunci obţinem
contradicţia n = k(n − k)(a − b)2 ≥ k(n − k) ≥ 2(n − 2) = 2n − 4 > n. Dacă
k = 1 sau k = n − 1 obţinem ecuaţia (n − 1)(a − b)2 = n care este imposibilă,
căci (n − 1, n) = 1, n − 1|n implică n − 1 = 1, n = 2, (a − b)2 = 2. Deci singura
valoare a lui n, pentru care ecuaţia din enunţ are soluţii ı̂ntregi, este n = 4.
(L. Panaitopol )

37. Fie z = ay cu a ∈ N∗ . Avem x2 + y 2 = a2 y 2 + 1. Dacă a = 1,


rezultă y = z şi deci z|xy 2 . Fie a ≥ 2; avem x2 − (a2 − 1)y 2 = 1. Această
ecuaţie Pell are soluţia minimală (a, 1) şi deci soluţiile ı̂n numere naturale
√ € √ Šn
sunt date de egalităţile xn + yn a2 − 1 = a + a2 − 1 cu n ∈ N∗ . Rezultă
xn = Cn0 an + Cn2 an−2 (a2 − 1) + . . . , yn = Cn1 an−1 + Cn3 an−3 (a2 − 1) + . . . . Avem
x1 = a, y1 = 1; x2 = 2a2 − 1, y2 = 2a . . . . Rezultă că pentru n impar avem
z
a|xn , iar pentru n par avem a|yn . Aşadar a|x sau a|y. Pentru a|x avem x,
y
z
adică z|xy. Pentru a|y avem y, adică z|y 2 . Rezultă de aici z|xy 2 .
y
(L. Panaitopol )

270
38. Dacă y = 0, rezultă z = 0 şi deci z|y 2 . Dacă y = z, atunci evident că
z|y 2 . Să presupunem că z = ay, a ≥ 2. Avem ecuaţia Pell x2 − (a2 − 1)y 2 = 1
€√ Š
a2 − 1 6∈ Q . Soluţia iniţială este x1 = a, y1 = 1 (x0 = 1, y0 = 0).
Avem formulele de recurenţă xn = axn−1 + (a2 − 1)yn−1 , yn = xn−1 + ayn−1 ,
∀ n ∈ N∗ , din care deducem prin inducţie că x2k−1 = a(2tk + 1), x2k = 2sk + 1,
y2k−1 = 2uk + 1, y2k = a · 2vk , unde tk , sk , uk , vk sunt numere naturale.
Cum y este par, deducem că ∃ k ∈ N astfel ı̂ncât y = y2k = a · 2vk . Rezultă că
y2 y2 y
= = = 2vk ∈ N şi problema este rezolvată.
z ay a

39. Avem n2 − 1 6∈ Q. Soluţia iniţială este x1 = n şi y1 = 1.
√ € √ Šk
Aşadar xk + yk n2 − 1 = n + n2 − 1 (curs, pagina 133). Avem xk =
nk + Ck2 nk−2 (n2 − 1) + . . . , yk = Ck1 nk−1 + Ck3 nk−3 (n2 − 1) . . . . Pentru k par
atunci n|yk , iar pentru k impar rezultă n|xk .

40. Avem n2 − 4 6∈ Q. Dacă n este par, se găseşte soluţia iniţială x1 =
„ Žk
n2 n √ n2 n√
−1, y1 = şi deci xk +yk n2 − 4 = −1+ n2 − 4 (curs, pagina
2 2 2 2
133),
„ Žk−1 „ Žk−3
 3
n2 n n2 n .n
yk = Ck1 −1 + Ck3 −1 (n2 − 4) + . . . .. .
2 2 2 2 2


Dacă n este impar, n ≥ 5, determinăm soluţia iniţială. Avem n2 − 4 =
n−3 n−3
(a0 ; a1 , a2 , a3 , a4 , a5 , a6 ) cu a0 = n − 1, a1 = 1, a2 = , a3 = 2, a4 =
2 2
a5 = 1, a6 = 2n − 2, a7 = 1 şi deci s = 6. Avem de asemenea P0 = n − 1,
(n + 1)(n − 2)
Q0 = 1, P1 = n − 1 + 1 = n, Q1 = 1, P2 = a2 P1 + P0 = ,
2
n−3 n−1 n3 − 2n2 − 3n + 4
Q2 = +1 = , P3 = n2 − 2, Q3 = n, P4 = ,
2 2 2
n2 − 2n − 1 n3 − 3n n2 − 1
Q4 = , P5 = = x1 , Q5 = = y1 (monografie,
2 2 „ 2 Žk
√ n(n2 − 3) n2 − 1√
paginile 16 şi 26), xk + yk n2 − 4 = + n2 − 4 şi de
2 2
 
. n2 − 1 n−1 .n−1 n
aici yk .. = (n + 1), adică yk .. = . Pentru n = 3 enunţul
2 2 2 2
este trivial.

271
41. Fie x1 = x2 = · · · = xn−2 = 1, xn−1 = x, xn = y. Ecuaţia devine
x2 + y 2 + n − 2= nxy. Pentru n = 2 ecuaţia devine (x − y)2 = 0 şi are soluţiile
(λ, λ), λ ∈ Z. Pentru n ≥ 3 ecuaţia se scrie (n−2)(x+y)2−(n+2)(x−y)2 = 4(n−2).
Alegem x − y = (n − 2)ξ şi notăm x + y = s. Avem s2 − (n2 − 4)ξ 2 = 4.
Ecuaţia are soluţia s = n, ξ = 1. Deoarece ∆ = n2 −4 > 0 şi n2 −4 6= k 2 , ecuaţia
u2 −∆v 2 = 1 este de tip Pell şi are soluţia nebanală (u0 , v0 ). din egalităţile sk +
√ € √ Š€ √ Šk
∆ξk = n + ∆ u0 + v0 ∆ , obţinem o infinitate de soluţii. Avem apoi
(
xk + yk = sk (n − 2)ξk + sk sk − (n − 2)ξk
şi deci xk = , yk = .
xk − yk = (n − 2)ξk 2 2
Este imediat că aceste numere sunt naturale.

42. Fie u, v ∈ N, (u, v) 6= (1, 0), soluţia minimală a ecuaţiei Pell x2 −


py 2 = 1. Vom arăta că u este par şi v este impar. Să presupunem că u este
u−1 u+1
impar. Din u2 − pv 2 = 1 deducem că v este par. Avem că · =
8 2 2
>
> u−1
 2   < = pα2
v u−1 u+1 u−1 u+1
p , , = 1, , ∈ N∗ , şi deci 2 sau
2 2 2 2 2 >
> u+1
: =β 2
8 2
>
> u−1
< = α2
2 , unde α, β ∈ N. În primul caz rezultă că β 2 − pα2 = 1 şi se
>
> u + 1
: = pβ 2
2
contrazice minimalitatea lui v, căci 2αβ = v (deci α, β ∈ N∗ ) şi 0 < α < 2αβ = v.
În cel de-al doilea caz pβ 2 − α2 = 1, α2 ≡ −1(p) şi am obţinut o contradicţie,
 
−1
căci = −1, deoarece p ≡ 3(4). Deci neapărat u este par şi v impar.
p (
2 u − 1 = α2
Atunci (u − 1, u + 1) = 1 şi din (u − 1)(u + 1) = pv rezultă că
u + 1 = pβ 2
(
u − 1 = pα2
sau , unde α, β ∈ N. În primul caz obţinem că pβ 2 − α2 = 2,
u + 1 = β2
   
−2 p−1 2
α2 ≡ −2 (mod p) şi am obţinut o contradicţie, căci = (−1) 2 =
p p
(−1)(+1) = −1 (am folosit faptul că p ≡ 7 (mod 8)). Deci singurul caz valabil
este al doilea şi din el deducem imediat că β 2 − pα2 = 2(α, β ∈ Z), ceea ce
trebuia demonstrat.
Observaţie. În acelaşi mod se arată că dacă p este un număr prim,
p ≡ 3 (mod 8), atunci ecuaţia x2 − py 2 = −2 are soluţii ı̂ntregi.

272
43. Problema se găseşte ı̂n curs la pagina 144. Prezentăm aici o soluţie

folosind ecuaţia Pell. Ştim că dacă d ∈ N, d 6∈ N şi scrierea ca fracţie continuă

infinită a lui d este (a0 ; a1 , a2 , . . . , as ), atunci ecuaţia x2 −dy 2 = −1 are soluţii
ı̂n Z dacă şi numai dacă s este impar. Pentru a ∈ N, a ≥ 2, fracţia continuă

a lui a2 − 1 este (a − 1; 1, 2a − 2), ceea ce ne arată că s = 2 şi ecuaţia
x2 − (a2 − 1)y 2 = −1 nu are soluţii ı̂ntregi. „
De asemenea, dacă a ∈ N, a impar, Ž
√ a−3 a−3
a ≥ 5, fracţia continuă a lui a2 − 4 este a − 1; 1, , 2, , 1, 2a − 2
2 2
(vezi soluţia problemei 40). Cum s = 6, deducem că ecuaţia x2 −y 2 (a2 −4) = −1
nu are soluţii ı̂ntregi. Revenim acum la problema iniţială. Să presupunem că
există x, y, z ∈ N, z 6= 3, astfel ı̂ncât x2 + y 2 + 1 = xyz. Analizăm ı̂ntâi cazul
z par. Evident, nu putem avea z = 0, căci x2 + y 2 + 1 > 0, şi nici z = 2, căci
x2 + y 2 + 1 − 2xy = (x − y)2 + 1 > 0. Deci z ≥ 4, z = 2k, k ≥ 2, k ∈ N.
Din x2 − yzx + y 2 + 1 = 0 şi x, y, z ∈ N, rezultă că discriminantul ecuaţiei
de grad 2 ı̂n x trebuie să fie pătrat. Deci y 2 z 2 − 4(y 2 + 1) = A2 . Cum z este
par, din ecuaţia precedentă deducem că A = 2B, B ∈ Z, y 2 k 2 − y 2 − 1 = B 2 ,
B 2−y 2 (k 2−1) = −1. Am obţinut o contradicţie, căci am observat ceva mai sus,
că ecuaţia B 2−y 2 (k 2−1) = −1 nu are soluţii pentru k ∈ N, k ≥ 2. Deci neapărat
z este impar. Să presupunem că x sau y ar fi număr par. Din cauza simetriei
ecuaţiei ı̂n x şi y, putem presupune că y este par. Din ecuaţia y 2 z 2 −4(y 2 +1) =
A2 , rezultă atunci din nou că A = 2B, y = 2C, C 2 z 2 − 4C 2 − 1 = B 2 ,
B 2 − C 2 (z 2 − 4) = −1 şi am obţinut iaraşi o contradicţie, căci am văzut că
dacă z este număr natural impar, z ≥ 5, ultima ecuaţie nu are soluţii ı̂ntregi.
Pentru z = 1, ecuaţia nu are evident soluţii, căci x2 −xy+y 2 +1 ≥ 1 > 0. Deci x
şi y sunt ambele impare (la fel ca z). Avem din nou egalitatea A2 −y 2 (z 2 −4) =
€ √ Š
−4 cu A impar. Notând N a + b z 2 − 4 = a2 − b2 (z 2 − 4), ∀ a, b ∈ Q,
€√ Š
ştim că N (z1 z2 ) = N (z1 )N (z2 ), ∀ z1 , z2 ∈ Q z 2 − 4 . Din cele de mai sus,
„ √ Ž „ √ Ž3 
A+y z −4 2 A+y z −4 2
N = −1 şi N = (−1)3 = −1. Dar
2 2

‚ √ Œ3 € Š√
A + y z2 − 4 A3 + 3Ay 2 (z 2 − 4) 3A2 y + y 3 (z 2 − 4) z 2 − 4
= +
2 8 8
€ Š € Š
A A2 + 3y 2 (z 2 − 4) y 3A2 + y 2 (z 2 − 4)
şi , sunt numere ı̂ntregi, căci
8 8
A2 +3y 2 (z 2 −4) ≡ 1+3(1−4) = −8 ≡ 0(8) şi 3A2 +y 2 (z 2 −4) ≡ 3+(1−4) ≡ 0(8)
(am folosit că a2 ≡ 1(8) dacă a este impar).

273
Din cele de mai sus, ar rezulta că ecuaţia m2 − n2 (z 2 − 4) = −1 ar
avea
€
soluţii ı̂n Z, ceea
Š
ce am€văzut că este imposibil
Š
(soluţiile ar fi m =
2 2
A A + 3y (z − 4) 2 2 2 2
y 3A + y (z − 4)
şi n = ).
8 8

44. Împărţind cu cel mai mare divizor comun, putem presupune că
(x, y, z) = 1. Avem x < y < z, x + z = 2y. Deci x şi z sunt impare şi y
este par. Notăm z − x = 2a, z + x = 2b (a, b ∈ N∗ , (a, b) = 1, a < b) şi
x+z
avem că x = b − a, z = a + b, y = = b, (b − a)n + bn = (b + a)n ,
2
b
(u − 1)n + un = (u + 1)n , u = > 1, u fiind număr raţional. Dacă n este
a
impar, atunci u este soluţie a ecuaţiei un − 2Cn1 un−1 − · · · − 2 = 0. Cum
u este raţional, u > 1, deducem că u = 2, 1 + 2n = 3n , ecuaţie care este
imposibilă pentru n > 1. Dacă n este par, atunci u este soluţie a ecuaţiei
un − 2Cn1 un−1 − 2Cn3 un−3 − · · · − 2Cnn−1 u = 0. Cum u 6= 0, deducem că u
este rădăcină a ecuaţiei un−1 − 2Cn1 un−2 − 2Cn3 un−4 − · · · − 2n = 0. Cum
u > 0, u raţional, deducem că u ∈ N, u|2n, u ≤ 2n. Avem contradicţia
2Cn1 2Cn3 2n 2n
1 = + 3 + · · · + n−1 > ≥ 1. Pentru n = 2 obţinem că u = 4,
u u u u
b = 4a, a = 1 (deoarece (a, b) = 1), b = 4, x = 3, y = 4, z = 5. Soluţia
generală ı̂n acest caz este x = 3d, y = 4d, z = 5d, d ∈ N∗ .

45. Fie x, y ∈ Q o soluţie a ecuaţiei x2 +432 = y 3 ; evident că y > 0. Scriem


x k y m
= , = , unde k, m, n ∈ Z, n 6= 0. Pot presupune că k şi n sunt pare,
36 n 12 n
x 2k y 2m n+k n−k
scriind eventual = , = . Notăm u = ,v= , w = m.
36 2n 12 2n 2 2
   
n+k 3 n−k 3
Calculăm u3 +v 3 −w3 şi obţinem că u3 +v 3 −w3 = + −m3 =
2 2
n3 3nk 2 n3 3n x2 n2 n3 y 3 n3
+ − m3 = + · − = (432 + x2 − y 3 ) = 0.
4 4 4 4 362 123 1728
Deci u3 +v 3 = w3 şi conform Marii Teoreme a lui Fermat pentru exponentul
3 (curs, pagina 142), avem că u · v · w = 0. Însă w = 0 implică m = 0, y = 0,
ceea ce este imposibil. Dacă u = 0, avem că n = −k şi x = −36, iar dacă v = 0,
avem că n = k şi x = 36. Deci x = ±36 şi y 3 = 362 +432 = 1728 = 123 , y = 12.
Am demonstrat că singura soluţie a ecuaţiei x2 + 432 = y 3 este x = ±36 şi
y = 12.

274
46. Să presupunem că ∃ x, y ∈ N∗ astfel ı̂ncât 4xy − x − y = z 2 . Atunci
(4x − 1)(4y − 1) = (2z)2 + 1. Cum 4x − 1 ≥ 3 şi 4x − 1 ≡ 3 (mod 4), rezultă că
există un număr prim p ≡ 3 (mod 4) astfel ı̂ncât p|4x − 1. Din ecuaţia de mai
sus rezultă p|(2z)2 + 12 , p|1; contradicţie. Pentru partea a doua a enunţului,
să observăm că x = −1, y = −5n2 − 2n, z = −5n − 1, n ∈ N, verifică ecuaţia
din enunţ.

47. Dacă y este impar, atunci x este par şi obţinem contradicţia 1 ≡ y 2 =
x5 + 7 ≡ 3 (mod 4), Deci y este par şi x impar. Ecuaţia se rescrie x5 + 25 =
y 2 + 52 . Dacă x ≡ 3(4), atunci x5 + 32 ≡ x ≡ 3 (mod 4) şi deducem că există
un divizor prim p ≡ 3 (mod 4) al lui x5 + 32 (din y 2 = x5 + 7 rezultă că x > 0
şi deci x5 + 32 > 32). Dacă x ≡ 1 (mod 4), atunci x + 2 ≡ 3 (mod 4), x + 2 ≥ 3
şi există deci un divizor prim p ≡ 3 (mod 4) al lui x + 2, deci şi al lui x5 + 25 ,
căci x + 2|x5 + 25 . În ambele cazuri am găsit un număr prim p ≡ 3 (mod 4) cu
p|x5 + 25 = y 2 + 52 . Deducem că p|5; contradicţie.
(I. Cucurezeanu)

48. Să presupunem că ∃ x, y ∈ Z astfel ı̂ncât x2 − y 3 = 7. Dacă y este


par, atunci x2 ≡ 7 (mod 4); contradicţie. Deci y = 2m + 1 şi x2 + 1 = y 3 + 8 =
(y + 2)(y 2 − 2y + 4). Dar y 2 − 2y + 4 = (y − 1)2 + 3 ≥ 3 şi y 2 − 2y + 4 =
(y − 1)2 + 3 = 4m2 + 3 ≡ 3 (mod 4). Există deci un divizor prim p ≡ 3 (mod 4)
al numărului y 2 − 2y + 4. Obţinem contradicţia p|y 2 − 2y + 4|x2 + 1, p|1. Deci
ecuaţia nu are soluţii.

49. Presupunem că x este par. Atunci x2 ≡ 0, 4 (mod 16), y este par şi
obţinem contradicţia 0 ≡ y 5 = x2 + 36 = 4, 8 (mod 16). Deci x, y sunt impare,
y ≥ 3 (dacă y = 1 ar rezulta că x2 = −35). Avem că y ≡ y 5 = x2 + 36 ≡ x2 ≡ 1
(mod 4) şi x2 + 22 = y 5 − 32 = (y − 2)(y 4 + 2y 3 + 4y 2 + 8y + 16). Cum y − 2 ≡ 3
(mod 4), y − 2 ≥ 3, deducem că există un divizor prim p ≡ 3 (mod 4) al lui
y − 2. Deducem contradicţia p|x2 + 22 , p|2, p = 2.

50. Să presupunem că există o astfel de soluţie. Împărţind cu cel mai mare
divizor comun, putem presupune că (x, y, z, w) = 1. Făcând un raţionament
modulo 4, rezultă că w este impar şi unul din numerele x, y, z este impar.
Să presupunem că z este impar. Avem x2p + y 2p = (w2 − z 2 )M , unde M =
p−1
X
(w2 )j (z 2 )p−1−j . Deoarece z şi w sunt numere impare, rezultă că w2 ≡ z 2 ≡ 1
j=0
(mod 4) şi M ≡ p ≡ 3 (mod 4), M ≥ p ≥ 3. Deducem că ∃ q număr prim,
q ≡ 3 (mod 4) şi q apare la exponent impar ı̂n descompunerea lui M .

275
Deducem că q|(xp )2 +(y p )2 , q|x, q|y, q 6= p (deoarece p¤|¤xyzw). Deci exponentul
lui q ı̂n (xp )2 + (y p )2 este par. Deducem că q|w2 − z 2 , w2 ≡ z 2 (mod q), M ≡
pz 2(p−1) ≡ 0 (mod q). Cum p 6= q, rezultă că q|z şi q|w. Contradicţie, căci
q|(x, y, z, w) = 1.

51. Să presupunem că x este par. Atunci 2|y, 8|y 3 − 16 = x2 , 4|x, 16|y 3 ,
4|y. Avem x = 4x1 , y = 4y1 şi x21 + 1 = 4y13 . Rezultă o contradicţie, deoarece
x1 este impar şi 0 ≡ 4y13 = x21 + 1 ≡ 2 (mod 4). Deci x şi y sunt impare şi
1 ≡ x2 +16 = y 3 ≡ y (mod 8). Avem x2 +8 = y 3 −8 = (y−2)(y 2 +2y+4). Avem
că y − 2 ≡ 7 (mod 8), y − 2 > 0 şi deci există un divizor prim q al lui y − 2 de
 
2 2
−8
forma 8k+5 sau 8k+7. Rezultă că q|y−2|x +8, x ≡ −8 (mod q), = 1.
q
   
−8 −2 q−1 q 2 −1
Aceasta este o contradicţie, căci = = (−1) 2 (−1) 8 = −1
q q
(deoarece q ≡ 5, 7 (mod 8)).

52. Notăm cu h1 < h2 < · · · < hn−1 distanţele de la vârful conului la


planele mulţimii M1 şi cu i1 < i2 < · · · < in−1 distanţele de la vr̂ful conului
 3
hk k
la planele mulţimii M2 ; notăm cu h ı̂nălţimea conului. Avem că = şi
h n
 2
ik k
= , ∀ k = 1, n − 1. Dacă M1 ∩M2 6= ∅, atunci ∃ k, j ∈ {1, 2, . . . , n−1}
h n
astfel ı̂ncât hk = ij . Rezultă din formulele precedente că avem de rezolvat
ecuaţia j 3 = nk 2 . Arătăm că dacă există p prim astfel ı̂ncât p3 |n, atunci
ecuaţia are soluţii. Avem n = p3 · a, 1 ≤ a ≤ n − 1. Alegem k = a, j = pa
(1 ≤ j = pa < p3 a = n) şi ele verifică ecuaţia. Să presupunem acum că dacă
n = pa11 · pa22 · . . . · par r , pj prim ∀ j = 1, r, pj 6= pk pentru j 6= k, ai ∈ {1, 2},
∀ i = 1, r, atunci ecuaţia nu are soluţii şi deci M1 ∩ M2 = ∅ ı̂n acest caz. Să
presupunem că există j, k ∈ {1, 2, . . . , n − 1} astfel ı̂ncât j 3 = nk 2 . Atunci
k = l3 · pb11 · pb22 · . . . · pbrr , l ∈ N∗ , ai + 2bi = 3ci , ∀ i = 1, r, bi , ci ∈ N. Dacă
ai = 1, atunci bi ≥ 1, iar dacă ai = 2, atunci bi ≥ 2 (din ai + 2bi ≡ 0 (mod 3)
şi bi ∈ N). Deci bi ≥ ai ∀ i = 1, r şi obţinem contradicţia k ≥ n. Condiţia
necesară şi suficientă ca M1 ∩ M2 6= ∅ este ca n să se dividă cu cubul unui
număr prim.
(L. Panaitopol )

53. Fie xk , yk , k = 1, n distanţele de la A la aceste paralele şi s


fie h
 2
xk k k
ı̂nălţimea triunghiului. Avem = , de unde deducem că xk = h .
h n n

276
2
Se arată uşor că valoarea comună a perimetrelor este p = √ (3yk+1 − yk ),
3
2
∀ k = 1, n − 1 (notăm yn = h) şi că p = √ 3y1 . Deducem formula de recurenţă
3
2 2
√ (3yk+2 − yk+1 ) = √ (3yk+1 − yk ), 3yk+2 − yk+1 = 3yk+1 − yk , 3yk+2 =
3 3
2 2 4
4yk+1 − yk . De asemenea, √ (3y2 − y1 ) = √ 3y1 , y2 = y1 . Din formula de
3 3 3
4 y1 3k − 1
recurenţă şi din y2 = y1 , deducem imediat prin inducţie că yk = · k−1 ,
3 2 3
y 3n−1 2h · 3n−1
1
∀ k ∈ N∗ . Punem k = n şi rezultă că h = yn = · n−1 , y1 = ,
2 3 3n − 1
n−k
3k − 1
yk = h · 3 .
3n − 1
Să presupunem că M1 ∩ M2 6= ∅; există deci i, k ∈ {1, 2, . . . , n−1} astfel
ı̂ncât xi = yk . Din formulele de mai sus rezultă că i(3n −1)2 = 9n−k (3k −1)2 n.
„ Ž2
3n − 1
Fie (n, k) = d. Atunci (3n − 1, 3k − 1) = 3d − 1 şi i =
3d − 1
„ Ž2 „ Ž „ Ž
3k − 1 3n − 1 3k − 1 3n − 1 n−k
9n−k n. Deoarece , = 1 şi ,9 = 1,
3d − 1 3d − 1 3d − 1 3d − 1
„ Ž2 „ Ž2
3n − 1 3n − 1
rezultă că n şi n ≥ > (3n−d )2 . În ultima inegali-
3d − 1 3d − 1
n
tate am folosit că d < n. De fapt d ≤ (căci d|n, d ≤ k < n) şi avem că
€ 2
n Š2
n > (3n−d )2 ≥ 3n− 2 = 3n . Am obţinut o contradicţie, căci se arată imediat
prin inducţie că 3n > n, ∀ n ∈ N∗ .
(L. Panaitopol )

54. Fie m = dm1 , n = dn1 , (m1 , n1 ) = 1. Introducând ı̂n ecuaţie şi


simplificând cu d, obţinem m1 + dn21 + d2 = d2 m1 n1 . Deci d|m1 , m1 = dm2
n21 + d
şi m2 + n21 + d = d2 m2 n1 , m2 = 2 . Deoarece m2 d2 ∈ N∗ şi m2 d2 =
d n1 − 1
n1 + d3 n1 + d3 n 1 + d3
n1 + 2 , rezultă că 2 ∈ N∗ şi 2 ≥ 1.
d n1 − 1 d n1 − 1 d n1 − 1
Presupunem ı̂n continuare că d ≥ 3. Din ultima inegalitate rezultă că
d3 + 1 d2 − d + 1 1 1
n1 ≤ 2 = =d+ ≤d+ .
d −1 d−1 d−1 2

277
Cum n1 ∈ N∗ , avem că n1 ≤ d. Însă 1 = (m1 , n1 ) = (dm2 , n1 ), d ≥ 3 şi deci
nu putem avea n1 = d. Concluzionăm că n1 ≤ d − 1. Obţinem următoarele
n21 + d (d − 1)2 + d d2 − d + 1
inegalităţi contradicţorii: 1 ≤ m2 = 2 ≤ ≤ <1
d n1 − 1 d2 n1 − 1 d2 − 1
n1 + d3
(deoarece d ≥ 3). Deci neapărat d = 1 sau d = 2. Din d = 1 şi 2 ∈ N∗ ,
d n1 − 1
n1 + 1 2
rezultă că ∈ N, ∈ N∗ , n1 = 2 sau n1 = 3, m2 = 5. Dacă d = 2,
n1 − 1 n1 − 1
n1 + d3 n1 + 8 4n1 + 32 33
din 2 = ∈ N, rezultă că = 1+ ∈ N∗ şi deci
d n1 − 1 4n1 − 1 4n1 − 1 4n1 − 1
n1 = 1 sau n1 = 3 şi m2 = 1. Soluţiile ecuaţiei sunt (m, n) = (5, 2), (5, 3), (4, 2),
(4, 6).

55. Dacă a = 0, k = b2 şi enunţul este evident. La fel dacă b = 0.


2a2 2
Dacă a = b ∈ N∗ , k = 2
= 2− , a = b = 1, k = 1 şi enunţul
1+a 1 + a2
este clar. Deci putem presupune că a, b ∈ N∗ , a 6= b. Alegem dintre toate
a2 + b2
perechile (a, b) cu a, b ∈ N∗ , a < b, = k, pe cea pentru care b este
1 + ab
(ka − b)2 + a2
minim. Avem că = k, ka − b < a (inegalitatea ka − b < a este
1 + (ka − b)a
a+b a2 + b2 a + b
echivalentă cu k < ⇔ < ⇔ a3 + ab2 < a + b + a2 b + ab2 ⇔
a 1 + ab a
a3 < a + b + a2 b; ultima inegalitate este evidentă deoarece 0 < a < b) şi
din cauza minimalităţii lui b trebuie neapărat ca ka − b ≤ 0 şi deci a3 ≤ b.
a4 + 1 + a2 b2 − 1 a4 + 1 a4 + 1
Cum a2 k = = ab − 1 + ∈ N, rezultă că ∈ N∗ ,
1 + ab 1 + ab ab + 1
a4 + 1 3 3 3
a2 + a6
≥ 1, a ≥ b. Cum avem şi a ≤ b, rezultă că b = a , k = = a2 ,
ab + 1 1 + a4
adică k este pătrat perfect.
c2 + a2
Observaţie. Faptul că c = ka − b satisface egalitatea = k, rezultă
1 + ca
2 2
din faptul că ecuaţia de grad 2, x − akx + a − k = 0, are ca soluţie pe b
şi atunci cea de-a doua soluţie c trebuie să satisfacă relaţia c + b = ak (din
relaţiile lui Viète).

278
CAPITOLUL 16

Ecuaţii diofantice exponenţiale

1. Nu putem avea y ≥ 3 căci ar rezulta contradicţia 3x = 2y + 7 ≡


7 (mod 8) (3x ≡ 1, 3 (mod 7)). Deci y ∈ {0, 1, 2} şi se observă imediat că
singura soluţie a problemei este y = 1, x = 2.
2. Dacă y = 0, atunci x = 1. Pentru y ≥ 1 avem 1 ≡ 1 + 3y = 2x ≡ (−1)x
(mod 3) şi deci x = 2u, u ∈ N. Rezultă că 3y = 22u − 1 = (2u − 1) (2u + 1),
€ Š
2u − 1 = 3a , 2u + 1 = 3b , a < b, a + b = y, 3b − 3a = 2 = 3a 3b−a − 1 , a = 0,
b = y, 3y − 1 = 2, y = 1, u = 1, x = 2. Deci soluţiile problemei sunt x = 1,
y = 0 şi x = 2, y = 1.
3. Dacă y = 0 atunci x = 1. Dacă y ≥ 1, atunci 3x − 1 = 2y+1 ≡
€ Š
0 (mod 4) şi deci x trebuie să fie par; x = 2k, k ∈ N. 2y+1 = 3k − 1 (3k + 1),
3k − 1 = 2a , 3k + 1 = 2b , a, b ∈ N, a < b, a + b = y + 1. Scăzând ultimele două
€ Š
egalităţi rezultă că 2 = 2b − 2a = 2a 2b−a − 1 , a = 1, b − a = 1. Deci a = 1,
b = 2, y = 2, x = 2.
4. Pentru p = 2 avem că mk = 22 + 32 = 13 şi rezultă că k = 1.
Presupunem că p ≥ 3. Atunci 2p + 3p ≡ 2p + (5 − 2)p ≡ 2p − 2p ≡ 0 (mod 5)
şi deci 5|m. Dacă k ≥ 2 atunci 52 |mk = 2p + 3p . Dar 2p + 3p ≡ 2p + (5 − 2)p ≡
p · 2p−1 · 5 (mod 52 ) şi deducem că p = 5. Dar 25 + 35 = 32 + 243 = 275 =
52 · 11 6= mk (deoarece k ≥ 2). Deci k = 1.
5. 3x = (y − 2x )(y + 2x ). Deci y + 2x = 3a , y − 2x = 3b , a, b ∈ N, a > b,
a + b = x. Scăzând cele două egalităţi anterioare rezultă că 2x+1 = 3a − 3b =
3b (3a−b − 1). Deci b = 0 şi a = x şi avem de rezolvat ecuaţia 2x+1 = 3x − 1,
 x  x
3 1
x ∈ N. Această ecuaţie se mai poate scrie şi sub forma 2 = − .
2 2
 x  x
3 1
Deoarece funcţia f (x) = − este strict crescătoare şi f (2) = 2
2 2
rezultă că x = 2 este singura soluţie. y 2 = 32 + 42 = 25, y = 5.
6. Dacă x ≥ 1 atunci 5z = 3x + 4y ≡ 1 (mod 3) şi deci z = 2k. 3x =
52k −22y= (5k −2y )(5k +2y ), 5k −2y = 3a , 5k +2y = 3b , a, b ∈ N, a < b, a+b = x.

279
Scăzând cele două egalităţi rezultă că 2y+1 = 3b − 3a = 3a (3b−a − 1); a = 0,
b = x, 2y+1 = 3x − 1. Am rezolvat această ultimă ecuaţie (problema 3) şi ştim
că ea are doar soluţiile y = 0, x = 1 şi y = x = 2. Dacă introducem ı̂n ecuaţia
iniţială y = 0 şi x = 1, obţinem contradicţia 5z = 4. Dacă y = x = 2 obţinem
soluţia z = 2. Mai trebuie analizat cazul x = 0 şi ecuaţia 1 + 4y = 5z . Privind
această egalitate ca o congruenţă modulo 3 rezultă că z este impar. Evident
y = 0 nu este soluţie. Dacă y = 1, atunci z = 1. Presupunem acum că y ≥ 2.
€ Š
Avem egalităţile 4y = 5z − 1 = (5 − 1) 5z−1 + 5z−2 + . . . + 5 + 1 , 4y−1 =
5z−1 + 5z−2 + . . . + 5 + 1. Deoarece y ≥ 2, termenul stâng al ultimei egalităţi
este par. Deoarece z este impar, termenul drept al ultimei egalităţi este impar.
Deci ultima egalitate nu poate avea loc. Am demonstrat că singurele soluţii
ale ecuaţiei sunt x = 0, y = z = 1 şi x = y = z = 2.

7. Am rezolvat ecuaţia 3a + 4b = 5c ı̂n problema 6. Pentru a rezolva


problema din enunţ este suficient să analizăm cazul x impar. Presupunem
că x ≥ 3 şi avem că 1 ≡ 5z = 2x + 3y ≡ (−1)y (mod 4), y par. De aici
deducem şi că 1 ≡ 2x + 3y = 5z (mod 8), z par. Nu putem avea y = 0 căci
ar rezulta contradicţia 3|2x + 1 = 5z (x este impar). Deci y ≥ 1 şi −1 ≡
(−1)x ≡ 2x + 3y = 5z ≡ (−1)z = 1 (mod 3); contradicţie. Deci x = 1 şi trebuie
să rezolvăm ecuaţia 2 + 3y = 5z . Avem că 1 ≡ 5z = 2 + 3y ≡ 2 + (−1)y
(mod 4) şi deci y este impar. Dacă y ≥ 2 atunci 2 ≡ 5z (mod 9) şi rezultă
că z ≡ 5 (mod 6) (avem 56 ≡ 1 (mod 9)). În egalitatea 2 + 3y = 5z trecem
la o congruenţă modulo 7 şi obţinem (ţinând cont că 56 ≡ 1 (mod 7), 55 ≡
3 (mod 7)) că 2 + 3y = 5z ≡ 55 ≡ 3 (mod 7) (am folosit şi că z ≡ 5 (mod 6)).
Deci 3y ≡ 1 (mod 7) şi 6|y; contradicţie căci y este impar. Deci y = z = 1 este
singura soluţie a ecuaţiei 2 + 3y = 5z . Ţinând cont de cele de mai sus şi de
problema 6 rezultă că singurele soluţii ale ecuaţiei sunt x = y = z = 1, x = 2,
y = 0, z = 1, x = 4, y = z = 2.

8. Dacă y = 0 atunci 1 ≡ 13z = 5x + 1 (mod 3), 3|5x ; am obţinut o


contradicţie. Pentru y = 1 să presupunem că x ≥ 1. Atunci 2 ≡ 5x + 12 =
13z ≡ 3z (mod 5), z ≡ 3 (mod 4) (deoarece ordinul lui 3 ı̂n (Z∗5 , ·) este 4).
Avem 132 ≡ −1 (mod 17), 134 ≡ 1 (mod 17) şi 5x = 13z − 12 ≡ 133 − 12 ≡
(−4)3 − 12 = −76 ≡ 9 (mod 17). Ţinând cont că ordinul lui b 5 ı̂n (Z∗17 , ·) este 16
deducem că x = 16t + 10. Pe de altă parte z ≥ 1, 5x = 13z − 12 ≡ 1 (mod 13)
şi 4|x deoarece ordinul lui 5̃ ı̂n (Z∗13 , ·) este 4. Am ajuns la o contradicţie căci
4¤|¤x = 16t + 10. Deci pentru y = 1 avem x = 0 şi z = 1. Să presupunem
acum că y ≥ 2. Avem (−1)x ≡ 5x + 12y = 13z ≡ 1 (mod 3) şi deci x = 2u.

280
Pe de altă parte 1 ≡ 5x + 12y = 13z ≡ 5z (mod 8) şi deci z = 2v. Avem
22y · 3y = 12y = 132v − 52u = (13v − 5u ) (13( v + 5u ). Deoarece 13v + 5u ≡

13v + 5u = 2 · 3a
2 (mod 4) din identitatea precedentă rezultă că , unde
13v − 5u = 22y−1 · 3b
a, b ∈ N, a + b = y. Avem 2 · 5u = 2 · 3a − 22y−1 · 3b , 5u = 3a − 22y−2 · 3b .
Nu putem avea simultan a ≥ 1 şi b ≥ 1 căci ar rezulta că 3|5u , ceea ce
constituie o contradicţie. Nu putem avea nici a = 0 căci ar rezulta contradicţia
13v + 5u = 2 < 22y−1 · 3b = 13v − 5u (am2folosit că y ≥ 2). Deci b = 0, a = y
!y−1 3
4
şi 5u = 3y − 22y−2 = 3y − 4y−1 = 3y−1 43 − 5. Pentru y ≥ 5 avem
3
!y−1 !4
4 4
3− ≤3− < 0 (căci 243 < 256) şi nu poate avea loc egalitatea
3 3
2 !y−1 3
4
5u = 3y−1 43 − 5. Deci 2 ≤ y ≤ 4. Introducând aceste valori ale lui
3
y ı̂n egalitatea 5u = 3y − 4y−1 obţinem că singura soluţie este y = 2, u = 1,
x = 2, z = 2. Am arătat că singurele soluţii ale ecuaţiei 5x + 12y = 13z sunt
x = 0, y = z = 1 şi x = y = z = 2.
9. Evident că x ≥ 1 şi deci 0 ≡ 3x = 2y + 5 ≡ (−1)y + 5 (mod 3). De
aici deducem că y = 2u, u ∈ N. Evident că y ≥ 2 şi deci 1 ≡ 2y + 5 = 3x ≡
(−1)x (mod 4), x = 2v, v ∈ N. Avem, 5 = 32v − 22u = (3v − 2u ) (3v + 2u ),
3v − 2u = 1, 3v + 2u = 5, 2 · 3v = 6, 3v = 3, v = 1, u = 1, x = y = 2.
10. Pentru y = 0 avem x = 3. Să presupunem că y ≥ 1. Avem că
1 ≡ 7 = 2x − 3y ≡ 2x ≡ (−1)x (mod 3) şi deci x = 2u, u ∈ N. Avem că
2x = 7 + 3y ≥ 8, x ≥ 3 şi deci 2x ≡ 0 (mod 8). Dacă y este impar atunci
3 ≡ 3y = 2x − 7 ≡ −7 ≡ 1 (mod 8) şi am obţinut o contradicţie. Deci y = 2v
şi ecuaţia din enunţ se rescrie 7 = 2x − 3y = 22u − 32v = (2u − 3v )(2u + 3v ).
Avem că 2u − 3v = 1 şi 2u + 3v = 7 de unde deducem că 2u+1 = 8, u = 2,
3v = 3, v = 1, x = 4, y = 2. Deci singurele soluţii ale ecuaţiei sunt x = 3,
y = 0 şi x = 4, y = 2.
11. Nu putem avea că p şi q sunt simultan impare căci ar rezulta că pr −q s
este par. Deci unul dintre cele două numere prime este 2; să presupunem că
q = 2. Avem de rezolvat ecuaţiile pr ± 1 = 2s . Să presupunem că r este impar
şi pr + 1 = 2s . Deducem că p + 1|pr + 1|2s , p + 1 = 2t , t < s (deoarece
pr + 1
r ≥ 2) şi = 2s−t este număr par. Aceasta este o contradicţie căci
p+1

281
pr + 1
= pr−1 − pr−2 + pr−3 . . . − p + 1 este număr impar (deoarece p şi r
p+1
sunt impare). La fel aratăm că ecuaţia pr − 1 = 2s nu are soluţii dacă r
este impar. Deci r = 2m. Dacă p2m + 1 = 2s obţinem o contradicţie căci
0 ≡ 2s = p2m + 1 ≡ 2 (mod 4). Avem de rezolvat ecuaţia p2m − 1 = 2s care se
mai scrie (pm − 1)(pm + 1) = 2s , pm − 1 = 2a , pm + 1 = 2b , a < b, a + b = s.
Avem că 2 = 2b − 2a = 2a (2b−a − 1), a = 1, b − a = 1, b = 2, s = 3, pm = 3,
m = 1, p = 3. Deci singura soluţie a ecuaţiei este p = 3, r = 2, q = 2, s = 3
(şi p = 2, r = 3, q = 3, s = 2).

12. Avem soluţiile (0, 0), (1, ±1), (2, ±3). Dacă x = 2k + 1, k ≥ 1, avem
(4 + 1)2k+1 − 42k+1 = y 2 şi deci M8 + 5 = y 2 ; contradicţie. Dacă x = 2k, k ≥ 2
avem (5k − 4k )(5k + 4k ) = y 2 . Cum (5k − 4k , 5k + 4k ) = 1 avem 5k − 4k = a2
şi 5k + 4k = b2 . Rezultă (b − 2k )(b + 2k ) = 5k şi deci b − 2k = 5s şi b + 2k = 5t .
Rezultă 2k+1 = 5s (5t−s − 1). Aşadar s = 0 şi de aici 2k+1 = 5t − 1. Cum
k ≥ 2 avem (4 + 1)t − 1 = M8 şi deci M8 + 4t = M8, adică t = 2u. Din
2k+1 = (5u − 1)(5u + 1) rezultă 5u − 1 = 2m şi 5u + 1 = 2n , adică 2n − 2m = 2
şi deci m = 1 şi contradicţia 5u = 3.
(N. Papacu)

13. Dacă x este impar atunci şi y este impar şi x2 + y 2 = 2 (mod 4).
Deducem că x = 1 şi y = 1. Dacă x este par fie x = 2a · x1 , y = 2a · y1 , unde
x1 , y1 ∈ N, 2¤|¤(x1 , y1 ). Avem 2x−2a = x21 + y12 şi x21 + y12 ≡ 1, 2 (mod 4). Deci
x − 2a ≤ 1. Cum x − 2a este număr natural par şi x − 2a ≤ 1, atunci x = 2a
şi pentru a ≥ 3 obţinem contradicţia 2a = x = 2a · x1 ≥ 2a > 2a (ultima
inegalitate se demonstrază uşor prin inducţie după a ≥ 3). Deci a ∈ {0, 1, 2}
şi x = 2a ∈ {0, 2, 4}. Am arătat că soluţiile ecuaţiei din enunţ sunt (x, y) =
(0, 1), (1, 1), (2, 0), (4.0).
(L. Panaitopol )

14. Rezultă imediat că x = 2X, y = 2Y şi deci X 2 +1 = 2n−1 Y n . Rezultă


că X este impar şi deci X 2 +1 = M4+2. Aşadar n−1 = 1, deci n = 2. Ecuaţia
√ √
se scrie X 2 − 2Y 2 = −1. Rezultă de aici că Xm + Ym 2 = (1 + 2)m cu m
impar.

15. Pentru x = 0 rezultă y = 0. Pentru x = 1 avem y = 1, p = 2.


Pentru x = 2 avem p = 3, y = 2. Presupunem ı̂n continuare că x ≥ 3. Avem
(x+1)(x2 −x+1) = py , x+1 = pα , x2 −x+1 = pβ . Deoarece x2 −x+1 > x+1
pentru x ≥ 3 deducem că 1 ≤ α < β.

282
Avem pβ = (x + 1)2 − 3x = p2α − 3x. Rezultă că p|3. Cum p¤|¤x (deoarece
p|x + 1) deducem că p = 3 şi avem identitatea 3β = 32α − 3x. Deoarece β ≥ 2,
2α ≥ 2, rezultă din ultima egalitate că 9|3x, 3|x; contradicţie căci 3|x şi 3|x+1.
Soluţiile ecuaţiei sunt cele listate mai sus.

16. Dacă m este impar atunci y m−1 + y m−2 + . . . + y + 1 este un divizor


impar al lui 2n . Rezultă că y m−1 + y m−2 + . . . + y + 1 = 1 şi contradicţia y = 0,
2n + 1 = 0 (dacă y ≥ 1 atunci avem inegalitatea y m−1 + y m−2 + . . . + y + 1 ≥
m > 1). Pentru m par deducem din ecuaţia 2n = y m − 1 că y − 1 şi y + 1 sunt
divizori ai lui 2n . Deci y − 1 = 2a , y + 1 = 2b , a < b. Avem 2b − 2a = 2, a = 1,
b = 2, y = 3. Ecuaţia se rescrie 2n + 1 = 3m şi din problema 3 ştim că n = 3
şi m = 2.

17. Fie x = 2i · a, y = 2j · b, unde a şi b sunt numere impare. Deoarece


ecuaţia este simetrică ı̂n x şi y, putem presupune că i ≤ j. Avem că im ≤ n,
am +bm 2(j−i)m = 2n−im . Dacă j > i atunci rezultă că 2n−im = am +bm ·2(j−i)m
este un număr impar, n = im şi am obţinut o contradicţie căci 1 = am + bm ·
2(j−i)m ≥ 2. Deci i = j şi ecuaţia se rescrie am + bm = 2n−im . Dacă m
este par atunci 2n−im = am + bm ≡ 2 (mod 4), n − im = 1, am + bm = 2,
a = b = 1 şi soluţia problemei este x = y = 2i , n = im + 1. Dacă m este
impar atunci (a + b)(am−1 − am−2 b + . . . − abm−2 + bm−1 ) = 2n−im . Deoarece
am−1 − am−2 b + . . . − abm−2 + bm−1 este număt impar (fiind suma a m numere
impare, m fiind şi el impar) rezultă că a + b = 2n−im = am + bm . Deducem că
a(am−1 − 1) + b(bm−1 − 1) = 0, a = b = 1 şi obţinem aceeaşi soluţie ca mai
sus x = y = 2i , n = im + 1.
(L. Panaitopol )

18. Să presupune că m este par. Dacă m = 0 obţinem contradicţia 3 = 6n .


Pentru m ≥ 2, m par, avem 6n = 3m + 4m + 5m ≡ 1 + 0 + 1 = 2 (mod 4). Dar
6n ≡ 2 (mod 4) doar pentru n = 1. Obţinem contradicţia 6 = 3m + 4m + 5m ≥
32 +42 +52 > 6. Să presupunem acum că m este impar. Pentru m = 1 obţinem
contradicţia 6n = 3+4+5 = 12. Deci putem presupune că m este impar, m ≥ 3.
Avem 6n = 3m + 4m + 5m ≡ 3m + (8 − 3)m ≡ m · 3m−1 · 8 ≡ 8 (mod 16). Pentru
n ≥ 4 rezultă contradicţia 8 ≡ 6n ≡ 0 (mod 16). Deci n ∈ {0, 1, 2, 3} , m ≥ 3.
Deoarece 3m + 4m + 5m ≥ 33 + 43 + 53 = 63 deducem că m = n = 3 este
singura soluţie a ecuaţiei.
(L. Panaitopol )

283
19. Să presupunem că (n + 1)(n + 2) · . . . · (n + k) = mk , n ∈ N,
m ∈ N∗ . Avem că (n + 1)k < (n + 1)(n + 2) · . . . · (n + k) = mk < (n + k)k ,
n + 1 < m < n + k, m = n + a, 2 ≤ a ≤ k − 1. Ecuaţia din enunţ se re-
scrie (n + 1)(n + 2) · . . . · (n + a − 1)(n + a + 1) . . . (n + k) = (n + a)k−1 .
Deoarece n + a + 1 ≥ 2, există un număr prim p care divide pe n + a + 1.
Din ultima identitate rezultă că p|n + a. Am obţinut o contradicţie pentru că
(n + a, n + a + 1) = 1.

!m
2j −1
20. Să presupunem că n ≥ 2, m ≥ 2. Avem inegalităţile <
n2
2j −1
, pentru orice j = 1, n. Adunând aceste inegalităţi obţinem o
n2 !m
n
X2j − 1 1 Xn n2
contradicţie, căci 1 = < 2 (2j − 1) = 2 = 1. Deci n = 1
j=1
n2 n j=1 n
∗ ∗
sau m = 1. Se observă imediat că n = 1, m ∈ N şi n ∈ N , m = 1 sunt soluţii
n
X
ale problemei (deoarece (2j − 1) = n2 ) şi conform cu remarcile anterioare
j=1
sunt singurele.
(L. Panaitopol )

21. Să demonstrăm ı̂ntâi că dacă a > b > 0 sunt numere reale, k ≥ 2 este
număr natural şi a + b = 2, atunci ak + bk > 2. Avem 0 < b < 1 < a < 2,
k−1
X
b − 1 = −(a − 1) şi ak + bk − 2 = ak − 1 + bk − 1 = (a − 1) · aj +
j=0
k−1
X k−1
X
(b − 1) bj = (a − 1) (aj − bj ) > 0. Să presupunem ı̂n continuare că
j=0 j=1
!m
2j − 1
n ≥ 2, m ≥ 2. Folosind observaţia precedentă deducem că +
n
" #m
2n − (2j − 1) n+1
> 2 oricare ar fi j = 1, n, j 6= . Dacă n = 2k − 1
n 2
!m " #m
2k − 1 2n − (2k − 1)
atunci + = 2. Sumând inegalităţile de mai sus
n n
„ Žm
n
X 2j − 1 n
X
pentru j = 1, n obţinem că 2 > 2n, (2j − 1)m > nm+1 ;
j=1
n j=1
contradicţie. Deci n = 1 sau m = 1.

284
Se observă imediat că n = 1, m ∈ N∗ şi n ∈ N∗ , m = 1 sunt soluţii ale
n
X
problemei (am folosit că (2j − 1) = n2 ) şi conform cu cele de mai sus, sunt
j=1
singurele.
(L. Panaitopol )

22. Să presupunem că (n − 1)n(n + 1) = mk , unde n ∈ N, n ≥ 2, m ∈ N∗ .


Avem n(n2 − 1) = mk , (n, n2 − 1) = 1 şi deci n = ak , n2 − 1 = bk . Rezultă
k−1
X
că 1 = a2k − bk = (a2 − b) · (a2 )j bk−1−j . Deoarece a, b ∈ N∗ rezultă că
j=0
k−1
X
(a2 )j bk−1−j ≥ k şi din identitatea precedentă rezultă că 1 este multiplu de
j=0
un număr mai mare strict decât 1; am obţinut o contradicţie.

23. x = 0 nu este soluţie şi putem presupune că x ≥ 1. Avem 2y ≡ 7y =


2+5x ≡ 2 (mod 5) şi de aici deducem că y ≡ 1 (mod 4). Cum 74 = (50−1)2 ≡ 1
(mod 25), rezultă că 7y ≡ 7 (mod 25) şi 5x + 2 ≡ 7 (mod 25), 5x ≡ 5 (mod 25).
Dacă x ≥ 2 avem că 5x ≡ 0 (mod 25) şi am obţinut o contradicţie. Deci singura
soluţie este x = 1, y = 1.

24. Să presupunem că y ≥ 5. Rezultă că 3x ≡ 11 (mod 32). Cum ordinul
lui 3 ı̂n U (Z32 , ·) este 8 deducem imediat că x = 8k + 7, k ∈ N. Cum 38 ≡
1 (mod 41) rezultă că 2y ≡ 3x − 11 ≡ 37 − 11 ≡ −33 − 11 = −38 ≡ 3 (mod 41),
! ! !y !
3 2y 2 3
= = = 1. Aceasta este o contradicţie căci =
41 41 41 41
! !
41 2
= = −1. Deci y ≤ 4 şi se verifică uşor că singura soluţie a problemei
3 3
este y = 4, x = 3.

25. Să presupunem că y ≥ 2. Rezultă că 11x ≡ −2 ≡ 11 (mod 13),


x 6= 0, 11x−1 ≡ 1 (mod 13) şi cum ordinul lui 11 ı̂n (Z∗13 , ·) este 12 deducem că
x − 1 = 12k, k ∈ N, x = 12k + 1. Avem că 11x = (13 − 2)12k+1 ≡ −212k+1 +
(12k + 1) · 13 · 212k (mod 132 ) şi cum 2 + 11x ≡ 0 (mod 132 ), 212 ≡ 1 + 3 · 13
(mod 132 ) deducem că 2−2(1+3·13)k +(12k+1)·13(1+3·13)k ≡ 0 (mod 132 ),
2 − 2(1 + 3 · 13k) + 13(12k + 1) ≡ 0 (mod 132 ). Din ultima congruenţă rezultă
că −6 · 13k + 13(12k + 1) ≡ 0 (mod 132 ), −6k + (12k + 1) ≡ 0 (mod 13),
6k ≡ −1 ≡ 12 (mod 13), k ≡ 2 (mod 13), x = 12(13t + 2) + 1, t ∈ N.

285
       
11 53 53 9
Avem că = · (−1)26·5 = = = 1 şi deci 1126 ≡
53 11 11 11
1 (mod 53) conform criteriului lui Euler. Avem că 11 · 13y = 22 + 11x+1 ≡
y
!  
26 6t+1 6t+1
11 · 13 23
22 + (11 ) ≡ 22 + 1 ≡ 23 (mod 53) şi deci = =
53 53
           
53 53 7 23 23 2
· (−1)11·26 = = = · (−1)11·3 = − =− = −1.
23 23 23 7 7 7
         
13 53 26·6
53 1 11 · 13y
Dar = · (−1) = = = 1 şi deci =
53 13 13 13 53
   y    
11 13 11 · 13y 23
· = 1. Mai sus am obţinut că = = −1 şi deci a
53 53 53 53
rezultat o contradicţie. Deducem că y = 0 sau y = 1. Evident y = 0 nu este
soluţie şi rezultă că singura soluţie a ecuaţiei din enunţ este x = y = 1.
(A. Gica)

26. Să presupunem că n este impar. Atunci xn + 2n = y 2 + 2n−1 şi n ≥ 3.


Dacă x ≡ 3 (mod 4), atunci xn + 2n ≡ 3 (mod 4), iar dacă x ≡ 1 (mod 4)
atunci x + 2|xn + 2n , x + 2 ≡ 3 (mod 4). Cum x + 2 ≥ 3, xn + 2n ≥ 3,
deducem existenţa unui divizor prim p ≡ 3 (mod 4) al lui xn + 2n . Rezultă
 n−1
2 n−1
că p|xn + 2n = y 2 + 2 2 şi obţinem contradicţia p|2 2 . Deci n este
par; n = 2k, k ∈ N. Avem 2n−1
= − y2 = (y − x2k + xk )(y
y − xk = 2a , xk ),
y + xk = 2b , a < b, a + b = n − 1 şi a ≥ 1 (deoarece y şi x sunt impare). Rezultă
că 2xk = 2b − 2a , xk = 2b−1 − 2a−1 . Deoarece x este impar şi b − 1 > a − 1 ≥ 0
n
deducem că a = 1, b = n − 2 şi avem de rezolvat ecuaţia x 2 = 2n−3 − 1.
Deoarece 1 = a < b = n − 2 avem că n > 3, n ≥ 4. Dacă x ≥ 5 obţinem
n n
contradicţia 2n−3 > 2n−3 − 1 = x 2 > 4 2 = 2n > 2n−3 . Dacă x = 3 obţinem
n
contradicţia 0 ≡ 3 2 = 2n−3 − 1 ≡ (−1)n−3 − 1 ≡ −2 ≡ 1 (mod 3). Deci x = 1,
2n−3 = 2, n − 3 = 1, n = 4. Am arătat că singura soluţie a ecuaţiei din enunţ
este n = 4, x = 1, y = 3.
(I. Cucurezeanu)

27. Pentru n impar avem xn + 2n = 2n+1 + y 2 . Deoarece x este impar


deducem că există p ≡ 3 (mod 4), p prim, p|xn + 2n (dacă x ≡ 3 (mod 4),
atunci xn + 2n ≡ 3 (mod 4), iar dacă x ≡ 1 (mod 4), atunci x + 2 ≡ 3 (mod 4)
 n+1
2 n+1
şi (x + 2)|xn + 2n ). Atunci p|xn + 2n = y 2 + 2 2 , p|2 2 ; contradicţie.
Deci n este par, n = 2k, k ∈ N∗ , (xk − y)(xk + y) = 2n , xk − y = 2a ,
xk + y = 2b , 1 ≤ a < b, a + b = n (a ≥ 1 deoarece xk − y este par).

286
Avem xk = 2a−1 + 2b−1 şi a = 1 căci xk este număr impar. Deci b = n − 1,
xk = 1 + 2n−2 , n > 2 (deoarece 1 = a < b = n − 1). Dacă x ≥ 5, atunci
2n > 1 + 2n−2 = xk ≥ 5k > 4k = 2n ; contradicţie. Pentru x = 3 avem
contradicţia 0 ≡ 3k = 1 + 2n−2 ≡ 1 + (−1)2k−2 ≡ 2 (mod 3). Deci x = 1 şi
obţinem contradicţia 2n−2 = 0. Deci ecuaţia din enunţ nu are soluţii.
(I. Cucurezeanu)

28. Dacă x este par atunci y este impar şi obţinem contradicţia 0 ≡ xp +
2p = p2 + y 2 ≡ 2 (mod 4). Deci x este impar şi la fel ca ı̂n problema precedentă
deducem existenţa unui divizor prim q ≡ 3 (mod 4) al lui xp + 2p = p2 + y 2 .
Rezultă că q|p, q = p ceea ce constituie o contradicţie căci q ≡ 3 (mod 4), iar
p ≡ 1 (mod 4).
(I. Cucurezeanu)

29. Să presupunem că y este impar. Atunci x este par şi xn ≡ 0 (mod 4)
deoarece n ≥ 3. Am obţinut o contradicţie căci 0 ≡ xn = y 2 + 5 ≡ 1 + 5 ≡ 2
(mod 4). Deci y este neapărat par şi x impar: x = 2z + 1. Rezultă că 1 ≡
5 + y 2 = xn = (2z + 1)n ≡ 1 + 2nz (mod 4) şi deci 2nz ≡ 0 (mod 4), nz ≡ 0
(mod 2), 2|z (deoarece n este impar). Cum z este par deducem că x ≡ 1
(mod 4). Avem identitatea 4 + y 2 = xn − 1 = (x − 1)(xn−1 + xn−2 + . . . + x + 1).
Deoarece xn−1 + xn−2 + . . . + x + 1 ≡ n ≡ 3 (mod 4) şi xn−1 + xn−2 + . . . + x +
1 ≥ 3 deducem că există un număr prim p ≡ 3 (mod 4) astfel ı̂ncât p divide
xn−1 + xn−2 + . . . + x + 1. Din identitatea de mai sus obţinem că p|y 2 + 22 şi
p|y, p|2. Am obţinut o contradicţie şi deci ecuaţia nu are soluţii.

30. Dacă a ≥ 2 şi b ≥ 2 obţinem o contradicţie căci aq−1 ≥ 2q−1 > q


pentru q ≥ 3 (ultima inegalitate se demonstrează prin inducţie) şi deci aq >
aq, bq > bq, aq + bq > q(a + b). Unul din numere trebuie deci să fie 1; să
presupunem că b = 1. Avem de rezolvat ecuaţia aq = qa + q − 1. Demonstrăm
prin inducţie după q că aq > qa + q − 1, oricare ar fi q ≥ 4, a ≥ 2. Pentru
q = 4 trebuie să arătăm că a4 − 4a > 3. Aceasta este adevărat deoarece
a4 − 4a = a(a3 − 4) ≥ 2(8 − 4) = 8 > 3. Presupunem că aq > qa + q − 1.
Deducem că aq+1 > qa2 + aq − a şi ar fi suficient să arătăm că qa2 + aq − a ≥
(q + 1)a + q, ceea ce este echivalent cu q(a2 − 1) ≥ 2a. Ultima inegalitate
este adevărată deoarece q(a2 − 1) ≥ 4a2 − 4 > 2a (ultima inegalitate are loc
deoarece 4a2 − 2a = 2a(2a − 1) ≥ 4 · 3 = 12 > 4). Deci q = 3, a3 + 1 = 3(a + 1),
a2 − a + 1 = 3, a2 − a − 2 = 0, (a + 1)(a − 2) = 0, a = 2. Am arătat că singura
soluţie este q = 3, (a, b) = (2, 1), (1, 2).

287
31. Dacă m ≥ n, atunci xn + y n = (x + y)m ≥ (x + y)n > xn + y n şi am
obţinut o contradicţie. Deci m < n. Fie d = (x, y), x = da, y = db, (a, b) = 1.
Ecuaţia se rescrie
dn−m (an + bn ) = (a + b)m . (1)
Arătăm că n este impar. Să presupunem că n = 2k, k ∈ N∗ . Fie p un divizor
prim al lui an + bn . Din (1) deducem că p|a + b. Deci b ≡ −a (mod p), 0 ≡
an + bn ≡ an + (−a)n ≡ 2an (mod p). Nu putem avea p|a căci din p|a + b ar
rezulta că p|b, ceea ce contrazice faptul că (a, b) = 1. Din p¤|¤a, 0 ≡ 2an (mod p)
deducem că p = 2 şi a2k + b2k = 2t . Dacă t ≥ 2 rezultă o contradicţie căci
a ≡ b ≡ 1 (mod 2), 2 ≡ a2k + b2k = 2t ≡ 0 (mod 4). Rezultă că a2k + b2k = 2,
a = b = 1, x = y = d. Am obţinut o contradicţie. Deci n este ı̂ntr-adevăr
aq + bq
impar. Fie q un divizor prim al lui n şi p un divizor prim al lui .
a+b
Deducem că p|aq + bq |an + bn şi din (1) rezultă că p|a + b. Notăm s = a + b.
q−1
P
Avem că (−1)k sq−k · ak Cqk = (s − a)q + aq = bq + aq şi rezultă că
k=0
aq+ bq
= sq−1 − Cq1 sq−2 a + Cq2 sq−3 a2 + . . . + Cqq−1 aq−1 . (2)
a+b
aq + bq
Din identitatea de mai sus şi din p| , p|s, deducem că p|q · aq−1 .
a+b
aq + bq
Cum p¤|¤a (deoarece p|a + b şi (a, b) = 1) rezultă că p = q şi = qr ,
a+b
r ∈ N∗ . Din q|s, q|Cqj oricare ar fi j = 1, q − 1, q ≥ 3, deducem din (2) că
r
aq + bq
q = ≡ qaq−1 (mod q 2 ). Cum q¤|¤a rezultă că r = 1 şi aq + bq = q(a + b).
a+b
Din problema precedentă ştim că q = 3 şi a = 2, b = 1. Din cele de mai
sus n = 3h şi (1) devine dn−m (2n + 1) = 3m ; deci d ∈ {1, 3}. Dacă d = 3
atunci 2n + 1 = 32m−n şi din problema 3 rezultă că n = 3, 2m − n = 2.
Deducem contradicţia 2m = 5. Deci d = 1, 2n + 1 = 3m şi din problema 3
rezultă că n = 3, m = 2 şi singura soluţie a problemei este n = 3, m = 2,
(x, y) = (1, 2), (2, 1).
32. Dacă n = 0, atunci y ∈ N∗ şi x = y + 2. Să presupunem că n ∈ N∗ .
Putem presupune şi că x > y. Într-adevăr, x 6= y iar dacă y > x atunci din
ecuaţie deducem că n este impar şi y n + xn = (y − x)n+1 . Schimbându-l ı̂n
acest caz pe x cu y putem deci presupunem că x > y. Punem d = (x, y),
x = ad, y = bd, (a, b) = 1 şi ecuaţia din enunţ se rescrie
an + bn = d(a − b)n+1 . (1)

288
Notăm a−b = c, c ∈ N∗ , (b, c) = 1 şi din (1) rezultă că (b+c)n +bn −dcn+1 = 0,
c|2bn , c|2, c ∈ {1, 2}. Dacă c = 1, atunci d = bn + (b + 1)n şi x = (b + 1)[bn +
(b+1)n ], y = b[bn +(b+1)n ]. Dacă c = 2, atunci a = b+2, b = 2k −1 (deoarece
(2k + 1)n + (2k − 1)n
(a, b) = 1), k ∈ N∗ şi a = 2k+1. Cum d = ∈ N∗ deducem
2n+1
că n este impar. Într-adevăr, dacă n este par atunci (2k + 1)n + (2k − 1)n ≡ 2
(mod 4) ceea ce contrazice faptul că 4|2n+1 |(2k + 1)n + (2k − 1)n . Deci n este
impar. Avem că
(2k)n + Cn2 (2k)n−2 + . . . + Cnn−3 (2k)3 + Cnn−1 (2k)
d= . (2)
2n
Notând k = 2j · h, unde h este număr natural impar şi j ∈ N, rezultă că
exponentul lui 2 ı̂n (2k)n +Cn2 (2k)n−2 +. . .+Cnn−3 (2k)3 +Cnn−1 (2k) coincide cu
exponentul lui 2 ı̂n 2k (deoarece n este impar) şi deci este egal cu j +1. Din (2)
deducem că j+1 ≥ n (deoarece d ∈ N∗ ), j ≥ n−1. În acest caz 2k = 2n ·t, t ∈ N
(2n t + 1)n + (2n t − 1)n (2n t + 1)n + (2n t − 1)n
şi y = (2n t − 1) · , x = (2n t + 1) ,
2n+1 2n+1
n fiind impar.
(L. Panaitopol )

33. Valoarea x = 0 nu convine căci ar rezulta că y = −1. Pentru x = 1,


obţinem y = 0. Pentru x = 2, avem de rezolvat ecuaţia 2y = y 2 +y+2. Se arată
uşor, prin inducţie, că 2y > y 2 + y + 2 pentru orice y ≥ 6. Singura soluţie a
ecuaţiei 2y = y 2 + y + 2 este deci y = 5. Pentru y = 0 obţinem x = 1. Nu există
soluţie pentru y = 1. Pentru y = 2, avem de rezolvat ecuaţia x2 − x − 2 = 2x .
Se arată prin inducţie că 2x > x2 − x − 2 oricare ar fi x ∈ N, deci ultima
ecuaţie nu are soluţie. Căutăm ı̂n cele ce urmează soluţii ale ecuaţiei cu x ≥ 3,
ln x
y ≥ 3. Deoarece funcţia f (x) = este strict descrescătoare pe intervalul
x
ln x ln y
[e, ∞), deducem că xy = y x + x + y > y x , y ln x > x ln y, > , x < y,
x y
y ≥ x + 1 (ultima inegalitate are loc deoarece x şi y sunt numere naturale).
Scriem y = x + k, k ∈ N∗ şi avem de rezolvat ecuaţia xx+k − (x + k)x =
!x
k
x+k 2x + k
2x + k care se poate scrie sub forma x − = . Avem că
x xx
!x
2x + k 6+k x+k
≤ < 3k − ek < xk − şi deci ultima ecuaţie nu are
xx 33 x
!x
x+k
soluţii. Am folosit mai sus faptul că < ek (consecinţă a inegalităţii
x

289
!z
1 6+k
1+ < e) şi inegalitatea < 3k − ek . Ultima inegalitate se deduce din
z 33
!k
6+k 7 3 3 7
≤ < −1 ≤ −1 (am folosit mai sus inegalitatea +e < 3).
3 3 · ek 33 · e e e 33
Din toate consideraţiile precedente rezultă că singurele soluţii ale problemei
sunt x = 1, y = 0 şi x = 2, y = 5.

34. Dacă n = 0, a = 1. Dacă n = 1, a = 2. În cele ce urmează presupunem


că n ∈ N, n ≥ 2. Din (a + n)(a − n) = 3n deducem că a − n = 3b , a + n = 3c ,
b, c ∈ N, b < c, b + c = n. Scăzând cele două egalităţi precedente rezultă că
2n = 3c − 3b = 3b (3c−b − 1). Nu putem avea b = 0, căci ı̂n acest caz ar rezulta
că c = n, 2n = 3n − 1 = (1 + 2)n − 1 > 1 + 2Cn1 − 1 = 2n; am obţinut o
contradicţie. Deci b ≥ 1, n = 3b · α, α ∈ N∗ , 3¤|¤α, 2α + 1 = 3c−b = 3n−2b =
b b
33 α−2b > 33 (α−1) ≥ 33(α−1) . Inegalitatea 3b α − 2b > 3b (α − 1) este echivalentă
cu 3b > 2b, care este adevărtă pentru orice b ∈ N∗ . Deci 2α + 1 ≥ 33(α−1) .
Însă pentru α ≥ 2 avem că 33(α−1) = 27α−1 ≥ 1 + 26(α − 1) > 2α + 1 (ultima
26
inegalitate este adevărată deoarece α ≥ 2 > ), ceea ce ne arată că α = 1,
24
n=3 ,3=3b n−2b b
, n − 2b = 1, n = 2b + 1, 3 = 2b + 1, b = 1 (căci pentru b ≥ 2
b
avem că 3 > 2b + 1). Rezultă că n = 3 şi a = 6 ı̂n acest caz. Am arătat deci
că soluţiile ecuaţiei sunt n = 0, a = 1; n = 1, a = 2; n = 3, a = 6.

35. Dacă a = 1, atunci b = 1. Dacă a > 1, atunci b > 1 şi rezultă


imediat că a şi b au aceeaşi factori primi. Fie a = pα1 1 . . . pαr r , b = pβ1 1 . . . pβr r ,
p1 < p2 < . . . < pr , pi prim αi , βi ∈ N∗ pentru orice i = 1, r. Din egalitatea
2 2
ab = ba rezultă că exponenţii lui pi ı̂n ab şi ba sunt egali: αi b2 = βi a pentru
αi a β (k−2) β (k−2)
orice i = 1, r. Notăm k = = 2 = pα1 1 −2β1 . . . prαr −2βr = p1 1 . . . pr r .
βi b
βi b2
Evident că k 6= 1 şi k 6= 2. Să presupunem că k < 1. Atunci t = = =
αi a
p2β
1
1 −α1
. . . p2β
r
r −αr , 2β −α = β (2−k) > 0 şi t ∈ N∗ . Dar p2β1 −α1 . . . p2βr −αr ≥
i i i 1 r
p2β
1
1 −α1
≥ 2 α1 (2t−1) ≥ 22t−1 > t şi se obţine o contradicţie (inegalitatea 22t−1 >

t pentru t ∈ N∗ se arată imediat prin inducţie). Dacă 1 < k < 2 se ajunge


β (k−2) β (k−2)
imediat la o contradicţie din egalitatea k = p1 1 . . . pr r . Dacă k >

2 atunci αi − 2βi = βi (k − 2) > 0, αi − 2βi ∈ N şi k ∈ N. Dacă există
1 ≤ i ≤ r astfel ı̂ncât pi ≥ 5, atunci k = p1α1 −2β1 . . . pαr r −2βr ≥ pαi i −2βi ≥
5αi −2βi = 5βi (k−2) ≥ 5k−2 > k oricare ar fi k ∈ N, k ≥ 3. Ultima inegalitate
se demonstrează prin inducţie şi ajungem la o contradicţie. Deci r ≤ 2 şi

290
dacă r = 2, atunci neapărat p1 = 2, p2 = 3. În acest ultim caz k = 2β1 (k−2) ·
3β2 (k−2) ≥ 6k−2 > k (ultima inegalitate rezultă prin inducţie după k ≥ 3). Deci
neapărat r = 1 şi p1 = 2 sau p1 = 3. Dacă p1 = 2 avem ecuaţia 2β1 (k−2) = k.
Cum 2k−2 > k pentru k ≥ 5 (din nou inducţie), rezultă că k = 3 sau k = 4.
Soluţia convenabilă este k = 4, β1 = 1, α1 = 4. Dacă p1 = 3 avem ecuaţia
3β1 (k−2) = k. Cum 3k−2 > k pentru k ≥ 4 (din nou inducţie), obţinem că
k = 3, β1 = 1, α1 = 3. Concluzionând toate aceste raţionamente avem că
soluţiile problemei sunt (a, b) = (1, 1), (16, 2), (27, 3).

36. Evident că x ≥ 2. Avem că xn+1 −Cn0 xn −Cn1 xn−1 −. . .−Cnn−1 x = 2002
şi de aici deducem că
x|2002 = 2 · 7 · 11 · 13. (1)

Tot din identitatea precedentă rezultă şi că

x2 |nx + 2002. (2)

Din ecuaţia din enunţ obţinem şi că (−1)n+1 ≡ 2001 (mod x + 1),

x + 1|2001 + (−1)n . (3)

Să presupunem că x este par. Din relaţia (2) deducem că n este impar şi
din (3) rezultă că x + 1|2000. Cum x + 1 este impar, deducem că x + 1|125,
x ∈ {4, 24, 124}. Raţionând modulo 4 ı̂n relaţia (2) se observă că nici una din
aceste trei valori nu convine. Deci x este impar. Din (1) rezultă că x aparţine
mulţimii {7, 11, 13, 77, 91, 143, 1001}. Din (3) deducem că x + 1 este divizor
al lui 2000 sau 2002. Folosind această observaţie se exclud valorile 11, 77,
91, 143, 1001. Dacă x = 7, atunci din (3) rezultă că n este impar. Obţinem
contradicţia 0 ≡ 2001 = 7n+1 − 8n ≡ 1 − (−1)n ≡ 2 (mod 3). Deci x = 13.
Din (2) deducem că 132 |13n + 2002, 13|n + 154, n ≡ 2 (mod 13), iar din (3)
deducem că n este par. Deci „n = „ 2 + 26t,
Žn Žt ∈ N. Să presupunem că t ≥ 2 şi
14
13n+1 − 14n = 2001 = 13n 13 − . Dar n ≥ 54 (deoarece t ≥ 2) şi
13
„ Žn „ Ž54
14 1 54 54 · 53
≥ 1+ ≥ 1+ + > 1 + 4, 15 + 8, 46 > 13. Am obţinut
13 13 13 2 · 132
„ „ Žn Ž
14
o contradicţie, căci 13n 13 − = 2001 ar trebui să fie negativ. Dacă
13
t = 1, atunci n = 28 şi obţinem contradicţia 1 ≡ 2001 = 1329 − 1428 ≡ 3 − 1 ≡

291
2 (mod 5). Rezultă că t = 0, n = 2. Cum 133 − 142 = 2001, toate consideraţiile
precedente ne arată că x = 13, n = 2 este singura soluţie a problemei.

37. Dacă n = 2m, m ∈ N atunci 89 = x2 − (10m )2 , x − 10m = 1,


x + 10m = 89, 2 · 10m = 88, 10m = 44; contradicţie. Dacă n = 2m + 1, m ∈ N

atunci x2 − 10y 2 = 89, y = 10m . Avem că x + y 10 > 1; există deci k ∈ N
√ √ √ √
astfel ı̂ncât (x + y 10)(19 − 6 10)k+1 < 1 ≤ (x + y 10) · (19 − 6 10)k
√ √ √
(deoarece lim (19 − 6 10)k = 0 şi x + y 10 > 1). Notăm A + B 10 =
√ k→+∞ √
(x + y 10)(19 − 6 10)k , A, B ∈ Z. Deoarece 192 − 62 · 10 = 1 deducem
√ √
că A2 − 10B 2 = x2 − 10y 2 = 89. Avem şi 1 ≤ A + B 10 < 19 + 6 10. Dar
√ 89 √ √ √
0 < A−B 10 = √ ≤ 89, −89 ≤ B 10−A < 0, 2B 10 < 19+6 10,
√ A + B 10
19 + 6 10 √ − 88
B< √ < 7, 2B 10 ≥ −88, B ≥ √ > −14, B ∈ [−13, 6] ∩ Z. Se
2 10 2 10
observă că A = 27, B = −8 şi A = 33, B = −10 sunt singurele soluţii care
convin. Deci
√ √ √
x + y 10 = (33 − 10 10)(19 + 6 10)k (1)
sau
√ √ √
x + y 10 = (27 − 8 10)(19 + 6 10)k (2)
√ √ √
Dacă ar avea loc ecuaţia (1) atunci x + y 10 ≡ −10 10 ≡ − 10 (mod 3) şi
deci 1 ≡ 10m = y ≡ −1 (mod 3); contradicţie. Notăm

ak = 19k + Ck2 19k−2 · 62 · 10 + Ck4 19k−4 · 64 · 102 + . . . (3)

bk = Ck1 19k−1 · 6 + Ck3 19k−3 · 63 · 10 + . . . (4)


√ k √ √ √
(19+6 10) ≡ ak +bk 10. Ecuaţia (2) se rescrie x+y 10 = (27−8 10)(ak +

bk 10), 10m = y = 27bk − 8ak . Să presupunem că m ≥ 3. Atunci bk ≡ 0
(mod 8), 6 · 19k−1 Ck1 ≡ 0 (mod 8), 3 · 19k−1 k ≡ 0 (mod 4), k = 4h, h ∈ N.
k
Rezultă că ak ≡ 6k · 10 2 ≡ 64h · 102h (mod 19), bk ≡ 0 (mod 19), 10m = y =
! ! ! ! !
10m − 8ak −8 64h 102h
27bk − 8ak ≡ −8ak (mod 19), = = =
19 19 19 19 19
! ! ! !m !m
−2 2 10m 10 −9
= − = 1. Deci 1 = = = =
19 19 19 19 19
!m
−1
= (−1)m , m = 2t, t ∈ N. Rezultă că ecuaţia din enunţ se rescrie
19

104t+1 + 89 = x2 . (5)

292
Dar 102 ≡ −1 (mod 101), 104 ≡ 1 (mod 101) şi deci x2 ≡ 104t+1 + 89 ≡ 10 +
! !
x2 99
89 = 99 (mod 101). Am obţinut o contradicţie căci 1 = = =
101 101
! !
−2 2
= = −1. Deci m ∈ {0, 1, 2} şi n ∈ {1, 3, 5}. Se observă uşor
101 101
că doar n = 3 este soluţie a ecuaţiei şi din cele de mai sus este singura
(n = 3, x = 33).
(A. Gica)

293
CAPITOLUL 17

Diverse

1. Dacă ak = m2 , avem an+k = ak +nr = m2 +nr. Alegem n = 2mh+rh2 ,


h ∈ N.
n
X
2. Avem ai ai ∈ {−1, 1} cu an+1 = a1 şi dacă s = ai ai+1 = 0, rezultă că
i=1
n este par. Fie k numărul schimbărilor de semn din secvenţa a1 , a2 , . . . , an , a1 .
n
Y
Rezultă p = (ai ai+1 ) = (−1)k deoarece numărul schimbărilor de semn este
i=1
n
Y
numărul de cazuri ı̂n care ai ai+1 = −1. Pe de altă parte, p = a2i = 1 şi din
i=1
.
(−1)k = 1 rezultă k = 2h. Deoarece s = 0, rezultă n = 2k = 4h şi deci n .. 4.
Reciproc, dacă n = 4h, putem alege ai , i ∈ 1, n astfel ca s = 0. Considerăm
secvenţa 1, 1, −1, −1, 1, 1, −1, −1, . . . , 1, 1, −1, −1 şi rezultă s = 0.

3. Arătăm ı̂ntâi prin inducţie că a2k+1 ≡ 4 (mod 9) şi a2k+2 ≡ 7 (mod 9),
∀ k ∈ N. a1 = 4 şi a2 = 7 şi verificarea este făcută. Presupunem enunţul
adevărat pentru k şi ı̂l demonstrăm pentru k + 1. Avem a2k+3 = a22k+2 −
3a2k+2 + 3 ≡ 49 − 21 + 3 = 31 ≡ 4 (mod 9) şi a2k+4 = a22k+3 − 3a2k+3 + 3 ≡
16 − 12 + 3 ≡ 7 (mod 9). În particular, rezultă că 3¤|¤an , ∀ n ∈ N∗ . Înmulţim
egalităţile aj+1 − 3 = aj (aj − 3) pentru j = 1, m − 1 şi obţinem (ţinând cont
că aj 6= 3 ∀ j = 1, m − 1) că am − 3 = a1 a2 . . . am−1 , am = a1 a2 . . . am−1 + 3.
a) Fie m > n ≥ 1 şi să presupunem că există p prim astfel ı̂ncât p|am şi
p|an . Din am = a1 a2 . . . am−1 + 3, m − 1 ≥ n, p|am , p|an , deducem că p|3,
p = 3 şi am obţinut contradicţia 3|an .
b) an an+1 − 1 = (an − 1)3 .
c) Să presupunem că an = a3 , a ∈ Z. Atunci an = a3 ≡ 0, 1, 8 (mod 9),
ceea ce contrazice congruenţa an ≡ 4, 7 (mod 9) demonstrată mai sus.
1 1 1 1 2
d) Demonstrăm prin inducţie că + +··· + < − , ∀ n ≥ 2.
a1 a2 an 2 an+1
11 1 1 1 2 27
Pentru n = 2 trebuie să verificăm inegalitatea = + < − = ,
28 4 7 2 31 62

295
1 1 1 1 2
care este adevărată. Presupunem că + + ··· + < − . Atunci
a1 a2 an 2 an+1
n+1
X 1 1 2 1 1 1 1 2 2 1
< − + = − < − ⇔ < ⇔ an+2 >
j=1
aj 2 an+1 an+1 2 an+1 2 an+2 an+2 an+1
2an+1 ⇔ a2n+1 − 5an+1 + 3 > 0. Ultima inegalitate este adevărată deoarece

5 + 13
an+1 = a1 a2 . . . an + 3 ≥ 31 > . Am demonstrat pasul de inducţie.
2
Afirmaţia de la punctul d) devine ı̂n acest moment evidentă.
(L. Panaitopol )

  !
n
X X 1 n X 1 1 X∞ 1
4. Avem ak = ≤ n 2
< n + <
k=2 p prim
p p p prim
p 4 k=1 (2k + 1)2
p≤n p≤n
! !
1 1X ∞ 1 1 1 n
n + =n + = şi deducem din inegalitatea mediilor
4 4 k=1 k(k + 1) 4 4 2
√ a2 + a3 + · · · + an n
că a2 a3 . . . an ≤
n−1 < . Deducem că a2 a3 . . . an ≤
n−1 2(n − 1)
 n−1  
1 1 3 1 m
1 + < (am folosit că 1 + < e < 3 ∀ m ∈ N∗ ).
2n−1 n−1 2n−1 m
1 1 1 1
Avem a2 = , a3 = , a4 = , a5 = şi din cele de mai sus rezultă că
2 3 2 5


X X∞
1 1 1 1 1 23 3 229
a2 a3 . . . ak < + + + +3 j
= + = < 1.
k=2
2 6 12 60 j=5
2 30 16 240

(L. Panaitopol )

5. Dacă 0 < a1 ≤ a2 ≤ · · · ≤ an , 0 < b1 ≤ b2 ≤ · · · ≤ bn şi σ ∈ Sn , atunci


n
X Xn Xn 1 2
aj bn+1−j ≤ aj bσ(j) ≤ aj bj . De aici deducem că + +· · ·+
j=1 j=1 j=1
n+1 n+2
n 1 2 n
≤ nx ≤ + + ··· + şi că n(1 − sn ) ≤ nx ≤ (2n + 1)sn − n,
2n 2n 2n − 1 n+1
2n + 1 1 1 1
1 − sn ≤ x ≤ sn − 1, unde sn = + + ··· + . Pentru a
n n+1 n+2 2n
2n + 1
demonstra enunţul este suficient să arătăm că 0, 3 < 1−sn şi sn−1 < 0, 4
n
pentru n ≥ 51. Să observăm că sn este un şir crescător convergent la ln 2.

296
1 1 1
Într-adevăr, sn+1 − sn = > 0 şi sn = 1 + + · · · + −
(2n + 1)(2n + 2) 2 2n
   
1 1 1 1
ln 2n − 1 + + · · · + − ln n + ln 2. Cum lim 1 + + · · · + − ln m = γ
2 n m→∞ 2 m
(unde γ este constanta lui Euler), rezultă din cele de mai sus că sn este strict
crescător şi lim sn = ln 2. Deci sn < ln 2 = 0, 6931 · · · < 0, 7 ∀ n ∈ N∗
n→∞
2n + 1
şi 0, 3 < 1 − sn . Pentru a demonstra că sn < 1, 4, este suficient să
n
1, 4 · n
observăm că sn < ln 2 < ∀ n ≥ 51. Ultima inegalitate este echivalentă
2n + 1
cu ln 2 < n(1, 4 − 2 ln 2) ∀ n ≥ 51. Pentru n ≥ 51 avem că n(1, 4 − 2 ln 2) ≥
51(1, 4 − 1, 3864) = 51 · 0, 0136 = 0, 6936 > ln 2 şi enunţul este demonstrat.
(L. Panaitopol )
√ n √
6. a) (1 + i a) = an + i abn , unde bn = Cn1 − Cn3 a + Cn5 a2 . . . . Găsim
√ √ n+1
o formulă de recurenţă pentru an . Avem an+1 + i abn+1 = (1 + i a) =
√ √
(an + i abn ) (1 + i a), an+1 = an − abn , bn+1 = an + bn . Din prima formulă
an − an+1
obţinem că bn = . Înlocuind bn şi bn+1 ı̂n cea de-a doua formulă,
a
an+1 − an+2 an − an+1
obţinem = an + , an+2 = 2an+1 − (a + 1)an . Arătăm
a a
că 2n−1 |an ∀ n ∈ N prin inducţie după n. 20 |a1 , 21 |a2 = 1 − a = 2 − 4k.

.
Presupunem că 2n−1 |a şi 2n |a
n n+1 . Deoarece a + 1 = 4k .. 4, deducem din
formula de recurenţă an+2 = 2an+1 − (a + 1)an , că 2n+1 |an+2 .
(Dorel Miheţ)
€ √ Š2n+1 √
b) 1 + 8 = a + b 8, a = 1 + C2n+12 4
8 + C2n+1 82 + . . . .
€ √ Š2n+1 √ € √ Š2n+1 € √ Š2n+1
Avem 1 − 8 = a − b 8 şi 1 + 8 1− 8 = (−7)2n+1 =
−72n+1 = a2 − 8b2 . Dacă am avea că 5|b, atunci ultima egalitate ne-ar conduce
la congruenţa −72n+1 ≡ a2 (mod 5). Cum 5¤|¤a, rezultă că a2 ≡ 1, 4 (mod 5).
Deci 1, 4 ≡ a2 ≡ −7 · (7n )2 ≡ 3, 2 (mod 5). Am obţinut o contradicţie.
Deci 5¤|¤b.
7. Vom arăta enunţul prin inducţie după n. Avem a0 = a1 = 1, a2 = 4,
a3 = −11. Să presupunem că a0 , a1 , . . . , an+2 sunt numere ı̂ntregi şi vrem să
arătăm că an+3 este ı̂ntreg. Să presupunem că 5|an şi 5|an+1 . Fie j ≥ 3 cel
mai mic număr natural cu proprietatea că 5|ak , ∀ k = j, n + 1. Din aj =
5 − a2j−1
, aj aj−2 = 5 − a2j−1 , aj−2 , aj−1 , aj ∈ Z, 5|aj , deducem că 5|aj−1 ;
aj−2
contradicţie cu definiţia lui j. Deci nu putem avea 5|an şi 5|an+1 . Arătăm

297
acum că (an , an+1 ) = 1. Să presupunem că există un număr prim p astfel
ı̂ncât p|an , p|an+1 . Din an+2 · an = 5 − a2n+1 , an , an+1 , an+2 ∈ Z, p|an , p|an+1 ,
deducem că p = 5, ceea ce constituie o contradicţie. Deci (an , an+1 ) = 1 şi
5 − a2n+2
pentru a arăta că an+3 = este ı̂ntreg, este suficient să arătăm că
a
€ Š € n+1 Š € Š2
a2n 5 − a2n+2 a2n 5 − a2n+2 5a2n − 5 − a2n+1 5a2n − 25
∈ Z. Dar = = +
an+1 an+1 an+1 an+1
10an+1 − an+1 3 = −5an−1 + 10an+1 − a3n+1 ∈ Z şi enunţul este demonstrat.
(L. Panaitopol )

8. Fie q prim şi N = 102q + 10q + 1 ≥ 10101. Avem că N ≡ 1 + 1 + 1 = 3


(mod 9). Există p prim astfel ı̂ncât p|N şi p 6= 3, 37. În caz contrar am avea că
N = 3 · 37t , t ∈ N∗ . Se obţine o contradicţie căci 3 · 37t ≡ 3 (mod 4), pe când
N = 102q +10q +1 ≡ 1 (mod 4). Există deci p cu proprietăţile indicate (evident
1 1
că p 6= 2, 5). Perioada lui este 3q. Perioada f a lui este ordinul lui 10 ı̂n
p p
(Z∗p , ·). Din p|N rezultă că 0 ≡ 102q +10q +1 (mod p) şi 103q ≡ 1 (mod p). Deci
f |3q şi f ∈ {1, 3, q, 3q}. Dacă f = 1, 3, rezultă că p = 3 sau p = 37, ceea ce este
imposibil. Dacă f = q, atunci 0 ≡ 102q + 10q + 1 ≡ 3 (mod p) şi contradicţia
p = 3. Deci f = 3q. La fiecare q prim se asociază un prim p ca mai sus şi ı̂n
acest fel rezultă infinitatea numerelor prime cu proprietăţile cerute (la numere
1
diferite q corespund numere diferite p, căci are perioada 3q).
p
Pentru partea a doua a problemei, fie p un număr prim, p 6= 2, 3, 5 astfel
1 1
ı̂ncât perioada lui este 3k şi = 0, (a1 , a2 , . . . , a3k ). Notăm bj = 0, aj aj+1 . . .
p p
( j−1 )
10 1
şi evident că aj < 10bj şi bj = . Deoarece perioada lui este 3k, avem
p p
că p|103k − 1 şi p¤|¤10k − 1. Cum 103k − 1 = (10k − 1)(102k + 10k + 1), din cele
de mai€ sus rezultă c㊠p|102k + 10k + 1. Fie 1 ≤ i ≤ k. bi + bi+k + bi+2k =
" i−1 # " i−1+k # " i−1+2k #
10i−1 1 + 10k + 102k 10 10 10
− − − ∈ Z, deoarece
p p p p
p|102k + 10k + 1. Deci bi + bi+k + bi+2k ∈ N. Cum 0 < bi + bi+k + bi+2k < 3,
rezultă că bi +bi+k +bi+2k ≤ 2. Cum aj < 10bj , rezultă că 0 ≤ ai +ai+k +ai+2k <
10(bi + bi+k + bi+2k ) ≤ 20. Deci maximul căutat este cel mult 19. Deoarece
1
= 0, (142857), 4 + 8 + 7 = 19, rezultă că maximul căutat este chiar 19.
7

298
p−1
!
Y Cpk
9. Arătăm că E = . Cum p|Cpk ∀ k = 1, p − 1, enunţul rezultă
k=1
p
imediat.

p−1
Y Cpk [(p − 1)!]p−1 [(p − 1)!]p−1
= p−1
! p−1
!= !2 .
k=1
p Y Y p−1
Y
k! (p − k)! k!
k=1 k=1 k=1
k apare la numărător la puterea p − 1 iar la numitor este prezent ı̂n k!,
(k + 1)!, . . . (p − 1)! şi deci k apare la numitor la puterea 2(p − k). Deci
p−1 p−1 p−1
Y Cpk Y Y
= k p−1−2(p−k) = k 2k−p−1 = E.
k=1
p k=1 k=1
(L. Panaitopol )

an+1 + 2n+1 + 1
10. Notăm x = ∈ N∗ . Evident că x ≥ 2. Avem
an + 2 n + 1
an (a − x) = 2n (x − 2) + x − 1. Din ultima egalitate şi din faptul că x ≥ 2,
deducem an (a − x) > 0, a > x, a ≥ x + 1 ≥ 3. Avem egalitatea
 ‹n
a x−1
(a − x) = (x − 2) + n . (1)
2 2
 n  !n  
a a a n(a − 2)
Cum = 1+ −1 ≥ 1+n −1 = 1+ , din (1)
2 2 2 2
" #  
x−1 n(a − 2) 1 n
deducem că (x − 2) + n ≥ (a − x) 1 + şi x 2 + n + (a − 2) ≥
2 2 2 2
1 an(a − 2)
2+ n
+a+ . Deoarece a − 1 ≥ x, din ultima inegalitate rezultă
2 2 !
1 n(a − 2) 1 an(a − 2) a−2
că (a − 1) 2 + n + ≥ 2+ n+a+ şi a − 2 + n ≥
2 2 2 2 2
n(a − 2) n(a − 2) 2
2+ > . Înmulţind ultima inegalitate cu > 0, obţinem
2 2 a−2
1 a3 + 9
n < 2 + n−1 ≤ 3 şi deci n = 1 sau n = 2. Pentru n = 2 obţinem 2 =
2 a +5
∗ 3 2 3 2
x ∈ N . Dar x ≤ a − 1 şi deducem a + 9 ≤ (a + 5)(a − 1) = a − a + 5a − 5,
a2 −5a+14 ≤ 0. Ultima inegalitate este imposibilă, căci discriminantul ecuaţiei
a2 + 5
este ∆ = 25 − 4 · 14 = −31 < 0. Deci n = 1 şi x = ∈ N∗ . Dar
a+3
14
x = a−3+ şi deducem că a + 3|14, a + 3 ≥ 4. Deci a + 3 = 7 sau
a+3

299
a + 3 = 14, de unde deducem că singurele soluţii ale problemei sunt n = 1 şi
a = 4 sau a = 11.
Observaţie. Acelaşi enunţ apare cu o altă demonstraţie şi ı̂n Capitolul 3,
problema 22.
(L. Panaitopol )

11. Evident că b = 0, a ∈ N, respectiv b = 1, a par, a ≥ 2 şi a = 0,


b ∈ N, b 6= 1, reprezintă soluţii ale problemei. În continuare vom presupune
a2
că b ≥ 2 şi a ∈ N∗ . Cum ∈ N∗ , deducem că 2ab2 − b3 + 1 ≥ 1
2ab2 − b3 + 1
a2
şi ≥ 1. Din prima inegalitate rezultă că 2a ≥ b, iar din cea de-a
2ab2 − b3 + 1
doua rezultă că a2 − 2ab2 + b3 − 1 ≥ 0. Deoarece rădăcinile ecuaţiei de grad 2,
√ √
x2 − 2b2 x + b3 − 1 = 0, sunt x1 = b2 − b4 − b3 + 1 şi x2 = b2 + b4 − b3 + 1,

inecuaţia a2 − 2b2 a + b3 − 1 ≥ 0 ne asigură că a ≤ b2 − b4 − b3 + 1 sau a ≥
√ √
b2 + b4 − b3 + 1. Să presupunem că a ≤ b2 − b4 − b3 + 1. Să presupunem şi că
a2 a2
2a > b. Avem că 2a−b ≥ 1, 2ab2 −b3 +1 ≥ b2 +1 şi 1 ≤ ≤ ,
√ 2ab2 − b3 +√ 1 b2 + 1
a2 ≥ b2 +1, a ≥ b2 + 1 > b. Ţinând cont de inegalitatea a ≤ b2 − b4 − b3 + 1,
√ √
obţinem inegalităţile b < a ≤ b2 − b4 − b3 + 1, b4 − b3 + 1 < b2 − b, b4 −
b3 + 1 < b4 − 2b3 + b2 , b3 − b2 + 1 < 0, b2 (b − 1) + 1 < 0; ultima inegalitate
constituie o contradicţie căci b ≥ 2. Deci ı̂n acest caz, neapărat 2a = b şi
b
soluţiile problemei sunt constituite din toate numerele pare b şi a = .
√ 2
Să presupunem acum că a ≥ b2 + b4 − b3 + 1; vom arăta că ı̂n acest caz b este
b4 − b √
de asemenea par şi a = . Să observăm ı̂ntâi că a ≥ b2 + b4 − b3 + 1 ≥
2
a2 2a2 b2
b2 + (b2 − b) = 2b2 − b. Deoarece ∈ N, avem că =
2ab2 − b3 + 1 2ab2 − b3 + 1
ab3 − a ab3 − a
a+ este un număr natural. Deci ∈ N. Evident că
2ab2 − b3 + 1 2ab2 − b3 + 1
2ab3 − 2a b4 − b − 2a
şi = b + este număr natural, de unde deducem
2ab2 − b3 + 1 2ab2 − b3 + 1
b4 − b − 2a − 2ab2 + b6 − b3 b6 − 2b3 + 1
că ∈ Z. Avem şi că = −1 +
2ab2 − b3 + 1 2ab2 − b3 + 1 2ab2 − b3 + 1
3
(b − 1) 2
este număr ı̂ntreg, de unde rezultă că ≥ 1 (am ţinut cont că
2ab2 − b3 + 1
b ≥ 2). Din ultima inegalitate obţinem că b6 − 2b3 + 1 ≥ 2ab2 − b3 + 1,
b6 − b3 ≥ 2ab2 , 2a ≤ b4 − b. Să presupunem că b4 − b > 2a. În acest caz, din

300
2ab3 − 2a b4 − b − 2a
cele de mai sus rezultă că = b+ ≥ b+1, 2ab3 −2a ≥
2ab2 − b3 + 1 2ab2 − b3 + 1
(b + 1)(2ab2 − b3 + 1) = 2ab3 − b4 + b + 2ab2 − b3 + 1, 2a(b2 + 1) ≤ b4 + b3 − b − 1,
b4 + b3 − b − 1 b2 + b
a ≤ ≤ . Am obţinut ı̂n acest moment o contradicţie,
2(b2 + 1) 2
b2 + b b2 + b
căci ≥ a ≥ 2b2 − b, inegalitatea ≥ 2b2 − b fiind imposibilă pentru
2 2
b4 − b
b ≥ 2. Singura posibilitate rămâne aceea ca a = . Introducând ı̂n relaţia
2
(b4 − b)2 b2 (b3 − 1)2 b2
iniţială această formulă, deducem că = =
4(b6 − b3 − b3 + 1) 4(b3 − 1)2 4
trebuie să fie număr natural. Rezultă că b este par şi totul este demonstrat ı̂n
acest moment.

12. Fie A = {a1 < a2 < a3 < a4 < a5 < a6 < a7 }. Să presupunem că
ai+1
enunţul n-ar fi adevărat. Atunci > 2 ∀ i = 1, 6, ai+1 > 2ai , ai+1 ≥ 2ai + 1
ai
∀ i = 1, 6. a7 ≥ 2a6 + 1 ≥ 22 a5 + 22 − 1 ≥ 23 a4 + 23 − 1 ≥ · · · ≥ 26 a1 + 26 − 1.
Se obţine contradicţia a7 ≥ 64 + 63 = 127.

13. Există n + 1 elemente ı̂n A de aceeaşi paritate (ı̂n caz contrar, ar


exista cel mult n elemente pare ı̂n A, cel mult n elemente impare ı̂n A şi deci
cel mult 2n elemente ı̂n A; aceasta contrazice faptul că |A| = 2n + 1). Fie
aceste elemente 1 ≤ b1 < b2 < · · · < bn+1 ≤ 3n.
( )
b1 + b2 b1 + b3 b1 + bn+1
Notăm B = , ,..., . Evident că B ⊆ {1, 2, . . . , 3n}
2 2 2
deoarece b1 ≡ bj (2) ∀ j = 1, n + 1. Dacă cumva A ∩ B = ∅, obţinem
contradicţia 3n = |{1, 2, . . . , 3n}| ≥ |A ∪ B| = |A| + |B| = 2n + 1 + n = 3n + 1.
b1 + bj
Deci A ∩ B 6= ∅ şi ∃ 2 ≤ j ≤ n + 1 astfel ı̂ncât ∈ A. Cum b1 , bj ∈ A,
2
rezultă că media aritmetică a celor două numere din A este tot ı̂n A.

14. Cazul n = 2t. A = {1, 3, 5, . . . , 2t−1}, |A| = t şi 6 ∃ a, b, c ∈ A a. ı̂. a+


b = c. Fie A ⊆ {1, 2, . . . , 2t} astfel ı̂ncât |A| ≥ t + 1. Fie a1 < a2 < · · · < at+1 ,
t+1 elemente din A. |{a1 , . . . , at+1 }|+|{at+1 −at , at+1 −at−1 , . . . , at+1 −a1 }| =
t + 1 + t = 2t + 1 > |{1, 2, . . . 2t}|. Cum {a1 , . . . , at+1 } ∪ {at+1 − at , . . . , at+1 −
a1 } ⊆ {1, . . . , 2t|}, rezultă că cele două submulţimi au cel puţin un element
comun şi deci ∃ a, b, c ∈ A astfel ı̂ncât a + b = c. La fel se raţionează ı̂n cazul
n = 2t + 1.

301
15. Notăm P (n) propoziţia din enunţ. Vom demonstra enunţul prin
inducţie. Pentru n = 1 avem X = {−1, 0, 1} şi avem 1 + (−1) = 0. Fie X
cu n + 3 elemente x cu |x| ≤ n + 1. Dacă n + 1 ∈ X, celelalte numere din in-
tervalul −n − 1, n + 1 se ı̂mpart ı̂n perechile (−n − 1, 0), (−n, 1), . . . , (−1, n).
Cum ı̂n X există n + 3 elemente, rezultă că există k ∈ 0, n cu k ∈ X şi
k − n − 1 ∈ X. Notăm a = k − n − 1, b = n + 1, c = k şi avem a + b = c cu
a, b, c ∈ X. Procedăm analog dacă −(n+1) ∈ X. Dacă {−n−1, n+1}∩X = ∅,
atunci pentru x ∈ X avem |x| ≤ n şi problema se reduce la P (n).

n(n + 1)
16. Să presupunem că n are calităţile din enunţ. Atunci =
X X X X X 2
1 + 2 + ··· + n = x= x+ x+ x=3 x. Deci 3|n sau
x∈A∪B∪C x∈A x∈B x∈C x∈A
n ≡ 2(3). Se observă că n = 2 şi n = 3 nu sunt soluţii pentru problemă. Deci
condiţiile necesare sunt n ≡ 0, 2 (mod 3) şi n ≥ 5. Arătăm că sunt şi suficiente.
Se foloseşte o inducţie cu pasul 6. Presupunem enunţul adevărat pentru n şi
notăm cu A, B, C mulţimile corespunzătoare. Atunci A1 = A ∪ {n + 1, n + 6},
B2 = B ∪{n+2, n+5}, C1 = C ∪{n+3, n+4} sunt mulţimile corespunzătoare
lui n + 6. Pentru n = 5, alegem A = {1, 4}, B = {2, 3}, C = {5}, pentru n = 6
alegem A = {1, 6}, B = {2, 5}, C = {3, 4}, pentru n = 8 alegem A = {1, 3, 8},
B = {2, 4, 6}, C = {5, 7} şi pentru n = 9, A = {1, 5, 9}, B = {2, 6, 7},
C = {3, 4, 8}.

17.
¦ ©
A = a2 , (a + 5)2 , (a + 8)2 , (a + 9)2 , (a + 13)2 , (a + 16)2 |a = 18t + 1, t = 0, k − 1 .
¦ ©
B = (a + 1)2 , (a + 4)2 , (a + 6)2 , (a + 11)2 , (a + 14)2 , (a + 15)2 |a = 18t+1, t = 0, k−1 .
¦ ©
A = (a + 2)2 , (a + 3)2 , (a + 7)2 , (a + 10)2 , (a + 12)2 , (a + 17)2 |a = 18t+1, t = 0, k−1 .

18. Să presupunem că ∃ A ⊆ {1, 2, . . . , 2n}, |A| = n + 1 şi a¤|¤b ∀ a, b ∈ A,


a 6= b. Notăm elementele lui A cu a1 < a2 < · · · < an+1 şi alegem acea mulţime
A cu proprietăţile de mai sus şi a1 maxim. Deoarece an+1 ≤ 2n, rezultă că a1 ≤
n. Considerăm B = {2a1 , a2 , . . . , an+1 }. Avem că B ⊆ {1, 2, . . . , 2n} şi |B| =
n + 1 deoarece 2a1 6= aj ∀ j = 2, n + 1 (deoarece a1 ¤|¤aj ∀ j = 2, n + 1). În plus,
B are proprietatea că b¤|¤c ∀ b, c ∈ B, b 6= c. Aceasta rezultă din proprietăţile
lui A. Dacă 2a1 |aj , rezultă că a1 |aj , ceea ce contravine proprietăţilor lui A.
Dacă aj |2a1 (j ≥ 2), atunci 2a1 = kaj . k 6= 1 căci a1 ¤|¤aj şi k 6= 2 căci a1 6= aj .
2a1 2a1
Deci k ≥ 3, aj = ≤ < a1 şi se contrazice faptul că a1 este cel mai mic
k 3
element al lui A. Deci B are aceleaşi proprietăţi ca şi A şi, ı̂n plus, min B =

302
min{2a1 , a2 } > a1 , lucru care contrazice maximalitatea lui a1 . Problema este
astfel rezolvată.

19. Vom arăta că maximul căutat este 1959.


Fie A = {45, 46, 47, . . . , 2003}. Evident că |A| = 1959 şi a · b 6∈ A pentru
∀ a, b ∈ A (aceasta deoarece 452 = 2025 > 2003).
Mai avem de arătat că ∀ A ⊆ {1, 2, . . . , 2003}, |A| = 1960, există a, b ∈ A cu
proprietatea că a · b ∈ A.
Presupunem prin absurd că ab 6∈ A ∀ a, b ∈ A. Notăm elementele mulţimii
A cu ai , i = 1, 1960, 1 ≤ a1 < a2 < · · · < a1960 ≤ 2003.
2003
Fie 1 ≤ k ≤ 1960 astfel ı̂ncât ak ≤ < ak+1 . Cum mulţimea
a1
{1, 2, . . . , a1 − 1} ∪ {a21 , a1 a2 , . . . , a1 ak } este inclusă ı̂n {1, 2, . . . , 2003}\A, de-
ducem că a1 − 1 + k ≤ |{1, 2, . . . , 2003}\A}| = 2003 − 1960 = 43. Pe de altă
( " #)
2003
parte, 1, 2, . . . , \{a1 , a2 , . . . , ak } ⊆ {1, 2, . . . , 2003}\A şi are cel mult
a1
" # " #
2003 2003
43 de elemente. De aici deducem inegalitatea −k ≤ 43, k ≥ −43.
a1 a1
Anterior demonstrasem că 43 ≥ a1 − 1 + k. Combinând cele două inegalităţi,
" #
2003 2003
obţinem 43 ≥ a1 − 1 + k ≥ a1 + − 44 > a1 + − 45. Deoarece
a1 a1
2003
|A| = 1960, a1 ≤ 44. Derivata funcţiei f : [1, 44] → R, f (x) = x + − 45
x
2003
este f 0 (x) = 1 − < 0(442 < 2003) şi f este deci descrescătoare.
x2
2003
43 > f (a1 ) ≥ f (44) = 44 + − 45 > 44 + 45 − 45 = 44; contradicţie.
44
Observaţie. Cu aceeaşi tehnică se poate arăta că ∀ n ∈ N, n ≥ 2, avem că

n − [ n] = max{k|∃ A ⊆ {1, 2, . . . , n}, |A| = k şi ab 6∈ A, ∀ a, b ∈ A}.

20. a) Cazurile n = 3 şi n = 4 se tratează separat. Fie n ≥ 5.


Presupunem bi−1 + bi+1 = 2bi ∀ i ∈ 1, n − 1 şi deci i(ai−1 + ai+1 − 2ai ) =
ai−1 − ai+1 , adică i|ai−1 − ai+1 . Pentru i = n − 1 rezultă n − 1|an−2 − an . Cum
an−2 6= an şi ai ∈ 1, n, rezultă că |an − an−2 | = n − 1 şi deci: 1) an−2 = 1,
an = n sau 2) an−2 = n, an = 1. Pentru i = n − 2 rezultă |an−3 − an−1 | = n − 2
şi cum an−3 , an−1 ∈ 2, n − 1, rezultă contradicţia |an−1 − an−3 | > n − 3.
b) Dacă b1 , b2 , . . . , bn sunt ı̂n progresie geometrică, avem (b1 bn )n =
4
(b1 b2 . . . bn )2 = (n!)4 şi deci b1 bn = (n!) n . Fie n ≥ 3 şi p prim maxim, p ≤ n.

303
 
n
Din postulatul lui Bertrand rezultă că p ∈ , n şi deci 2p > n, adică p2 ¤|¤n!
2
4
şi deci pentru ca b1 bn ∈ N este necesar ca ∈ N şi deci n = 4. Pentru n = 4
n
avem b1 b2 b3 b4 = (b1 b4 )2 = (1 · 2 · 3 · 4)2 şi deci b1 b4 = 23 · 3, adică a1 a4 = 6 şi
deci a1 , a4 ∈ {2, 3} şi deci a2 , a3 ∈ {1, 4}.
În nici unul dintre cazuri nu putem avea b1 , b2 , b3 , b4 ı̂n progresie geometrică.
(L. Panaitopol )

21. Descompunem ı̂n fracţii simple


n! A1 A2 An
= + + ··· + . (1)
(x + 1)(x + 2) . . . (x + n) x+1 x+2 x+n
Înmulţim cu x + k şi ı̂nlocuim ı̂n identitatea obţinută x = −k. Obţinem
n!
Ak = =
(−k + 1)(−k + 2) . . . (−k + (k − 1))(−k + (k + 1) . . . (−k + n)
(−1)k−1 n!
= n! = (−1)k−1 k = (−1)k−1 k Cnk .
(k − 1)!(n − k)! k!(n − k)!
Punem ı̂n (1) x = x1 − 1 şi obţinem identitatea
Cn1 2Cn2 3Cn3 (n − 1)Cnn−1
− + + · · · + (−1)n−2 =
x1 x2 x3 xn−1
n! (−1)n n
= + = an . (2)
x1 x2 . . . xn xn
Deoarece xk |kCnk ∀ k = 1, n − 1, deducem din (2) că an ∈ Z. Să presupunem
că x1 > 2; rezultă xk > k + 1 ∀ k = 1, n.
n! n 1 n
Din (2) deducem că |an | < + = + = 1. Cum
2 · 3 · . . . · (n + 1) n + 1 n + 1 n + 1
n! (−1)n+1 n
|an | ∈ N şi |an | < 1, rezultă an = 0, = , n este impar şi
x1 x2 . . . xn xn
x1 x2 . . . xn−1 = (n − 1)!. Ultima egalitate este imposibilă căci x1 x2 . . . xn−1 >
2 · 3 · . . . · n > (n − 1)!.
(L. Panaitopol )

22. a) Fie m, a1 , a2 , . . . , am cu proprietăţile cerute. Nu putem avea ai ≥ 5


pentru 1 ≤ i ≤ m. Dacă s-ar ı̂ntâmpla aşa ceva, ı̂nlocuim numerele a1 , . . . , am
cu a1 , . . . , ai−1 , ai+1 , . . . , am , 2, ai − 2. Suma este aceeaşi, dar produsul este
mai mare strict decât cel iniţial, deoarece 2(ai − 2) > ai (fiindcă ai ≥ 5). Deci
ai ∈ {1, 2, 3, 4} ∀ i = 1, m. Deoarece 2 + 2 = 4, 2 · 2 = 4, putem presupune

304
că ai ∈ {1, 2, 3} ∀ i = 1, m. Nu putem avea ai = 1, căci ı̂nlocuim numerele
noastre cu (a1 + 1), a2 , . . . , ai−1 , ai+1 , . . . , am . Suma este aceeaşi, dar produsul
ar fi mai mare strict decât cel iniţial. Deci ai ∈ {2, 3} ∀ i = 1, m. Nu putem
avea trei de 2, căci 2 + 2 + 2 = 3 + 3, dar 2 · 2 · 2 < 3 · 3. Din aceste observaţii
obţinem că:
i) dacă n = 3k, atunci m = k, a1 = a2 = · · · = ak = 3, ii) dacă n = 3k + 2,
m = k + 1, a1 = a2 = · · · = ak = 3, ak+1 = 2, iii) dacă n = 3k + 4, m = k + 2,
a1 = a2 = · · · = ak = 3, ak+1 = ak+2 = 2.
b) Fie m, a1 , a2 , . . . , am cu proprietăţile cerute. Se arată prin inducţie după
k ≥ 2 că oricare ar fi a ∈ N, a ≥ 3, are loc inegalitatea ak > (a − 1)k + 2k . De
aici deducem că nu avem un ai care să fie mai mare sau egal cu 3. Dacă am avea
ai ≥ 3, atunci ı̂nlocuim pe aki cu (ai −1)k +2k +(aki −(ai −1)k −2k )1k . Suma se
păstrează aceeaşi, dar produsul devine mai mare strict deoarece 2(ai − 1) > ai .
Deci ai ∈ {1, 2} ∀ i = 1, m. 1 apare cel mai mult de 2k − 1 ori. Avem că
   
n n
m = k
+ n − 2 k , a1 = a2 = · · · = a” n — = 2, a” n — = ··· =
2 2k 2k 2k
+1
a” n — ” — = 1.
k n
+n−2
2k 2k

23. Să presupunem că enunţul n-ar fi adevărat. Ar exista deci k ∈ N,


0 ≤ k ≤ n − 2, astfel ı̂ncât k 2 + k + n = pt, unde p este prim şi t ∈ N,
t ≥ 2. Alegându-l pe p cel mai mic divizor prim al lui k 2 + k + n, avem că
p2 ≤ pt = k 2 + k + n ≤ (n − 2)2 + (n − 2) + n = n2 − 2n + 2 < n2 şi p < n.
Alegem acum pe p ca fiind cel mai mic număr prim cu proprietatea că p < n
.
şi că ∃ 0 ≤ k ≤ n − 2 astfel ı̂ncât k 2 + k + n..p (presupunerea făcută mai
sus ne asigură existenţa lui p). De asemenea, alegem pe k, ca fiind cel mai
mic număr natural din intervalul [0, n − 2], cu proprietatea că p|k 2 + k + n.
p−1
Evident că 0 ≤ k ≤ p − 1. De fapt, rezultă că k ≤ (p 6= 2; dacă p = 2,
2
rezultă că 2|n. Însă din enunţ ştim că n = 02 + 0 + n este prim. Deci n = 2 şi
p+1
enunţul este evident ı̂n acest caz). Dacă cumva ≤ k ≤ p − 1, obţinem o
2
2 2
contradicţie, căci (p − k − 1) + (p − k − 1) + n ≡ k + k + n ≡ 0 (mod p) şi
p+1 p−3
0 ≤ p−1−k ≤ p−1− = < k; aceasta contravine minimalităţii
2 2
2 ∗
lui k. Avem că k + k + n = pt, t ∈ N . Nu putem avea t = 1, căci am obţine
contradicţia n > p = k 2 + k + n ≥ n. Deci t > 1. Datorită minimalităţii lui
 
2 2
p−1 2 p−1
p deducem că t ≥ p. Deci p ≤ pt = k + k + n ≤ + + n,
2 2

305
s
p2 1 4n − 1
p2 ≤ − + n, 3p2 ≤ 4n − 1, p ≤ . Pe de altă parte, conform
4 4 s 3
n p−1
ipotezei din enunţ, avem că <k≤ . Combinând aceste inegalităţi
3 2s
s 4n − 1 s
n p−1 −1 n 1 1
cu precedenta, obţinem că ≤ ≤ 3 = − − <
s 3 2 2 3 12 2
n 1
− . Am obţinut o contradicţie şi problema este rezolvată.
3 12
Observaţie. Există doar 6 numere n cu proprietatea din enunţ. Ele sunt
n = 2, 3, 5, 11, 17, 41. Acest lucru a fost conjecturat de Gauss ı̂n 1801 şi demon-
strat independent de Stark şi Baker ı̂n 1967.

b2n + bn+1 + 3b − 5 yn
24. Avem b ≥ 6 şi xn = = . Pentru b = 10 avem
b−1 b−1
! 2
10n + 5
xn = . Dacă xn este pătrat pentru orice n, rezultă că xn xn+1 =
3
yn yn+1
, adică yn yn+1 este pătrat perfect pentru orice n. Verificările făcute
(b − 1)2
pentru b ∈ 0, 9 arată că xn nu este pătrat perfect pentru orice n. Rămâne deci
b ≥ 11. Avem
 2
2n n+1 b n
yn = b +b + 3b − 5 < b + (1)
2
şi deci
 2  2 ‚ Œ2
b n n+1 b 2n+1 (b + 1) n+1 b2
yn yn+1 < b + b + = b + b + . (2)
2 2 2 4
!2
b + 1 n+1
Stabilim o inegalitate de forma yn yn+1 > b2n+1 + b −x . Avem
2
€ Š€ Š
yn yn+1 > b2n + bn+1 b2n+2 + bn+2 şi alegem x pentru care

!2
€ Š€ Š b + 1 n+1
2n n+1 2n+2 n+2 2n+1
b +b b +b > b + b −x ⇔
2
!  2
2n+1
b + 1 n+1 b+1
⇔ 2x b + b + b2n+3 > b2n+2 + x2 .
2 2

306
Putem alege x = b3 şi avem
‚ Œ2
2n+1 bn+1 (b + 1)
yn yn+1 > b + − b3 . (3)
2
Cum yn · yn+1 este pătrat perfect, rezultă că există an ∈ Z astfel ca
‚ Œ2
2n+1 bn+1 (b + 1)
yn yn+1 = b + + an . (4)
2
Din (2) şi (3) rezultă pentru orice n
b2
> an > −b3 . (5)
4
Relaţia (4) se va scrie
‚ Œ ‚ Œ2
bn+1 (b + 1) bn+1 (b + 1)
a2n + 2an b 2n+1
+ + b 2n+1
+ =
2 2
€ Š€ Š
= b2n + bn+1 + 3b − 5 b2n+2 + bn+2 + 3b − 5 (6)
Rezultă că bn |a2n − (3b − 5)2 pentru orice n. Cum an este mărginit (vezi
5), rezultă a2n = (3b − 5)2 , adică an = ±(3b − 5). Revenind la (6),
n+1 (b + 1)
!
2n+1
b b2n+2 (b + 1)2
avem ±2(3b − 5) b + + = b2n+3 + (3b − 5)·
2 4
€ Š
b2n+2 + b2n + bn+2 + bn+1 . Împărţim cu b2n şi facem n → ∞ ı̂n relaţia
b2 (b + 1)2
precedentă. Obţinem ±2(3b − 5)b + = b3 + (3b − 5)(b2 + 1). Avem
4
±8b(3b−5)+b2 (b+1)2 = 4b3 +4(3b−5)(b2 +1). b este deci rădăcina ı̂ntreagă a
unui polinom cu coeficienţi ı̂ntregi f (x) = x4 − 14x3 + ex2 + f x + 20. Deducem
că b|20. Cum b ≥ 11, rezultă b = 20. Din egalitatea precedentă rezultă că
55 = 3b − 5|b2 (b + 1)2 − 4b3 = 144400, ceea ce constituie o contradicţie.
(L. Panaitopol )
Soluţia a doua. a) Avem xn = b2n−1 + b2n−2 + · · · + bn+1 + 2 · bn + 2 ·
bn−1 + · · · + 2 · b + 5 = yn2 ∀ n ≥ n0 . Arătăm că b este par. Să presupunem
că b = 4k + 1. Alegem n ≥ n0 , n ≡ 1 (mod 4) şi avem yn2 ≡ xn ≡ (n − 1) +
2n + 5 = 3n + 4 ≡ 3 (mod 4), ceea ce constituie o contradicţie, căci t2 ≡ 1
(mod 4) pentru t impar. Dacă b = 4k + 3 alegem n impar, n ≥ n0 şi avem
€ Š € Š
yn2 ≡ xn ≡ b2n−1 + b2n−2 + · · · + b + 1 + bn + bn−1 + · · · + b + 1 + 3 ≡
(−1 + 1 − 1 · · · − 1 + 1) + (−1 + 1 · · · − 1 + 1) + 3 ≡ 3 (mod 4) şi am obţinut
din nou o contradicţie. Deci b este par, b ≥ 6.
b) Arătăm că b − 1 = 3k . Să presupunem că ∃ p prim, p 6= 3, astfel
ı̂ncât p|b − 1. Deoarece b este par, deducem că b − 1 este impar şi p 6= 2. Fie

307
 
a
a ≥ 3n0 + 4 astfel ı̂ncât = −1. Deoarece p 6= 3, numerele a, a + p, a + 2p
p
sunt necongruente modulo 3 şi unul din ele, pe care-l notăm c, este congruent
cu 1 modulo 3. Avem c = 3n + 4 ≥ a ≥ 3n0 + 4 şi deci n ≥ n0 .
Avem că b ≡ 1 (mod p) şi yn2 = xn ≡ 3n + 4 = c ≡ a (mod p) şi obţinem
!  
yn2 a
contradicţia 1 = = = −1. Deci b = 3k + 1, k ≥ 2.
p p
c) Arătăm că numărul k de mai sus este par.
Să presupunem că k = 2t + 1. Luăm n impar, n ≥ n0 , ı̂n formula
2n−1
X n
X
yn2 = xn = bj + bj + 3 ≡ 3 (mod (b + 1)). Deducem că simbolul Ja-
j=0 j=0
 
3
cobi este 1. Avem că b + 1 ≡ 32t+1 + 2 ≡ 3 + 2 ≡ 1 (mod 4) şi
b+1
b + 1 = 32t+1 + 2 ≡ 2 (mod 3). Folosim proprietatea analogă legii de recipro-
 
3
citate pentru simbolul Jacobi (curs, pagina 111) şi deducem că 1 = =
b+1
     
3−1 b
·
b+1 b+1 2
(−1) 2 2 = = = −1. Am obţinut o contradicţie şi am
3 3 3
demonstrat deci că k este par.
b2n − 1 bn+1 − 1
d) Arătăm că b = 10. Avem că yn2 = xn = + + 3,
b−1 b−1
k
zn2 = (b − 1)yn2 = b2n + bn+1 + 3b − 5, unde zn = a · yn , a = 3 2 (am arătat mai
 
k 2 n
b 2 b2
sus că b − 1 = 3 , k fiind număr par). Deci zn = b + + 3b − 5 − .
2 4
b2
Din toate consideraţiile precedente rezultă că ecuaţia z 2 − x2 = 3b − 5 − are
4
k b
o infinitate de soluţii ı̂n N (z = zn = 3 2 yn şi x = bn + ). Acest lucru se poate
2
b2
ı̂ntâmpla doar dacă 3b − 5 − = 0. Avem b2 − 12b + 20 = 0, (b − 2)(b − 10) = 0
4
şi deci singurele soluţii sunt b = 2 şi b = 10. Nu convine decât b = 10 căci
b ≥ 6.
(Soluţie A. Gica)
Observaţie. În ambele soluţii s-a folosit ipoteza mai slabă: ”xn este pătrat
perfect pentru n suficient de mare”.

25. Presupunem prin absurd că există trei puncte necoliniare A, B, C ı̂n
mulţimea M. Să arătăm ı̂ntâi că pe dreapta BC există un număr finit de puncte

308
din M. Să presupunem contrariul: (An )n∈N ⊆ M ∩ BC. −AB < AAi − BAi <
AB, ∀ i ∈ N. Deoarece AAi − BAi ∈ Z, deducem că ∃ i < j astfel ı̂ncât
AAi − BAi = AAj − BAj . Pentru comoditate presupunem că i = 1 şi j = 2.
A1 A2 = BA2 − BA1 = AA2 − AA1 ; contradicţie cu inegalitatea triunghiului.
Deci |M ∩ BC| < ∞. Fie (Bn )n∈N ⊆ M\BC. Deoarece −BC < Bn B − Bn C <
BC şi Bn B − Bn C ∈ Z, există un subşir nk astfel ı̂ncât Bnk B − Bnk C = α,
−BC < α < BC. Pentru comoditate, subşirul (Bnk )k ı̂l notăm tot cu (Bn )n .
Există deci un şir infinit Bn ⊆ M ∩ H1 , unde H1 este hiperbola de focare B, C
şi constantă α. Fie două puncte ı̂n M ∩ H1 . Să presupunem că acestea sunt A
şi D. Facem acelaşi raţionament ca mai sus cu (Bn )n ı̂n loc de M şi cu A, D
ı̂n loc de B, C. Deducem existenţa unui subşir (Bnk )k ⊆ (Bn )n ∩ H2 , unde H2
este hiperbola de focare A, D şi constantă β (−AD < β < AD). Deducem că
H1 ∩ H2 ar fi o mulţime infinită, căci conţine şirul de puncte (Bnk )k . Acest
lucru este ı̂nsă imposibil, căci două hiperbole distincte au cel mult 4 puncte
comune. H1 şi H2 sunt distincte fiindcă perechile de focare (B, C şi A, D)
sunt distincte. Am obţinut o contradicţie şi deci enunţul teoremei lui Erdös
este adevărat.

309

S-ar putea să vă placă și